You are on page 1of 186

especially that such law failed to provide a scheme to justly

compensate the discount.


GOVERNMENTAL POWERS and INDIVIDUAL RIGHTS
AND FREEDOM (THE BILL OF RIGHTS)

ISSUE: WON Section 4(a) of the Expanded Senior Citizens


Act is unconstitutional or not violative of Article 3 Section 9 of
A. FUNDAMENTAL POWERS AND THE BILL OF the Constitution which provides that private property shall not
RIGHTS be taken for public use without just compensation and the
equal protection clause of Article 3 Section 1.

1. CARLOS SUPERDRUG CORP. vs. DSWD, ET. AL


HELD: The permanent reduction in their total revenues is a
forced subsidy corresponding to the taking of private property
FACTS: Petitioners, belonging to domestic corporations and for public use or benefit. This constitutes compensable taking
proprietors operating drugstores in the Philippines, are for which petitioners would ordinarily become entitled to a just
praying for preliminary injunction assailing the constitutionality compensation. Just compensation is defined as the full and
of Section 4(a) of Republic Act (R.A.) No. 9257, otherwise fair equivalent of the property taken from its owner by the
known as the “Expanded Senior Citizens Act of 2003.” On expropriator. The measure is not the taker’s gain but the
February 26, 2004, R.A. No. 9257, amending R.A. No. 7432, owner’s loss. The word just is used to intensify the meaning of
was signed into law by President Gloria Macapagal-Arroyo the word compensation, and to convey the idea that the
and it became effective on March 21, 2004. Section 4(a) of the equivalent to be rendered for the property to be taken shall be
Act states: real, substantial, full and ample.

SEC. 4. Privileges for the Senior Citizens. – The The law grants a twenty percent discount to senior citizens for
senior citizens shall be entitled to the following: medical and dental services, and diagnostic and laboratory
fees; admission fees charged by theaters, concert halls,
(a) the grant of twenty percent (20%) discount from circuses, carnivals, and other similar places of culture, leisure
all establishments relative to the utilization of and amusement; fares for domestic land, air and sea travel;
services in hotels and similar lodging establishments, utilization of services in hotels and similar lodging
restaurants and recreation centers, and purchase of establishments, restaurants and recreation centers; and
medicines in all establishments for the exclusive use purchases of medicines for the exclusive use or enjoyment of
or enjoyment of senior citizens, including funeral and senior citizens. As a form of reimbursement, the law provides
burial services for the death of senior citizens; that business establishments extending the twenty percent
discount to senior citizens may claim the discount as a tax
The establishment may claim the discounts granted under (a),
deduction.
(f), (g) and (h) as tax deduction based on the net cost of the
goods sold or services rendered: Provided, That the cost of The law is a legitimate exercise of police power which, similar
the discount shall be allowed as deduction from gross income to the power of eminent domain, has general welfare for its
for the same taxable year that the discount is granted. object. Police power is not capable of an exact definition, but
Provided, further, That the total amount of the claimed tax has been purposely veiled in general terms to underscore its
deduction net of value added tax if applicable, shall be comprehensiveness to meet all exigencies and provide
included in their gross sales receipts for tax purposes and enough room for an efficient and flexible response to
shall be subject to proper documentation and to the provisions conditions and circumstances, thus assuring the greatest
of the National Internal Revenue Code, as amended. benefits. Accordingly, it has been described as “the most
essential, insistent and the least limitable of powers, extending
The DSWD, on May 8, 2004, approved and adopted the
as it does to all the great public needs.” It is “[t]he power
Implementing Rules and Regulations of RA No. 9275, Rule VI,
vested in the legislature by the constitution to make, ordain,
Article 8 which contains the proviso that the implementation of
and establish all manner of wholesome and reasonable laws,
the tax deduction shall be subject to the Revenue Regulations
statutes, and ordinances, either with penalties or without, not
to be issued by the BIR and approved by the DOF. With the
repugnant to the constitution, as they shall judge to be for the
new law, the Drug Stores Association of the Philippines
good and welfare of the commonwealth, and of the subjects
wanted a clarification of the meaning of tax deduction. The
of the same.”
DOF clarified that under a tax deduction scheme, the tax
deduction on discounts was subtracted from Net Sales
together with other deductions which are considered as
operating expenses before the Tax Due was computed based
on the Net Taxable Income. On the other hand, under a tax
credit scheme, the amount of discounts which is the tax credit
item, was deducted directly from the tax due amount.

The DOH issued an Administrative Order that the twenty


percent discount shall include both prescription and non-
prescription medicines, whether branded or generic. It stated
that such discount would be provided in the purchase of
medicines from all establishments supplying medicines for the
exclusive use of the senior citizens.

Drug store owners assail the law with the contention that
granting the discount would result to loss of profit and capital

Page 1 of 186
2. Drug Stores Association of the Philippines vs The establishments may claim the discounts granted in
National Council on Disability Affairs sub-sections (a), (b), (c), (f) and (g) as tax deductions based
on the net cost of the goods sold or services rendered:
Provided, however, That the cost of the discount shall be
allowed as deduction from gross income for the same taxable
FACTS: On March 24, 1992, Republic Act (R.A.) No. 7277,
year that the discount is granted: Provided, further, That the
entitled "An Act Providing for the Rehabilitation, Self-
total amount of the claimed tax deduction net of value-added
Development and Self-Reliance of Disabled Persons and their
tax if applicable, shall be included in their gross sales receipts
Integration into the Mainstream of Society and for Other
for tax purposes and shall be subject to proper documentation
Purposes," otherwise known as the "Magna Carta for Disabled
and to the provisions of the National Internal Revenue Code
Persons," was passed into law. The law defines "disabled
(NIRC), as amended.
persons", "impairment" and "disability" as follows:
The Implementing Rules and Regulations (IRR) of R.A. No.
SECTION 4. Definition of Terms. - For purposes of
9442 was jointly promulgated by the Department of Social
this Act, these terms are defined as follows:
Welfare and Development (DSWD), Department of Education,
(a) Disabled Persons are those suffering from Department of Finance (DOF), Department of Tourism,
restriction of different abilities, as a result of a mental, Department of Transportation and Communication,
physical or sensory impairment, to perform an Department of the Interior and Local Government (DILG) and
activity in the manner or within the range considered Department of Agriculture. Insofar as pertinent to this petition,
normal for a human being; the salient portions of the IRR are hereunder quoted:

(b) Impairment is any loss, diminution or aberration


of psychological, physiological, or anatomical
RULE IV. PRIVILEGES AND INCENTIVES FOR THE
structure of function;
PERSONS WITH DISABILITY
(c) Disability shall mean (1) a physical or mental
Section 6. Other Privileges and Incentives. Persons with
impairment that substantially limits one or more
disability shall be entitled to the following:
psychological, physiological or anatomical function
of an individual or activities of such individual; (2) a 6.1.d. Purchase of Medicine - at least twenty percent
record of such an impairment; or (3) being regarded (20%) discount on the purchase of medicine for the
as having such an impairment. exclusive use and enjoyment of persons with
disability. All drugstores, hospital, pharmacies,
On April 30, 2007, Republic Act No. 9442[7] was enacted
clinics and other similar establishments selling
amending R.A. No. 7277. The Title of R.A. No. 7277 was
medicines are required to provide at least twenty
amended to read as "Magna Carta for Persons with Disability"
percent (20%) discount subject to the guidelines
and all references on the law to "disabled persons" were
issued by DOH and PHILHEALTH.
amended to read as "persons with disability" (PWD).
Specifically, R.A. No. 9442 granted the PWDs a twenty (20) 6.11 The abovementioned privileges are available
percent discount on the purchase of medicine, and a tax only to persons with disability who are Filipino
deduction scheme was adopted wherein covered citizens upon submission of any of the following as
establishments may deduct the discount granted from gross proof of his/her entitlement thereto subject to the
income based on the net cost of goods sold or services guidelines issued by the NCWDP in coordination with
rendered: DSWD, DOH and DILG.
CHAPTER 8. Other Privileges and Incentives. SEC. 6.11.1 An identification card issued by the city or
32. Persons with disability shall be entitled to the municipal mayor or the barangay captain of the place
following: where the person with disability resides;
(d) At least twenty percent (20%) discount for the 6.11.2 The passport of the persons with disability
purchase of medicines in all drugstores for the concerned; or
exclusive use or enjoyment of persons with
disability; 6.11.3 Transportation discount fare Identification
Card (ID) issued by the National Council for the
The abovementioned privileges are available only to Welfare of Disabled Persons (NCWDP). However,
persons with disability who are Filipino citizens upon upon effectivity of this Implementing Rules and
submission of any of the following as proof of his/her Regulations, NCWDP will already adopt the
entitlement thereto: Identification Card issued by the Local Government
Unit for purposes of uniformity in the implementation.
(i) An identification card issued by the city or
NCWDP will provide the design and specification of
municipal mayor or the barangay captain of
the identification card that will be issued by the Local
the place where the person with disability
Government Units.
resides;
(ii) The passport of the person with disability 6.14. Availment of Tax Deductions by Establishment
concerned; or Granting Twenty Percent. 20% Discount - The
(iii) Transportation discount fare Identification establishments may claim the discounts granted in
Card (ID) issued by the National Council for sub-sections (6.1), (6.2), (6.4), (6.5) and (6.6) as tax
the Welfare of Disabled Persons (NCWDP). deductions based on the net cost of the goods sold
or services rendered: Provided, however, that the
cost of the discount shall be allowed as deduction
from gross income for the same taxable year that the

Page 2 of 186
discount is granted: Provided, further, That the total can only deduct the 20% discount from their gross income
amount of the claimed tax deduction net of value- subject to some conditions.
added tax if applicable, shall be included in their
gross sales receipts for tax purposes and shall be On May 20, 2009, the DOH issued A.O. No. 2009-0011
subject to proper documentation and to the specifically stating that the grant of 20% discount shall be
provisions of the National Internal Revenue Code, as provided in the purchase of branded medicines and
amended. unbranded generic medicines from all establishments
dispensing medicines for the exclusive use of the PWDs. It
On April 23, 2008, the National Council on Disability Affairs also detailed the guidelines for the provision of medical and
(NCDA) issued Administrative Order (A.O.) No. 1, Series of related discounts and special privileges to PWDs pursuant to
2008, prescribing guidelines which should serve as a R.A. 9442.
mechanism for the issuance of a PWD Identification Card
(IDC) which shall be the basis for providing privileges and On July 28, 2009, petitioners filed a Petition for Prohibition with
discounts to bona fide PWDs in accordance with R.A. 9442: application for a Temporary Restraining Order and/or a Writ of
Preliminary Injunction before the Court of Appeals to annul
IV. INSTITUTIONAL ARRANGEMENTS and enjoin the implementation of the following laws:

The Local Government Unit of the City or Municipal Office 1) Section 32 of R.A. No. 7277 as amended by R.A.
shall implement these guidelines in the issuance of the PWD- No. 9442;
IDC
2) Section 6, Rule IV of the Implementing Rules and
D. Issuance of the appropriate document to confirm the Regulations of R.A. No. 9442;
medical condition of the applicant is as follows:
3) NCDA A.O. No. 1;
Disability Document Issuing Entity
Apparent Medical Licensed Private 4) DOF Revenue Regulation No. 1-2009;
Disability Certificate or Government
5) DOH A.O. No. 2009-0011.
Physician
School Licensed On July 26, 2010, the CA rendered a Decision upholding the
Assessment Teacher duly constitutionality of R.A. 7277 as amended, as well as the
signed by the
assailed administrative issuances. However, the CA
School Principal
suspended the effectivity of NCDA A.O. No. 1 pending proof
Certificate of Head of the of respondent NCDA's compliance with filing of said
Disability Business administrative order with the Office of the National
Establishment or Administrative Register (ONAR) and its publication in a
Head of Non- newspaper of general circulation. The dispositive portion of
Government the Decision states:
Organization
Non-Apparent Medical Licensed Private WHEREFORE, the petition is PARTLY GRANTED.
Disability Certificate or Government The effectivity of NCDA Administrative Order No. 1 is
Physician hereby SUSPENDED pending Respondent's
compliance with the proof of filing of NCDA
Administrative Order No. 1 with the Office of the
E. PWD Registration Forms and ID Cards shall be issued and National Administrative Register and its publication
signed by the City or Municipal Mayor, or Barangay Captain. in a newspaper of general circulation.

Respondent NCDA filed a motion for reconsideration before


the CA to lift the suspension of the implementation of NCDA
V. IMPLEMENTING GUIDELINES AND PROCEDURES
A.O. No. 1 attaching thereto proof of its publication in the
Any bonafide person with permanent disability can apply for Philippine Star and Daily Tribune on August 12, 2010, as well
the issuance of the PWD-IDC. His/her caregiver can assist in as a certification from the ONAR showing that the same was
the application process. Procedures for the issuance of the ID filed with the said office on October 22, 2009. Likewise,
Cards are as follows: petitioners filed a motion for reconsideration of the CA
Decision.
A. Completion of the Requirements. Complete and/or
make available the following requirements: In a Resolution dated November 19, 2010, the CA dismissed
petitioners' motion for reconsideration and lifted the
Two "1x1" recent ID pictures with the names, and suspension of the effectivity of NCDA A.O. No. 1 considering
signatures or thumbmarks at the back of the picture the filing of the same with ONAR and its publication in a
newspaper of general circulation.
One (1) Valid ID
Hence, the instant petition.
Document to confirm the medical or disability
condition (See Section IV, D for the required
document).
ISSUE/S:
On December 9, 2008, the DOF issued Revenue Regulations
No. 1-2009 prescribing rules and regulations to implement WHETHER OR NOT CA SERIOUSLY ERRED ON A
R.A. 9442 relative to the tax privileges of PWDs and tax QUESTION OF SUBSTANCE WHEN IT RULED THAT THE
incentives for establishments granting the discount. Section 4 MANDATED PWD DISCOUNT IS A VALID EXERCISE OF
of Revenue Regulations No. 001-09 states that drugstores POLICE POWER. ON THE CONTRARY, IT IS AN INVALID
EXERCISE OF THE POWER OF EMINENT DOMAIN

Page 3 of 186
BECAUSE IT FAILS TO PROVIDE JUST COMPENSATION individuals are subjected to restraints and burdens in order to
TO PETITIONERS AND OTHER SIMILARLY SITUATED secure the general comfort, health, and prosperity of the state.
DRUGSTORES; A legislative act based on the police power requires the
concurrence of a lawful subject and a lawful method. In more
WHETHER OR NOT CA SERIOUSLY ERRED WHEN IT familiar words, (a) the interests of the public generally, as
RULED THAT SECTION 32 OF RA 7277 AS AMENDED BY distinguished from those of a particular class, should justify
RA 9442, NCDA AO 1 AND THE OTHER IMPLEMENTING the interference of the state; and (b) the means employed are
REGULATIONS DID NOT VIOLATE THE DUE PROCESS reasonably necessary for the accomplishment of the purpose
CLAUSE; and not unduly oppressive upon individuals.
WHETHER OR NOT THE CA SERIOUSLY ERRED WHEN IT R.A. No. 7277 was enacted primarily to provide full support to
RULED THAT THE DEFINITIONS OF DISABILITIES UNDER the improvement of the total well-being of PWDs and their
SECTION 4(A), SECTION 4(B) AND SECTION 4(C) OF RA integration into the mainstream of society. The priority given
7277 AS AMENDED BY RA 9442, RULE 1 OF THE to PWDs finds its basis in the Constitution:
IMPLEMENTING RULES AND REGULATIONS[23] OF RA
7277, SECTION 5.1 OF THE IMPLEMENTING RULES AND ARTICLE XII: NATIONAL ECONOMY AND
REGULATIONS OF RA 9442, NCDA AO 1 AND DOH AO PATRIMONY
2009-11 ARE NOT VAGUE, AMBIGUOUS AND
UNCONSTITUTIONAL; xxxx

WHETHER OR NOT CA SERIOUSLY ERRED WHEN IT Section 6. The use of property bears a social
RULED THAT THE MANDATED PWD DISCOUNT DOES function, and all economic agents shall contribute to
NOT VIOLATE THE EQUAL PROTECTION CLAUSE. the common good. Individuals and private groups,
including corporations, cooperatives, and similar
collective organizations, shall have the right to own,
establish, and operate economic enterprises, subject
HELD: We deny the petition. to the duty of the State to promote distributive justice
and to intervene when the common good so
The CA is correct when it applied by analogy the case of
demands.
Carlos Superdrug Corporation et al. v. DSWD, et al. wherein
We pronounced that Section 4 of R.A. No. 9257 which grants ARTICLE XIII: SOCIAL JUSTICE AND HUMAN
20% discount on the purchase of medicine of senior citizens RIGHTS
is a legitimate exercise of police power:
xxxx
The law is a legitimate exercise of police power which, similar
to the power of eminent domain, has general welfare for its Section 11. The State shall adopt an integrated and
object. Police power is not capable of an exact definition, but comprehensive approach to health development
has been purposely veiled in general terms to underscore its which shall endeavor to make essential goods,
comprehensiveness to meet all exigencies and provide health and other social services available to all the
enough room for an efficient and flexible response to people at affordable cost. There shall be priority for
conditions and circumstances, thus assuring the greatest the needs of the underprivileged, sick, elderly,
benefits. Accordingly, it has been described as the most disabled, women, and children. The State shall
essential, insistent and the least limitable of powers, extending endeavor to provide free medical care to paupers.
as it does to all the great public needs. It is [t]he power vested
in the legislature by the constitution to make, ordain, and Thus, R.A. No. 7277 provides:
establish all manner of wholesome and reasonable laws,
SECTION 2. Declaration of Policy. The grant of the
statutes, and ordinances, either with penalties or without, not
rights and privileges for disabled persons shall be
repugnant to the constitution, as they shall judge to be for the
guided by the following principles:
good and welfare of the commonwealth, and of the subjects
of the same. (a). Disabled persons are part of the Philippine
society, thus the Senate shall give full support to the
For this reason, when the conditions so demand as
improvement of the total well-being of disabled
determined by the legislature, property rights must bow to the
persons and their integration into the mainstream of
primacy of police power because property rights, though
society.
sheltered by due process, must yield to general welfare.
Toward this end, the State shall adopt policies
Police power as an attribute to promote the common good
ensuring the rehabilitation, self-development and
would be diluted considerably if on the mere plea of petitioners
self-reliance of disabled persons.
that they will suffer loss of earnings and capital, the questioned
provision is invalidated. Moreover, in the absence of evidence It shall develop their skills and potentials to enable
demonstrating the alleged confiscatory effect of the provision them to compete favorably for available
in question, there is no basis for its nullification in view of the opportunities.
presumption of validity which every law has in its favor.
(b). Disabled persons have the same rights as other
Police power is the power of the state to promote public people to take their proper place in society. They
welfare by restraining and regulating the use of liberty and should be able to live freely and as independently as
property. On the other hand, the power of eminent domain is possible. This must be the concern of everyone - the
the inherent right of the state (and of those entities to which family, community and all government and non-
the power has been lawfully delegated) to condemn private government organizations.
property to public use upon payment of just compensation. In
the exercise of police power, property rights of private

Page 4 of 186
Disabled person's rights must never be perceived as resides; (ii) the passport of the PWD; or (iii) transportation
welfare services by the Government. discount fare identification card issued by NCDA. Petitioners,
thus, maintain that none of the said documents has any
xxxx relation to a medical finding of disability, and the grant of the
discount is allegedly without any process for the determination
(d). The State also recognizes the role of the private
of a PWD in accordance with law.
sector in promoting the welfare of disabled persons
and shall encourage partnership in programs that Section 32 of R.A. No. 7277, as amended by R.A. No. 9442,
address their needs and concerns. must be read with its IRR which stated that upon its effectivity,
NCWDP (which is the government agency tasked to ensure
To implement the above policies, R.A. No. 9442 which
the implementation of RA 7277), would adopt the IDC issued
amended R.A. No. 7277 grants incentives and benefits
by the local government units for purposes of uniformity in the
including a twenty percent (20%) discount to PWDs in the
implementation. Thus, NCDA A.O. No. 1 provides the
purchase of medicines; fares for domestic air, sea and land
reasonable guidelines in the issuance of IDCs to PWDs as
travels including public railways and skyways; recreation and
proof of their entitlement to the privileges and incentives under
amusement centers including theaters, food chains and
the law and fills the details in the implementation of the law.
restaurants. This is specifically stated in Section 4 of the IRR
of R.A. No. 9442: As stated in NCDA A.O. No. 1, before an IDC is issued by the
city or municipal mayor or the barangay captain, or the
Section 4. Policies and Objectives - It is the objective
Chairman of the NCDA,[42] the applicant must first secure a
of Republic Act No. 9442 to provide persons with
medical certificate issued by a licensed private or government
disability, the opportunity to participate fully into the
physician that will confirm his medical or disability condition. If
mainstream of society by granting them at least
an applicant is an employee with apparent disability, a
twenty percent (20%) discount in all basic services.
"certificate of disability" issued by the head of the business
It is a declared policy of RA 7277 that persons with
establishment or the head of the non-governmental
disability are part of Philippine society, and thus the
organization is needed for him to be issued a PWD-IDC. For
State shall give full support to the improvement of
a student with apparent disability, the "school assessment"
their total wellbeing and their integration into the
issued by the teacher and signed by the school principal
mainstream of society. They have the same rights as
should be presented to avail of a PWD-ID.
other people to take their proper place in society.
They should be able to live freely and as Petitioners' insistence that Part IV (D) of NCDA Administrative
independently as possible. This must be the concern Order No. 1 is void because it allows allegedly non-competent
of everyone the family, community and all persons like teachers, head of establishments and heads of
government and non-government organizations. Non-Governmental Organizations (NGOs) to confirm the
Rights of persons with disability must never be medical condition of the applicant is misplaced. It must be
perceived as welfare services. Prohibitions on stressed that only for apparent disabilities can the teacher or
verbal, non-verbal ridicule and vilification against head of a business establishment validly issue the mentioned
persons with disability shall always be observed at all required document because, obviously, the disability is easily
times. seen or clearly visible. It is, therefore, not an unqualified grant
of authority for the said non-medical persons as it is simply
Hence, the PWD mandatory discount on the purchase of
limited to apparent disabilities. For a non-apparent disability or
medicine is supported by a valid objective or purpose as
a disability condition that is not easily seen or clearly visible,
aforementioned. It has a valid subject considering that the
the disability can only be validated by a licensed private or
concept of public use is no longer confined to the traditional
government physician, and a medical certificate has to be
notion of use by the public, but held synonymous with public
presented in the procurement of an IDC. Relative to this issue,
interest, public benefit, public welfare, and public
the CA validly ruled, thus:
convenience. As in the case of senior citizens, the discount
privilege to which the PWDs are entitled is actually a benefit We agree with the Office of the Solicitor General's (OSG)
enjoyed by the general public to which these citizens belong. ratiocination that teachers, heads of business establishments
The means employed in invoking the active participation of the and heads of NGOs can validly confirm the medical condition
private sector, in order to achieve the purpose or objective of of their students/employees with apparent disability for
the law, is reasonably and directly related. Also, the means obvious reasons as compared to non-apparent disability
employed to provide a fair, just and quality health care to which can only be determined by licensed physicians. Under
PWDs are reasonably related to its accomplishment, and are the Labor Code, disabled persons are eligible as apprentices
not oppressive, considering that as a form of reimbursement, or learners provided that their handicap are not as much as to
the discount extended to PWDs in the purchase of medicine effectively impede the performance of their job. We find that
can be claimed by the establishments as allowable tax heads of business establishments can validly issue
deductions pursuant to Section 32 of R.A. No. 9442 as certificates of disability of their employees because aside from
implemented in Section 4 of DOF Revenue Regulations No. the fact that they can obviously validate the disability, they also
1-2009. Otherwise stated, the discount reduces taxable have medical records of the employees as a pre-requisite in
income upon which the tax liability of the establishments is the hiring of employees. Hence, Part IV (D) of NCDA AO No.
computed. 1 is logical and valid.
Further, petitioners aver that Section 32 of R.A. No. 7277 as Furthermore, DOH A.O. No. 2009-11 prescribes additional
amended by R.A. No. 9442 is unconstitutional and void for guidelines for the 20% discount in the purchase of all
violating the due process clause of the Constitution since medicines for the exclusive use of PWD. To avail of the
entitlement to the 20% discount is allegedly merely based on discount, the PWD must not only present his I.D. but also the
any of the three documents mentioned in the provision, doctor's prescription stating, among others, the generic name
namely: (i) an identification card issued by the city or municipal of the medicine, the physician's address, contact number and
mayor or the barangay captain of the place where the PWD professional license number, professional tax receipt number

Page 5 of 186
and narcotic license number, if applicable. A purchase booklet capabilities by the administrative agency charged with
issued by the local social/health office is also required in the implementing a particular statute.
purchase of over-the-counter medicines. Likewise, any single
dispensing of medicine must be in accordance with the Lastly, petitioners contend that R.A. No. 7227, as amended by
prescription issued by the physician and should not exceed a R.A. No. 9442, violates the equal protection clause of the
one (1) month supply. Therefore, as correctly argued by the Constitution because it fairly singles out drugstores to bear the
respondents, Section 32 of R.A. No. 7277 as amended by R.A. burden of the discount, and that it can hardly be said to
No. 9442 complies with the standards of substantive due "rationally" meet a legitimate government objective which is
process. the purpose of the law. The law allegedly targets only retailers
such as petitioners, and that the other enterprises in the drug
We are likewise not persuaded by the argument of petitioners industry are not imposed with similar burden. This same
that the definition of "disabilities" under the subject laws is argument had been raised in the case of Carlos Superdrug
vague and ambiguous because it is allegedly so general and Corp., et al. v. DSWD, et al., and We reaffirm and apply the
broad that the person tasked with implementing the law will ruling therein in the case at bar:
undoubtedly arrive at different interpretations and applications
of the law. Aside from the definitions of a "person with The Court is not oblivious of the retail side of the
disability" or "disabled persons" under Section 4 of R.A. No. pharmaceutical industry and the competitive pricing
7277 as amended by R.A. No. 9442 and in the IRR of RA component of the business. While the Constitution
9442, NCDA A.O. No. 1 also provides: protects property rights, petitioners must accept the
realities of business and the State, in the exercise of
Identification Cards shall be issued to any bonafide PWD with police power, can intervene in the operations of a
permanent disabilities due to any one or more of the following business which may result in an impairment of
conditions: psychosocial, chronic illness, learning, mental, property rights in the process.
visual, orthopedic, speech and hearing conditions. This
includes persons suffering from disabling diseases resulting to Moreover, the right to property has a social dimension. While
the person's limitations to do day to day activities as normally Article XIII of the Constitution provides the precept for the
as possible such as but not limited to those undergoing protection of property, various laws and jurisprudence,
dialysis, heart disorders, severe cancer cases and such other particularly on agrarian reform and the regulation of contracts
similar cases resulting to temporary or permanent disability. and public utilities, continuously serve as a reminder that the
right to property can be relinquished upon the command of the
Similarly, DOH A.O. No. 2009-0011 defines the different State for the promotion of public good.
categories of disability as follows:
Under the equal protection clause, all persons or things
Rule IV, Section 4, Paragraph B of the Implementing similarly situated must be treated alike, both in the privileges
Rules and Regulations (IRR) of this Act required the conferred and the obligations imposed. Conversely, all
Department of Health to address the health concerns persons or things differently situated should be treated
of seven (7) different categories of disability, which differently. In the case of ABAKADA Guro Party List, et al. v.
include the following: (1) Psychological and Hon. Purisima, et al.,We held:
behavioral disabilities (2) Chronic illness with
disabilities (3) Learning (cognitive or intellectual) Equality guaranteed under the equal protection
disabilities (4) Mental disabilities (5) Visual/seeing clause is equality under the same conditions and
disabilities (6) Orthopedic/moving, and (7) among persons similarly situated; it is equality
communication deficits. among equals, not similarity of treatment of persons
who are classified based on substantial differences
Elementary is the rule that when laws or rules are clear, when in relation to the object to be accomplished. When
the law is unambiguous and unequivocal, application not things or persons are different in fact or
interpretation thereof is imperative. However, where the circumstance, they may be treated in law differently.
language of a statute is vague and ambiguous, an In Victoriano v. Elizalde Rope Workers' Union, this
interpretation thereof is resorted to. A law is deemed Court declared:
ambiguous when it is capable of being understood by
reasonably well-informed persons in either of two or more The guaranty of equal protection of the laws is not a
senses. The fact that a law admits of different interpretations guaranty of equality in the application of the laws
is the best evidence that it is vague and ambiguous. upon all citizens of the State. It is not, therefore, a
requirement, in order to avoid the constitutional
In the instant case, We do not find the aforestated definition of prohibition against inequality, that every man,
terms as vague and ambiguous. Settled is the rule that courts woman and child should be affected alike by a
will not interfere in matters which are addressed to the sound statute. Equality of operation of statutes does not
discretion of the government agency entrusted with the mean indiscriminate operation on persons merely as
regulation of activities coming under the special and technical such, but on persons according to the circumstances
training and knowledge of such agency.[48] As a matter of surrounding them. It guarantees equality, not identity
policy, We accord great respect to the decisions and/or of rights. The Constitution does not require that
actions of administrative authorities not only because of the things which are different in fact be treated in law as
doctrine of separation of powers but also for their presumed though they were the same. The equal protection
knowledge, ability, and expertise in the enforcement of laws clause does not forbid discrimination as to things that
and regulations entrusted to their jurisdiction. The rationale for are different. It does not prohibit legislation which is
this rule relates not only to the emergence of the multifarious limited either in the object to which it is directed or by
needs of a modern or modernizing society and the the territory within which it is to operate.
establishment of diverse administrative agencies for
addressing and satisfying those needs; it also relates to the The equal protection of the laws clause of the Constitution
accumulation of experience and growth of specialized allows classification. Classification in law, as in the other

Page 6 of 186
departments of knowledge or practice, is the grouping of 3. Southern Luzon Drug Corporation vs. DSWD
things in speculation or practice because they agree with one
another in certain particulars. A law is not invalid because of
simple inequality. The very idea of classification is that of
FACTS: On April 23, 1992, R.A. No. 7432, entitled "An Act to
inequality, so that it goes without saying that the mere fact of
Maximize the Contribution of Senior Citizens to Nation-
inequality in no manner determines the matter of
Building, Grant Benefits and Special Privileges and For Other
constitutionality. All that is required of a valid classification is
Purposes," was enacted. Under the said law, a senior citizen,
that it be reasonable, which means that the classification
who must be at least 60 years old and has an annual income
should be based on substantial distinctions which make for
of not more than P60,000.00, may avail of the privileges
real differences, that it must be germane to the purpose of the
provided in Section 4 thereof, one of which is 20% discount on
law; that it must not be limited to existing conditions only; and
the purchase of medicines. The said provision states:
that it must apply equally to each member of the class. This
Court has held that the standard is satisfied if the classification Sec. 4. Privileges for the Senior Citizen. – x x x:
or distinction is based on a reasonable foundation or rational
basis and is not palpably arbitrary. a) the grant of twenty percent (20%) discount from all
establishments relative to utilization of transportation
In the exercise of its power to make classifications for the services, hotels and similar lodging establishment,
purpose of enacting laws over matters within its jurisdiction, restaurants and recreation centers and purchase of
the state is recognized as enjoying a wide range of discretion. medicine anywhere in the country: Provided, That
It is not necessary that the classification be based on scientific private establishments may claim the cost as tax
or marked differences of things or in their relation. Neither is it credit[.]
necessary that the classification be made with mathematical
nicety. Hence, legislative classification may in many cases To recoup the amount given as discount to qualified senior
properly rest on narrow distinctions, for the equal protection citizens, covered establishments can claim an equal amount
guaranty does not preclude the legislature from recognizing as tax credit which can be applied against the income tax due
degrees of evil or harm, and legislation is addressed to evils from them.
as they may appear.
On February 26, 2004, then President Gloria Macapagal-
The equal protection clause recognizes a valid classification, Arroyo signed R.A. No. 9257, amending some provisions of
that is, a classification that has a reasonable foundation or R.A. No. 7432. The new law retained the 20% discount on the
rational basis and not arbitrary. With respect to R.A. No. 9442, purchase of medicines but removed the annual income ceiling
its expressed public policy is the rehabilitation, self- thereby qualifying all senior citizens to the privileges under the
development and self-reliance of PWDs. Persons with law. Further, R.A. No. 9257 modified the tax treatment of the
disability form a class separate and distinct from the other discount granted to senior citizens, from tax credit to tax
citizens of the country. Indubitably, such substantial distinction deduction from gross income, computed based on the net cost
is germane and intimately related to the purpose of the law. of goods sold or services rendered. The pertinent provision,
Hence, the classification and treatment accorded to the PWDs as amended by R.A. No. 9257, reads as follows:
fully satisfy the demands of equal protection. Thus, Congress
SEC. 4. Privileges for the Senior Citizens. – The
may pass a law providing for a different treatment to persons
senior citizens shall be entitled to the following:
with disability apart from the other citizens of the country.
(a) the grant of twenty percent (20%) discount from
Subject to the determination of the courts as to what is a
all establishments relative to the utilization of
proper exercise of police power using the due process clause
services in hotels and similar lodging establishments,
and the equal protection clause as yardsticks, the State may
restaurants and recreation centers, and purchase of
interfere wherever the public interests demand it, and in this
medicines in all establishments for the exclusive use
particular, a large discretion is necessarily vested in the
or enjoyment of senior citizens, including funeral and
legislature to determine, not only what interests of the public
burial services for the death of senior citizens;
require, but what measures are necessary for the protection
of such interests. Thus, We are mindful of the fundamental The establishment may claim the discounts granted
criteria in cases of this nature that all reasonable doubts under (a), (f), (g) and (h) as tax deduction based on
should be resolved in favor of the constitutionality of a statute. the net cost of the goods sold or services rendered:
The burden of proof is on him who claims that a statute is Provided, That the cost of the discount shall be
unconstitutional. Petitioners failed to discharge such burden allowed as deduction from gross income for the
of proof. same taxable year that the discount is granted.
Provided, further, That the total amount of the
WHEREFORE, the petition is DENIED. The Decision of the
claimed tax deduction net of value added tax if
Court of Appeals dated July 26, 2010, and the Resolution
applicable, shall be included in their gross sales
dated November 19, 2010, in CA-G.R. SP No. 109903 are
AFFIRMED. receipts for tax purposes and shall be subject to
proper documentation and to the provisions of the
National Internal Revenue Code, as amended.

On May 28, 2004, the DSWD issued the Implementing Rules


and Regulations (IRR) of R.A. No. 9257. Article 8 of Rule VI
of the said IRR provides:

Article 8. Tax Deduction of Establishments. - The


establishment may claim the discounts granted
under Rule V, Section 4 – Discounts for
Establishments; Section 9, Medical and Dental
Services in Private Facilities and Sections 10 and 11

Page 7 of 186
– Air, Sea and Land Transportation as tax deduction Undeniably, the success of the senior citizens
based on the net cost of the goods sold or services program rests largely on the support imparted by
rendered. Provided, That the cost of the discount petitioners and the other private establishments
shall be allowed as deduction from gross income for concerned. This being the case, the means
the same taxable year that the discount is granted; employed in invoking the active participation of the
Provided, further, That the total amount of the private sector, in order to achieve the purpose or
claimed tax deduction net of value-added tax if objective of the law, is reasonably and directly
applicable, shall be included in their gross sales related. Without sufficient proof that Section 4(a) of
receipts for tax purposes and shall be subject to R.A. No. 9257 is arbitrary, and that the continued
proper documentation and to the provisions of the implementation of the same would be
National Internal Revenue Code, as amended; unconscionably detrimental to petitioners, the Court
Provided, finally, that the implementation of the tax will refrain from quashing a legislative act.
deduction shall be subject to the Revenue
Regulations to be issued by the Bureau of Internal WHEREFORE, the petition is DISMISSED for lack of
Revenue (BIR) and approved by the Department of merit.
Finance (DOF).
On August 1, 2007, Carlos Superdrug filed a motion for
The change in the tax treatment of the discount given to senior reconsideration of the foregoing decision. Subsequently, the
citizens did not sit well with some drug store owners and Court issued Resolution dated August 21, 2007, denying the
corporations, claiming it affected the profitability of their said motion with finality.
business. Thus, on January 13, 2005, Carlos Superdrug
Meanwhile, on March 24, 1992, R.A. No. 7277 pertaining to
Corporation (Carlos Superdrug), together with other
the "Magna Carta for Disabled Persons" was enacted,
corporation and proprietors operating drugstores in the
codifying the rights and privileges of PWDs. Thereafter, on
Philippines, filed a Petition for Prohibition with Prayer for
April 30, 2007, R.A. No. 9442 was enacted, amending R.A.
Temporary Restraining Order (TRO) and/or Preliminary
No. 7277. One of the salient amendments in the law is the
Injunction before this Court, entitled Carlos Superdrug
insertion of Chapter 8 in Title 2 thereof, which enumerates the
Corporation v. DSWD,[5] docketed as G.R. No. 166494,
other privileges and incentives of PWDs, including the grant
assailing the constitutionality of Section 4(a) of R.A. No. 9257
of 20% discount on the purchase of medicines. Similar to R.A.
primarily on the ground that it amounts to taking of private
No. 9257, covered establishments shall claim the discounts
property without payment of just compensation. In a Decision
given to PWDs as tax deductions from the gross income,
dated June 29, 2007, the Court upheld the constitutionality of
based on the net cost of goods sold or services rendered.
the assailed provision, holding that the same is a legitimate
Section 32 of R.A. No. 9442 reads:
exercise of police power. The relevant portions of the decision
read, thus: CHAPTER 8. Other Privileges and Incentives
The law is a legitimate exercise of police power SEC. 32. Persons with disability shall be entitled to
which, similar to the power of eminent domain, has the following:
general welfare for its object. Police power is not
capable of an exact definition, but has been xxxx
purposely veiled in general terms to underscore its
(c) At least twenty percent (20%) discount for the
comprehensiveness to meet all exigencies and
purchase of medicines in all drugstores for the
provide enough room for an efficient and flexible
exclusive use or enjoyment of persons with disability;
response to conditions and circumstances, thus
assuring the greatest benefits. Accordingly, it has xxxx
been described as "the most essential, insistent and
the least limitable of powers, extending as it does to The establishments may claim the discounts granted
all the great public needs." It is "[t]he power vested in in sub-sections (a), (b), (c), (e), (f) and (g) as tax
the legislature by the constitution to make, ordain, deductions based on the net cost of the goods sold
and establish all manner of wholesome and or services rendered: Provided, however, That the
reasonable laws, statutes, and ordinances, either cost of the discount shall be allowed as deduction
with penalties or without, not repugnant to the from gross income for the same taxable year that the
constitution, as they shall judge to be for the good discount is granted: Provided, further, That the total
and welfare of the commonwealth, and of the amount of the claimed tax deduction net of value-
subjects of the same." added tax if applicable, shall be included in their
gross sales receipts for tax purposes and shall be
For this reason, when the conditions so demand as subject to proper documentation and to the
determined by the legislature, property rights must provisions of the National Internal Revenue Code
bow to the primacy of police power because property (NIRC), as amended.
rights, though sheltered by due process, must yield
to general welfare. Pursuant to the foregoing, the IRR of R.A. No. 9442 was
promulgated by the DSWD, Department of Education, DOF,
Moreover, the right to property has a social Department of Tourism and the Department of Transportation
dimension. While Article XIII of the Constitution and Communications. Sections 5.1 and 6.1.d thereof provide:
provides the precept for the protection of property,
various laws and jurisprudence, particularly on Sec. 5. Definition of Terms. For purposes of these
agrarian reform and the regulation of contracts and Rules and Regulations, these terms are defined as
public utilities, continuously serve as a reminder that follows:
the right to property can be relinquished upon the
command of the State for the promotion of public 5.1. Persons with Disability are those individuals
good. defined under Section 4 of RA 7277, "An Act

Page 8 of 186
Providing for the Rehabilitation, Self-Development Unyielding, the petitioner filed the instant petition, raising the
and Self-Reliance of Persons with Disability as following assignment of errors, to wit:
amended and their integration into the Mainstream of
Society and for Other Purposes." This is defined as
a person suffering from restriction or different
ISSUES:
abilities, as a result of a mental, physical or sensory
impairment, to perform an activity in a manner or Whether or not the Petition for Prohibition may be filed to
within the range considered normal for human being. question the constitutionality of a law
Disability shall mean: (1) a physical or mental
impairment that substantially limits one or more Whether or not the case constitute stare decisis
psychological, physiological or anatomical function
Whether or not the 20% Sales Discount for Senior Citizens
of an individual or activities of such individual; (2) a
PWDs does not violate the petitioner’s right to equal protection
record of such an impairment; or (3) being regarded
of the law
as having such an impairment.
Whether or not the definitions of Disabilities and PWDs are
xxxx
vague and violates the petitioners right to due process of law
6.1.d Purchase of Medicine – At least twenty percent
(20%) discount on the purchase of medicine for the
exclusive use and enjoyment of persons with RULING:
disability. All drug stores, hospital, pharmacies,
clinics and other similar establishments selling 1. Yes. Prohibition may be filed to question the
medicines are required to provide at least twenty constitutionality of a law. Generally, the office of
percent (20%) discount subject to the guidelines prohibition is to prevent the unlawful and oppressive
issued by DOH and PHILHEALTH. exercise of authority and is directed against
proceedings that are done without or in excess of
On February 26, 2008, the petitioner filed a Petition for jurisdiction, or with grave abuse of discretion, there
Prohibition with Application for TRO and/or Writ of Preliminary being no appeal or other plain, speedy, and adequate
Injunction with the CA, seeking to declare as unconstitutional remedy in the ordinary course of law. It is the remedy
(a) Section 4(a) of R.A. No. 9257, and (b) Section 32 of R.A. to prevent inferior courts, corporations, boards, or
No. 9442 and Section 5.1 of its IRR, insofar as these persons from usurping or exercising a jurisdiction or
provisions only allow tax deduction on the gross income based power with which they have not been vested by the law.
on the net cost of goods sold or services rendered as This is, however, not the lone office of an action for
compensation to private establishments for the 20% discount prohibition. In Diaz, et al. v. The Secretary of Finance,
that they are required to grant to senior citizens and PWDs. et al., prohibition was also recognized as a proper
Further, the petitioner prayed that the respondents be remedy to prohibit or nullify acts of executive officials
permanently enjoined from implementing the assailed that amount to usurpation of legislative authority. And,
provisions. in a number of jurisprudence, prohibition was allowed
as a proper action to assail the constitutionality of a law
or prohibit its implementation.
Ruling of the CA

On June 17, 2011, the CA dismissed the petition, reiterating 2. No. The Court agrees that the ruling in Carlos
the ruling of the Court in Carlos Superdrug particularly that Superdrug does not constitute stare decisis to the
Section 4(a) of R.A. No. 9257 was a valid exercise of police instant case, not because of the petitioner's submission
power. Moreover, the CA held that considering that the same of financial statements which were wanting in the first
question had been raised by parties similarly situated and was case, but because it had the good sense of including
resolved in Carlos Superdrug, the rule of stare decisis stood questions that had not been raised or deliberated in the
as a hindrance to any further attempt to relitigate the same former case of Carlos Superdrug, i.e., validity of the
issue. It further noted that jurisdictional considerations also 20% discount granted to PWDs, the supposed
compel the dismissal of the action. It particularly emphasized vagueness of the provisions of R.A. No. 9442 and
that it has no original or appellate jurisdiction to pass upon the violation of the equal protection clause.
constitutionality of the assailed laws, the same pertaining to
the Regional Trial Court (RTC). Even assuming that it had
concurrent jurisdiction with the RTC, the principle of hierarchy 3. Yes. The subject laws do not violate the equal
of courts mandates that the case be commenced and heard protection clause. The equal protection clause is not
by the lower court. The CA further ruled that the petitioner infringed by legislation which applies only to those
resorted to the wrong remedy as a petition for prohibition will persons falling within a specified class. If the groupings
not lie to restrain the actions of the respondents for the simple are characterized by substantial distinctions that make
reason that they do not exercise judicial, quasi-judicial or real differences, one class may be treated and
ministerial duties relative to the issuance or implementation of regulated differently from another." For a classification
the questioned provisions. Also, the petition was wanting of to be valid, (1) it must be based upon substantial
the allegations of the specific acts committed by the distinctions, (2) it must be germane to the purposes of
respondents that demonstrate the exercise of these powers the law, (3) it must not be limited to existing conditions
which may be properly challenged in a petition for prohibition. only, and (4) it must apply equally to all members of the
same class.
The petitioner filed its Motion for Reconsideration of the
Decision dated June 17, 2011 of the CA, but the same was
denied in a Resolution dated November 25, 2011.

Page 9 of 186
4. No. The definitions of "disabilities" and "PWDs" are 4. Crisostomo B. Aquino vs. Municipality Of Malay, Aklan
clear and unequivocal. Section 4(a) of R.A. No. 7277,
the precursor of R.A. No. 94421 defines "disabled
persons" as follows:
IN THE EXERCISE OF POLICE POWER AND THE
(a) Disabled persons are those suffering from GENERAL WELFARE CLAUSE, PROPERTY RIGHTS OF
restriction or different abilities, as a result of a mental, INDIVIDUALS MAY BE SUBJECTED TO RESTRAINTS AND
physical or sensory impairment, to perform an activity BURDENS IN ORDER TO FULFIL THE OBJECTIVES OF
in the manner or within the range considered normal for THE GOVERNMENT.
a human being[.]

On the other hand, the term "PWDs" is defined in


FACTS: In a Petition for Review on Certiorari under Rule 45,
Section 5.1 of the IRR of R.A. No. 9442 as follows:
petitioner Crisostomo Aquino, the president and chief
5.1. Persons with Disability are those individuals executive officer of Boracay Island West Cove, assailed the
defined under Section 4 of [R.A. No.] 7277 [or] An Act decisions of the Municipality of Malay, Aklan for denying his
Providing for the Rehabilitation, Self-Development and application for zoning compliance and issuing demolition
Self-Reliance of Persons with Disability as amended closure and demolition order and the CA for denying his
and their integration into the Mainstream of Society and petition for certiorari for being the improper remedy.
for Other Purposes. This is defined as a person
Petitioner’s application for zoning compliance and building
suffering from restriction or different abilities, as a result
permit covering the construction of an additional three-storey
of a mental, physical or sensory impairment, to perform
hotel over a parcel of land covered by a Forest Land Use
an activity in a manner or within the range considered
Agreement for Tourism Purposes (FLAgT) were denied by the
normal for human being. Disability shall mean (1) a
Municipal Zoning Administrator on the ground that the
physical 1or mental impairment that substantially limits
proposed construction site was within the "no build zone"
one or more psychological, physiological or anatomical
demarcated in a Municipal Ordinance. Thereafter, the
function of an individual or activities of such individual;
respondent issued a 1) Cease and Desist Order enjoining the
(2) a record of such an impairment; or (3) being
expansion of the resort, and 2) EO No. 10 ordering the closure
regarded as having such an impairment.
and demolition of Boracay West Cove’s hotel. EO 10 was
In view of the foregoing disquisition, Section 4(a) of Republic partially implemented on June 10, 2011. Also, respondents
Act No. 9257 and Section 32 of Republic Act No. 9442 are demolished the improvements introduced by Boracay West
hereby declared CONSTITUTIONAL. Cove.

Petitioner alleged that the order was issued and executed with
grave abuse of discretion. He argued that judicial proceedings
should first be conducted before the respondent mayor could
order the demolition of the company’s establishment.

On the other hand, respondents argued that the demolition


needed no court order because the municipal mayor has the
express power under the Local Government Code (LGC) to
order the removal of illegally constructed buildings.

ISSUES:

Whether or not the procedural due process (of due notice and
hearing) was complied with.

Whether or not judicial proceedings should first be conducted


before the respondent mayor could order the demolition of the
company’s establishment.

RULING:

1. Yes, the procedural due process (of due notice and


hearing) was complied with.

First, basic is the rule that public officers enjoy the


presumption of regularity in the performance of their duties.
The burden is on the petitioner herein to prove that Boracay
West Cove was deprived of the opportunity to be heard before
EO 10 was issued. Regrettably, copies of the Cease and
Desist Order issued by the LGU and of the assailed EO 10
itself were never attached to the petition before this Court,
which documents could have readily shed light on whether or
not petitioner has been accorded the 10-day grace period
provided in Section 10 of the Ordinance. In view of this fact,
the presumption of regularity must be sustained.

Page 10 of 186
Second, as quoted by petitioner in his petition before the CA, 5. FERRER, JR. v. BAUTISTA
the assailed EO 10 states that petitioner received notices from
the municipality government on March 7 and 28, 2011,
requiring Boracay West Cove to comply with the zoning
FACTS: Quezon City Council enacted Ordinance No. SP-
ordinance and yet it failed to do so. If such was the case, the
2095, S-2011, or the Socialized Housing Tax of Quezon City
grace period can be deemed observed and the establishment
which allowed the imposition of special assessment (1/2 of the
was already ripe for closure and demolition by the time EO 10
assessed value of land in excess of P100k).
was issued in June.
Ordinance No. SP-2235, S-20135 was enacted on December
16, 2013 and took effect ten days after when it was approved
2. No, petitioner’s right to due process was not violated for by the City Mayor relative to Garbage Collection Fees
lack of judicial proceedings prior to the issuance of imposing fees depending on the amount of the land or floor
demolition order. area.

The government may enact legislation that may interfere with Jose Ferrer, as a property in Quezon City questioned the
personal liberty, property, lawful businesses and occupations validity of the city ordinances. Ferrer claims that the city has
to promote the general welfare. One such piece of legislation no power to impose the tax. The SH Tax violates the rule on
is the Local Government Code (LGC), which authorizes city equality because it burdens real property owners with
and municipal governments, acting through their local chief expenses to provide funds for the housing of informal settlers.
executives, to issue demolition orders and to hear issues The SH Tax is confiscatory or oppressive.
involving property rights of individuals and to come out with an
On the validity of the garbage fees imposition, Ferrer claims
effective order or resolution thereon. Pertinent herein is Sec.
that it violates the rule on double taxation. And it violates the
444 (b)(3)(vi) of the LGC, which empowered the mayor to
rule on equality because the fees are collected from only
order the closure and removal of illegally constructed
domestic households and not from restaurants, food courts,
establishments for failing to secure the necessary permits.
fast food chains, and other commercial dining places that
Though the court agreed with Petitioner’s contention that, spew garbage much more than residential property owners.
under the LGC, the Sanggunian does not have the power to
authorize the extrajudicial condemnation and destruction of a
nuisance per accidens, still the SC ruled that the LGU may ISSUE: Whether or not the ordinances were valid.
nevertheless properly order the hotel’s demolition without due
hearing thereon in a tribunal, because in the exercise of police
power and the general welfare clause provided in the
HELD:
Constitution, property rights of individuals may be subjected
to restraints and burdens in order to fulfil the objectives of the 1st ordinance: Socialized Housing Tax of Quezon City is valid.
government.
Cities have the power to tax
Hence, the right to due process was satisfied in the case at
bar. It must be noted that local government units such as cities has
the power to tax. The collection for the socialized housing tax
is valid. It must be noted that the collections were made to
accrue to the socialized housing programs and projects of the
city.

The imposition was for a public purpose (exercise of power of


taxation + police power)

In this case, there was both an exercise of the power to tax


(primary) and police power (incidental). Removing slum areas
in Quezon City is not only beneficial to the underprivileged and
homeless constituents but advantageous to the real property
owners as well.

The situation will improve the value of the their property


investments, fully enjoying the same in view of an orderly,
secure, and safe community, and will enhance the quality of
life of the poor, making them law-abiding constituents and
better consumers of business products.

There is no violation of the rule on equality

Note: There is a substantial distinction between: real property


owner and an informal settler. In fact, the Supreme Court said
that the disparity is so obvious. It is inherent in the power to
tax that a State is free to select the subjects of taxation.
Inequities which result from a singling out of one particular
class for taxation or exemption infringe no constitutional
limitation.

All these requisites are complied with: An ordinance based on


reasonable classification does not violate the constitutional

Page 11 of 186
guaranty of the equal protection of the law. The requirements imposed regardless of whether the resident is from a
for a valid and reasonable classification are: (1) it must rest on condominium or from a socialized housing project.
substantial distinctions; (2) it must be germane to the purpose
of the law; (3) it must not be limited to existing conditions only; The classifications are not germane to the purpose of the
and (4) it must apply equally to all members of the same class. ordinance

The ordinance is not oppressive or confiscatory The declared purpose is: "promoting shared responsibility with
the residents to attack their common mindless attitude in over-
The ordinance is also not oppressive since the tax rate being consuming the present resources and in generating waste."
imposed is consistent with the UDHA (Urban Development
and Housing Act of 1992). While the law authorizes LGUs to Instead of simplistically categorizing the payee into land or
collect SHT on properties with an assessed value of more than floor occupant of a lot or unit of a condominium, socialized
P50,000.00, the questioned ordinance only covers properties housing project or apartment, respondent City Council should
with an assessed value exceeding P100,000.00. As well, the have considered factors that could truly measure the amount
ordinance provides for a tax credit equivalent to the total of wastes generated and the appropriate fee for its collection.
amount of the special assessment paid by the property owner Factors include, among others, household age and size,
beginning in the sixth (6th) year of the effectivity of the accessibility to waste collection, population density of the
ordinance. barangay or district, capacity to pay, and actual occupancy of
the property.

2nd ordinance: The imposition of garbage fee is invalid.


SC:
Note: There was no violation of double taxation but there was
a violation of the rule on equity. → Validity of Socialized Housing Tax of Quezon City is upheld.

There is no violation of double taxation: the garbage fees → Ordinance No. SP-2235, S-2013, which collects an annual
are not taxes garbage fee on all domestic households in Quezon City, is
unconstitutional and illegal
In Progressive Development Corporation v. Quezon City, the
Court declared that:

"if the generating of revenue is the primary purpose


and regulation is merely incidental, the imposition is
a tax; but if regulation is the primary purpose, the fact
that incidentally revenue is also obtained does not
make the imposition a tax."

Contention of Ferrer: That the imposition of garbage fee is


tantamount to double taxation because garbage collection is
a basic and essential public service that should be paid out
from property tax, business tax, transfer tax, amusement tax,
community tax certificate, other taxes, and the IRA of the
Quezon City Government. All these are valid taxes. The
garbage fees are license fees.

Footnote: In order to constitute double taxation in the


objectionable or prohibited sense the same property must be
taxed twice when it should be taxed but once; both taxes must
be imposed on the same property or subject-matter, for the
same purpose, by the same State, Government, or taxing
authority, within the same jurisdiction or taxing district, during
the same taxing period, and they must be the same kind or
character of tax.

There is a violation of the rule on equality: no substantial


distinction

There is no substantial distinction between an occupant of a


lot, on one hand, and an occupant of a unit in a condominium,
socialized housing project or apartment, on the other hand.

Most likely, garbage output produced by these types of


occupants is uniform and does not vary to a large degree;
thus, a similar schedule of fee is both just and equitable.

The garbage fees or rates are unjust and inequitable

A resident of a 200 sq. m. unit in a condominium or socialized


housing project has to pay twice the amount than a resident
of a lot similar in size; unlike unit occupants, all occupants of
a lot with an area of 200 sq. m. and less have to pay a fixed
rate of Php100.00; and the same amount of garbage fee is

Page 12 of 186
6. PHILIPPINE CARE HEALTH PROVIDERS, INC. v. equally and uniformly, lest the tax collector kill the
COMMISSIONER OF INTERNATL REVENUE "hen that lays the golden egg."

Legitimate enterprises enjoy the constitutional protection not


to be taxed out of existence. Incurring losses because of a tax
FACTS: On January 27, 2000, the respondent CIR sent imposition may be an acceptable consequence but killing the
petitioner assessment of deficiency taxes, both Value-Added business of an entity is another matter and should not be
Tax (VAT) and documentary stamp tax (DST) in the total allowed. It is counter-productive and ultimately subversive of
amount of P224,702,641.18 for taxable years 1996 and 1997. the nation’s thrust towards a better economy which will
ultimately benefit the majority of our people.
Petitioner protested such assessment in a letter, but the
respondent did not act on the protest which led the petitioner Moreover, HMOs play an important role in society as partners
to file a petition in the Court of Tax Appeals (CTA) seeking the of the State in achieving its constitutional mandate of providing
cancellation of said assessments. CTA partially granted the citizens with affordable health services.
petition wherein the petitioner is ordered to pay the deficiency
VAT and set aside the DST deficiency tax. Also, the DST assessment of the petitioner for the years 1996
and 1997 became moot and academic since it availed tax
Respondent appealed in Court of Appeals (CA) with regard to amnesty under RA 9480 on December 10, 2007. Thus,
the cancellation of DST assessment. CA granted the petition. petitioner is entitled to immunity from payment of taxes for
The Court affirmed CA’s decision. Hence, petitioner filed a taxable year 2005 and prior years.
motion for reconsideration.

ISSUE: Whether or not the petitioner is liable to pay the DST


on its health care agreement pursuant to Sec.185 of the
National Internal Revenue Code of 1997

RULING: Petition granted. Petitioner is not contemplated to


be included in “or other branch insurance” covered by Section
185 of NIRC because it is a Health Maintenance Organization
(HMO) and not an insurance company. HMOs primary
purpose is rendering service to its member by lowering prices
and reducing the cost rather than the risk of medical health.
On the other hand, insurance businesses undertakes for a
consideration to indemnify its clients against loss, damage or
liability arising from unknown or contingent event. The term
“indemnify” therein presuppose that a liability or claim has
already been incurred. In HMOs, there is no indemnity
precisely because the member merely avails of medical
services to be paid or already paid in advance at a pre-agreed
price under the agreements.

The Power To Tax Is Not The Power To Destroy

As a general rule, the power to tax is an incident of sovereignty


and is unlimited in its range, acknowledging in its very nature
no limits, so that security against its abuse is to be found only
in the responsibility of the legislature which imposes the tax
on the constituency who is to pay it.51 So potent indeed is the
power that it was once opined that "the power to tax involves
the power to destroy."

Petitioner claims that the assessed DST to date which


amounts to ₱376 million53 is way beyond its net worth of ₱259
million.54 Respondent never disputed these assertions. Given
the realities on the ground, imposing the DST on petitioner
would be highly oppressive. It is not the purpose of the
government to throttle private business. On the contrary, the
government ought to encourage private enterprise.55
Petitioner, just like any concern organized for a lawful
economic activity, has a right to maintain a legitimate
business. As aptly held in Roxas, et al. v. CTA, et al.:

The power of taxation is sometimes called also the


power to destroy. Therefore it should be exercised
with caution to minimize injury to the proprietary
rights of a taxpayer. It must be exercised fairly,

Page 13 of 186
7. NATIONAL POWER CORPORATION v. CITY OF Ruling in favor of the local government in both instances, we
CABANATUAN ruled that the franchise tax in question is imposable despite
any exemption enjoyed by MERALCO under special laws, viz:

“It is our view that petitioners correctly rely on


FACTS: Petitioner is a government-owned and controlled provisions of Sections 137 and 193 of the LGC to
corporation created under Commonwealth Act No. 120, as support their position that MERALCO’s tax
amended. exemption has been withdrawn. The explicit
language of section 137 which authorizes the
For many years now, petitioner sells electric power to the
province to impose franchise tax ‘notwithstanding
residents of Cabanatuan City, posting a gross income of
any exemption granted by any law or other special
P107,814,187.96 in 1992.7 Pursuant to section 37 of
law’ is all-encompassing and clear. The franchise tax
Ordinance No. 165-92,8 the respondent assessed the
is imposable despite any exemption enjoyed under
petitioner a franchise tax amounting to P808,606.41,
special laws.
representing 75% of 1% of the latter’s gross receipts for the
preceding year. Section 193 buttresses the withdrawal of extant tax exemption
privileges. By stating that unless otherwise provided in this
Petitioner refused to pay the tax assessment arguing that the
Code, tax exemptions or incentives granted to or presently
respondent has no authority to impose tax on government
enjoyed by all persons, whether natural or juridical, including
entities. Petitioner also contended that as a non-profit
government-owned or controlled corporations except (1) local
organization, it is exempted from the payment of all forms of
water districts, (2) cooperatives duly registered under R.A.
taxes, charges, duties or fees in accordance with sec. 13 of
6938, (3) non-stock and non-profit hospitals and educational
Rep. Act No. 6395, as amended.
institutions, are withdrawn upon the effectivity of this code, the
The respondent filed a collection suit in the RTC, demanding obvious import is to limit the exemptions to the three
that petitioner pay the assessed tax due, plus surcharge. enumerated entities. It is a basic precept of statutory
Respondent alleged that petitioner’s exemption from local construction that the express mention of one person, thing,
taxes has been repealed by section 193 of the LGC, which act, or consequence excludes all others as expressed in the
reads as follows: familiar maxim expressio unius est exclusio alterius. In the
absence of any provision of the Code to the contrary, and we
“Sec. 193. Withdrawal of Tax Exemption Privileges.- find no other provision in point, any existing tax exemption or
Unless otherwise provided in this Code, tax incentive enjoyed by MERALCO under existing law was
exemptions or incentives granted to, or presently clearly intended to be withdrawn.
enjoyed by all persons, whether natural or juridical,
including government owned or controlled Reading together sections 137 and 193 of the LGC, we
corporations, except local water districts, conclude that under the LGC the local government unit may
cooperatives duly registered under R.A. No. 6938, now impose a local tax at a rate not exceeding 50% of 1% of
non-stock and non-profit hospitals and educational the gross annual receipts for the preceding calendar based on
institutions, are hereby withdrawn upon the effectivity the incoming receipts realized within its territorial jurisdiction.
of this Code.” The legislative purpose to withdraw tax privileges enjoyed
under existing law or charter is clearly manifested by the
RTC upheld NPC’s tax exemption. On appeal the CA reversed language used on (sic) Sections 137 and 193 categorically
the trial court’s Order on the ground that section 193, in withdrawing such exemption subject only to the exceptions
relation to sections 137 and 151 of the LGC, expressly enumerated. Since it would be not only tedious and
withdrew the exemptions granted to the petitioner. impractical to attempt to enumerate all the existing statutes
providing for special tax exemptions or privileges, the LGC
provided for an express, albeit general, withdrawal of such
ISSUE: W/N the respondent city government has the authority exemptions or privileges. No more unequivocal language
to issue Ordinance No. 165-92 and impose an annual tax on could have been used.” (emphases supplied)
“businesses enjoying a franchise
Doubtless, the power to tax is the most effective instrument to
raise needed revenues to finance and support myriad
activities of the local government units for the delivery of basic
HELD: YES. Taxes are the lifeblood of the government, for services essential to the promotion of the general welfare and
without taxes, the government can neither exist nor endure. A the enhancement of peace, progress, and prosperity of the
principal attribute of sovereignty, the exercise of taxing power people. As this Court observed in the Mactan case, “the
derives its source from the very existence of the state whose original reasons for the withdrawal of tax exemption privileges
social contract with its citizens obliges it to promote public granted to government-owned or controlled corporations and
interest and common good. The theory behind the exercise of all other units of government were that such privilege resulted
the power to tax emanates from necessity; without taxes, in serious tax base erosion and distortions in the tax treatment
government cannot fulfill its mandate of promoting the general of similarly situated enterprises.” With the added burden of
welfare and well-being of the people. devolution, it is even more imperative for government entities
to share in the requirements of development, fiscal or
Section 137 of the LGC clearly states that the LGUs can otherwise, by paying taxes or other charges due from them.
impose franchise tax “notwithstanding any exemption granted
by any law or other special law.” This particular provision of
the LGC does not admit any exception. In City Government of
San Pablo, Laguna v. Reyes, MERALCO’s exemption from
the payment of franchise taxes was brought as an issue before
this Court. The same issue was involved in the subsequent
case of Manila Electric Company v. Province of Laguna.75

Page 14 of 186
B. DUE PROCESS the course of the proceedings, respondent is actually a male
person and hence his birth certificate has to be corrected to
reflect his true sex/gender, change of sex or... gender is
1. REPUBLIC v. CAGANDAHAN allowed under Rule 108, and respondent substantially
complied with the requirements of Rules 103 and 108 of the
Rules of Court.

FACTS: On December 11, 2003, respondent Jennifer Section 3, Rule 108 provides that the civil registrar and all
Cagandahan filed a Petition for Correction of Entries in Birth persons who have or... claim any interest which would be
Certificate before the RTC, Branch 33 of Siniloan, Laguna. affected thereby shall be made parties to the proceedings.
Likewise, the local civil registrar is required to be made a party
In her petition, she alleged that she was born on January 13, in a proceeding for the correction of name in the civil registry.
1981 and was registered as a female in the Certificate of Live He is an indispensable party without whom no final...
Birth but while growing up, she developed secondary male determination of the case can be had.[12] Unless all possible
characteristics and was diagnosed to have Congenital indispensable parties were duly notified of the proceedings,
Adrenal Hyperplasia (CAH) which is a... condition where the same shall be considered as falling much too short of the
persons thus afflicted possess both male and female requirements of the rules.
characteristics. She further alleged that she was diagnosed to
have clitoral hyperthropy in her early years and at age six,
underwent an ultrasound where it was discovered that she has
ISSUE: Simply stated, the issue is whether the trial court erred
small ovaries. At age... thirteen, tests revealed that her
in ordering the correction of entries in the birth certificate of
ovarian structures had minimized, she has stopped growing
respondent to change her sex or gender, from female to male,
and she has no breast or menstrual development. She then
on the ground of her medical condition known as CAH, and
alleged that for all interests and appearances as well as in
her name from "Jennifer" to "Jeff,"... under Rules 103 and 108
mind and emotion, she has become a male person. Thus, she
of the Rules of Court.
prayed that... her birth certificate be corrected such that her
gender be changed from female to male and her first name be
changed from Jennifer to Jeff.
RULING: The determination of a person's sex appearing in his
To prove her claim, respondent testified and presented the birth certificate is a legal issue and the court must look to the
testimony of Dr. Michael Sionzon of the Department of statutes. In this connection, Article 412 of the Civil Code
Psychiatry, University of the Philippines-Philippine General provides:
Hospital.
ART. 412. No entry in a civil register shall be
He explained that genetically respondent is female but changed or corrected without a judicial order.
because her body secretes male hormones, her female
organs did not develop normally and she has two sex organs Respondent here has simply let nature take its course and has
- female and male. He testified that this condition is very rare, not taken unnatural steps to arrest or interfere with what he
that respondent's uterus is not fully developed... because of was born with. And accordingly, he has already ordered his
lack of female hormones, and that she has no monthly period. life to that of a male. Respondent could have undergone
He further testified that respondent's condition is permanent treatment and taken steps, like taking... lifelong medication, to
and recommended the change of gender because respondent force his body into the categorical mold of a female but he did
has made up her mind, adjusted to her chosen role as male, not. He chose not to do so.
and the gender change would be... advantageous to her.
In so ruling we do no more than give respect to (1) the diversity
Civil Register of Pakil, Laguna is hereby ordered to make the of nature; and (2) how an individual deals with what nature has
following corrections in the birth [c]ertificate of Jennifer handed out. In other words, we respect respondent's
Cagandahan upon payment of the prescribed fees:... a) By congenital condition and his mature decision to be a male. Life
changing the name from Jennifer Cagandahan to JEFF is already difficult for the... ordinary person. We cannot but
CAGANDAHAN; and... b) By changing the gender from respect how respondent deals with his unordinary state and
female to MALE. thus help make his life easier, considering the unique
circumstances in this case.
It is likewise ordered that petitioner's school records, voter's
registry, baptismal certificate, and other pertinent records are As for respondent's change of name under Rule 103, this
hereby amended to conform with the foregoing corrected data. Court has held that a change of name is not a matter of right
but of judicial discretion, to be exercised in the light of the
this petition by the Office of the Solicitor General (OSG) reasons adduced and the consequences that will follow.
seeking a reversal of the abovementioned ruling.
Considering the consequence that respondent's change of
THE REQUIREMENTS OF RULES 103 AND 108 OF THE name merely recognizes his preferred gender, we find merit in
RULES OF COURT HAVE NOT BEEN COMPLIED WITH respondent's change of name. Such a... change will conform
CORRECTION OF ENTRY UNDER RULE 108 DOES NOT with the change of the entry in his birth certificate from female
ALLOW CHANGE OF "SEX" OR "GENDER" IN THE BIRTH to male.
CERTIFICATE, WHILE RESPONDENT'S MEDICAL
CONDITION, i.e., CONGENITAL ADRENAL HYPERPLASIA Republic's petition is DENIED. The Decision dated January
DOES NOT MAKE HER A "MALE."... respondent counters 12, 2005 of the Regional Trial Court, Branch 33 of Siniloan,
that although the Local Civil Registrar of Pakil, Laguna was Laguna, is AFFIRMED
not formally named a party in the Petition for Correction of
Birth Certificate, nonetheless the Local Civil Registrar was
furnished a copy of the Petition, the Order to publish on
December 16, 2003 and all pleadings, orders or processes in

Page 15 of 186
ISSUE: Whether or not Cagandahan’s sex as appearing in her 2. BRIGIDO B. QUIAO v. RITA C. QUIAO
birth certificate be changed.

FACTS: Respondent Rita C. Quiao (Rita) filed a complaint for


RULING: The Supreme Court affirmed the decision of the legal separation against herein petitioner Brigido B. Quiao
lower court. It held that, in deciding the case, the Supreme (Brigido). The judgment finds petitioner-husband as the guilty
Court considered “the compassionate calls for recognition of party and forfeited his share in the conjugal property in favor
the various degrees of intersex as variations which should not of the common children.
be subject to outright denial.” The Supreme Court made use
of the available evidence presented in court including the fact The petitioner is saying that since the property relations
that private respondent thinks of himself as a male and as to between the spouses is governed by the regime of Conjugal
the statement made by the doctor that Cagandahan’s body Partnership of Gains under the Civil Code, the petitioner
produces high levels of male hormones (androgen), which is acquired vested rights over half of the properties of the
preponderant biological support for considering him as being Conjugal Partnership of Gains, pursuant to Article 143 of the
male.” Civil Code, which provides: All property of the conjugal
partnership of gains is owned in common by the husband and
The Supreme Court further held that they give respect to (1) wife.[60] Thus, since he is one of the owners of the properties
the diversity of nature; and (2) how an individual deals with covered by the conjugal partnership of gains, he has a vested
what nature has handed out. That is, the Supreme Court right over half of the said properties, even after the
respects the respondent’s congenital condition and his mature promulgation of the Family Code; and he insisted that no
decision to be a male. Life is already difficult for the ordinary provision under the Family Code may deprive him of this
person. vested right by virtue of Article 256 of the Family Code which
prohibits retroactive application of the Family Code when it will
The Court added that a change of name is not a matter of right prejudice a person's vested right.
but of judicial discretion, to be exercised in the light of the
reasons and the consequences that will follow.

ISSUE: WON the forfeiture violated the vested right of the


petitioner?

HELD: NO.

A vested right is one whose existence, effectivity and extent


do not depend upon events foreign to the will of the holder, or
to the exercise of which no obstacle exists, and which is
immediate and perfect in itself and not dependent upon a
contingency. The term vested right expresses the concept of
present fixed interest which, in right reason and natural justice,
should be protected against arbitrary State action, or an
innately just and imperative right which enlightened free
society, sensitive to inherent and irrefragable individual rights,
cannot deny.

To be vested, a right must have become a title legal or


equitable to the present or future enjoyment of property.

The concept of vested right is a consequence of the


constitutional guaranty of due process that expresses a
present fixed interest which in right reason and natural justice
is protected against arbitrary state action; it includes not only
legal or equitable title to the enforcement of a demand but also
exemptions from new obligations created after the right has
become vested. Rights are considered vested when the right
to enjoyment is a present interest, absolute, unconditional,
and perfect or fixed and irrefutable.

From the foregoing, it is clear that while one may not be


deprived of his vested right, he may lose the same if there is
due process and such deprivation is founded in law and
jurisprudence.

In the present case, the petitioner was accorded his right to


due process. First, he was well-aware that the respondent
prayed in her complaint that all of the conjugal properties be
awarded to her. In fact, in his Answer, the petitioner prayed
that the trial court divide the community assets between the
petitioner and the respondent as circumstances and evidence
warrant after the accounting and inventory of all the
community properties of the parties. Second, when the

Page 16 of 186
Decision dated October 10, 2005 was promulgated, the 3. ANONYMOUS v. RADAM
petitioner never questioned the trial court's ruling forfeiting
what the trial court termed as net profits, pursuant to Article
129(7) of the Family Code. Thus, the petitioner cannot claim
FACTS: Ma. Victoria Radam, utility worker in the Office of the
being deprived of his right to due process.
Clerk of Court of the Regional Trial Court of Alaminos City in
Also, the petitioner's claim of a vested right has no basis Pangasinan, was charged with immorality. The unnamed
considering that even under Article 176 of the Civil Code, his complainant alleged that respondent was unmarried but got
share of the conjugal partnership profits may be forfeited if he pregnant and gave birth outside wedlock. The complainant
is the guilty party in a legal separation case. claimed that respondent’s behavior tainted the image of the
judiciary.

In connection with the complaint, Judge Abella conducted a


discreet investigation to verify the allegations against
respondent. He find out that truthfulness of the allegation. He
was told by the respondent that they are unable to marry
because they plan to migrate to Canada. In a further
investigation it was revealed that the father of her son is
unknown, as shown by the child’s Certificate of Live Birth.

In this connection, Judge Abella recommends that respondent


MA. VICTORIA RADAM be accordingly found GUILTY of
IMMORAL CONDUCT or ACT UNBECOMING A COURT
EMPLOYEE. A suspension of one (1) month or a fine of
Php5,000.00 is respectfully recommended, with warning that
a repetition of the same or similar act in the future will be dealt
with more severely. After reviewing the findings and
recommendation of Judge Abella, the Office of the Court
Administrator (OCA) recommended that, respondent be
absolved of the charge of immorality because her alleged
misconduct (that is, giving birth out of wedlock) did not affect
the character and nature of her position as a utility worker.
However, it proposed that she be held liable for conduct
unbecoming a court employee and imposed a fine ofP5,000
for stating in the birth certificate of her child Christian Jeon that
the father was "unknown" to her.

ISSUE: whether or not respondent may be held liable in


relation to her entry in her child’s birth certificate regarding her
son’s father?

RULING: No. The essence of due process in an


administrative proceeding is the opportunity to explain one’s
side, whether written or verbal.

This presupposes that one has been previously apprised of


the accusation against him or her. Here, respondent was
deprived of both with regard to her alleged unbecoming
conduct in relation to a certain statement in the birth certificate
of her child. She was indicted only for alleged immorality for
giving birth out of wedlock. It was the only charge of which she
was informed. Judge Abella’s investigation focused solely on
that matter. She was neither confronted with it nor given the
chance to explain it. To hold her liable for a totally different
charge of which she was totally unaware will violate her right
to due process. Case dismissed

Page 17 of 186
4. MACIAS v. MACIAS judge who cannot live up to this exacting requirement has
no business sitting on the bench. Considering the
proliferation of complaints of abuses and immorality
committed by judges, it is only proper that the Court be
FACTS: This involves an administrative complaint filed by
ever vigilant in requiring impeccable conduct from the
complainant Margie C. Macias charging her husband, Mariano
members of its bench.
Joaquin S. Macias (Judge Macias), with immorality and
conduct prejudicial to the best interest of the service. The However, in this case, we are not convinced that
complaint was filed on March 7, 2001, when respondent complainant was able to prove, by substantial evidence,
was still sitting as the presiding judge of Branch 28 of that respondent committed the acts complained of. Basic
the Regional Trial Court (RTC) of Liloy, Zamboanga del is the rule that in administrative proceedings, complainant
Norte. bears the onus of establishing the averments of her
complaint. If complainant fails to discharge this burden,
Complainant alleged that sometime in 1998, respondent
respondent cannot be held liable for the charge.
engaged in an illicit liaison and immoral relationship with
a certain Judilyn Seranillos (Seranillos), single and in her Under Sections 8 and 11 of Rule 140 of the Rules of
early 20s. The relationship continued until the time of the Court, a judge found guilty of immorality can be dismissed
filing of the complaint. from the service, if still in the active service, or may forfeit
all or part of his retirement benefits, if already retired,
The witness for respondent was Judge Macias himself. He
and disqualified from reinstatement or appointment to any
denied the allegations of Mutia and Zozobrado. He said that
public office including government-owned or controlled
complainant also filed a complaint for concubinage against
corporations. We have already ruled that if a judge is to
him, but the same was dismissed by the Regional State
be disciplined for a grave offense, the evidence against
Prosecutor for lack of sufficient evidence. He believed
him should be competent and derived from direct
that complainant’s accusations were brought about by her
knowledge. This quantum of evidence, complainant failed to
psychiatric condition characterized as severe paranoia.
satisfy.

Nevertheless, we agree with the findings of the


Issue: Whether or not substantial evidence is sufficient to Investigating Justice that although the charges of
establish the guilt of a judge charged with a grave offense immorality and conduct prejudicial to the best interest of
the service were not satisfactorily proven by complainant,
respondent cannot be completely exonerated. Mutia’s
testimony that he saw Judge Macias having dinner with
Ruling: Yes. While substantial evidence would ordinarily
Seranillos and entering a bedroom with her may not
suffice to support a finding of guilt, the rule is a bit
satisfactorily prove the charge of immorality, but this act
different where the proceedings involve judges charged
certainly suggested an appearance of impropriety, Judge
with grave offense. Administrative proceedings against
Macias being a married man. Such behavior undeniably
judges are, by nature, highly penal in character and are
constituted unbecoming conduct, a light offense punishable
to be governed by the rules applicable to criminal cases.
by a fine not less than ₱1,000.00 but not more than
In more recent rulings, however, the Court applied ₱10,000.00. In light of the circumstances affecting not only
substantial evidence as the normative quantum of proof the reputation of Judge Macias himself but the image and
necessary in resolving administrative complaints against reputation of the whole judiciary as well, we find it
judges. In order to diffuse confusion, a clarification has to be reasonable to impose upon him the maximum fine of
made. First, the pronouncements in Horilleno and Alcuizar ₱10,000.00.
may be said to have been superseded by the Court’s
WHEREFORE, premises considered, the administrative
recent rulings in Gutierrez v. Belen, Reyes v. Paderanga,
complaint for immorality and conduct prejudicial to the
and Naval v. Panday.
best interest of the service against respondent Judge
Second, members of the judiciary are not a class of their Mariano Joaquin S. Macias of RTC, Branch 28, of Liloy,
own, sui generis, in the field of public service as to Zamboanga del Norte is DISMISSED for insufficiency of
require a higher degree of proof for the administrative evidence. However, respondent is held administratively
cases filed against them other than, perhaps, the fact liable for UNBECOMING CONDUCT and FINED in the
that because of the nature of the responsibility judges amount of ₱10,000.00 to be deducted from his retirement
have, they are required to live up to a higher standard benefits.
of integrity, probity and morality. When we dismiss a public
officer or employee from his position or office for the
commission of a grave offense in connection with his
office, we merely require that the complainant prove
substantial evidence. When we disbar a disgraceful lawyer,
we require that complainant merely prove a clear
preponderance of evidence to establish liability. There
appears no compelling reason to require a higher degree
of proof when we deal with cases filed against judges.

Judges play a vital role in the dispensation of justice. In this


jurisdiction, the integrity demanded of a judge does not
commence only when he dons the habiliments of a
magistrate or ends when he sheds off his judicial robe.
The nature of the position requires nothing less than a
24-hour daily obeisance to this mandate of integrity. Any

Page 18 of 186
5. OFFICE OF THE COURT ADMINISTRATOR v. INDAR ISSUE: Whether or not there was due process when the court
handled Judge Indar’s case.

FACTS: This is an administrative complaint for gross


misconduct and dishonesty against respondent Judge Cader RULING: Yes. It is settled that technical rules of procedure
P. Indar, Al Haj (Judge Indar). and evidence are not strictly applied to administrative
proceedings. Thus, administrative due process cannot be fully
There have been reports by the Local Civil Registrars of equated with due process in its strict judicial sense. It is
Manila and Quezon City to the Office of the Court enough that the party is given the chance to be heard before
Administrator (OCA) that they have received an alarming the case against him is decided.
number of decisions, resolutions, and orders on annulment of
marriage cases allegedly issued by Judge Indar. In this case, Judge Indar was given ample opportunity to
controvert the charges against him. While there is no proof
The Audit Team found that the list of cases submitted by the that Judge Indar personally received the notices of hearing
Local Civil Registrars of Manila and Quezon City do not issued by the Investigating Justices, the first two notices of
appear in the records of cases received, pending or disposed hearing were received by one Mustapha Randang of the Clerk
by RTC. of Court, RTC-Cotabato, while one of the notices was received
by a certain Mrs. Asok, who were presumably authorized and
Likewise, the annulment decisions did not exist in the records
capable to receive notices on behalf of Judge Indar.
of RTC-Cotabato, Branch 14. The Audit Team further
observed that the case numbers in the list submitted by the The Uniform Rules on Administrative Cases in the Civil
Local Civil Registrars are not within the series of case Service, which govern the conduct of disciplinary and
numbers recorded in the docket books nondisciplinary proceedings in administrative cases, clearly
provide that technical rules of procedure and evidence do not
The audit team asked for confirmation on the authenticity of
strictly apply to administrative proceedings.
Judge Indar’s decision, dated 23 May 2007, in Spec. Proc. No.
06-581, entitled “Chona Chanco Aguiling v. Alan V. Aguiling,” “Section 3. Technical Rules in Administrative
for Declaration of Nullity of Marriage. As regards this case, the Investigations.· Administrative investigations shall be
Audit Team found that Spec. Proc. No. 06-584 does not exist conducted without necessarily adhering strictly to the
in the records of cases filed, pending or disposed by RTC- technical rules of procedure and evidence applicable to
Shariff Aguak. judicial proceedings.”
Meanwhile, in compliance with DCA Villasor’s Indorsement In other words, Judge Indar, who had sworn to faithfully uphold
and in response to the Australian Embassy letter, Judge Indar the law, issued decisions on the questioned annulment of
explained, in a Letter dated 10 March 2010, that “this court is marriage cases, without any showing that such cases
a Court of General Jurisdiction and can therefore act even on underwent trial and complied with the statutory and
cases involving Family Relations. Hence, the subject decision jurisprudential requisites for voiding marriages. Such act
rendered by this Court annulling the marriage of your client is undoubtedly constitutes gross misconduct.
VALID and she is free to marry.”
In this case, Judge Indar issued Decisions on numerous
In a Resolution dated 4 May 2010, the Court En Banc annulment of marriage cases when in fact he did not conduct
preventively suspended Judge Indar pending investigation of any judicial proceedings on the cases. Not even the filing of
this case. the petitions occurred. Judge Indar made it appear in his
Decisions that the annulment cases complied with the
Judge Indar failed to attend the hearing as rescheduled and
stringent requirements of the Rules of Court and the strict
to submit the affidavit as required.
statutory and jurisprudential conditions for voiding marriages,
Justice Borreta proceeded to determine Judge Indar’s when quite the contrary is true, violating Canon 3 of the Code
administrative liability, and found the latter guilty of serious of Judicial Conduct which mandates that a judge perform
misconduct and dishonesty. official duties honestly.

According to Justice Borreta, Judge Indar’s act of issuing Considering that Judge Indar is guilty of gross misconduct and
decisions on annulment of marriage cases without complying dishonesty, constituting violations of the Lawyer’s Oath, and
with the stringent procedural and substantive requirements of Canons 1 and 7 and Rule 1.01 of the Code of Professional
the Rules of Court for such cases clearly violates the Code of Responsibility, Judge Indar deserves disbarment.
Judicial Conduct. Judge Indar made it appear that the
annulment cases underwent trial, when the records show no
judicial proceedings occurred.

Moreover, Judge Indar’s act of affirming in writing before the


Australian Embassy the validity of a decision he allegedly
rendered, when in fact that case does not appear in the court’s
records, constitutes dishonesty. Justice Borreta
recommended the dismissal of Judge Indar from service, and
the investigation of Atty. Silongan, who is not included as
respondent in this case, on her participation in the certification
of the authenticity of the spurious Decisions.

Page 19 of 186
6. GO v. COLEGIO DE SAN JUAN DE LETRAN ISSUE: WON petitionser’s right to due process was violated.

FACTS: October 2001, Mr. George Isleta, the Head of RULING: NO. In Ateneo de Manila University v. Capulong, the
Letran’s Auxiliary Services Department, received information Court held that Guzman v. National University, is the authority
that certain fraternities were recruiting new members among on the procedural rights of students in disciplinary cases. In
Letran’s high school students. He also received a list of the Guzman, we laid down the minimum standards in the
students allegedly involved. School authorities started an imposition of disciplinary sanctions in academic institutions, as
investigation, including the conduct of medical examinations follows:
on the students whose names were on the list.
[I]t bears stressing that due process in disciplinary cases
The school physician, reported that six (6) students bore involving students does not entail proceedings and hearings
injuries, probable signs of blunt trauma of more than two similar to those prescribed for actions and proceedings in
weeks, on the posterior portions of their thighs. Mr. Rosarda, courts of justice. The proceedings in student discipline cases
the Assistant Prefect for Discipline, conferred with the may be summary; and crossexamination is not, contrary to
students and asked for their explanations in writing. petitioners’ view, an essential part thereof. There are withal
minimum standards which must be met to satisfy the demands
Four (4) students, namely admitted that they were neophytes of procedural due process; and these are, that (1) the students
of the Tau Gamma Fraternity and were present in a hazing rite must be informed in writing of the nature and cause of any
held on October 3, 2001 in the house of one Dulce in Tondo, accusation against them; (2) they shall have the right to
Manila. They also identified the senior members of the answer the charges against them, with the assistance of
fraternity present at their hazing. These included Kim Go, then counsel, if desired; (3) they shall be informed of the evidence
a fourth year high school student. against them; (4) they shall have the right to adduce evidence
in their own behalf; and (5) the evidence must be duly
At the Parents-Teachers Conference, Rosarda informed Kim’s
considered by the investigating committee or official
mother, petitioner Mrs. Angelita Go (Mrs. Go), that students
designated by the school authorities to hear and decide the
had positively identified Kim as a fraternity member. Mrs. Go
case.
expressed disbelief as her son was supposedly under his
parents’ constant supervision. Since disciplinary proceedings may be summary, the
insistence that a “formal inquiry” on the accusation against
Mr. Rosarda thereafter spoke to Kim and asked him to explain
Kim should have been conducted lacks legal basis. It has no
his side. Kim responded through a written statement; he
factual basis as well. While the petitioners state that Mr. and
denied that he was a fraternity member. He stated that at that
Mrs. Go were “never given an opportunity to assist Kim,”56
time, he was at Dulce’s house to pick up a gift, and did not
the records show that the respondents gave them two (2)
attend the hazing. On the same day, Mr. Rosarda requested
notices, dated December 19, 2001 and January 8, 2002, for
Kim’s parents (by notice) to attend a conference on January
conferences on January 8, 2002 and January 15, 2002.57 The
8, 2002 to address the issue of Kim’s fraternity membership.
notices clearly state: “Dear Mr./Mrs. Go, We would like to seek
Both Mrs. Go and petitioner Mr. Eugene Go (Mr. Go) did not
your help in correcting Kim’s problem on: Discipline & Conduct
attend the conference.
Offense: Membership in Fraternity.” Thus, the respondents
The respondents found substantial basis in the neophytes’ had given them ample opportunity to assist their son in his
statements that Kim was a senior fraternity member. Letran disciplinary case. “Where a party was afforded an opportunity
decided to allow the fourth year students to graduate from to participate in the proceedings but failed to do so, he cannot
Letran. Students who were not in their fourth year were [thereafter] complain of deprivation of due process.”
allowed to finish the current school year but were barred from
The petitioners nevertheless argue that the respondents
subsequent enrollment in Letran.
defectively observed the written notice rule because they had
Mr. Rosarda conveyed to Mrs. Go and Kim, in their conference requested, and received, Kim’s written explanation at a time
on January 15, 2002, the decision to suspend Kim from when the respondents had not yet issued the written notice of
January 16, 2002 to February 18, 2002. Incidentally, Mr. Go the accusation against him.
did not attend this conference. On even date, Mrs. Go
We see no merit in this argument as the petitioners apparently
submitted a request for the deferment of Kim’s suspension to
hew to an erroneous view of administrative due process.
January 21, 2002 so that he could take a previously scheduled
Jurisprudence has clarified that administrative due process
examination. The request was granted.
cannot be fully equated with due process in the strict judicial
The respondents proposed that the students and their parents sense. The very nature of due process negates any concept
sign a pro-forma agreement to signify their conformity with of inflexible procedures universally applicable to every
their suspension. Mr. and Mrs. Go refused to sign. They also imaginable situation. Thus, we are hard pressed to believe
refused to accept the respondents’ finding that Kim was a that Kim’s denial of his fraternity membership before formal
fraternity member. They likewise insisted that due process notice was given worked against his interest in the disciplinary
had not been observed. case. What matters for due process purpose is notice of what
is to be explained, not the form in which the notice is given.
Subsequently, the petitioners filed a complaint for damages
before the RTC of Caloocan City claiming that the The raison d’etre of the written notice rule is to inform the
respondents had unlawfully dismissed Kim. Mr. and Mrs. Go student of the disciplinary charge against him and to enable
also sought compensation for the “business opportunity him to suitably prepare a defense. We are not convinced that
losses” they suffered while personally attending to Kim’s Kim’s right to explain his side as exercised in his written denial
disciplinary case. had been violated or diminished. The essence of due process,
it bears repeating, is simply the opportunity to be heard. And
Kim had been heard. His written explanation was received,

Page 20 of 186
indeed even solicited, by the respondents. Thus, he cannot 7. CUDIA v. SUPERINTENDENT OF THE PHILIPPINE
claim that he was denied the right to adduce evidence in his MILITARY ACADEMY
behalf.

FACTS: Cadet 1 CL Cudia was a member of Siklab Diwa


Class of 2014 of the PMA, the country's premiere military
academy located at Fort Gregorio del Pilar in Baguio City. He
belonged to the "A" Company and was the Deputy Baron of
his class. As claimed by petitioners and petitioner-intervenor
(hereinafter collectively called "petitioners," unless otherwise
indicated), he was supposed to graduate with honors as the
class salutatorian, receive the Philippine Navy Saber as the
top Navy cadet graduate, and be commissioned as an ensign
of the Philippine Navy.

Professor Juanita Beron 5th period class issued a


Delinquency Report (DR) against Cadet 1 CL Cudia because
he was "late for two (2) minutes in his Eng 412 class. the DRs
reached the Department of Tactical Officers. They were
logged and transmitted to the Company Tactical Officers
(CTO) for explanation of the concerned cadets. Two days
later, Cadet lCL Cudia received his DR. Cadet lCL Cudia was
informed that Maj. Hindang reported him to the HC21 for
violation of the Honor Code. the HC constituted a team to
conduct a preliminary investigation on the reported honor
violation of Cadet 1 CL Cudia. The formal investigation against
Cadet 1 CL Cu di a then ensued. the OIC of the HC forwarded
the Formal Investigation Report to the Staff Judge Advocate
(SJA) for review. Special Orders No. 26 was issued by the
PMA Headquarters placing Cadet 1 CL Cudia on indefinite
leave of absence without pay and allowances effective
February 10, 2014 pending approval of his separation by the
AFPGHQ, barring him from future appointment and/or
admission as cadet, and not permitting him to qualify for any
entrance requirements to the PMA. Vice Admiral Abogado
approved the recommendation to dismiss Cadet 1 CL Cudia.
Cadet lCL Cudia submitted a letter to the Office of the
Commandant of Cadets requesting for reinstatement by the
PMA of his status as a cadet.

Cadet lCL Cudia made his personal appeal letter to Maj. Gen.
Lopez. On even date, the AFP Chief of Staff ordered a
reinvestigation following the viral Facebook post of Annavee
demanding the intervention of the military leadership. Cadet 1
CL Cudia and his family engaged the services of the Public
Attorney's Office (PAO) in Baguio City. the Spouses Cudia
filed a letter-complaint before the CHR-Cordillera
Administrative Region (CAR) Office against the HC members
and Maj. Gracilla for alleged violation of the human rights of
Cadet lCL Cudia, particularly his rights to due process,
education, and privacy of communication. CHR-CAR issued
its Resolution with respect to CHR-CAR Case.The
Commission on Human Rights-CAR Office finds PROBABLE
CAUSE FOR HUMAN RIGHTS VIOLATIONS against the
officers and members of the PMA Honor Committee and ..
certain PMA officials, specifically for violations of the rights of
CADET ALDRIN JEFF P. CUDIA to dignity, due process,
education, privacy/privacy of communication, and good life

ISSUES:

1. Whether or not the PMA committed grave abuse of


discretion in dismissing Cudia in utter disregard of his right to
due process and in holding that he violated the (Honor
Codethrough lying

Page 21 of 186
2. Whether or not the court can interfere ,with military affair 8. AGABON v. NLRC

RULING: FACTS: Riviera Home Improvements, Inc. is engaged in the


business of selling and installing ornamental and construction
1.No. The determination of ,whether the PMA cadet has rights materials. It employed petitioners Virgilio Agabon and Jenny
to due process, education,and property should be placed in Agabon as gypsum board and cornice installers on January 2,
the context of the Honor Code.All the administrative 1992 until February 23, 1999 when they were dismissed for
reremedies ,were exhausted. A student of a military academy abandonment of work. Thus, Petitioners then filed a complaint
must be prepared to subordinate his private interest for the for illegal dismissal and payment of money claims. Petitioners
proper functioning of the institution" The PMA may impose also claim that private respondent did not comply with the twin
disciplinary measures and punishments as it deems fit and requirements of notice and hearing. Private respondent, on
consistent ,with the peculiar needs of the institution" PMA the other hand, maintained that petitioners were not dismissed
hasre!ulatory authority to administratively dismiss erring but had abandoned their work
cadets. PMA has a right to invoke academic freedom in the
enforcement of the internal rules and regulations.

ISSUE: Whether or not petitioners were illegally dismissed.

2. Yes. The court is part of the checks and balance machinery


mandated by Article VIII of the Constitution. The courts
mandate according to Section 1 Article 8 is expanded that the RULING: Petitioners’ dismissal was for a just cause. They had
duty of the courts is not only to settle actual controversies abandoned their employment and were already working for
involving rights ,which are legally demandable and another employer. Abandonment is the deliberate and
enforceable but also to determine ,whether or not there has unjustified refusal of an employee to resume his employment.
been a grave abuse of discretion on the part of any branch or It is a form of neglect of duty, hence, a just cause for
instrumentality of the Government even if the latter does not termination of employment by the employer.
exercise judicial' quasi-judicial' or ministerial functions. No one
To dismiss an employee, the law requires not only the
is above the law' including the military' especially in violations
existence of a just and valid cause but also enjoins the
of Constitutionally guaranteed rights.
employer to give the employee the opportunity to be heard and
to defend himself. Thus, the procedures for dismissal should
be observed. Procedurally, (1) if the dismissal is based on a
just cause under Article 282, the employer must give the
employee two written notices and a hearing or opportunity to
be heard if requested by the employee before terminating the
employment: a notice specifying the grounds for which
dismissal is sought a hearing or an opportunity to be heard
and after hearing or opportunity to be heard, a notice of the
decision to dismiss; and (2) if the dismissal is based on
authorized causes under Articles 283 and 284, the employer
must give the employee and the Department of Labor and
Employment written notices 30 days prior to the effectivity of
his separation.

From the foregoing rules four possible situations may be


derived: (1) the dismissal is for a just cause under Article 282
of the Labor Code, for an authorized cause under Article 283,
or for health reasons under Article 284, and due process was
observed; (2) the dismissal is without just or authorized cause
but due process was observed; (3) the dismissal is without just
or authorized cause and there was no due process; and (4)
the dismissal is for just or authorized cause but due process
was not observed.

The present case squarely falls under the fourth situation. The
dismissal should be upheld because it was established that
the petitioners abandoned their jobs to work for another
company. Private respondent, however, did not follow the
notice requirements and instead argued that sending notices
to the last known addresses would have been useless
because they did not reside there anymore. Unfortunately for
the private respondent, this is not a valid excuse because the
law mandates the twin notice requirements to the employee’s
last known address. Thus, it should be held liable for non-
compliance with the procedural requirements of due process.
That in cases involving dismissals for cause but without
observance of the twin requirements of notice and hearing, the
better rule is to abandon the Serrano doctrine and to follow
Wenphil by holding that the dismissal was for just cause but

Page 22 of 186
imposing sanctions on the employer. Such sanctions, 9. MICHAEL H. v. GERALD D.
however, must be stiffer than that imposed in Wenphil.

Where the dismissal is for a just cause, as in the instant case,


the lack of statutory due process should not nullify the FACTS: Gerald D. was the presumptive father of Victoria D.
dismissal, or render it illegal, or ineffectual. However, the since she was born to his wife Carole D.. However, Carole had
employer should indemnify the employee for the violation of an adulterous partner, Michael H., who obtained blood tests
his statutory rights in the form of nominal damages, i.e. indicating that he was likely the biological father. When
P30,000.00. Michael obtained visitation rights in a California state court,
Gerald argued that Michael had no ground under California
law to challenge Gerald's paternity since more than two years
had passed since Victoria's birth. According to Cal. Evid. Code
621, the child is "presumed to be a child of the marriage" and
another man can only challenge this presumption within two
years of birth. The court ruled in favor of Gerald and canceled
Michael's visitation rights. Michael claimed that Code 621
violated his Fourteenth Amendment due process rights by
denying him an opportunity to establish his paternity. A
California Court of Appeals upheld the constitutionality of
Code 621.

ISSUE: Does Cal. Evid. Code 621 violate the Due Process
Clause by denying a possible biological father the chance to
establish his paternity of a child after two years have passed
since the child's birth?

RULING: NO. Based on its analysis of common-law tradition,


the plurality opinion found that a possible biological father
does not have a fundamental right to obtain parental rights
after the presumptive father has exercised significant
responsibility over the child. Therefore due process protection
does not apply. Code 621 was based on common-law
precedent which showed "an aversion to declaring children
illegitimate" and supported "the interest in promoting the
'peace and tranquility of States and families.'" Restricting
Michael's parental rights achieved this by granting Gerald the
sole responsibility to play the role of Victoria's father.

Page 23 of 186
10. WASHINGTON V. GLUCKSBERG, 521 U.S. 702 asserted right has no place in our Nation's traditions, given the
(1997) country's consistent, almost universal, and continuing
rejection of the right, even for terminally ill, mentally competent
adults. To hold for respondents, the Court would have to
reverse centuries of legal doctrine and practice, and strike
FACTS: It as always been a crime to assist a suicide in the
down the considered policy choice of almost every State.
State of Washington. The State's present law makes
"[p]romoting a suicide attempt" a felony, and provides: "A The constitutional requirement that Washington's assisted
person is guilty of [that crime] when he knowingly causes or suicide ban be rationally related to legitimate government
aids another person to attempt suicide." Dr. Harold interests is unquestionably met here. These interests include
Glucksberg -- along with four other physicians who prohibiting intentional killing and preserving human life;
occasionally treat terminally ill, suffering patients, declare that preventing the serious public health problem of suicide,
they would assist these patients in ending their lives if not for especially among the young, the elderly, and those suffering
the State's assisted suicide ban. They, along with three from untreated pain or from depression or other mental
gravely ill plaintiffs who have since died and a nonprofit disorders; protecting the medical profession's integrity and
organization that counsels people considering physician ethics and maintaining physicians' role as their patients'
assisted suicide, filed this suit against petitioners, the State healers; protecting the poor, the elderly, disabled persons, the
and its Attorney General, seeking a declaration that the ban terminally ill, and persons in other vulnerable groups from
is, on its face, unconstitutional. They assert a liberty interest indifference, prejudice, and psychological and financial
protected by the Fourteenth Amendment's Due Process pressure to end their lives; and avoiding a possible slide
Clause which extends to a personal choice by a mentally towards voluntary and perhaps even involuntary euthanasia.
competent, terminally ill adult to commit physician assisted The relative strengths of these various interests need not be
suicide. The Federal District Court agreed, concluding that weighed exactingly, since they are unquestionably important
Washington's assisted suicide ban is unconstitutional and legitimate, and the law at issue is at least reasonably
because it places an undue burden on the exercise of that related to their promotion and protection.
constitutionally protected liberty interest. The en banc Ninth
Circuit affirmed.

ISSUE: WON Washington's ban on physician assisted-suicide


violate the Fourteenth Amendment's Due Process Clause by
denying competent terminally ill adults the liberty to choose
death over life. NO

HELD: An examination of our Nation's history, legal traditions,


and practices demonstrates that Anglo American common law
has punished or otherwise disapproved of assisting suicide for
over 700 years; that rendering such assistance is still a crime
in almost every State; that such prohibitions have never
contained exceptions for those who were near death; that the
prohibitions have in recent years been reexamined and, for
the most part, reaffirmed in a number of States; and that the
President recently signed the Federal Assisted Suicide
Funding Restriction Act of 1997, which prohibits the use of
federal funds in support of physician assisted suicide.

In light of that history, this Court's decisions lead to the


conclusion that respondents' asserted "right" to assistance in
committing suicide is not a fundamental liberty interest
protected by the Due Process Clause. The Court's established
method of substantive due process analysis has two primary
features: First, the Court has regularly observed that the
Clause specially protects those fundamental rights and
liberties which are, objectively, deeply rooted in this Nation's
history and tradition. Second, the Court has required a "careful
description" of the asserted fundamental liberty interest. The
Ninth Circuit's and respondents' various descriptions of the
interest here at stake--e.g., a right to "determin[e] the time and
manner of one's death," the "right to die," a "liberty to choose
how to die," a right to "control of one's final days," "the right to
choose a humane, dignified death," and "the liberty to shape
death"--run counter to that second requirement. Since the
Washington statute prohibits "aiding another person to
attempt suicide," the question before the Court is more
properly characterized as whether the "liberty" specially
protected by the Clause includes a right to commit suicide
which itself includes a right to assistance in doing so. This

Page 24 of 186
11. LAWRENCE et al. v. TEXAS 539 U.S. 558 (2003) and implicitly invalidated similar sodomy statutes in 13 other
Argued March 26, 2003—Decided June 26, 2003; states.
JUSTICE KENNEDY
Resolution of this case depends on whether petitioners were
free as adults to engage in private conduct in the exercise of
their liberty under the Due Process Clause. For this inquiry the
FACTS: In Houston, Texas, officers of the Harris County US Supreme Court deems it necessary to reconsider its
Police Department were dispatched to a private residence in Bowers holding. The Bowers Court's initial substantive
response to a reported weapons disturbance. They entered statement-"The issue presented is whether the Federal
an apartment where one of the petitioners, John Geddes Constitution confers a fundamental right upon homosexuals to
Lawrence, resided. The right of the police to enter does not engage in sodomy ... ," To say that the issue in Bowers was
seem to have been questioned. The officers observed simply the right to engage in certain sexual conduct demeans
Lawrence and another man, Tyron Garner, engaging in a the claim the individual put forward, just as it would demean a
sexual act. The two petitioners were arrested, held in custody married couple were it said that marriage is just about the right
over night, and charged and convicted before a Justice of the to have sexual intercourse. Although the laws involved in
Peace. Bowers and here purport to do no more than prohibit a
particular sexual act, their penalties and purposes have more
The complaints described their crime as "deviate sexual
far-reaching consequences, touching upon the most private
intercourse, namely anal sex, with a member of the same sex
human conduct, sexual behavior, and in the most private of
(man)." The applicable state law is Tex. Penal Code Ann. §
places, the home. They seek to control a personal relationship
21.06(a). It provides: "A person commits an offense if he
that, whether or not entitled to formal recognition in the law, is
engages in deviate sexual intercourse with another individual
within the liberty of persons to choose without being punished
of the same sex." The statute defines "[d]eviate sexual
as criminals. The liberty protected by the Constitution allows
intercourse" as follows: (A) any contact between any part of
homosexual persons the right to choose to enter upon
the genitals of one person and the mouth or anus of another
relationships in the confines of their homes and their own
person; or (B) the penetration of the genitals or the anus of
private lives and still retain their dignity as free persons.
another person with an object." § 21.01(1).
Having misapprehended the liberty claim presented to it, the
The petitioners exercised their right to a trial de novo in Harris
Bowers Court stated that proscriptions against sodomy have
County Criminal Court. They challenged the statute as a
ancient roots. It should be noted, however, that there is no
violation of the Equal Protection Clause of the Fourteenth
longstanding history in U.S. of laws directed at homosexual
Amendment and of a like provision of the Texas Constitution.
conduct as a distinct matter. Early American sodomy laws
Those contentions were rejected. The petitioners, having
were not directed at homosexuals as such but instead sought
entered a plea of nolo contendere, were each fined $200 and
to prohibit non-procreative sexual activity more generally,
assessed court costs of $141.25.
whether between men and women or men and men.
The Court of Appeals for the Texas Fourteenth District Moreover, early sodomy laws seem not to have been enforced
considered the petitioners' federal constitutional arguments against consenting adults acting in private. Instead, sodomy
under both the Equal Protection and Due Process Clauses of prosecutions often involved predatory acts against those who
the Fourteenth Amendment. After hearing the case en banc could not or did not consent: relations between men and minor
the court, in a divided opinion, rejected the constitutional girls or boys, between adults involving force, between adults
arguments and affirmed the convictions. In affirming, the implicating disparity in status, or between men and animals.
State Court of Appeals held, inter alia, that the statute was not The longstanding criminal prohibition of homosexual sodomy
unconstitutional under the Due Process Clause of the upon which Bowers placed such reliance is as consistent with
Fourteenth Amendment. Then, petitioners filed a petition for a general condemnation of non-procreative sex as it is with an
certiorari in the U.S. Supreme Court. established tradition of prosecuting acts because of their
homosexual character. The Bowers Court was, of course,
making the broader point that for centuries there have been
powerful voices to condemn homosexual conduct as immoral,
ISSUES: (1) WON the petitioners' criminal convictions under
but the US Supreme Court's obligation is to define the liberty
the Texas "Homosexual Conduct" law—which criminalizes
of all, not to mandate its own moral code. The Nation's laws
sexual intimacy by same-sex couples, but not identical
and traditions in the past half century are most relevant here.
behavior by different-sex couples—violate the Fourteenth
They show an emerging awareness that liberty gives
Amendment guarantee of equal protection of the laws.
substantial protection to adult persons in deciding how to
(2) WON the petitioners' criminal convictions for adult conduct their private lives in matters pertaining to sex.
consensual sexual intimacy in their home violate their vital
Bowers' deficiencies became even more apparent in the years
interests in liberty and privacy protected by the Due Process
following its announcement. The 25 States with laws
Clause of the Fourteenth Amendment
prohibiting the conduct referenced in Bowers are reduced now
(3) WON Bowers v. Hardwick should be overruled. to 13, of which 4 enforce their laws only against homosexual
conduct. In those States, including Texas, that still proscribe
sodomy (whether for same-sex or heterosexual conduct),
there is a pattern of nonenforcement with respect to
HELD: The main question before the Court is the validity of a consenting adults acting in private. Planned Parenthood of
Texas statute making it a crime for two persons of the same Southeastern Pa. v. Casey, 505 U.S. 833, 844 (1992)
sex to engage in certain intimate sexual conduct. On June 26, confirmed that the Due Process Clause protects personal
2003, the Supreme Court released its 6–3 decision striking decisions relating to marriage, procreation, contraception,
down the questioned Texas statute. Five justices held it family relationships, child rearing, and education-and Romer
violated due process guarantees, and a sixth, Sandra Day v. Evans, 517 U. S. 620, 624-which struck down class-based
O'Connor, held it violated equal protection guarantees. The legislation directed at homosexuals-cast Bowers' holding into
five-member majority opinion overruled Bowers v. Hardwick

Page 25 of 186
even more doubt. The stigma the Texas criminal statute 12. OBERGEFELL ET AL. v. HODGES, DIRECTOR, OHIO
imposes, moreover, is not trivial. DEPARTMENT OF HEALTH, ET AL.

Although the offense is but a minor misdemeanor, it remains


a criminal offense with all that imports for the dignity of the
persons charged, including notation of convictions on their FACTS: The petitioners, 14 same-sex couples and two men
records and on job application forms, and registration as sex whose same-sex partners are deceased, filed suits in Federal
offenders under state law. Where a case's foundations have District Courts in their home States, claiming that respondent
sustained serious erosion, criticism from other sources is of state officials violate the Fourteenth Amendment by denying
greater significance. In the United States, criticism of Bowers them the right to marry or to have marriages lawfully
has been substantial and continuing, disapproving of its performed in another State given full recognition. Each District
reasoning in all respects, not just as to its historical Court ruled in their favor. Respondents appealed the
assumptions. And, to the extent Bowers relied on values decisions to the United States Court of Appeals. USCA
shared with a wider civilization, the case's reasoning and reversed the judgments of the District Courts. USCA held that
holding have been rejected by the European Court of Human a State has no constitutional obligation to license same-sex
Rights, and that other nations have taken action consistent marriages or to recognize same-sex marriages performed out
with an affirmation of the protected right of homosexual adults of State. The petitioners sought certiorari before the SC of US.
to engage in intimate, consensual conduct. There has been
no showing that in this country the governmental interest in
circumscribing personal choice is somehow more legitimate or ISSUE: WON the Fourteenth Amendment requires a State to
urgent. Stare decisis is not an inexorable command. Payne v. license/recognize a same-sex marriage licensed and
Tennessee, 501 U.S. 808, 828. Bowers' holding has not performed in a State which does grant that right. YES
induced detrimental reliance of the sort that could counsel
against overturning it once there are compelling reasons to do
so. Bowers causes uncertainty, for the precedents before and
HELD: The Fourteenth Amendment requires a State to
after it contradict its central holding.
license a marriage between two people of the same sex.
In his dissenting opinion in Bowers, JUSTICE STEVENS
The fundamental liberties protected by the Fourteenth
concluded that (1) the fact a State's governing majority has
Amendment’s Due Process Clause extend to certain personal
traditionally viewed a particular practice as immoral is not a
choices central to individual dignity and autonomy, including
sufficient reason for upholding a law prohibiting the practice,
intimate choices defining personal identity and beliefs. Courts
and (2) individual decisions concerning the intimacies of
must exercise reasoned judgment in identifying interests of
physical relationships, even when not intended to produce
the person so fundamental that the State must accord them
offspring, are a form of “liberty” protected by due process. That
its respect. History and tradition guide and discipline the
analysis should have controlled Bowers, and it controls here.
inquiry but do not set its outer boundaries. When new insight
Bowers was not correct when it was decided, is not correct
reveals discord between the Constitution’s central protections
today, and is hereby overruled. This case does not involve
and a received legal stricture, a claim to liberty must be
minors, persons who might be injured or coerced, those who
addressed.
might not easily refuse consent, or public conduct or
prostitution. It does involve two adults who, with full and Applying these tenets, the Court has long held the right to
mutual consent, engaged in sexual practices common to a marry is protected by the Constitution. For example, Loving v.
homosexual lifestyle. Petitioners' right to liberty under the Due Virginia, 388 U. S. 1, 12, invalidated bans on interracial
Process Clause gives them the full right to engage in private unions, and Turner v. Safley, 482 U. S. 78, 95, held that
conduct without government intervention. Casey, supra, at prisoners could not be denied the right to marry. To be sure,
847. The Texas statute furthers no legitimate state interest these cases presumed a relationship involving opposite-sex
which can justify its intrusion into the individual's personal and partners, as did Baker v. Nelson, 409 U. S. 810, a one-line
private life. summary decision issued in 1972, holding that the exclusion
of same-sex couples from marriage did not present a
The judgment of the Court of Appeals for the Texas
substantial federal question. But other, more instructive
Fourteenth District was reversed, and the case was remanded
precedents have expressed broader principles. In assessing
for further proceedings not inconsistent with this (US Supreme
whether the force and rationale of its cases apply to same-sex
Court’s) opinion.
couples, the Court must respect the basic reasons why the
right to marry has been long protected. This analysis compels
the conclusion that same-sex couples may exercise the right
to marry.

Four principles and traditions demonstrate that the reasons


marriage is fundamental under the Constitution apply with
equal force to same-sex couples. First, the right to personal
choice regarding marriage is inherent in the concept of
individual autonomy. Decisions about marriage are among the
most intimate that an individual can make. This is true for all
persons, whatever their sexual orientation. Second, the right
to marry is fundamental because it supports a two-person
union unlike any other in its importance to the committed
individuals. Same-sex couples have the same right as
opposite-sex couples to enjoy intimate association. Third, right
to marry safeguards children and families and thus draws
meaning from related rights of childrearing, procreation, and

Page 26 of 186
education. Without the recognition, stability, and predictability 13. WHITE LIGHT CORPORATION, TITANIUM
marriage offers, children suffer the stigma of knowing their CORPORATION and STA. MESA TOURIST &
families are somehow lesser. They also suffer the significant DEVELOPMENT CORPORATION vs. CITY OF
material costs of being raised by unmarried parents, relegated MANILA, represented by DE CASTRO, MAYOR
to a more difficult and uncertain family life. The marriage laws ALFREDO S. LIM
at issue thus harm and humiliate the children of same-sex
couples. Finally, the Court’s cases and the Nation’s traditions
make clear that marriage is a keystone of the Nation’s social
FACTS: On December 3, 1992, City Mayor Alfredo S. Lim
order. States have contributed to the fundamental character
(Mayor Lim) signed into law Ordinance No. 7774 entitled, "An
of marriage by placing it at the center of many facets of the
Ordinance Prohibiting Short-Time Admission, Short-Time
legal and social order. There is no difference between same
Admission Rates, and Wash-Up Rate Schemes in Hotels,
and opposite-sex couples with respect to this principle, yet
Motels, Inns, Lodging Houses, Pension Houses, and Similar
same-sex couples are denied the constellation of benefits that
Establishments in the City of Manila"
the States have linked to marriage. It is demeaning to lock
same-sex couples out of a central institution of the Nation’s Malate Tourist and Development Corporation (MTDC) filed a
society, for they too may aspire to the transcendent purposes complaint for declaratory relief with the RTC praying that the
of marriage. Ordinance, insofar as it includes motels and inns as among its
prohibited establishments, be declared invalid and
unconstitutional.

MTDC claimed that as owner and operator of the Victoria


Court in Malate, Manila it was authorized by Presidential
Decree (P.D.) No. 259 to admit customers on a short time
basis as well as to charge customers wash up rates for stays
of only three hours.

Petitioners White Light Corporation (WLC), Titanium


Corporation (TC) and Sta. Mesa Tourist and Development
Corporation (STDC) filed a motion to intervene and to admit
attached complaint-in-intervention7 on the ground that the
Ordinance directly affects their business interests as
operators of drive-in-hotels and motels in Manila. The three
companies are components of the Anito Group of Companies
which owns and operates several hotels and motels in Metro
Manila.

RTC rendered a decision declaring the Ordinance null and


void.

Version of the Respondents

Before the Court of Appeals, the City asserted that the


Ordinance is a valid exercise of police power pursuant to
Section 458 (4)(iv) of the Local Government Code which
confers on cities, among other local government units, the
power:

[To] regulate the establishment, operation and


maintenance of cafes, restaurants, beerhouses,
hotels, motels, inns, pension houses, lodging houses
and other similar establishments, including tourist
guides and transports.

The Ordinance, it is argued, is also a valid exercise of the


power of the City under Article III, Section 18(kk) of the
Revised Manila Charter, thus:

"to enact all ordinances it may deem necessary and


proper for the sanitation and safety, the furtherance
of the prosperity and the promotion of the morality,
peace, good order, comfort, convenience and
general welfare of the city and its inhabitants, and
such others as be necessary to carry into effect and
discharge the powers and duties conferred by this
Chapter; and to fix penalties for the violation of
ordinances which shall not exceed two hundred
pesos fine or six months imprisonment, or both such
fine and imprisonment for a single offense.

Page 27 of 186
Lacking a concurrence of these requisites, the police measure
shall be struck down as an arbitrary intrusion into private
Version of the Petitioner rights. As held in Morfe v. Mutuc, the exercise of police power
is subject to judicial review when life, liberty or property is
Petitioners argued that the Ordinance is unconstitutional and
affected. However, this is not in any way meant to take it away
void since it violates the right to privacy and the freedom of
from the vastness of State police power whose exercise
movement; it is an invalid exercise of police power; and it is
enjoys the presumption of validity.
an unreasonable and oppressive interference in their
business. The behavior which the Ordinance seeks to curtail is in fact
already prohibited and could in fact be diminished simply by
The Court of Appeals reversed the decision of the RTC and
applying existing laws. Less intrusive measures such as
affirmed the constitutionality of the Ordinance.
curbing the proliferation of prostitutes and drug dealers
TC, WLC and STDC come to this Court via petition for review through active police work would be more effective in easing
on certiorari. In their petition and Memorandum, petitioners in the situation. So would the strict enforcement of existing laws
essence repeat the assertions they made before the Court of and regulations penalizing prostitution and drug use. These
Appeals. They contend that the assailed Ordinance is an measures would have minimal intrusion on the businesses of
invalid exercise of police power. the petitioners and other legitimate merchants. Further, it is
apparent that the Ordinance can easily be circumvented by
merely paying the whole day rate without any hindrance to
those engaged in illicit activities. Moreover, drug dealers and
ISSUE: WON Ordinance No. 7774 be declared void and
prostitutes can in fact collect "wash rates" from their clientele
unconstitutional. YES
by charging their customers a portion of the rent for motel
rooms and even apartments.

HELD: Ordinance N. 7774 is void and unconstitutional. WHEREFORE, the Petition is GRANTED. The Decision of the
Court of Appeals is REVERSED, and the Decision of the
Police power, while incapable of an exact definition, has been Regional Trial Court of Manila, Branch 9, is REINSTATED.
purposely veiled in general terms to underscore its Ordinance No. 7774 is hereby declared
comprehensiveness to meet all exigencies and provide UNCONSTITUTIONAL. No pronouncement as to costs.
enough room for an efficient and flexible response as the
conditions warrant. Police power is based upon the concept of
necessity of the State and its corresponding right to protect
itself and its people. Police power has been used as
justification for numerous and varied actions by the State.
These range from the regulation of dance halls, movie
theaters, gas stations and cockpits. The awesome scope of
police power is best demonstrated by the fact that in its
hundred or so years of presence in our nation’s legal system,
its use has rarely been denied.

The apparent goal of the Ordinance is to minimize if not


eliminate the use of the covered establishments for illicit sex,
prostitution, drug use and alike. These goals, by themselves,
are unimpeachable and certainly fall within the ambit of the
police power of the State. Yet the desirability of these ends do
not sanctify any and all means for their achievement. Those
means must align with the Constitution, and our emerging
sophisticated analysis of its guarantees to the people. The Bill
of Rights stands as a rebuke to the seductive theory of
Macchiavelli, and, sometimes even, the political majorities
animated by his cynicism.

That the Ordinance prevents the lawful uses of a wash rate


depriving patrons of a product and the petitioners of lucrative
business ties in with another constitutional requisite for the
legitimacy of the Ordinance as a police power measure. It
must appear that the interests of the public generally, as
distinguished from those of a particular class, require an
interference with private rights and the means must be
reasonably necessary for the accomplishment of the purpose
and not unduly oppressive of private rights.71 It must also be
evident that no other alternative for the accomplishment of the
purpose less intrusive of private rights can work. More
importantly, a reasonable relation must exist between the
purposes of the measure and the means employed for its
accomplishment, for even under the guise of protecting the
public interest, personal rights and those pertaining to private
property will not be permitted to be arbitrarily invaded.

Page 28 of 186
14. GOVERNMENT SERVICE INSURANCE SYSTEM, acquires a vested right to benefits that have become due as
Cebu City Branch vs. MILAGROS O. provided under the terms of the public employees' pension
MONTESCLAROS, (G.R. No. 146494; July 14, 2004) statute. No law can deprive such person of his pension rights
without due process of law, that is, without notice and
opportunity to be heard.
FACTS: Sangguniang Bayan member Nicolas Montesclaros In addition to retirement and disability benefits, PD 1146 also
married Milagros Orbiso on 10 July 1983. Nicolas was a 72- provides for benefits to survivors of deceased government
year old widower when he married Milagros who was then 43 employees and pensioners. Under PD 1146, the dependent
years old. spouse is one of the beneficiaries of survivorship benefits. A
widow's right to receive pension following the demise of her
Nicolas filed with the GSIS an application for retirement
husband is also part of the husband's contractual
benefits under Presidential Decree No. 1146 or the Revised
compensation.
Government Service Insurance Act of 1977 ("PD 1146"). In his
retirement application, Nicolas designated his wife Milagros as
his sole beneficiary. GSIS approved Nicolas' application for
retirement "effective 17 February 1984," granting a lump sum Denial of Due Process
payment of annuity for the first five years and a monthly
annuity thereafter. Nicolas died on 22 April 1992. Milagros The proviso is contrary to Section 1, Article III of the
filed with GSIS a claim for survivorship pension under PD Constitution, which provides that "[n]o person shall be
1146. On 8 June 1992, GSIS denied the claim because under deprived of life, liberty, or property without due process of law,
Section 18 of PD 1146, the surviving spouse has no right to nor shall any person be denied the equal protection of the
survivorship pension if the surviving spouse contracted the laws." The proviso is unduly oppressive in outrightly denying
marriage with the pensioner within three years before the a dependent spouse's claim for survivorship pension if the
pensioner qualified for the pension. According to GSIS, dependent spouse contracted marriage to the pensioner
Nicolas wed Milagros on 10 July 1983, less than one year from within the three-year prohibited period. There is outright
his date of retirement on "17 February 1984." confiscation of benefits due the surviving spouse without
giving the surviving spouse an opportunity to be heard. The
On 2 October 1992, Milagros filed with the trial court a special proviso undermines the purpose of PD 1146, which is to
civil action for declaratory relief questioning the validity of assure comprehensive and integrated social security and
Section 18 of PD 1146 disqualifying her from receiving insurance benefits to government employees and their
survivorship pension. dependents in the event of sickness, disability, death, and
retirement of the government employees.
Trial court: rendered judgment declaring Milagros eligible for
survivorship pension. The "whereas" clauses of PD 1146 state:

CA: affirmed WHEREAS, the Government Service Insurance


System in promoting the efficiency and welfare of the
employees of the Government of the Philippines,
administers the laws that grant to its members social
ISSUE: WON Section 18 of PD 1146 violates due process.
security and insurance benefits;
YES
WHEREAS, it is necessary to preserve at all times
the actuarial solvency of the funds administered by
HELD: SEC. 18. Death of a Pensioner. Upon the death of a the System; to guarantee to the government
pensioner, the primary beneficiaries shall receive the employee all the benefits due him; and to expand
applicable pension mentioned under paragraph (b) of section and increase the benefits made available to him and
seventeen of this Act: Provided, That, the dependent spouse his dependents to the extent permitted by available
shall not be entitled to said pension if his marriage with the resources;
pensioner is contracted within three years before the
WHEREAS, provisions of existing laws have
pensioner qualified for the pension. When the pensioner dies
impeded the efficient and effective discharge by the
within the period covered by the lump sum, the survivorship
System of its functions and have unduly hampered
pension shall be paid only after the expiration of the said
the System from being more responsive to the
period. This shall also apply to the pensioners living as of the
dramatic changes of the times and from meeting the
effectivity of this Act, but the survivorship benefit shall be
increasing needs and expectations of the Filipino
based on the monthly pension being received at the time of
public servant;
death. (Emphasis supplied)
WHEREAS, provisions of existing laws that have
Under PD 1146, the primary beneficiaries are (1) the
prejudiced, rather than benefited, the government
dependent spouse until such spouse remarries, and (2) the
employee; restricted, rather than broadened, his
dependent children.
benefits, prolonged, rather than facilitated the
payment of benefits, must now yield to his paramount
welfare;
Retirement Benefits as Property Interest
WHEREAS, the social security and insurance
Where the employee retires and meets the eligibility benefits of government employees must be
requirements, he acquires a vested right to benefits that is continuously re-examined and improved to assure
protected by the due process clause. Retirees enjoy a comprehensive and integrated social security and
protected property interest whenever they acquire a right to insurance programs that will provide benefits
immediate payment under pre-existing law. Thus, a pensioner responsive to their needs and those of their

Page 29 of 186
dependents in the event of sickness, disability, 15. REPUBLIC OF THE PHILIPPINES vs. LIBERTY D.
death, retirement, and other contingencies; and to ALBIOS (G.R. No. 198780; October 16, 2013)
serve as a fitting reward for dedicated public service;

WHEREAS, in the light of existing economic


conditions affecting the welfare of government FACTS: Fringer, an American citizen, and Albios were
employees, there is a need to expand and improve married before Judge Ofelia I. Calo of the Mandaluyong City
the social security and insurance programs (MeTC).
administered by the Government Service Insurance
Albios filed with the RTC a petition for declaration of nullity of
System, specifically, among others, by increasing
her marriage with Fringer. She alleged that immediately after
pension benefits, expanding disability benefits,
their marriage, they separated and never lived as husband
introducing survivorship benefits, introducing
and wife because they never really had any intention of
sickness and income benefits, and eventually
entering into a married state or complying with any of their
extending the compulsory coverage of these
essential marital obligations. She described their marriage as
programs to all government employees regardless of
one made in jest and, therefore, null and void ab initio .
employment status.
The RTC declared the marriage void ab initio. The RTC was
of the view that the parties married each other for convenience
PD 1146 has the following purposes: only. Giving credence to the testimony of Albios, it stated that
she contracted Fringer to enter into a marriage to enable her
a. to preserve at all times the actuarial solvency of to acquire American citizenship; that in consideration thereof,
the funds administered by the System; she agreed to pay him the sum of $2,000.00; that after the
ceremony, the parties went their separate ways; that Fringer
b. to guarantee to the government employee all the returned to the United States and never again communicated
benefits due him; and with her; and that, in turn, she did not pay him the $2,000.00
because he never processed her petition for citizenship. The
c. to expand, increase, and improve the social
RTC, thus, ruled that when marriage was entered into for a
security and insurance benefits made available to
purpose other than the establishment of a conjugal and family
him and his dependents such as:
life, such was a farce and should not be recognized from its
- increasing pension benefits inception.

- expanding disability benefits CA affirmed the RTC ruling which found that the essential
requisite of consent was lacking.
- introducing survivorship benefits

- introducing sickness income benefits


ISSUE: Is a marriage, contracted for the sole purpose of
- extending compulsory membership to all acquiring American citizenship in consideration of $2,000.00,
government employees irrespective of status25 void ab initio on the ground of lack of consent? NO

The law extends survivorship benefits to the surviving and HELD: Under Article 2 of the Family Code, for consent to be
qualified beneficiaries of the deceased member or pensioner valid, it must be (1) freely given and (2) made in the presence
to cushion the beneficiaries against the adverse economic of a solemnizing officer. A "freely given" consent requires that
effects resulting from the death of the wage earner or the contracting parties willingly and deliberately enter into the
pensioner. marriage.

Based on the above, consent was not lacking between Albios


and Fringer. In fact, there was real consent because it was not
vitiated nor rendered defective by any vice of consent. Their
consent was also conscious and intelligent as they understood
the nature and the beneficial and inconvenient consequences
of their marriage, as nothing impaired their ability to do so.
That their consent was freely given is best evidenced by their
conscious purpose of acquiring American citizenship through
marriage. Such plainly demonstrates that they willingly and
deliberately contracted the marriage. There was a clear
intention to enter into a real and valid marriage so as to fully
comply with the requirements of an application for citizenship.
There was a full and complete understanding of the legal tie
that would be created between them, since it was that precise
legal tie which was necessary to accomplish their goal.

The avowed purpose of marriage under Article 1 of the Family


Code is for the couple to establish a conjugal and family life.
The possibility that the parties in a marriage might have no
real intention to establish a life together is, however,
insufficient to nullify a marriage freely entered into in
accordance with law. The same Article 1 provides that the
nature, consequences, and incidents of marriage are

Page 30 of 186
governed by law and not subject to stipulation. A marriage 16. SOUTHERN HEMISPHERE ENGAGEMENT
may, thus, only be declared void or voidable under the NETWORK, INC., on behalf of the South-South
grounds provided by law. There is no law that declares a Network (SSN) for Non-State Armed Group
marriage void if it is entered into for purposes other than what Engagement, and ATTY. SOLIMAN M. SANTOS, JR.
the Constitution or law declares, such as the acquisition of vs. ANTI-TERRORISM COUNCIL, THE EXECUTIVE
foreign citizenship. Therefore, so long as all the essential and SECRETARY, THE SECRETARY OF JUSTICE, THE
formal requisites prescribed by law are present, and it is not SECRETARY OF FOREIGN AFFAIRS, THE
void or voidable under the grounds provided by law, it shall be SECRETARY OF NATIONAL DEFENSE, THE
declared valid. SECRETARY OF THE INTERIOR AND LOCAL
GOVERNMENT, THE SECRETARY OF FINANCE,
THE NATIONAL SECURITY ADVISER, THE CHIEF OF
STAFF OF THE ARMED FORCES OF THE
PHILIPPINES, AND THE CHIEF OF THE PHILIPPINE
NATIONAL POLICE (G.R. No. 178552
October 5, 2010)

FACTS: Six petitions for certiorari and prohibition were filed


challenging the constitutionality of RA 9372, otherwise known
as the Human Security Act. Impleaded as respondents in the
various petitions are the Anti-Terrorism Councilcomposed of,
at the time of the filing of the petitions, Executive Secretary
Eduardo Ermita as Chairperson, Justice Secretary Raul
Gonzales as Vice Chairperson, and Foreign Affairs Secretary
Alberto Romulo, Acting Defense Secretary and National
Security Adviser Norberto Gonzales, Interior and Local
Government Secretary Ronaldo Puno, and Finance Secretary
Margarito Teves as members. All the petitions, except that of
the IBP, also impleaded Armed Forces of the Philippines
(AFP) Chief of Staff Gen. Hermogenes Esperon and
Philippine National Police (PNP) Chief Gen. Oscar Calderon.

ISSUE: WON RA 9372 is unconstitutional. NO

HELD: Petitioners assail for being intrinsically vague and


impermissibly broad the definition of the crime of terrorism
under RA 9372 in that terms like "widespread and
extraordinary fear and panic among the populace" and
"coerce the government to give in to an unlawful demand" are
nebulous, leaving law enforcement agencies with no standard
to measure the prohibited acts.

A statute or act suffers from the defect of vagueness when it


lacks comprehensible standards that men of common
intelligence must necessarily guess at its meaning and differ
as to its application. It is repugnant to the Constitution in two
respects: (1) it violates due process for failure to accord
persons, especially the parties targeted by it, fair notice of the
conduct to avoid; and (2) it leaves law enforcers unbridled
discretion in carrying out its provisions and becomes an
arbitrary flexing of the Government muscle. The overbreadth
doctrine, meanwhile, decrees that a governmental purpose to
control or prevent activities constitutionally subject to state
regulations may not be achieved by means which sweep
unnecessarily broadly and thereby invade the area of
protected freedoms.

As distinguished from the vagueness doctrine, the


overbreadth doctrine assumes that individuals will understand
what a statute prohibits and will accordingly refrain from that
behavior, even though some of it is protected.

Distinguished from an as-applied challenge which considers


only extant facts affecting real litigants, a facial invalidation is
an examination of the entire law, pinpointing its flaws and
defects, not only on the basis of its actual operation to the

Page 31 of 186
parties, but also on the assumption or prediction that its very C. EQUAL PROTECTION
existence may cause others not before the court to refrain
from constitutionally protected speech or activities.
1. LOUIS "BAROK" C. BIRAOGO v. THE PHILIPPINE
Justice Mendoza accurately phrased the subtitle in his TRUTH COMMISSION OF 2010. CONSOLIDATED
concurring opinion that the vagueness and over-breadth WITH G.R. No. 193036
doctrines, as grounds for a facial challenge, are not applicable
to penal laws. A litigant cannot thus successfully mount a
facial challenge against a criminal statute on either vagueness
or over-breadth grounds. Since a penal statute may only be FACTS: For consideration before the Court are two
assailed for being vague as applied to petitioners, a limited consolidated cases both of which essentially assail the validity
vagueness analysis of the definition of "terrorism" in RA 9372 and constitutionality of Executive Order No. 1, dated July 30,
is legally impermissible absent an actual or imminent charge 2010, entitled "Creating the Philippine Truth Commission of
against them. 2010."

In insisting on a facial challenge on the invocation that the law In, G.R. No. 192935, Biraogo assails Executive Order No. 1
penalizes speech, petitioners contend that the element of for being violative of the legislative power of Congress under
"unlawful demand" in the definition of terrorism must Section 1, Article VI of the Constitution as it usurps the
necessarily be transmitted through some form of expression constitutional authority of the legislature to create a public
protected by the free speech clause. office and to appropriate funds therefor.

Before a charge for terrorism may be filed under RA 9372, The second case, G.R. No. 193036, is a special civil action for
there must first be a predicate crime actually committed to certiorari and prohibition filed by petitioners Edcel C. Lagman,
trigger the operation of the key qualifying phrases in the other Rodolfo B. Albano Jr., Simeon A. Datumanong, and Orlando
elements of the crime, including the coercion of the B. Fua, Sr. (petitioners-legislators) as incumbent members of
government to accede to an "unlawful demand." Given the the House of Representatives.
presence of the first element, any attempt at singling out or
The Philippine Truth Commission (PTC) is a mere ad hoc body
highlighting the communicative component of the prohibition
formed under the Office of the President with the primary task
cannot recategorize the unprotected conduct into a protected
to investigate reports of graft and corruption committed by
speech.
third-level public officers and employees, their co-principals,
Petitioners notion on the transmission of message is entirely accomplices and accessories during the previous
inaccurate, as it unduly focuses on just one particle of an administration, and thereafter to submit its finding and
element of the crime. Almost every commission of a crime recommendations to the President, Congress and the
entails some mincing of words on the part of the offender like Ombudsman. Though it has been described as an
in declaring to launch overt criminal acts against a victim, in "independent collegial body," it is essentially an entity within
haggling on the amount of ransom or conditions, or in the Office of the President Proper and subject to his control.
negotiating a deceitful transaction. Doubtless, it constitutes a public office, as an ad hoc body is
one.
As earlier reflected, petitioners have established neither an
actual charge nor a credible threat of prosecution under RA To accomplish its task, the PTC shall have all the powers of
9372. Even a limited vagueness analysis of the assailed an investigative body under Section 37, Chapter 9, Book I of
definition of "terrorism" is thus legally impermissible. The the Administrative Code of 1987. It is not, however, a quasi-
Court reminds litigants that judicial power neither judicial body as it cannot adjudicate, arbitrate, resolve, settle,
contemplates speculative counseling on a statutes future or render awards in disputes between contending parties. All
effect on hypothetical scenarios nor allows the courts to be it can do is gather, collect and assess evidence of graft and
used as an extension of a failed legislative lobbying in corruption and make recommendations. It may have
Congress. subpoena powers but it has no power to cite people in
contempt, much less order their arrest. Although it is a fact-
NOTE: A facial invalidation of a statute is allowed only in free finding body, it cannot determine from such facts if probable
speech cases, wherein certain rules of constitutional litigation cause exists as to warrant the filing of an information in our
are rightly excepted. courts of law. Needless to state, it cannot impose criminal, civil
or administrative penalties or sanctions.
In Estrada vs. Sandiganbayan it was held that:

A facial challenge is allowed to be made to a vague


statute and to one which is overbroad because of ISSUES: WON EO No. 1 unconstitutional.
possible “chilling effect” upon protected speech. The
possible harm to society in permitting some
unprotected speech to go unpunished is outweighed
HELD: The Court disagrees with the OSG in questioning the
by the possibility that the protected speech of others
legal standing of the petitioners-legislators to assail Executive
may be deterred and perceived grievances left to
Order No. 1. Evidently, their petition primarily invokes
fester because of possible inhibitory effects of overly
usurpation of the power of the Congress as a body to which
broad statutes.
they belong as members. This certainly justifies their resolve
This rationale does not apply to penal statutes. Criminal to take the cudgels for Congress as an institution and present
statutes have general in terrorem effect resulting from their the complaints on the usurpation of their power and rights as
very existence, and, if facial challenge is allowed for this members of the legislature before the Court.
reason alone, the State may well be prevented from enacting
laws against socially harmful conduct. In the area of criminal
law, the law cannot take chances as in the area of free speech. As held in Philippine Constitution Association v. Enriquez:

Page 32 of 186
To the extent the powers of Congress are impaired, WHEREAS, the transition towards the parliamentary
so is the power of each member thereof, since his form of government will necessitate flexibility in the
office confers a right to participate in the exercise of organization of the national government.
the powers of that institution.
Clearly, as it was only for the purpose of providing
An act of the Executive which injures the institution of manageability and resiliency during the interim, P.D. No.
Congress causes a derivative but nonetheless substantial 1416, as amended by P.D. No. 1772, became functus oficio
injury, which can be questioned by a member of Congress. In upon the convening of the First Congress, as expressly
such a case, any member of Congress can have a resort to provided in Section 6, Article XVIII of the 1987 Constitution.
the courts.
Invoking this authority, the President constituted the PTC to
Indeed, legislators have a legal standing to see to it that the primarily investigate reports of graft and corruption and to
prerogative, powers and privileges vested by the Constitution recommend the appropriate action. As previously stated, no
in their office remain inviolate. Thus, they are allowed to quasi-judicial powers have been vested in the said body as it
question the validity of any official action which, to their mind, cannot adjudicate rights of persons who come before it.
infringes on their prerogatives as legislators.
Contrary to petitioners apprehension, the PTC will not
The question, therefore, before the Court is this: Does the supplant the Ombudsman or the DOJ or erode their respective
creation of the PTC fall within the ambit of the power to powers. If at all, the investigative function of the commission
reorganize as expressed in Section 31 of the Revised will complement those of the two offices. As pointed out by the
Administrative Code? Section 31 contemplates Solicitor General, the recommendation to prosecute is but a
"reorganization" as limited by the following functional and consequence of the overall task of the commission to conduct
structural lines: (1) restructuring the internal organization of a fact-finding investigation. The actual prosecution of
the Office of the President Proper by abolishing, consolidating suspected offenders, much less adjudication on the merits of
or merging units thereof or transferring functions from one unit the charges against them, is certainly not a function given to
to another; (2) transferring any function under the Office of the the commission. The phrase, "when in the course of its
President to any other Department/Agency or vice versa; or investigation," under Section 2(g), highlights this fact and
(3) transferring any agency under the Office of the President gives credence to a contrary interpretation from that of the
to any other Department/Agency or vice versa. petitioners. The function of determining probable cause for the
filing of the appropriate complaints before the courts remains
Clearly, the provision refers to reduction of personnel, to be with the DOJ and the Ombudsman.
consolidation of offices, or abolition thereof by reason of
economy or redundancy of functions. These point to situations At any rate, the Ombudsmans power to investigate under R.A.
where a body or an office is already existent but a modification No. 6770 is not exclusive but is shared with other similarly
or alteration thereof has to be effected. The creation of an authorized government agencies. The same holds true with
office is nowhere mentioned, much less envisioned in said respect to the DOJ. Its authority under Section 3 (2), Chapter
provision. Accordingly, the answer to the question is in the 1, Title III, Book IV in the Revised Administrative Code is by
negative. no means exclusive and, thus, can be shared with a body
likewise tasked to investigate the commission of crimes.
To say that the PTC is borne out of a restructuring of the Office
of the President under Section 31 is a misplaced supposition, Although the purpose of the Truth Commission falls within the
even in the plainest meaning attributable to the term investigative power of the President, the Court finds difficulty
"restructure" an "alteration of an existing structure." Evidently, in upholding the constitutionality of Executive Order No. 1 in
the PTC was not part of the structure of the Office of the view of its apparent transgression of the equal protection
President prior to the enactment of Executive Order No. 1. clause.

In the same vein, the creation of the PTC is not justified by the The equal protection clause is aimed at all official state
Presidents power of control. Control is essentially the power actions, not just those of the legislature. Its inhibitions cover
to alter or modify or nullify or set aside what a subordinate all the departments of the government including the political
officer had done in the performance of his duties and to and executive departments, and extend to all actions of a state
substitute the judgment of the former with that of the latter. denying equal protection of the laws, through whatever
Clearly, the power of control is entirely different from the power agency or whatever guise is taken.
to create public offices. The former is inherent in the
Executive, while the latter finds basis from either a valid It, however, does not require the universal application of the
delegation from Congress, or his inherent duty to faithfully laws to all persons or things without distinction. What it simply
execute the laws. requires is equality among equals as determined according to
a valid classification. Indeed, the equal protection clause
The question is this, is there a valid delegation of power from permits classification. Such classification, however, to be valid
Congress, empowering the President to create a public office? must pass the test of reasonableness. The test has four
According to the OSG, the power to create a truth commission requisites: (1) The classification rests on substantial
pursuant to the above provision finds statutory basis under distinctions; (2) It is germane to the purpose of the law; (3) It
P.D. 1416, as amended by P.D. No. 1772. is not limited to existing conditions only; and (4) It applies
equally to all members of the same class. "Superficial
The Court, however, declines to recognize P.D. No. 1416 as differences do not make for a valid classification."
a justification for the President to create a public office. Said
decree is already stale, anachronistic and inoperable. P.D.
No. 1416 was a delegation to then President Marcos of the
authority to reorganize the administrative structure of the Applying these precepts to this case, Executive Order No. 1
national government including the power to create offices and should be struck down as violative of the equal protection
transfer appropriations pursuant to one of the purposes of the clause. The clear mandate of the envisioned truth commission
decree, embodied in its last "Whereas" clause: is to investigate and find out the truth "concerning the reported

Page 33 of 186
cases of graft and corruption during the previous 2. JESUS C. GARCIA v. THE HONORABLE RAY ALAN
administration only. The intent to single out the previous T. DRILON, Presiding Judge, Regional Trial Court-
administration is plain, patent and manifest. Mention of it has Branch 41, Bacolod City, and ROSALIE JAYPE-
been made in at least three portions of the questioned GARCIA, for herself and in behalf of minor children,
executive order. : JO-ANN, JOSEPH EDUARD, JESSE ANTHONE, all
surnamed GARCIA
In this regard, it must be borne in mind that the Arroyo
administration is but just a member of a class, that is, a class
of past administrations. It is not a class of its own. Not to
include past administrations similarly situated constitutes This is an appeal assailing the constitutionality of R.A. 9262
arbitrariness which the equal protection clause cannot (An Act Defining Violence Against Women and Their Children,
sanction. Such discriminating differentiation clearly Providing for Protective Measures for Victims, Prescribing
reverberates to label the commission as a vehicle for Penalties Therefor, and for Other Purposes) as being violative
vindictiveness and selective retribution. of the equal protection and due process clauses of the
Constitution
The Philippine Supreme Court, according to Article VIII,
Section 1 of the 1987 Constitution, is vested with Judicial
Power that "includes the duty of the courts of justice to settle
FACTS: Rosalie Jaype-Garcia filed for herself and in behalf of
actual controversies involving rights which are legally
her minor children, a verified petition before the Regional Trial
demandable and enforceable, and to determine whether or
Court of Bacolod City for the issuance of a Temporary
not there has been a grave of abuse of discretion amounting
Protection Order (TPO) against her husband, Jesus C. Garcia
to lack or excess of jurisdiction on the part of any branch or
pursuant to R.A. 9262. She claimed to be a victim of physical
instrumentality of the government."
abuse; emotional, psychological, and economic violence as a
Furthermore, in Section 4(2) thereof, it is vested with the result of marital infidelity on the part of petitioner. Private
power of judicial review which is the power to declare a treaty, respondent described herself as a dutiful and faithful wife. On
international or executive agreement, law, presidential decree, the other hand, petitioner, who is of Filipino-Chinese descent,
proclamation, order, instruction, ordinance, or regulation is dominant, controlling, and demands absolute obedience
unconstitutional. This power also includes the duty to rule on from his wife and children. Things turned for the worse when
the constitutionality of the application, or operation of petitioner took up an affair with a bank manager of Robinson's
presidential decrees, proclamations, orders, instructions, Bank, Bacolod City. Petitioner's infidelity spawned a series of
ordinances, and other regulations. These provisions, fights that left private respondent physically and emotionally
however, have been fertile grounds of conflict between the wounded.
Supreme Court, on one hand, and the two co-equal bodies of
Finding reasonable ground to believe that an imminent danger
government, on the other. Many times the Court has been
of violence against respondent and her children exists or is
accused of asserting superiority over the other departments.
about to recur, the RTC issued a TPO on March 24, 2006 for
Thus, the Court, in exercising its power of judicial review, is thirty (30) days. The TPO has been continuously extended
not imposing its own will upon a co-equal body but rather and renewed by the RTC after appearing that the hearing
simply making sure that any act of government isdone in could not yet be finally terminated. During the pendency of
consonance with the authorities and rights allocated to it by Civil Case, petitioner filed before the Court of Appeals (CA) a
the Constitution. And, if after said review, the Court finds no petition for prohibition with prayer for injunction and temporary
constitutional violations of any sort, then, it has no more restraining order, challenging the constitutionality of R.A. 9262
authority of proscribing the actions under review. Otherwise, for being violative of the due process and the equal protection
the Court will not be deterred to pronounce said act as void clauses, and the validity of the modified TPO issued in the civil
and unconstitutional. case for being an unwanted product of an invalid law. On
January 24, 2007, the CA dismissed the petition for failure of
petitioner to raise the constitutional issue in his pleadings
before the trial court in the civil case, thus, appealing to the
Supreme Court. Petitioner contends that on the basis of
unsubstantiated allegations, and practically no opportunity to
respond, he is stripped of family, property, guns, money,
children, job, future employment and reputation, all in a matter
of seconds, without an inkling of what happened

ISSUE: WON R.A. 9262 is discriminatory, unjust and violative


of the equal protection clause. NO

HELD: The Supreme Court find that R.A. 9262 is based on a


valid classification, as such, did not violate the equal
protection clause by favoring women over men as victims of
violence and abuse to whom the State extends its protection.

R.A. 9262 rests on substantial distinctions - The unequal


power relationship between women and men; the fact that
women are more likely than men to be victims of violence; and
the widespread gender bias and prejudice against women all

Page 34 of 186
make for real differences justifying the classification under the 3. REPUBLIC OF THE PHILIPPINES vs MARELYN
law. TANEDO MANALO (G.R. No. 221029)

Women are the usual and most likely victims of violence - At


the time of the presentation of Senate Bill No. 2723, official
statistics on violence against women and children shows that FACTS: Marelyn Tanedo Manalo (Manalo) filed a petition for
female violence comprised more than 90% of all forms of cancellation of entry of marriage in the Civil Registry of San
abuse and violence and more than 90% of these reported Juan, Metro Manila, by virtue of a judgment of divorce
cases were committed by the women's intimate partners such rendered by a Japanese court. petitioner is previously married
as their husbands and live-in partners. On the other hand, no in the Philippines to a Japanese national named YOSHINO
reliable estimates may be obtained on domestic abuse and MINORO; That at present, by virtue of the said divorce decree,
violence against men in the Philippines because incidents petitioner and her divorced Japanese husband are no longer
thereof are relatively low. living together; that this petition is filed principally for the
purpose of causing the cancellation of entry of the marriage
Gender Bias and Prejudices - From the initial report to the between the petitioner and the said Japanese national,
police through prosecution, trial, and sentencing, crimes pursuant to Rule 108 of the Revised Rules of Court.
against women are often treated differently and less seriously
than other crimes. The trial court denied the petition for lack of merit. It opined
that, based on Article 15 of the New Civil Code, the Philippine
R.A. 9262 is not violative of the due process clause of the law "does not afford Filipinos the right to file for a divorce,
Constitution. The grant of a TPO ex parte cannot, therefore, whether they are in the country or living abroad, if they are
be challenged as violative of the right to due process. Just like married to Filipinos or to foreigners, or if they celebrated their
a writ of preliminary attachment which is issued without notice marriage in the Philippines or in another country" and that
and hearing because the time in which the hearing will take unless Filipinos "are naturalized as citizens of another country.
could be enough to enable the defendant to abscond or
dispose of his property, in the same way, the victim of VAWC CA overturned the RTC decision. It held that Article 26 of the
may already have suffered harrowing experiences in the Family Code of the Philippines (Family Code) is applicable
hands of her tormentor, and possibly even death, if notice and even if it was Manalo who filed for divorce against her
hearing were required before such acts could be prevented. It Japanese husband because the decree they obtained makes
is a constitutional commonplace that the ordinary the latter no longer married to the former, capacitating him to
requirements of procedural due process must yield to the remarry.
necessities of protecting vital public interests, among which is
protection of women and children from violence and threats to
their personal safety and security. ISSUE: WON Article 26(2) of the Family Code violates the
equal protection clause. YES
The Court also pointed out that when the TPO is issued ex
parte, the court shall likewise order that notice be immediately
given to the respondent directing him to file an opposition
within five (5) days from service. The opposition to the petition HELD: Conveniently invoking the nationality principle is
which the respondent himself shall verify, must be erroneous. Such principle, found under Article 15 of the City
accompanied by the affidavits of witnesses and shall show Code, is not an absolute and unbending rule. In fact, the mer
cause why a temporary or permanent protection order should e existence of Paragraph 2 of Article 26 is a testament that the
not be issued. It is clear from the foregoing rules that the State may provide for an exception thereto. Moreover, blind
respondent of a petition for protection order should be adherence to the nationality principle must be disallowed if it
apprised of the charges imputed to him and afforded an would cause unjust discrimination and oppression to certain
opportunity to present his side. classes of individuals whose rights are equally protected by
law. The courts have the duty to enforce the laws of divorce
as written by the Legislature only if they are constitutional.
WHEREFORE, the instant petition for review on certiorari is While the Congress is allowed a wide leeway in providing for
hereby DENIED for lack of merit. a valid classification and that its decision is accorded
recognition and respect by the court of justice, such
classification may be subjected to judicial review.44 The
deference stops where the classification violates a
fundamental right, or prejudices persons accorded special
protection by the Constitution.45 When these violations arise,
this Court must discharge its primary role as the vanguard of
constitutional guaranties, and require a stricter and more
exacting adherence to constitutional limitations.46 If a
legislative classification impermissibly interferes with the
exercise of a fundamental right or operates to the peculiar
disadvantage of a suspect class strict judicial scrutiny is
required since it is presumed unconstitutional, and the burden
is upon the government to prove that the classification is
necessary to achieve a compelling state interest and that it is
the least restrictive means to protect such interest.

"Fundamental rights" whose infringement leads to strict


scrutiny under the equal protection clause are those basic
liberties explicitly or implicitly guaranteed in the Constitution.

Page 35 of 186
It includes the right to free speech, political expression, press, courts will automatically grant the same. Besides, such
assembly, and forth, the right to travel, and the right to vote. proceeding is duplicitous, costly, and protracted. All to the
On the other hand, what constitutes compelling state interest prejudice of our kababayan.
is measured by the scale rights and powers arrayed in the
Constitution and calibrated by history. It is akin to the
paramount interest of the state for which some individual
liberties must give way, such as the promotion of public
interest, public safety or the general welfare. It essentially
involves a public right or interest that, because of its primacy,
overrides individual rights, and allows the former to take
precedence over the latter.

Although the Family Code was not enacted by the Congress,


the same principle applies with respect to the acts of the
President which have the force and effect of law unless
declared otherwise by the court. In this case, We find that
Paragraph 2 of Article 26 violates one of the essential
requisites of the equal protection clause. Particularly, the
limitation of the provision only to a foreign divorce decree
initiated by the alien spouse is unreasonable as it is based on
superficial, arbitrary, and whimsical classification.

A Filipino who is married to another Filipino is not similarly


situated with a Filipino who is married to a foreign citizen.
There are real, material and substantial differences between
them. Ergo, they should not be treated alike, both as to rights
conferred and liabilities imposed. Without a doubt, there are
political, economic cultural, and religious dissimilarities as well
as varying legal systems and procedures, all too unfamiliar,
that a Filipino national who is married to an alien spouse has
to contend with. More importantly, while a divorce decree
obtained abroad by a Filipino against another Filipino is null
and void, a divorce decree obtained by an alien against his
her Filipino spouse is recognized if made in accordance with
the national law of the foreigner.

On the contrary, there is no real and substantial difference


between a Filipino who initiated a foreign divorce proceedings
a Filipino who obtained a divorce decree upon the instance of
his or her alien spouse. In the eyes of the Philippine and
foreign laws, both are considered as Filipinos who have the
same rights and obligations in an alien land. The
circumstances surrounding them are alike. Were it not for
Paragraph 2 of Article 26, both are still married to their
foreigner spouses who are no longer their wives/husbands.
Hence, to make a distinction between them based merely on
the superficial difference of whether they initiated the divorce
proceedings or not is utterly unfair. Indeed, the treatment
gives undue favor to one and unjustly discriminate against the
other.

Further, the differentiation in Paragraph 2 Article 26 is


arbitrary. There is inequality in treatment because a foreign
divorce decree that was initiated and obtained by a Filipino
citizen against his or her alien spouse would not be recognized
even if based on grounds similar to Articles 35, 36, 37 and 38
of the Family Code. In filing for divorce based on these
grounds, the Filipino spouse cannot be accused of invoking
foreign law at whim, tantamount to insisting that he or she
should be governed with whatever law he or she chooses. The
dissent's comment that Manalo should be "reminded that all is
not lost, for she may still pray for the severance of her martial
ties before the RTC in accordance with the mechanism now
existing under the Family Code" is anything but comforting.
For the guidance of the bench and the bar, it would have been
better if the dissent discussed in detail what these
"mechanism" are and how they specifically apply in Manalo's
case as well as those who are similarly situated. If the dissent
refers to a petition for declaration of nullity or annulment of
marriage, the reality is that there is no assurance that our

Page 36 of 186
4. ANTONIO M. SERRANO, Petitioner, vs. Gallant The subject clause being unconstitutional, petitioner is entitled
MARITIME SERVICES, INC. and MARLOW to his salaries for the entire unexpired period of nine months
NAVIGATION CO., INC., Respondents. (G.R. No. and 23 days of his employment contract, pursuant to law and
167614; March 24, 2009 ) jurisprudence prior to the enactment of R.A. No. 8042.

WHEREFORE, the Court GRANTS the Petition. The subject


clause "or for three months for every year of the unexpired
FACTS: Petitioner Antonio Serrano was hired by respondents term, whichever is less" in the 5th paragraph of Section 10 of
Gallant Maritime Services, Inc. and Marlow Navigation Co., Republic Act No. 8042 is DECLARED UNCONSTITUTIONAL;
Inc., under a POEA-approved contract of employment for 12 and the December 8, 2004 Decision and April 1, 2005
months, as Chief Officer, with the basic monthly salary of Resolution of the Court of Appeals are MODIFIED to the effect
US$1,400, plus $700/month overtime pay, and 7 days paid that petitioner is AWARDED his salaries for the entire
vacation leave per month. On the date of his departure, unexpired portion of his employment contract consisting of
Serrano was constrained to accept a downgraded nine months and 23 days computed at the rate of
employment contract upon the assurance and representation US$1,400.00 per month.
of respondents that he would be Chief Officer by the end of
April 1998.

Respondents did not deliver on their promise to make Serrano


Chief Officer. Hence, Serrano refused to stay on as second
Officer and was repatriated to the Philippines, serving only two
months and 7 days, leaving an unexpired portion of nine
months and twenty-three days. Upon complaint filed by
Serrano before the Labor Arbiter (LA), the dismissal was
declared illegal. On appeal, the NLRC modified the LA
decision based on the provision of RA 8042. Serrano filed a
Motion for Partial Reconsideration, but this time he questioned
the constitutionality of the last clause in the 5th paragraph of
Section 10 of RA 8042.

ISSUE: WON the last clause in the 5th paragraph of Section


10 of RA 8042 is constitutional. NO

HELD: The subject clause "or for three months for every year
of the unexpired term, whichever is less" in the 5th paragraph
of Section 10 of Republic Act No. 8042 is DECLARED
UNCONSTITUTIONAL

The enactment of the subject clause in R.A. No. 8042


introduced a differentiated rule of computation of the money
claims of illegally dismissed OFWs based on their
employment periods, in the process singling out one category
whose contracts have an unexpired portion of one year or
more and subjecting them to the peculiar disadvantage of
having their monetary awards limited to their salaries for 3
months or for the unexpired portion thereof, whichever is less,
but all the while sparing the other category from such
prejudice, simply because the latter's unexpired contracts fall
short of one year.

The subject clause creates a sub-layer of discrimination


among OFWs whose contract periods are for more than one
year: those who are illegally dismissed with less than one year
left in their contracts shall be entitled to their salaries for the
entire unexpired portion thereof, while those who are illegally
dismissed with one year or more remaining in their contracts
shall be covered by the subject clause, and their monetary
benefits limited to their salaries for three months only.

Thus, the subject clause in the 5th paragraph of Section 10 of


R.A. No. 8042 is violative of the right of petitioner and other
OFWs to equal protection.

The subject clause does not state or imply any definitive


governmental purpose; and it is for that precise reason that
the clause violates not just petitioner's right to equal
protection, but also her right to substantive due process under
Section 1, Article III of the Constitution.

Page 37 of 186
5. SAMEER OVERSEAS PLACEMENT AGENCY, INC., 10 of Republic Act No. 8042 is declared unconstitutional and,
vs. JOY C. CABILES (G.R. No. 170139 August 5, therefore, null and void.
2014)

FACTS: Petitioner, Sameer Overseas Placement Agency,


Inc., is a recruitment and placement agency.

Sameer claims that on July 14, 1997, a certain Mr. Huwang


from Wacoal informed Joy, without prior notice, that she was
terminated and that “she should immediately report to their
office to get her salary and passport.” She was asked to
“prepare for immediate repatriation.” Joy claims that she was
told that from June 26 to July 14, 1997, she only earned a total
of NT$9,000. According to her, Wacoal deducted NT$3,000 to
cover her plane ticket to Manila.

Respondent Joy Cabiles was hired thus signed a one-year


employment contract for a monthly salary of NT$15,360.00.
Joy was deployed to work for Taiwan Wacoal, Co. Ltd.
(Wacoal) on June 26, 1997. She alleged that in her
employment contract, she agreed to work as quality control for
one year. In Taiwan, she was asked to work as a cutter.

On October 15, 1997, Joy filed a complaint for illegal dismissal


with the NLRC against petitioner and Wacoal. LA dismissed
the complaint. NLRC reversed LA’s decision. CA affirmed the
ruling of the National Labor Relations Commission finding
respondent illegally dismissed and awarding her three
months’ worth of salary, the reimbursement of the cost of her
repatriation, and attorney’s fees

ISSUE: WON Republic Act. No. 10022 which was declared


unconstitutional is a law. NO

HELD: A statute or provision which was declared


unconstitutional is not a law. It “confers no rights; it imposes
no duties; it affords no protection; it creates no office; it is
inoperative as if it has not been passed at all.”

In the hierarchy of laws, the Constitution is supreme. No


branch or office of the government may exercise its powers in
any manner inconsistent with the Constitution, regardless of
the existence of any law that supports such exercise. The
Constitution cannot be trumped by any other law. All laws
must be read in light of the Constitution. Any law that is
inconsistent with it is a nullity.

Thus, when a law or a provision of law is null because it is


inconsistent with the Constitution, the nullity cannot be cured
by reincorporation or reenactment of the same or a similar law
or provision. A law or provision of law that was already
declared unconstitutional remains as such unless
circumstances have so changed as to warrant a reverse
conclusion.

The Court observed that the reinstated clause, this time as


provided in Republic Act. No. 10022, violates the
constitutional rights to equal protection and due process.96
Petitioner as well as the Solicitor General have failed to show
any compelling change in the circumstances that would
warrant us to revisit the precedent.

The Court declared, once again, the clause, “or for three (3)
months for every year of the unexpired term, whichever is
less” in Section 7 of Republic Act No. 10022 amending Section

Page 38 of 186
6. United States vs Windsor 570 U.S. 744, 26 June 2013 test, Judge Jacobs wrote, it is unconstitutional under the equal
protection guarantees of the Fifth Amendment.

On September 11, 2012, following Windsor's petition for


Facts: The Defense of Marriage Act (DOMA), enacted in certiorari before judgment and before the Second Circuit's
1996, states that, for the purposes of federal law, the words ruling, the Department of Justice filed its own petition for
"marriage" and "spouse" refer to legal unions between one certiorari before judgment with the Supreme Court. After the
man and one woman. Since that time, some states have appellate ruling on October 18, the parties filed supplemental
authorized same-sex marriage. In other cases regarding the briefs. On December 7, the Supreme Court granted certiorari
DOMA, federal courts have ruled it unconstitutional under the in the case, now United States v. Windsor, accepting the
Fifth Amendment, but the courts have disagreed on the DOJ's petition.
rationale.
Issue: Whether or not the Defense of Marriage Act, which
In 2007, Edith Windsor and Thea Clara Spyer, both residents defines the term "marriage" under federal law as a "legal union
of New York, married in Toronto, Ontario, under the provisions between one man and one woman" deprive same-sex couples
set forth in the Canadian Civil Marriage Act, after 40 years of who are legally married under state laws of their Fifth
romantic partnership. Canada's first openly gay judge, Justice Amendment rights to equal protection under federal law?
Harvey Brownstone, officiated. The State of New York
recognizes the marriage.
Held Yes. In a 5–4 decision issued on June 26, 2013, the
After Spyer's death in 2009, Edith Windsor as the widow and Supreme Court found Section 3 of DOMA to be
sole executor of the estate of her late spouse was required to unconstitutional, "as a deprivation of the liberty of the person
pay $363,053 in federal estate taxes on her inheritance of her protected by the Fifth Amendment". The Court held that states
wife's estate. Had federal law recognized the validity of their have the authority to define marital relationships and that
marriage, Windsor would have qualified for an unlimited DOMA goes against legislative and historical precedent by
spousal deduction and paid no federal estate taxes. undermining that authority. The result is that DOMA denies
same-sex couples the rights that come from federal
In May 2008, New York Governor David Paterson had ordered recognition of marriage, which are available to other couples
state agencies to recognize same-sex marriages performed in with legal marriages under state law. The Court held that the
other jurisdictions. Some lower-level state courts had made purpose and effect of DOMA is to impose a "disadvantage, a
similar rulings, but whether the state's highest court would give separate status, and so a stigma" on same-sex couples in
such a ruling the force of law, as Windsor's claim for a refund violation of the Fifth Amendment's guarantee of equal
required, remained uncertain and was disputed throughout protection.
her lawsuit.
The Court held that the Constitution prevented the federal
District Court government from treating state-sanctioned heterosexual
On November 9, 2010 Windsor filed suit in district court marriages differently from state-sanctioned same-sex
seeking a declaration that the Defense of Marriage Act was marriages, and that such differentiation "demean[ed] the
unconstitutional. At the time the suit was filed, the couple, whose moral and sexual choices the Constitution
government's position was that DOMA must be defended. On protects".
February 23, 2011, the President and the Attorney General
announced that they would not defend DOMA. On April 18, Justice Anthony Kennedy authored the majority opinion which
2011, the Bipartisan Legal Advisory Group (BLAG) of the held that to strike down a central part of DOMA cited the
House of Representatives filed a petition to intervene in principles of state autonomy, equal protection and liberty, but
defense of DOMA and motioned to dismiss the case. The the constitutional basis for striking down the law was not
district court denied the motion, and later held that DOMA was entirely clear, as it had elements of federalism, equal
unconstitutional. protection and due process.

New York Attorney General Eric Schneiderman filed a brief The answer may be found in Windsor's brief, in which she
supporting Windsor's claim on July 26, 2011, arguing that argues that DOMA operates to say "that married gay couples
DOMA Section 3 could not survive the scrutiny used for aren't genuinely married at all but are instead 'similarly
classifications based on sex and constitutes "an intrusion on situated' to unmarried people"
the power of the state to define marriage". On June 6, 2012,
Judge Barbara S. Jones ruled that a rational basis review of The Court wrote:
Section 3 of DOMA showed it to be unconstitutional, as it
violated plaintiff's rights under the equal protection guarantees DOMA seeks to injure the very class New York seeks to
of the Fifth Amendment, and ordered that Windsor receive the protect. By doing so it violates basic due process and equal
tax refund due to her. Where BLAG had argued that the Spyer- protection principles applicable to the Federal Government.
Windsor marriage was not recognized by New York law at the
time of Spyer's death – a prerequisite for Windsor's claim When New York adopted a law to permit same-sex marriage,
against the IRS – Jones cited the "informal opinion letters" of it sought to eliminate inequality; but DOMA frustrates that
the state's governor, attorney general, and comptroller to the objective through a system-wide enactment with no identified
contrary along with several opinions in New York appellate connection to any particular area of federal law. DOMA writes
courts. inequality into the entire United States Code.

Court of Appeals DOMA instructs all federal officials, and indeed all persons
On October 18, the Second Circuit Court of Appeals upheld with whom same-sex couples interact, including their own
the lower court's ruling that Section 3 of DOMA is children, that their marriage is less worthy than the marriages
unconstitutional. The majority opinion stated, "It is easy to of others. The federal statute is invalid, for no legitimate
conclude that homosexuals have suffered a history of purpose overcomes the purpose and effect to disparage and
discrimination." Thus they were part of a quasi-suspect class to injure those whom the State, by its marriage laws, sought
that deserves any law restricting its rights to be subjected to to protect in personhood and dignity. By seeking to displace
intermediate scrutiny. Because DOMA could not pass that this protection and treating those persons as living in

Page 39 of 186
marriages less respected than others, the federal statute is in D. SEARCHES AND SEIZURES
violation of the Fifth Amendment.

1. UNITED STATES vs Jeffrey GRUBBS 547 U.S. 90


(2006)

FACTS: On April 17, 2002, United States Postal Inspector


Gary Welsh (“Welsh”) presented an “Application and Affidavit
for Anticipatory Search Warrant” to a federal magistrate judge.
The application sought authority to conduct a search of
Grubbs' residence on the basis of an order Grubbs allegedly
placed for a videotape entitled “Lolita Mother and Daughter.”
Grubbs allegedly ordered the videotape from a website that
advertised for sale numerous videos depicting illegal child
pornography. Welsh averred that Grubbs sent him a letter
which contained $45 in cash and a note stating:  “I hope this
makes it to you please send film asap thanks Jeff Grubbs.” On
the basis of this evidence, the magistrate judge issued an
anticipatory search warrant.

The face of the warrant stated:


Affidavit(s) having been made before me by _ who
has reason to believe that on the premises known as
residence of Jeffrey Grubbs, [Address] as more
particularly described in Attachment A to the
attached Affidavit, in the Eastern District of California
there is now concealed a certain person or property,
namely the records and materials described in
Attachment B to the attached Affidavit. I am
satisfied that the affidavit(s) and any recorded
testimony establish probable cause to believe that
the person or property so described is now
concealed on the person or premises above-
described and establish grounds for the issuance of
this warrant.

As revealed by the “now concealed” language, the in artfully


drafted warrant approved by the magistrate was written on a
form “forthwith” search warrant. The only indication that the
warrant was an anticipatory search warrant was the word
“ANTICIPATORY,” handwritten at the top of the page above
the words “SEARCH WARRANT.” The warrant itself did not
state what triggering conditions needed to occur in order to
make the warrant valid; nor did it state the criminal activity of
which Grubbs was suspected.

The warrant relied on a 25-page affidavit to satisfy the


specificity and particularity requirements of the Fourth
Amendment. According to the affidavit, the warrant would
become operative once the videotape Grubbs ordered was
“received by a person(s)” and “taken into the residence.”
Pages five and nineteen of the affidavit set forth these
“triggering events,” or conditions precedent, upon which a
search would become authorized. The affidavit also had two
attachments:  Attachment A described the premises to be
searched; Attachment B listed the items to be seized,
including the videotape and packing material, Grubbs' Web TV
components, and various other items.

The search took place two days later. At approximately 7:20


A.M., an undercover postal inspector delivered the videotape
to Grubbs' residence. Grubbs' wife accepted the delivery of
the package, signed for it, and took it into the house. A few
minutes later, Postal Inspector Thomas Brucklacher saw
Grubbs leaving. At approximately 7:24 A.M., Brucklacher and
Inspector Esteban approached Grubbs and, after identifying
themselves, told him to remain where he was standing.
Grubbs asked Brucklacher why he and the other inspectors
were there. Brucklacher did not answer, but instead referred
him to Inspector Welsh, who was then approaching the
residence. Meanwhile, Inspector Esteban performed a
patdown search of Grubbs.

Page 40 of 186
Shortly after Grubbs was detained outside of the house, constitutional defect. The district judge admitted that “it is
Inspector Welsh arrived at the premises with a number of logical that officers would be required to actually present the
other law enforcement personnel. In all, there were ultimately affidavit setting forth the triggering event to the people whose
ten officers and inspectors at the scene. Welsh allegedly 9971 property they are searching in order to provide those people
announced “Police/Search Warrant” at the front door. Grubbs' with information regarding the parameters of the search.”
wife, Ms. Bradstreet, disputed hearing that announcement, but However, after concluding that no case from our circuit had
did testify that she heard a knock and answered the door. ruled on the precise question, the court declined to apply that
Welsh briefly entered the house to help several other officers logic “in the absence of specific guidance from the Ninth
perform a “protective sweep.” During that “protective sweep,” Circuit.”  Thus, it upheld the search, even though none of the
the officers searched the house for other people and stopped persons whose residence was searched were shown the
to prepare sketches of the interior. They permitted Grubbs' affidavit that identified the triggering event.
children to leave for school after searching their backpacks.
After assisting the officers inside, Welsh went back outside to After filing a motion for reconsideration, which the district court
speak with Grubbs, who was on the sidewalk with other denied, Grubbs entered a conditional guilty plea to the sole
officers. Welsh identified himself, and stated either “You know charge of the indictment-receiving a visual depiction of a minor
why we're here” or “Do you know why we're here?”  Grubbs engaged in sexually explicit conduct. He reserved his right to
replied “yeah,” and said that what the officers were looking for appeal the denial of his motion to suppress. The district court
was in the garage. Welsh told Grubbs that he was not under sentenced him to thirty-three months imprisonment, a three-
arrest, but that they were there to serve a search warrant, and year term of supervised release, a fine of $3,700, and a $100
that they should go inside the house to talk. special assessment. Grubbs timely appealed.

Grubbs and Welsh, accompanied by Officer Esteban, entered Jeffrey Grubbs appeals following his conditional guilty plea on
the house together and sat down at the dining room table. It a charge of receiving a visual depiction of a minor engaged in
was not until 7:53 A.M., approximately 30 minutes after the sexually explicit conduct. He contends that the district court
search began, that Welsh presented Grubbs with the search should have granted his motion to suppress evidence,
warrant. The copy of the search warrant provided to Grubbs including his statements, because the anticipatory search
included the two attachments, which described the place to be warrant that authorized the search of his premises was invalid
searched and the items to be seized, but did not include the under the Fourth Amendment. To resolve Grubbs' claim, we
affidavit that contained the “triggering events” or conditions must determine whether a facially defective anticipatory
precedent that would serve to make the warrant operative. search warrant may be cured by information contained within
Welsh contended that he had a copy of the affidavit with him an affidavit when that affidavit is not presented to the person
at all times during the search, and that his team had all read or persons whose property is to be searched. We answer that
the affidavit on the previous evening. However, the question in the negative, and hold that the search of Grubbs'
government concedes that the affidavit was not presented to premises violated the Fourth Amendment.
Mr. Grubbs or Ms. Bradstreet, and that no copy of the affidavit
was left at the residence following the search.
ISSUE: Whether or not “anticipatory" search warrants under
After the warrant was presented, Welsh reminded Grubbs that the Fourth Amendment is constitutional
he was not under arrest, advised him of his Miranda rights,
and asked if he understood those rights. Grubbs said that he
did and agreed to speak to Welsh. The interview lasted HELD: The Fourth Amendment states that “no Warrants shall
approximately 55 minutes. In it, Grubbs admitted that he had issue, but upon probable cause, supported by Oath or
ordered the pornography. He further admitted that he affirmation, and particularly describing the place to be
possessed child pornography in various digital forms in his searched, and the persons or things to be seized.” The
home. At the conclusion of the interview, Grubbs was arrested requirement that warrants “particularly describe the place to
and handcuffed. The officers seized the videotape in question be searched, and the persons or things to be seized” is most
along with several other items, including Grubbs' computer often described as the “particularity requirement.” As the
and several computer diskettes.6 Supreme Court has recently explained, that requirement
“applies with equal force to searches whose only defect is a
Within a few days, a grand jury returned an indictment lack of particularity in the warrant.”
charging Grubbs with receiving a visual depiction of a minor
engaged in sexually explicit conduct. Grubbs filed a motion to The Groh Court considered a warrant that “failed to identify
suppress evidence, in which he challenged the admissibility of any of the items” to be seized. Despite the fact that the officers
all of the seized evidence and his statements to Welsh. conducting the search had presented to the reviewing
Grubbs made three principal claims:  (1) that the agents' magistrate a detailed affidavit setting forth sufficient probable
failure to present the affidavit to Grubbs or his wife rendered cause for the search, the warrant itself did not explicitly
the warrant inoperative; (2) that the agents violated by failing “incorporate by reference the itemized list [of things to be
to present the search warrant at the outset of the search; and seized] contained in the application.” The officers in Groh left
(3) that his statement that the video was in the garage should the residents of the searched home a copy of the search
be excluded as the product of an impermissible custodial warrant, “but not a copy of the application, which had been
interrogation. The first and third claims alleged constitutional sealed.”
violations.
The Court found that the officers' conduct directly conflicted
Following an evidentiary hearing, the district court denied the with the purpose of the Fourth Amendment's particularity
motion to suppress in a written order. With respect to Grubbs' requirement: The Fourth Amendment by its terms requires
first claim, the Fourth Amendment claim, the district court held particularity in the warrant, not in the supporting documents.
that the anticipatory warrant could constitutionally be executed And for good reason:  “The presence of a search warrant
even though it failed to designate the triggering event for the serves a high function,” and that high function is not
implementation of the anticipatory search. It did so on the necessarily vindicated when some other document,
basis that the warrant incorporated the affidavit by reference, somewhere, says something about the objects of the search,
and that the affidavit was in the immediate presence of the but the contents of that document are neither known to the
officers while they searched Grubbs' residence. The court did person whose home is being searched nor available for her
not consider the officers' failure to present the affidavit to the inspection․
residents of the home to be searched as constituting a

Page 41 of 186
We have long held, moreover, that the purpose of the in Groh, the district court was correct when it opined that “it is
particularity requirement is not limited to the prevention of logical that officers would be required to actually present the
general searches. A particular warrant also “assures the affidavit setting forth the triggering event to the people whose
individual whose property is searched or seized of the lawful property they are searching in order to provide those people
authority of the executing officer, his need to search, and the with information regarding the parameters of the search.”
limits of his power to search.” Likewise, the district court was right to conclude that the
“underlying reasoning” of Hotal supports the rule that “the
Our cases have long been in accord with the Supreme Court's affidavit setting forth the triggering event for an anticipatory
reasoning in Groh. We have held that a search warrant is warrant must be presented to the people whose property is
invalid when it does not contain a specific description of the being searched.” The district court, however, was unwilling to
types of items to be seized. And, while we have permitted impose such a requirement in this case without further explicit
facially defective warrants to be “cured” by an affidavit that (a) guidance from us.
is incorporated within the four corners of the warrant and (b)
“accompanies” the warrant, we have unequivocally held that We believe that our prior cases unambiguously require
the defect is not cured if the officers fail to present the affidavit- officers to present any curative document-be it an affidavit,
that is, an affidavit that is not shown to the persons being attachment, or other instrument that supplies the particularity
subjected to the search does not have a curative effect on a and specificity demanded by the Fourth Amendment-to the
facially defective warrant. persons whose property is to be subjected to the search. To
the extent that there is any question that our cases have
As we explained in McGrew, we require affidavits to adopted that rule, we do so explicitly now. Anticipatory search
accompany warrants not only in order to limit officers' warrants are invalid absent “clear, explicit, and narrow”
discretion in conducting the search, but also in order to “inform triggering conditions. Those triggering conditions may be
the person subject to the search what items the officers listed either in the warrant itself or in attached documents, but
executing the warrant can seize.” If the officers conducting the whatever document contains them must be presented to the
search were not required to present the affidavit to the person whose property is being searched. Absent such
residents of the house being searched, law enforcement presentation, individuals would “stand [no] real chance of
personnel would be free to search as they like, and policing the officers' conduct,” because they would have no
homeowners and others would have no effective way to opportunity to check whether the triggering events by which
ensure that the search of their premises conformed to the the impartial magistrate has limited the officers' discretion
lawful constraints approved by an impartial magistrate. have actually occurred. In short, unless the officers “present”
Our cases have similarly held, without exception, that the the document containing the triggering events necessary to
particularity requirement of the Fourth Amendment applies render an anticipatory search warrant operative, the search
with full force to the conditions precedent to an anticipatory warrant is constitutionally invalid. In the absence of a proper
search warrant. An anticipatory search warrant is not valid presentation, “the search is rendered illegal because the
until the occurrence of one or more “triggering events”-in other warrant neither limits [the officers'] discretion nor gives the
words, the predicted future events that the magistrate homeowner the required information.”
determines will create sufficient probable cause to justify the
search. And, “when a warrant's execution is dependent on the In this case, there is no dispute that the officers failed to
occurrence of one or more conditions, the warrant itself must present the affidavit-the only document in which the triggering
state the conditions precedent to its execution and these conditions were listed-to Grubbs or Bradstreet. At no point
conditions must be clear, explicit, and narrow.” before, during, or after the search did the officers show or read
the affidavit to either of them. The copy of the warrant left with
The rationale for this rule is simple:  “a warrant conditioned on Ms. Bradstreet at the conclusion of the search did not include
a future event presents a potential for abuse above and the affidavit, nor did it otherwise include a list of the triggering
beyond that which exists in more traditional settings: conditions. The warrant was therefore inoperative, and the
 inevitably, the executing agents are called upon to determine search was illegal.
when and where the triggering event specified in the warrant
has actually occurred.” Absent a constitutionally valid warrant, the officers lacked the
legal authority to enter the defendant's home. The fact that the
We have, however, permitted the triggering conditions of an search ultimately may have been conducted in a manner
anticipatory search warrant to appear either on the face of the consistent with the application for the warrant is irrelevant. “If
warrant itself, or in the “attachments to the warrant] that those a warrant fails for lack of particularity or specificity, it is simply
executing the search maintain in their immediate possession unconstitutional-without regard to what actually occurred.” Nor
in order to guide their actions and to provide information to the is it significant that the officers may have possessed curative
person whose property is being searched.” Still, while an documents during the search, unless those documents were
affidavit may qualify as a valid curing “attachment” to an presented to the owners or occupants of the property:  “that
otherwise defective warrant, it counts as such only when the the applicant and the magistrate may understand the
affidavit actually “accompanies” the warrant. parameters of the search has no bearing on whether the
person to be searched is properly advised of [the officers']
The first requirement, that the application but not the warrant authority.” We therefore conclude that the officers in this case
itself identify the triggering event, does little if anything to limit did not execute a constitutionally valid warrant, and that they,
the discretion of the agents executing the warrant or to inform in effect, conducted a warrantless search. As a result, all
the subject of the search whether it was authorized, if the evidence obtained during that search, and following Welsh's
affidavit does not accompany the warrant. Indeed, that the announcement of “Police/Search Warrant,” must be
applicant and the magistrate may understand the parameters suppressed. “All evidence” includes all of the evidence seized
of the search has no bearing on whether the officers executing after the initial entry, as well as all of Grubbs' statements, all
the warrant do, or whether the person to be searched is of which were taken either during the illegal entry or as a direct
properly advised of their authority. causal result of it.
Thus, the failure to present the affidavit designating the
The question in this case is whether a curative affidavit that triggering events or conditions precedent to the operability of
contains the conditions precedent to an anticipatory search the search warrant rendered the warrant constitutionally
actually “accompanies” the warrant when the affidavit is not invalid and the search illegal. Because Grubbs entered a
shown to the person or persons being subjected to the search. conditional guilty plea, we are required to remand and allow
Given our prior holdings, and the Court's most recent decision him to withdraw his plea if he elects to do so. We therefore

Page 42 of 186
reverse the denial of Grubbs' suppression motion and remand 2. RETIRED SPO4 BIENVENIDO LAUD v. PEOPLE 741
for proceedings consistent with this opinion. SCRA 239 (2014)

FACTS: Philippine National Police (PNP), through Police


Senior Superintendent Fajardo, applied with the RTC for a
warrant to search three (3) caves located located inside the
Laud Compound in Purok 3, Barangay Maa, Davao City,
where the alleged remains of the victims summarily executed
by the so-called “Davao Death Squad” may be found. In
support of the application, a certain Ernesto Avasola was
presented to the RTC and there testified that he personally
witnessed the killing of six (6) persons in December 2005, and
was, in fact, part of the group that buried the victims.

Judge Peralta, acting as Vice Executive Judge of the Manila-


RTC, found probable cause for the issuance of a search
warrant, and thus, issued Search Warrant which was later
enforced by the elements of the PNP-Criminal Investigation
and Detection Group, in coordination with the members of the
Scene of the Crime Operatives on July 15, 2009. The search
of the Laud Compound caves yielded positive results for the
presence of human remains.

Petitioner, retired SPO4 Bienvenido Laud (Laud), filed an


Urgent Motion to Quash and to Suppress Illegally Seized
Evidence premised on the following grounds: (a) Judge
Peralta had no authority to act on the application for a search
warrant since he had been automatically divested of his
position as Vice Executive Judge when several administrative
penalties were imposed against him by the Court; (b) the
ManilaRTC had no jurisdiction to issue the Search Warrant
which was to be enforced in Davao City; (e) the search warrant
was issued despite lack of probable cause; and (g) there was
a violation of the rule requiring one-specific-offense and the
proper specification of the place to be searched and the
articles to be seized.

Manila-RTC Ruling
The RTC granted the motion of Laud. Respondents filed a
Motion for Reconsideration which was, however, denied. The
People failed to show any compelling reason to justify the
issuance of a search warrant by the Manila-RTC which was to
be implemented in Davao City where the offense was
allegedly committed, in violation of Section 2, Rule 126 of the
Rules of Court. The fact that the alleged offense happened
almost four (4) years before the search warrant application
was filed rendered doubtful the existence of probable cause;
the applicant, i.e., the PNP, violated the rule against forum
shopping as the subject matter of the present search warrant.

CA Ruling
CA granted the People's petition and thereby annulled and set
aside the Orders of the Manila-RTC for having been tainted
with grave abuse of discretion.

ISSUE #1:
WoN the administrative penalties imposed on Judge Peralta
invalidated the Search Warrant.

HELD #1:
No. Citing Section 5, Chapter III of A.M. No. 03802SC which
provides that “[t]he imposition upon an Executive Judge or
Vice Executive Judge of an administrative penalty of at least
a reprimand shall automatically operate to divest him of his
position as such”.

While the Court does agree that the imposition of said


administrative penalties did operate to divest Judge Peralta’s
authority to act as Vice Executive Judge, it must be qualified
that the abstraction of such authority would not, by and of
itself, result in the invalidity of Search Warrant considering that

Page 43 of 186
Judge Peralta may be considered to have made the issuance special criminal cases “shall be an exception to Section 2 of
as a de facto officer whose acts would, nonetheless, remain Rule 126 of the Rules of Court.” Perceptibly, the fact that a
valid. search warrant is being applied for in connection with a special
criminal case as above classified already presumes the
Undoubtedly, there is a de jure office of a 2nd Vice Executive existence of a compelling reason; hence, any statement to this
Judge. Judge Peralta also had a colorable right to the said effect would be superfluous and therefore should be
office as he was duly appointed to such position and was only dispensed with. By all indications, Section 12, Chapter V of
divested of the same by virtue of a supervening legal A.M. No. 03802SC allows the Manila and Quezon City RTCs
technicality — that is, the operation of Section 5, Chapter III of to issue warrants to be served in places outside their territorial
A.M. No. 03802SC; also, it may be said that there was general jurisdiction for as long as the parameters under the said
acquiescence by the public since the search warrant section have been complied with, as in this case. Thus, on
application was regularly endorsed to the sala of Judge these grounds, the Court finds nothing defective in the
Peralta by the Office of the Clerk of Court of the ManilaRTC preliminary issuance of Search Warrant.
under his apparent authority as 2nd Vice Executive Judge.
Finally, Judge Peralta’s actual physical possession of the said
office is presumed to be in good faith, as the contrary was not ISSUE #3: WoN the requirements of probable cause and
established. Accordingly, Judge Peralta can be considered to particular description were complied with, and the one-
have acted as a de facto officer when he issued Search specific- offense rule under Section 4, Rule 126 of the Rules
Warrant, hence, treated as valid as if it was issued by a de of Court was violated.
jure officer suffering no administrative impediment.
HELD #3:

ISSUE #2: Requirements of Probable Cause –YES


WoN the ManilaRTC had jurisdiction to issue the said warrant
despite non-compliance with the compelling reasons In order to protect the people’s right against unreasonable
requirement under Section 2, Rule 126 of the Rules of Court searches and seizures, Section 2, Article III of the 1987
Philippine Constitution provides that no search warrant shall
HELD #2: issue except upon probable cause to be determined
Yes. Section 12, Chapter V of A.M. No. 03802SC states the personally by the judge after examination under oath or
requirements for the issuance of search warrants in special affirmation of the complainant and the witnesses he may
criminal cases by the RTCs of Manila and Quezon City. These produce, and particularly describing the place to be searched
special criminal cases pertain to those “involving heinous and the persons or things to be seized:
crimes, illegal gambling, illegal possession of firearms and
ammunitions, as well as violations of the Comprehensive Complementarily, Section 4, Rule 126 of the Rules of Court
Dangerous Drugs Act of 2002, the Intellectual Property Code, states that a search warrant shall not be issued except upon
the Anti-Money Laundering Act of 2001, the Tariff and probable cause in connection with one specific offense.
Customs Code, as amended, and other relevant laws that may
hereafter be enacted by Congress, and included herein by the In this case, the existence of probable cause for the issuance
Supreme Court.” of Search Warrant is evident from the firsthand account of
Avasola who, in his deposition, stated that he personally
Search warrant applications for such cases may be filed by witnessed the commission of the aforestated crime and was,
“the National Bureau of Investigation (NBI), the Philippine in fact, part of the group that buried the victims.
National Police (PNP) and the AntiCrime Task Force
(ACTAF),” and “personally endorsed by the heads of such Verily, the facts and circumstances established from the
agencies.” As in ordinary search warrant applications, they testimony of Avasola, who was personally examined by Judge
“shall particularly describe therein the places to be searched Peralta, sufficiently show that more likely than not the crime of
and/or the property or things to be seized as prescribed in the Murder of six (6) persons had been perpetrated and that the
Rules of Court.” “The Executive Judges [of these RTCs] and, human remains in connection with the same are in the place
whenever they are on official leave of absence or are not sought to be searched.
physically present in the station, the Vice Executive Judges”
are authorized to act on such applications and “shall issue the In Santos v. Pryce Gases, Inc., the Court explained the
warrants, if justified, which may be served in places outside quantum of evidence necessary to establish probable cause
the territorial jurisdiction of the said courts.” for a search warrant.

The Court observes that all the above stated requirements “Probable cause for a search warrant is
were complied with in this case. defined as such facts and circumstances
which would lead a reasonably discrete and
As the records would show, the search warrant application prudent man to believe that an offense has
was filed before the ManilaRTC by the PNP and was endorsed been committed and that the objects sought
by its head, PNP Chief Jesus Ame Versosa, particularly in connection with the offense are in the
describing the place to be searched and the things to be place sought to be searched. A finding of
seized in connection with the heinous crime of Murder. Finding probable cause needs only to rest on
probable cause therefor, Judge Peralta, in his capacity as 2nd evidence showing that, more likely than not,
Vice Executive Judge, issued Search Warrant which, as the a crime has been committed and that it was
rules state, may be served in places outside the territorial committed by the accused. Probable cause
jurisdiction of the said RTC. demands more than bare suspicion; it
requires less than evidence which would
Notably, the fact that a search warrant application involves a justify conviction. The existence depends to
“special criminal case” excludes it from the compelling reason a large degree upon the finding or opinion
requirement under Section 2, Rule 126 of the Rules of Court. of the judge conducting the examination.
However, the findings of the judge should
As explicitly mentioned in Section 12, Chapter V of A.M. No. not disregard the facts before him nor run
03802SC, the rule on search warrant applications before the counter to the clear dictates of reason.”
Manila and Quezon City RTCs for the above mentioned

Page 44 of 186
In light of the foregoing, the Court finds that the quantum of offense; and the articles subject of search and seizure should
proof to establish the existence of probable cause had been come in handy merely to strengthen such evidence.
met. That a “considerable length of time” attended the search
warrant’s application from the crime’s commission does not, Consequently, the Court finds that the particular description
by and of itself, negate the veracity of the applicant’s claims or requirement — both as to the place to be searched and the
the testimony of the witness presented. As the CA correctly things to be seized — had been complied with.
observed, the delay may be accounted for by a witness’s fear
of reprisal and natural reluctance to get involved in a criminal One-Specific-Offense Rule –No
case. Ultimately, in determining the existence of probable
cause, the facts and circumstances must be personally The Court finds no violation of the one-specific-offense rule
examined by the judge in their totality, together with a judicious under Section 4, Rule 126 of the Rules of Court which, to note,
recognition of the variable complications and sensibilities was intended to prevent the issuance of scattershot warrants,
attending a criminal case. To the Court’s mind, the supposed or those which are issued for more than one-specific-offense.
delay in the search warrant’s application does not dilute the
probable cause finding made herein. In fine, the probable In Columbia Pictures, Inc. v. CA, the Court, however, settled
cause requirement has been sufficiently met. that a search warrant that covers several counts of a certain
specific offense does not violate the one-specific-offense rule.
Requirements of Particularity of Description –YES Hence, given that Search Warrant was issued only for one
specific offense — that is, of Murder, albeit for six (6) counts
The Court similarly concludes that there was compliance with — it cannot be said that Section 4, Rule 126 of the Rules of
the constitutional requirement that there be a particular Court had been violated.
description of “the place to be searched and the persons or
things to be seized.”

“[A] description of a place to be


searched is sufficient if the officer with
the warrant can, with reasonable effort,
ascertain and identify the place
intended and distinguish it from other
places in the community. Any
designation or description known to the
locality that points out the place to the
exclusion of all others, and on inquiry
leads the officers unerringly to it,
satisfies the constitutional requirement.”

Search Warrant evidently complies with the foregoing


standard since it particularly describes the place to be
searched, namely, the 3 caves located inside the Laud
Compound in Purok 3, Barangay Maa, Davao City.

For further guidance in its enforcement, the search warrant


even made explicit reference to the sketch contained in the
application. These, in the Court’s view, are sufficient enough
for the officers to, with reasonable effort, ascertain and identify
the place to be searched, which they in fact did.

The things to be seized were also particularly described,


namely, the remains of six (6) victims who were killed and
buried in the aforesaid premises. Laud’s posturing that human
remains are not “personal property” and, hence, could not be
the subject of a search warrant deserves scant consideration.

“Personal property” in Section 3, Rule 126 of the Rules of


Court actually refers to the thing’s mobility, and not to its
capacity to be owned or alienated. Considering that human
remains can generally be transported from place to place, and
considering further that they qualify under the phrase “subject
of the offense” given that they prove the crime’s corpus delicti,
it follows that they may be valid subjects of a search warrant
under the above cited criminal procedure provision.

A search warrant may be said to particularly describe the


things to be seized when the description therein is as specific
as the circumstances will ordinarily allow; or when the
description expresses a conclusion of fact — not of law — by
which the warrant officer may be guided in making the search
and seizure; or when the things described are limited to those
which bear direct relation to the offense for which the warrant
is being issued (Sec. 2, Rule 126, Revised Rules of Court) x x
x If the articles desired to be seized have any direct relation to
an offense committed, the applicant must necessarily have
some evidence, other than those articles, to prove the said

Page 45 of 186
3. LOS ANGELES COUNTY. V. RETTELE 550 US 609; accused of a crime that required an emergency search; and
MAY 21, 2007 (4) respondents were ordered out of bed naked and held at
gunpoint while the deputies searched their bedroom for the
suspects and a gun, we find that a reasonable jury could
FACTS: In 2001, Los Angeles County Sheriff’s Department conclude that the search and detention were ‘unnecessarily
Deputy Dennis Watters investigated a fraud and identity-theft painful, degrading, or prolonged,’ and involved ‘an undue
crime ring. There were 4 suspects of the investigation. One invasion of privacy.”
had registered a 9-millimeter Glock handgun. The 4 suspects
were known to be African-Americans. Turning to whether respondents’ Fourth Amendment rights
were clearly established, the majority held that a reasonable
Watters obtained a search warrant for 2 houses in Lancaster, deputy should have known the search and detention were
California, where he believed he could find the suspects. The unlawful. [main pinaglalaban talaga nila: Because
warrant authorized him to search the homes and 3 of the respondents were of a different race than the suspects the
suspects for documents and computer files. In support of the deputies were seeking, the Court of Appeals held that “after
search warrant an affidavit cited various sources showing the taking one look at respondents, the deputies should have
suspects resided at Retelle’s home. realized that respondents were not the subjects of the search
warrant and did not pose a threat to the deputies’ safety.]
[What Watters did not know was that one of the houses (the
first to be searched) had been sold in September to a Max
Rettele. He had purchased the home and moved into it 3 ISSUE: WoN there was a violation of the 4th Amendment
months earlier with his girlfriend Judy Sadler and Sadler’s 17 Rights of the respondents.
year-old son Chase Hall. All three, respondents here, are
Caucasians.]
HELD: No, When the deputies ordered respondents from their
Watters briefed the 6 other deputies in preparation for the bed, they had no way of knowing whether the African-
search of the houses. Watters informed them they would be American suspects were elsewhere in the house. The
searching for 3 African-American suspects, one of whom presence of some Caucasians in the residence did not
owned a registered handgun. Around 7:15am Watters and six eliminate the possibility that the suspects lived there as well.
other deputies knocked on the door and announced their As the deputies stated in their affidavits, it is not uncommon in
presence. Chase Hall answered. The deputies entered the our society for people of different races to live together. Just
house after ordering Hall to lie face down on the ground. as people of different races live and work together, so too
might they engage in joint criminal activity. The deputies, who
The deputies’ announcement awoke Rettele and Sadler. The were searching a house where they believed a suspect might
deputies entered their bedroom with guns drawn and ordered be armed, possessed authority to secure the premises before
them to get out of their bed and to show their hands. They deciding whether to continue with the search.
protested that they were not wearing clothes. Rettele stood up
and attempted to put on a pair of sweat pants, but deputies In Michigan v. Summers, this Court held that officers
told him not to move. Sadler also stood up and attempted, executing a search warrant for contraband may “detain the
without success, to cover herself with a sheet. Rettele and occupants of the premises while a proper search is
Sadler were held at gunpoint for one to two minutes before conducted.” In weighing whether the search in Summers was
Rettele was allowed to retrieve a robe for Sadler. He was then reasonable the Court first found that “detention represents
permitted to dress. Rettele and Sadler left the bedroom within only an incremental intrusion on personal liberty when the
three to four minutes to sit on the couch in the living room. search of a home has been authorized by a valid warrant.”
Against that interest, it balanced “preventing flight in the event
By that time the deputies realized they had made a mistake. that incriminating evidence is found”; “minimizing the risk of
They apologized to Rettele and Sadler, thanked them for not harm to the officers”; and facilitating “the orderly completion of
becoming upset, and left within five minutes. They proceeded the search.”
to the other house the warrant authorized them to search,
where they found 3 suspects. Those suspects were arrested In executing a search warrant officers may take reasonable
and convicted. action to secure the premises and to ensure their own safety
and the efficacy of the search. The test of reasonableness
Rettele and Sadler, individually and as guardians ad litem for under the Fourth Amendment is an objective one (addressing
Hall, filed this suit against Los Angeles County, the Los the reasonableness of a seizure of the person). Unreasonable
Angeles County Sheriff’s Department, Deputy Watters, and actions include the use of excessive force or restraints that
other members of the sheriff’s department. Respondents cause unnecessary pain or are imposed for a prolonged and
alleged petitioners violated their Fourth Amendment rights by unnecessary period of time.
obtaining a warrant in reckless fashion and conducting an
unreasonable search and detention. The orders by the police to the occupants, in the context of
this lawful search, were permissible, and perhaps necessary,
The District Court held that the warrant was obtained by to protect the safety of the deputies. Blankets and bedding can
proper procedures and the search was reasonable. conceal a weapon, and one of the suspects was known to own
a firearm, factors which underscore this point. The
On appeal respondents did not challenge the validity of the Constitution does not require an officer to ignore the possibility
warrant; they did argue that the deputies had conducted the that an armed suspect may sleep with a weapon within reach.
search in an unreasonable manner. The reports are replete with accounts of suspects sleeping
close to weapons.
The Court of Appeals for the Ninth Circuit reversed the District
Court’s decision. The deputies needed a moment to secure the room and
ensure that other persons were not close by or did not present
The CA’s majority held that “because (1) no African- a danger. Deputies were not required to turn their backs to
Americans lived in respondents’ home; (2) respondents, a allow Rettele and Sadler to retrieve clothing or to cover
Caucasian couple, purchased the residence several months themselves with the sheets. Rather, “the risk of harm to both
before the search and the deputies did not conduct an the police and the occupants is minimized if the officers
ownership inquiry; (3) the African-American suspects were not routinely exercise unquestioned command of the situation.”

Page 46 of 186
4. RODEL LUZ y ONG vs. PEOPLE OF THE
This is not to say, of course, that the deputies were free to PHILIPPINES (G.R. No. 197788 February 29, 2012)
force Rettele and Sadler to remain motionless and standing
for any longer than necessary. We have recognized that
“special circumstances, or possibly a prolonged detention” FACTS: PO2 Emmanuel L. Alteza, who was then assigned
might render a search unreasonable. There is no accusation at the Sub-Station 1 of the Naga City Police Station as a traffic
that the detention here was prolonged. The deputies left the enforcer, substantially testified that on March 10, 2003 at
home less than 15 minutes after arriving. The detention was around 3:00 o’clock in the morning, he saw the accused, who
shorter and less restrictive than the 2- to 3-hour handcuff was coming from the direction of Panganiban Drive and going
detention. And there is no allegation that the deputies to Diversion Road, Naga City, driving a motorcycle without a
prevented Sadler and Rettele from dressing longer than helmet; that this prompted him to flag down the accused for
necessary to protect their safety. Sadler was unclothed for no violating a municipal ordinance which requires all motorcycle
more than two minutes, and Rettele for only slightly more time drivers to wear helmet (sic) while driving said motor vehicle;
than that. Sadler testified that once the police were satisfied that he invited the accused to come inside their sub-station
that no immediate threat was presented, “they wanted us to since the place where he flagged down the accused is almost
get dressed and they were pressing us really fast to hurry up in front of the said sub-station; that while he and SPO1
and get some clothes on.” Rayford Brillante were issuing a citation ticket for violation of
municipal ordinance, he noticed that the accused was uneasy
The Fourth Amendment allows warrants to issue on probable and kept on getting something from his jacket; that he was
cause, a standard well-short of absolute certainty. Valid alerted and so, he told the accused to take out the contents of
warrants will issue to search the innocent, and people like the pocket of his jacket as the latter may have a weapon inside
Rettele and Sadler unfortunately bear the cost. Officers it; that the accused obliged and slowly put out the contents of
executing search warrants on occasion enter a house when the pocket of his jacket which was a nickel-like tin or metal
residents are engaged in private activity; and the resulting container about two (2) to three (3) inches in size, including
frustration, embarrassment, and humiliation may be real, as two (2) cellphones, one (1) pair of scissors and one (1) Swiss
was true here. When officers execute a valid warrant and act knife; that upon seeing the said container, he asked the
in a reasonable manner to protect themselves from harm, accused to open it; that after the accused opened the
however, the Fourth Amendment is not violated. container, he noticed a cartoon cover and something beneath
it; and that upon his instruction, the accused spilled out the
contents of the container on the table which turned out to be
four (4) plastic sachets, the two (2) of which were empty while
the other two (2) contained suspected shabu.

Petitioner claims that there was no lawful search and seizure,


because there was no lawful arrest. He claims that the finding
that there was a lawful arrest was erroneous, since he was not
even issued a citation ticket or charged with violation of the
city ordinance. Even assuming there was a valid arrest, he
claims that he had never consented to the search conducted
upon him.

In its 19 February 2009 Decision, the RTC convicted petitioner


of illegal possession of dangerous drugs committed on 10
March 2003. It found the prosecution evidence sufficient to
show that he had been lawfully arrested for a traffic violation
and then subjected to a valid search, which led to the
discovery on his person of two plastic sachets later found to
contain shabu.

Upon review, the CA affirmed the RTC’s Decision.

ISSUE: Was the search and seizure of the alleged subject


shabu invalid thus acquitted?

HELD: Petitioner must be acquitted. While he may have failed


to object to the illegality of his arrest at the earliest opportunity,
a waiver of an illegal warrantless arrest does not, however,
mean a waiver of the inadmissibility of evidence seized during
the illegal warrantless arrest. The Constitution guarantees the
right of the people to be secure in their persons, houses,
papers and effects against unreasonable searches and
seizures. Any evidence obtained in violation of said right shall
be inadmissible for any purpose in any proceeding. While the
power to search and seize may at times be necessary to the
public welfare, still it must be exercised and the law
implemented without contravening the constitutional rights of
citizens, for the enforcement of no statute is of sufficient
importance to justify indifference to the basic principles of
government. The subject items seized during the illegal arrest
are inadmissible. The drugs are the very corpus delicti of the
crime of illegal possession of dangerous drugs. Thus, their

Page 47 of 186
inadmissibility precludes conviction and calls for the acquittal issued if the information or charge was filed for an offense
of the accused. penalized by a fine only. It may be stated as a corollary that
neither can a warrantless arrest be made for such an offense.
First, there was no valid arrest of petitioner. When he was This ruling does not imply that there can be no arrest for a
flagged down for committing a traffic violation, he was not, ipso traffic violation. Certainly, when there is an intent on the part
facto and solely for this reason, arrested. Arrest is the taking of the police officer to deprive the motorist of liberty, or to take
of a person into custody in order that he or she may be bound the latter into custody, the former may be deemed to have
to answer for the commission of an offense. It is effected by arrested the motorist. In this case, however, the officer’s
an actual restraint of the person to be arrested or by that issuance (or intent to issue) a traffic citation ticket negates the
person’s voluntary submission to the custody of the one possibility of an arrest for the same violation. Even if one were
making the arrest. Neither the application of actual force, to work under the assumption that petitioner was deemed
manual touching of the body, or physical restraint, nor a formal "arrested" upon being flagged down for a traffic violation and
declaration of arrest, is required. It is enough that there be an while awaiting the issuance of his ticket, then the requirements
intention on the part of one of the parties to arrest the other, for a valid arrest were not complied with.
and that there be an intent on the part of the other to submit,
under the belief and impression that submission is necessary. This Court has held that at the time a person is arrested, it
Under R.A. 4136, or the Land Transportation and Traffic shall be the duty of the arresting officer to inform the latter of
Code, the general procedure for dealing with a traffic violation the reason for the arrest and must show that person the
is not the arrest of the offender, but the confiscation of the warrant of arrest, if any. Persons shall be informed of their
driver’s license of the latter. At the time that he was waiting for constitutional rights to remain silent and to counsel, and that
PO3 Alteza to write his citation ticket, petitioner could not be any statement they might make could be used against them.
said to have been "under arrest." There was no intention on It may also be noted that in this case, these constitutional
the part of PO3 Alteza to arrest him, deprive him of his liberty, requirements were complied with by the police officers only
or take him into custody. Prior to the issuance of the ticket, the after petitioner had been arrested for illegal possession of
period during which petitioner was at the police station may be dangerous drugs.
characterized merely as waiting time. In fact, as found by the
trial court, PO3 Alteza himself testified that the only reason In Berkemer, the U.S. Court also noted that the Miranda
they went to the police sub-station was that petitioner had warnings must also be given to a person apprehended due to
been flagged down "almost in front" of that place. Hence, it a traffic violation:
was only for the sake of convenience that they were waiting
The purposes of the safeguards prescribed by
there. There was no intention to take petitioner into custody.
Miranda are to ensure that the police do not coerce
Two features of an ordinary traffic stop mitigate the danger or trick captive suspects into confessing, to relieve
that a person questioned will be induced "to speak where he the "inherently compelling pressures" "generated by
would not otherwise do so freely," Miranda v. Arizona, 384 U. the custodial setting itself," "which work to undermine
S., at 467. First, detention of a motorist pursuant to a traffic the individual’s will to resist," and as much as
stop is presumptively temporary and brief. The vast majority possible to free courts from the task of scrutinizing
of roadside detentions last only a few minutes. A motorist’s individual cases to try to determine, after the fact,
expectations, when he sees a policeman’s light flashing whether particular confessions were voluntary.
behind him, are that he will be obliged to spend a short period Those purposes are implicated as much by in-
of time answering questions and waiting while the officer custody questioning of persons suspected of
checks his license and registration, that he may then be given misdemeanors as they are by questioning of persons
a citation, but that in the end he most likely will be allowed to suspected of felonies.
continue on his way. In this respect, questioning incident to an
Second, there being no valid arrest, the warrantless search
ordinary traffic stop is quite different from stationhouse
that resulted from it was likewise illegal. The following are the
interrogation, which frequently is prolonged, and in which the
instances when a warrantless search is allowed: (i) a
detainee often is aware that questioning will continue until he
warrantless search incidental to a lawful arrest; (ii) search of
provides his interrogators the answers they seek. See id., at
evidence in "plain view;" (iii) search of a moving vehicle; (iv)
451. Second, circumstances associated with the typical traffic
consented warrantless search; (v) customs search; (vi) a "stop
stop are not such that the motorist feels completely at the
and frisk" search; and (vii) exigent and emergency
mercy of the police. To be sure, the aura of authority
circumstances. None of the above-mentioned instances,
surrounding an armed, uniformed officer and the knowledge
especially a search incident to a lawful arrest, are applicable
that the officer has some discretion in deciding whether to
to this case.
issue a citation, in combination, exert some pressure on the
detainee to respond to questions. But other aspects of the It must be noted that the evidence seized, although alleged to
situation substantially offset these forces. Perhaps most be inadvertently discovered, was not in "plain view." It was
importantly, the typical traffic stop is public, at least to some actually concealed inside a metal container inside petitioner’s
degree. x x x pocket. Clearly, the evidence was not immediately apparent.
In both of these respects, the usual traffic stop is more Neither was there a consented warrantless search. Consent
analogous to a so-called "Terry stop," see Terry v. Ohio, 392 to a search is not to be lightly inferred, but shown by clear and
U. S. 1 (1968), than to a formal arrest. x x x The comparatively convincing evidence. It must be voluntary in order to validate
nonthreatening character of detentions of this sort explains the an otherwise illegal search; that is, the consent must be
absence of any suggestion in our opinions that Terry stops are unequivocal, specific, intelligently given and uncontaminated
subject to the dictates of Miranda. by any duress or coercion. While the prosecution claims that
petitioner acceded to the instruction of PO3 Alteza, this
It also appears that, according to City Ordinance No. 98-012,
alleged accession does not suffice to prove valid and
which was violated by petitioner, the failure to wear a crash
intelligent consent. In fact, the RTC found that petitioner was
helmet while riding a motorcycle is penalized by a fine only.
merely "told" to take out the contents of his pocket.
Under the Rules of Court, a warrant of arrest need not be

Page 48 of 186
Neither does the search qualify under the "stop and frisk" rule. 5. MARTINEZ vs. PEOPLE (G.R. No. 198694
While the rule normally applies when a police officer observes February 13, 2013)
suspicious or unusual conduct, which may lead him to believe
that a criminal act may be afoot, the stop and frisk is merely a
limited protective search of outer clothing for weapons. In
FACTS: At around 9:15PM of December 29, 2007, PO2
Knowles v. Iowa, the U.S. Supreme Court held that when a
Soque, PO2 Cepe and PO3 Zeta, Police officers assigned to
police officer stops a person for speeding and correspondingly
the Station Anti-Illegal Drugs (SAID) Section of the Malate
issues a citation instead of arresting the latter, this procedure Police Station 9 (Police Station 9), conducted a routine foot
does not authorize the officer to conduct a full search of the patrol along Balingkit Street, Malate, Manila. In the process,
car. The Court therein held that there was no justification for a they heard a man shouting "Putanginamo! Limangdaan na ba
full-blown search when the officer does not arrest the motorist. ito?"
Instead, police officers may only conduct minimal intrusions,
such as ordering the motorist to alight from the car or doing a For purportedly violating Section 844 of the Revised
patdown. Ordinance of the City of Manila which punishes breaches of
the peace, Ramon was apprehended and asked to empty his
pockets. In the course thereof, the police officers were able to
recover from him a small transparent plastic sachet containing
white crystalline substance suspected to be shabu. PO2
Soque confiscated the sachet and brought Ramon to Police
Station 9. Consequently, Ramon was charged with
possession of dangerous drugs under Section 11(3), Article II
of RA 9165.

In defense, Ramon denied the charge and gave his version of


the incident. He narrated that while walking along Balingkit
Street to borrow a welding machine from one Paez Garcia, a
man in civilian clothing approached and asked him if he is
Ramon Goco. Upon affirming his identity, he was immediately
handcuffed by the man who eventually introduced himself as
a police officer. Together, they boarded a tricycle (sidecar)
where the said officer asked him if he was carrying illegal
drugs. Despite his denial, he was still brought to a precinct to
be detained. Thereafter, one of the officers propositioned
Ramon and asked for P20,000.00 in exchange for his release.
When Ramon’s wife, Amalia Goco, was unable to produce the
P20,000.00, Ramon was brought to the Manila City Hall for
inquest proceedings.

RTC convicted Ramon as charged. CA affirmed the RTC


decision.

ISSUE: Whether or not Ramon could be convicted of the crime


of possession of dangerous drugs provided that there’s
warrantless search conducted.

HELD: Ramon Martinez was acquitted since the subject


shabu purportedly seized is inadmissible in evidence for being
the proverbial fruit of the poisonous tree.

Section 3(2), Article III of the Constitution provides that any


evidence obtained in violation of Section 2, Article III of the
Constitution shall be inadmissible for any purpose in any
proceeding. However, such exclusionary rule is not absolute.
The following are the traditional exceptions: customs
searches, searches of moving vehicles, seizure of evidence in
plain view, consented searches, "stop and frisk" measures
and searches incidental to a lawful arrest. As to the last
exception, it requires that the apprehending officer must have
been spurred by probable cause to arrest a person caught in
flagrante delicto. The term probable cause is a reasonable
ground of suspicion supported by circumstances sufficiently
strong in themselves to warrant a cautious man's belief that
the person accused is guilty of the offense with which he is
charged.

Enshrined in the fundamental law is a person’s right against


unwarranted intrusions by the government. Accordingly, so as
to ensure that the same sacrosanct right remains revered,
effects secured by government authorities in contravention of
the foregoing are rendered inadmissible in evidence for any
purpose, in any proceeding.

Page 49 of 186
Commonly known as the “exclusionary rule,” the proscription Ramon is inadmissible in evidence for being the proverbial
is not, however, an absolute and rigid one. As found in fruit of the poisonous tree as mandated by the above-
jurisprudence, the traditional exceptions are customs discussed constitutional provisions. In this regard, considering
searches, searches of moving vehicles, seizure of evidence in that the confiscated shabu is the very corpus delicti of the
plain view, consented searches, “stop and frisk” measures crime charged, Ramon’s acquittal should therefore come as a
and searches incidental to a lawful arrest. matter of course.

A valid warrantless arrest which justifies a subsequent search


is one that is carried out under the parameters of Section 5(a),
Rule 113 of the Rules of Court which requires that the
apprehending officer must have been spurred by probable
cause to arrest a person caught in flagrante delicto. To be
sure, the term probable cause has been understood to mean
a reasonable ground of suspicion supported by circumstances
sufficiently strong in themselves to warrant a cautious man’s
belief that the person accused is guilty of the offense with
which he is charged. Specifically with respect to arrests, it is
such facts and circumstances which would lead a reasonably
discreet and prudent man to believe that an offense has been
committed by the person sought to be arrested. In this light,
the determination of the existence or absence of probable
cause necessitates a re-examination of the factual incidents.

Records show that the Police officers arrested Ramon for


allegedly violating Section 844 of the Manila City Ordinance
which punishes Breaches of the Peace. The gravamen of the
offenses enumerated in the Ordinance is the disruption of
communal tranquillity. Thus, to justify a warrantless arrest
based thereon, it must be established that the apprehension
was effected after a reasonable assessment by the police
officer that a public disturbance is being committed.

The perusal of the testimony during the trial, negates the


presence of probable cause when the police officers
conducted their warrantless arrest of Ramon. Further, it bears
stressing that no one present at the place of arrest ever
complained that Ramon’s shouting disturbed the public. On
the contrary, a disinterested member of the community (a
certain Rosemarie Escobal) even testified that Ramon was
merely standing in front of the store of a certain Mang Romy
when a man in civilian clothes, later identified as the police
officer, approached Ramon, immediately handcuffed and took
him away.

In its totality, the Court observes that these facts and


circumstances could not have engendered a well-founded
belief that any breach of the peace had been committed by
Ramon at the time that his warrantless arrest was effected. All
told, no probable cause existed to justify Ramon’s warrantless
arrest.

Indeed, while it is true that the legality of arrest depends upon


the reasonable discretion of the officer or functionary to whom
the law at the moment leaves the decision to characterize the
nature of the act or deed of the person for the urgent purpose
of suspending his liberty, this should not be exercised in a
whimsical manner, else a person’s liberty be subjected to
ubiquitous abuse. As law enforcers, it is largely expected of
them to conduct a more circumspect assessment of the
situation at hand. The determination of probable cause is not
a blanket-license to withhold liberty or to conduct unwarranted
fishing expeditions. It demarcates the line between legitimate
human conduct on the one hand, and ostensible criminal
activity, on the other. In this respect, it must be performed
wisely and cautiously, applying the exacting standards of a
reasonably discreet and prudent man. Surely, as
constitutionally guaranteed rights lie at the fore, the duty to
determine probable cause should be clothed with utmost
conscientiousness as well as impelled by a higher sense of
public accountability.

Consequently, as it cannot be said that Ramon was validly


arrested, the warrantless search that resulted from it was also
illegal. Thus, the subject shabu purportedly seized from

Page 50 of 186
6. ONGCOMA HADJI HOMAR v. PEOPLE (GR No. HELD The prosecution failed to prove that a lawful
182534, September 02, 2015) warrantless arrest preceded the search conducted on the
petitioner's body.

The petitioner was charged for violation of Section 11, Article To constitute a valid in flagrante delicto arrest, two requisites
II of RA 9165. The Information states that on or about August must concur: (1) the person to be arrested must execute an
20, 2002, the petitioner was found to possess one heat-sealed overt act indicating that he has just committed, is actually
transparent plastic sachet containing 0.03 grams of committing, or is attempting to commit a crime; and (2) such
methylamphetamine hydrochloride, otherwise known overt act is done in the presence of or within the view of the
as shabu. The petitioner pleaded not guilty during arresting officer.
arraignment.
The prosecution was not able to discharge this burden,
PO1 Eric Tan (Tan) was the lone witness for the prosecution. particularly that Homar was actually committing a crime. They
As stated in the RTC decision, he testified that on August 20, did not identify the place where Homar allegedly crossed and
2002, at around 8:50 in the evening, their Chief, P/Chief Supt. that it was illegal to cross that area. He was also not charged
Alfredo C. Valdez, ordered him and civilian agent (C/A) with jaywalking.
Ronald Tangcoy (Tangcoy) to go to the South Wing, Roxas
Boulevard. While proceeding to the area onboard a mobile The police testified that they “accosted” Homar when he
hunter, they saw the petitioner crossing a "No Jaywalking" jaywalked. However, this is different from an actual arrest as
portion of Roxas Boulevard. They immediately accosted him contemplated by the Rules on warrantless arrests. No arrest
and told him to cross at the pedestrian crossing area. preceded the search because they did not intend to bring him
under custody or restrain his liberty. The lack of intent was
The petitioner picked up something from the ground, further proven by the absence of criminal charges against him.
prompting Tangcoy to frisk him resulting in the recovery of a Intent only came after they allegedly confiscated the shabu.
knife. Thereafter, Tangcoy conducted a thorough search on The shabu was not recovered immediately after the alleged
the petitioner's body and found and confiscated a plastic lawful arrest but only after the initial search. Intent to arrest is
sachet containing what he suspected as shabu. Tangcoy and indispensable because otherwise, any evidence obtained in
Tan executed a sinumpaang salaysay on the incident. violation thereof will be inadmissible.

The petitioner was the sole witness for the defense. He The Constitution guarantees the right of the people to be
testified that on August 20, 2002, he was going home at secure in their persons, houses, papers, and effects against
around 6:30 p.m. after selling imitation sunglasses and other unreasonable searches and seizures. Any evidence obtained
accessories at the BERMA Shopping Center. After crossing in violation of these rights shall be inadmissible for any
the overpass, a policeman and a civilian stopped and frisked purpose in any proceeding. While the power to search and
him despite his refusal. They poked a gun at him, accused him seize may at times be necessary to the public welfare, the
of being a holdupper, and forced him to go with them. They exercise of this power and the implementation of the law
also confiscated the kitchen knife, which he carried to cut should not violate the constitutional rights of the citizens.
cords. He was likewise investigated for alleged possession
of shabu and detained for one day. He was criminally charged To determine the admissibility of the seized drugs in evidence,
before the Metropolitan Trial Court of Parañaque City, Branch it is indispensable to ascertain whether or not the search which
77 for the possession of the kitchen knife but he was yielded the alleged contraband was lawful. There must be a
eventually acquitted. valid warrantless search and seizure pursuant to an equally
valid warrantless arrest, which must precede the search. For
RTC Ruling this purpose, the law requires that there be first a lawful arrest
The RTC convicted the petitioner. It ruled that PO1 Tan and before a search can be made — the process cannot be
C/A Tangcoy were presumed to have performed their duties reversed.
regularly in arresting and conducting a search on the
petitioner. The RTC also noted that PO1 Eric Tan was
straightforward in giving his testimony and he did not show
any ill motive in arresting the petitioner.

The RTC also did not believe the petitioner's defense of denial
and ruled that it is a common and standard defense ploy in
most prosecutions in dangerous drugs cases. This defense is
weak especially when it is not substantiated by clear and
convincing evidence as in this case.

CA Ruling
The CA dismissed the petition and affirmed the RTC's
findings.
The arrest falls under valid warrantless arrest; when the
person to be arrested has committed, is actually committing,
or is attempting to commit an offense in the presence of a
peace officer or a private person. In the present case, the
petitioner committed jaywalking in the presence of PO1 Tan
and C/A Tangcoy; hence, his warrantless arrest for jaywalking
was lawful. Consequently, the subsequent frisking and search
done on the petitioner's body which produced the knife and
the shabu were incident to a lawful arrest.

ISSUE: Whether or not the search was valid

Page 51 of 186
7. People v. Villareal 693 SCRA 549 (2013) considered criminal acts. In fact, even if appellant had been
exhibiting unusual or strange acts, or at the very least
appeared suspicious, the same would not have been sufficient
FACTS: As PO3 de Leon was driving his motorcycle on his in order for PO3 de Leon to effect a lawful warrantless arrest
way home, he saw appellant from a distance of about 8 to 10 under paragraph (a) of Section 5, Rule 113.
meters, holding and scrutinizing in his hand a plastic sachet of
shabu. Thus, PO3 de Leon, a member of the Station Anti- Neither has it been established that the rigorous conditions set
Illegal Drugs Special Operation Unit (SAIDSOU) in Caloocan forth in paragraph (b) of Section 5, Rule 113 have been
City, alighted from his motorcycle and approached the complied with, i.e., that an offense had in fact just been
appellant whom he recognized as someone he had previously committed and the arresting officer had personal knowledge
arrested for illegal drug possession. of facts indicating that the appellant had committed it. The
factual circumstances of the case failed to show that PO3 de
Upon seeing PO3 de Leon, appellant tried to escape but was Leon had personal knowledge that a crime had been
quickly apprehended with the help of a tricycle driver. Despite indisputably committed by the appellant. It is not enough that
appellant’s attempts to resist arrest, PO3 de Leon was able to PO3 de Leon had reasonable ground to believe that appellant
board appellant onto his motorcycle and confiscate the plastic had just committed a crime; a crime must in fact have been
sachet of shabu in his possession. When arraigned, appellant, committed first, which does not obtain in this case.
assisted by counsel de oficio, entered a plea of not guilty to
the offense charged. Without the overt act that would pin liability against appellant,
it is therefore clear that PO3 de Leon was merely impelled to
RTC ruled against Villareal. Moreover, the RTC found the apprehend appellant on account of the latter’s previous
plain view doctrine to be applicable, as the confiscated item charge for the same offense.
was in plain view of PO3 de Leon at the place and time of the
arrest. However, a previous arrest or existing criminal record, even
for the same offense, will not suffice to satisfy the exacting
CA affirmed the RTC ruling. requirements provided under Section 5, Rule 113 in order to
justify a lawful warrantless arrest. “Personal knowledge” of the
arresting officer that a crime had in fact just been committed
ISSUE: Whether or not there was a valid warrantless arrest is required. To interpret “personal knowledge” as referring to
based on the police officers personal knowledge of the a person’s reputation or past criminal citations would create a
criminal record of the appellant. dangerous precedent and unnecessarily stretch the authority
and power of police officers to effect warrantless arrests
based solely on knowledge of a person’s previous criminal
HELD: No, there was no valid warrantless arrest. infractions, rendering nugatory the rigorous requisites laid out
under Section 5.
Section 5, Rule 113 of the Revised Rules of Criminal
Procedure lays down the basic rules on lawful warrantless In fine, appellant’s acts of walking along the street and holding
arrests, either by a peace officer or a private person. something in his hands, even if they appeared to be dubious,
coupled with his previous criminal charge for the same
For the warrantless arrest under paragraph (a) of Section 5 to offense, are not by themselves sufficient to incite suspicion of
operate, two elements must concur: (1) the person to be criminal activity or to create probable cause enough to justify
arrested must execute an overt act indicating that he has just a warrantless arrest under Section 5. “Probable cause” has
committed, is actually committing, or is attempting to commit been understood to mean a reasonable ground of suspicion
a crime; and (2) such overt act is done in the presence or supported by circumstances sufficiently strong in themselves
within the view of the arresting officer. On the other hand, to warrant a cautious man’s belief that the person accused is
paragraph (b) of Section 5 requires for its application that at guilty of the offense with which he is charged. Specifically with
the time of the arrest, an offense had in fact just been respect to arrests, it is such facts and circumstances which
committed and the arresting officer had personal knowledge would lead a reasonably discreet and prudent man to believe
of facts indicating that the appellant had committed it. that an offense has been committed by the person sought to
be arrested, which clearly do not obtain in appellant’s case.
In both instances, the officer’s personal knowledge of the fact
of the commission of an offense is absolutely required. Under Thus, while it is true that the legality of an arrest depends upon
paragraph (a), the officer himself witnesses the crime while the reasonable discretion of the officer or functionary to whom
under paragraph (b), he knows for a fact that a crime has just the law at the moment leaves the decision to characterize the
been committed. nature of the act or deed of the person for the urgent purpose
of suspending his liberty, it cannot be arbitrarily or capriciously
A punctilious assessment of the factual backdrop of this case exercised without unduly compromising a citizen’s
shows that there could have been no lawful warrantless arrest. constitutionally-guaranteed right to liberty.
Consequently, there being no lawful warrantless arrest, the
The Court finds it inconceivable how PO3 de Leon, even with shabu purportedly seized from appellant is rendered
his presumably perfect vision, would be able to identify with inadmissible in evidence for being the proverbial fruit of the
reasonable accuracy, from a distance of about 8 to 10 meters poisonous tree. As the confiscated shabu is the very corpus
and while simultaneously driving a motorcycle, a negligible delicti of the crime charged, appellant must be acquitted and
and minuscule amount of powdery substance (0.03 gram) exonerated from all criminal liability.
inside the plastic sachet allegedly held by appellant. That he
had previously effected numerous arrests, all involving shabu,
is insufficient to create a conclusion that what he purportedly
saw in appellant’s hands was indeed shabu. Absent any other
circumstance upon which to anchor a lawful arrest, no other
overt act could be properly attributed to appellant as to rouse
suspicion in the mind of PO3 de Leon that he (appellant) had
just committed, was committing, or was about to commit a
crime, for the acts per se of walking along the street and
examining something in one’s hands cannot in any way be

Page 52 of 186
8. JEFFREY MIGUEL y REMEGIO vs. PEOPLE OF THE HELD: The Court ruled that the Bantay Bayan operatives
PHILIPPINES (G.R. No. 227038; July 31, 2017) conducted an illegal search on the person of petitioner.
Consequently, the marijuana purportedly seized from him on
account of such search is rendered inadmissible in evidence.
FACTS: An Information was filed before the RTC charging Hence, Miguel is acquitted.
Jeffrey Miguel of illegal possession of dangerous drugs,
defined and penalized under Section 11, Article II of Republic The Court is convinced that the acts of the Bantay Bayan - or
Act No. (RA) 9165 (Dangerous Drugs Act) any barangay-based or other volunteer organizations in the
nature of watch groups - relating to the preservation of peace
The prosecution alleged that at around 12:45 in the morning and order in their respective areas have the color of a state-
of May 24, 2010, a Bantay Bayan operative of Barangay San related function. As such, they should be deemed as law
Antonio Village, Makati City named Reynaldo Bahoyo was enforcement authorities for the purpose of applying the Bill of
doing his rounds when he purportedly received a report of a Rights under Article III of the 1987 Constitution to them.
man showing off his private parts at Kaong Street. BB Bahoyo
and fellow Bantay Bayan operative Mark Anthony Velasquez The law identifies three (3) instances when warrantless
then went to the said street and saw a visibly intoxicated arrests may be lawfully effected. These are: (a) an arrest of a
person, which they later identified as herein petitioner, suspect in flagrante delicto; (b) an arrest of a suspect where,
urinating and displaying his private parts while standing in based on personal knowledge of the arresting officer, there is
front of a gate enclosing an empty lot. probable cause that said suspect was the perpetrator of a
crime which had just been committed; and (c) an arrest of a
Bahoyo and Velasquez approached petitioner and asked him prisoner who has escaped from custody serving final
where he lived, and the latter answered Kaong Street. Bahoyo judgment or temporarily confined during the pendency of his
then said that he also lived in the same street but petitioner case or has escaped while being transferred from one
looked unfamiliar to him, so he asked for an identification card, confinement to another.
but petitioner failed to produce one. Velasquez then repeated
the request for an identification card, but instead, petitioner In warrantless arrests made pursuant to Section 5 (a), Rule
emptied his pockets, revealing a pack of cigarettes containing 113, two (2) elements must concur, namely: (a) the person to
one (1) stick of cigarette and two (2) pieces of rolled paper be arrested must execute an overt act indicating that he has
containing dried marijuana leaves, among others. This just committed, is actually committing, or is attempting to
prompted BB Bahoyo and Velasquez to seize the foregoing commit a crime; and (b) such overt act is done in the presence
items, take petitioner to the police station, and turn him, as or within the view of the arresting officer. On the other hand,
well as the seized items, over to SP03 Rafael Castillo. Section 5 (b), Rule 113 requires for its application that at the
time of the arrest, an offense had in fact just been committed
SP03 Castillo then inventoried, marked, and photographed and the arresting officer had personal knowledge of facts
the seized items, all in the presence of Bahoyo and indicating that the accused had committed it.
Velasquez, and thereafter, prepared an inventory report and a
request for qualitative examination of the seized two pieces of In both instances, the officer's personal knowledge of the fact
rolled paper and for petitioner to undergo drug testing. After of the commission of an offense is essential. Under Section 5
examination, it was confirmed that the aforesaid rolled paper (a), Rule 113 of the Revised Rules of Criminal Procedure, the
contained marijuana and that petitioner was positive for the officer himself witnesses the crime; while in Section 5 (b) of
presence of methamphetamine but negative for THC- the same, he knows for a fact that a crime has just been
metabolites, both dangerous drugs committed.

Petitioner alleged that he was just urinating in front of his The Court simply finds highly implausible the prosecution's
workplace when two Bantay Bayan operatives approached claim that a valid warrantless arrest was made on petitioner
and asked him where he lived. Upon responding that he lived on account of the alleged public display of his private parts
in Kaong Street, Bahoyo and Velasquez then frisked him, took because if it was indeed the case, then the proper charge
away his belongings, and thereafter, handcuffed and brought should have been filed against him. However, records are
him to the barangay hall. He was then detained for about an bereft of any showing that such charge was filed aside from
hour before being taken to the Ospital ng Makati and to the instant criminal charge for illegal possession of dangerous
another office where a bald police officer questioned him. drugs - thereby strengthening the view that no prior arrest was
Thereafter, he was taken back to the barangay hall where they made on petitioner which led to a search incidental thereto. As
showed him two sticks of marijuana joints allegedly recovered stressed earlier, there must first be a lawful arrest before a
from him. search can be made and that such process cannot be
reversed.
RTC found him guilty. It also found the warrantless arrest of
Miguel valid as petitioner was scandalously showing his
private parts at the time of his arrest. Therefore, the resultant
search incidental to such arrest which yielded the seized
marijuana in petitioner's possession was also lawful.

The CA ruled that the search made on petitioner which yielded


the seized marijuana was validly made as it was done
incidental to his arrest for exhibiting his private parts on public.
As such, the said seized marijuana is admissible in evidence
and, thus, sufficient to convict him for the crime charged. The
CA likewise held that the rule on chain of custody was duly
complied with and, thus, the integrity and evidentiary value of
the seized drugs were not compromised.

ISSUE: Whether or not the warrantless search and


subsequent arrest was valid?

Page 53 of 186
9. David Leon Riley vs California (2014) using "cloud computing" is not even "on the arrestee's
person." Nonetheless, the Court held that some warrantless
searches of cell phones might be permitted in an emergency:
FACTS: David Leon Riley belonged to the Lincoln Park gang when the government's interests are so compelling that a
of San Diego, California. David Leon Riley was pulled over on search would be reasonable.
August 22, 2009, for expired registrations tags. During the
stop, the San Diego Police Officer also found that Riley was Roberts wrote that it fails the warrantless search test
driving with a suspended driver's license. The San Diego established in Chimel v. California:
Police Department's policy at the time was to tow and impound
a vehicle after stopping a driver with a suspended license in “Digital data stored on a cell phone cannot itself
order to prevent the driver from driving again. Additionally, be used as a weapon to harm an arresting
department policy required the officers to perform an inventory officer or to effectuate the arrestee's escape.
search of the vehicle, which in this case led to the discovery Law enforcement officers remain free to
of two handguns under the hood of the vehicle. Later ballistic examine the physical aspects of a phone to
testing would confirm that the handguns were the weapons ensure that it will not be used as a weapon--say,
used in a gangland murder on August 2, 2009, for which Riley to determine whether there is a razor blade
had been a suspect. Although eyewitnesses to the shooting hidden between the phone and its case. Once
claimed that Riley could have been one of the shooters, they an officer has secured a phone and eliminated
declined to give a definitive positive identification of Riley as any potential physical threats, however, data on
one of the shooters. However, this was not known by Officer the phone can endanger no one.”
Dunnigan at the time of Riley's traffic stop. Because of the
discovery of the concealed and loaded handguns, along with Although possible evidence stored on a phone may be
gang paraphernalia, during the vehicle search, police placed destroyed with either remote wiping or data encryption,
Riley under arrest and searched his cell phone without a Roberts noted that is "the ordinary operation of a phone's
warrant. The cell phone search yielded information indicating security features, apart from any active attempt by a
that Riley was a member of the Lincoln Park gang; evidence defendant or his associates to conceal or destroy evidence
included pictures, cell phone contacts, text messages, and upon arrest." He then argues that a warrantless search is
video clips. Included in the photos was a picture of a different unlikely to make much of a difference:
vehicle that Riley owned, which was also the vehicle involved
in the August 2nd gang shooting. Based in part on the pictures “Cell phone data would be vulnerable to remote
and videos recovered from the cell phone, police charged wiping from the time an individual anticipates
Riley in connection with the gang shooting and sought an arrest to the time any eventual search of the
enhancement based on Riley's gang membership. phone is completed... likewise, an officer who
seizes a phone in an unlocked state might not
Riley's lawyer moved to suppress all the evidence the officers be able to begin his search in the short time
had obtained during the search of his cell phone on the remaining before the phone locks and data
grounds that the search violated his Fourth Amendment rights. becomes encrypted.”
The trial court rejected this argument and held that the search
was legitimate under the SITA doctrine. At trial, a gang expert Roberts then cites several common examples to turn off or
testified to Riley's membership in the Lincoln Park gang, the prevent the phone's security features. Furthermore, Roberts
rivalry between the gangs involved, and why the shooting argued that cell phones differ both quantitatively and
could have been gang-related. The jury convicted Riley on all qualitatively from other objects in a person's pocket:
three counts and sentenced to fifteen years to life in prison.
“Modern cell phones are not just another
On appeal, the California Court of Appeal affirmed the technological convenience. With all they contain
judgment based on the recent California Supreme Court and all they may reveal, they hold for many
decision People v. Diaz. In Diaz, the court held that the Fourth Americans “the privacies of life". The fact that
Amendment "search-incident-to-arrest" doctrine permits the technology now allows an individual to carry
police to conduct a full exploratory search of a cell phone such information in his hand does not make the
(even if it is conducted later and at a different location) information any less worthy of the protection for
whenever the phone is found near the suspect at the time of which the Founders fought.”
arrest.
Justice Samuel A. Alito, Jr. wrote an opinion concurring in part
and concurring in the judgment in which he expressed doubt
ISSUE: Whether or not the evidence admitted at trial from that the warrantless search exception following an arrest
Riley's cell phone discovered through a search that violated exists for the sole or primary purposes of protecting officer
his Fourth Amendment right to be free from unreasonable safety and preserving evidence. In light of the privacy interests
searches? at stake, however, he agreed that the majority's conclusion
was the best solution. Justice Alito also suggested that the
legislature enact laws that draw reasonable distinctions
HELD: Yes. Chief Justice John G. Roberts, Jr. wrote the regarding when and what information within a phone can be
opinion for the unanimous Court. The Court held that the reasonably searched following an arrest.
warrantless search exception following an arrest exists for the
purposes of protecting officer safety and preserving evidence,
neither of which is at issue in the search of digital data. The
digital data cannot be used as a weapon to harm an arresting
officer, and police officers have the ability to preserve
evidence while awaiting a warrant by disconnecting the phone
from the network and placing the phone in a "Faraday bag."
The Court characterized cell phones as minicomputers filled
with massive amounts of private information, which
distinguished them from the traditional items that can be
seized from an arrestee's person, such as a wallet. The Court
also held that information accessible via the phone but stored

Page 54 of 186
10. BIRCHFIELD VS NORTH DAKOTA (2016) produce a sample that can be preserved and used to obtain
further information beyond the subject’s blood alcohol level at
the time of the test. The Court also determined that
FACTS: Birchfield was a consolidation of three cases: criminalizing refusal to submit to a breath test is designed to
Birchfield v. North Dakota, Bernard v. Minnesota, and Beylund serve the government’s interest in preventing drunk driving,
v. Levi. Birchfield. which is greater than merely keeping currently drunk drivers
off the roads, and does so better than other alternatives.
Danny Birchfield drove into a ditch in Morton County, North However, the same rationale did not apply to criminalizing
Dakota. When police arrived on the scene, they believed refusal to submit to a blood test because of the greater degree
Birchfield was intoxicated. Birchfield failed both the field of intrusion and the available alternative of the breath test.
sobriety tests and the breath test. He was arrested, but he
refused to consent to a chemical test. Birchfield was charged The Court ruled in favor of Birchfield who was prosecuted for
with a misdemeanor for refusing to consent to a chemical test refusing a warrantless blood draw and ruled against Bernard
in violation of state law. He moved to dismiss the charge and who refused a warrantless breath test. Beylund, on the other
claimed that the state law violated his Fourth Amendment right hand consented to a blood test after police advised him that
against unreasonable search and seizure. he was required to do. The court therefore remanded
Beylund's case back to the state court "to reevaluate
In a similar case, police were called to the South St. Paul boat Beylund's consent given the partial inaccuracy of the officer's
launch where three men were attempting to pull their boat out advisory." The Supreme Court of North Dakota court
of the water and onto their truck. William Robert Bernard, Jr., subsequently avoided the issue by holding that, even
admitted he had been drinking and had the truck keys in his assuming the consent was involuntary, the Exclusionary Rule
hands, but he denied driving the truck and refused to perform does not apply in the administrative hearing context and thus
a field sobriety test. He was arrested on suspicion of driving affirmed suspension of his license for testing over the
while impaired (DWI) and taken to the police station, where he prohibited level set forth in the implied consent / administrative
refused to consent to a chemical test in violation of Minnesota license suspension statute.
state law. Bernard was charged with two counts of first-degree
test refusal pursuant to state law. In her partial concurrence and partial dissent, Justice Sonia
Sotomayor wrote that the Fourth Amendment’s prohibition
In a separate incident, Steve Beylund consented to a blood against warrantless searches should apply to breath tests
alcohol to test to confirm he was driving under the influence unless exigent circumstances justify one in a particular case.
after being informed it was a criminal offense in North Dakota In establishing exceptions to the warrant requirement, the
to refuse a blood alcohol test. The test confirmed he was over Court has routinely examined whether a legitimate
the legal limit, and Beylund was charged with driving under the government interest justified the search in light of the
influence. Beylund underwent a blood alcohol test consistent individual’s privacy interest and whether that determination
with North Dakota's implied consent law and challenged the should be made based on a case-by-case analysis or a
constitutionality of that law after an administrative hearing categorical rule. Based on this analysis, Justice Sotomayor
based on the test results led to the revocation of his license. argued that a categorical rule allowing warrantless breath
tests incident to arrest was unnecessary to protect the
All three men challenged the state statutes criminalizing government interest of preventing drunk driving because at
refusal to submit to a chemical test and argued that the that point the driver is off the road and a warrant could be
statutes violated their Fourth Amendment rights to be free obtained if necessary. Justice Ruth Bader Ginsburg joined in
from unreasonable searches and seizures when there was no the opinion concurring in part and dissenting in part.
probable cause that would support a warrant for the test. Both
the Supreme Court of Minnesota and the Supreme Court of In his separate opinion concurring in the judgment in part and
North Dakota determined that criminalizing the refusal to dissenting in part, Justice Clarence Thomas wrote that the
submit to a chemical test was reasonable under the Fourth search-incident-to-arrest exception to the Fourth
Amendment. Amendment’s warrant requirement should apply categorically
to all blood alcohol tests, including blood tests. By drawing an
arbitrary line between blood tests and breath tests, the
ISSUE: Is warrantless alcohol testing incident to drunk driving majority destabilized the law of exceptions to the warrant
arrests to determine blood alcohol content a violation of the requirement and made the jobs of both police officers and
Fourth Amendment? lower courts more difficult.

HELD: The Court held that both breath tests and blood tests
constitute a search under the Fourth Amendment.

A state statute may not criminalize the refusal to submit to a


blood test in the absence of a warrant because, while the
Fourth Amendment allows for warrantless breath tests
incident to an arrest for drunk driving, warrantless blood tests
incident to an arrest violate the Fourth Amendment.

Justice Samuel A. Alito, Jr. delivered the opinion for the 7-1
majority. The Court held that warrantless breath tests are
permissible under the search incident to arrest exception to
the Fourth Amendment’s warrant requirement because they
do not implicate significant privacy concerns. They involve
minimal physical intrusion to capture something that is
routinely exposed to the public, reveal a limited amount of
information, and do not enhance any embarrassment beyond
what the arrest itself causes. Blood tests, however, implicate
privacy interests because they are much more physically
invasive, they require the piercing of the skin, and they

Page 55 of 186
11. SOCIAL JUSTICE SOCIETY (SJS) VS. DANGEROUS a person’s constitutional right against unreasonable searches
DRUGS BOARD AND PHILIPPINE DRUG is also breached by said provisions.
ENFORCEMENT AGENCY (G.R. No. 157870,
November 03, 2008) (Pimentel v. COMELEC | G.R. No. 16158)
On Dec. 23, 2003, the COMELEC issued Resolution No.
6486, prescribing the rules and regulations for the mandatory
FACTS: In its Petition for Prohibition under Rule 65, petitioner drug testing of candidates for public office in connection with
Social Justice Society (SJS), a registered political party, seeks the May 2004 elections. Pimentel claims that Sec. 36 (g) of
to prohibit the Dangerous Drugs Board (DDB) and the RA 9165 and COMELEC Resolution No. 6486 illegally impose
Philippine Drug Enforcement Agency (PDEA) from enforcing an additional qualification on candidates for senator. He points
paragraphs (c), (d), (f), and (g) of Sec. 36 of RA 9165 on the out that, subject to the provisions on nuisance candidates, a
ground that they are constitutionally infirm. candidate for senator needs only to meet the qualifications laid
down in Sec. 3, Art. VI of the Constitution, to wit: (1)
In these kindred petitions, the constitutionality of Section 36 of citizenship, (2) voter registration, (3) literacy, (4) age, and (5)
Republic Act No. (RA) 9165, otherwise known as the residency. Beyond these stated qualification requirements,
Comprehensive Dangerous Drugs Act of 2002, insofar as it candidates for senator need not possess any other
requires mandatory drug testing of candidates for public office, qualification to run for senator and be voted upon and elected
students of secondary and tertiary schools, officers and as member of the Senate. The Congress cannot validly
employees of public and private offices, and persons charged amend or otherwise modify these qualification standards, as it
before the prosecutor’s office with certain offenses, among cannot disregard, evade, or weaken the force of a
other personalities, is put in issue. As far as pertinent, the constitutional mandate, or alter or enlarge the Constitution.
challenged section reads as follows:
(SJS v. DDM & PDEA | G.R. 157870)
SEC. 36. Authorized Drug Testing.—Authorized In its Petition for Prohibition under Rule 65, petitioner Social
drug testing shall be done by any government Justice Society (SJS), a registered political party, seeks to
forensic laboratories or by any of the drug prohibit the Dangerous Drugs Board (DDB) and the Philippine
testing laboratories accredited and monitored Drug Enforcement Agency (PDEA) from enforcing paragraphs
by the DOH to safeguard the quality of the test (c), (d), (f), and (g) of Sec. 36 of RA 9165 on the ground that
results. x x x The drug testing shall employ, they are constitutionally infirm. For one, the provisions
among others, two (2) testing methods, the constitute undue delegation of legislative power when they
screening test which will determine the positive give unbridled discretion to schools and employers to
result as well as the type of drug used and the determine the manner of drug testing. For another, the
confirmatory test which will confirm a positive provisions trench in the equal protection clause inasmuch as
screening test. x x x The following shall be they can be used to harass a student or an employee deemed
subjected to undergo drug testing: undesirable. And for a third, a person’s constitutional right
against unreasonable searches is also breached by said
(c) Students of secondary and tertiary provisions.
schools.—Students of secondary and tertiary
schools shall, pursuant to the related rules and (Atty. Laserna v. DDB & PDEA | G.R. 158633)
regulations as contained in the school’s student Petitioner Atty. Manuel J. Laserna, Jr., as citizen and
handbook and with notice to the parents, taxpayer, also seeks in his Petition for Certiorari and
undergo a random drug testing x x x; Prohibition under Rule 65 that Sec. 36(c), (d), (f), and (g) of
RA 9165 be struck down as unconstitutional for infringing on
(d) Officers and employees of public and private the constitutional right to privacy, the right against
offices.—Officers and employees of public and unreasonable search and seizure, and the right against self-
private offices, whether domestic or overseas, incrimination, and for being contrary to the due process and
shall be subjected to undergo a random drug equal protection guarantees.
test as contained in the company’s work rules
and regulations, x x x for purposes of reducing
the risk in the workplace. Any officer or ISSUE: Whether or not paragraphs (c), (d), (f), and (g) of Sec.
employee found positive for use of dangerous 36, RA 9165 unconstitutional? Specifically, do these
drugs shall be dealt with administratively which paragraphs violate the right to privacy, the right against
shall be a ground for suspension or termination, unreasonable searches and seizure, and the equal protection
subject to the provisions of Article 282 of the clause?
Labor Code and pertinent provisions of the Civil
Service Law;
HELD: The Court GRANTED the petition in G.R. No. 161658
(f) All persons charged before the prosecutor’s and declared Sec. 36(g) of RA 9165 and COMELEC
office with a criminal offense having an Resolution No. 6486 as UNCONSTITUTIONAL. It
imposable penalty of imprisonment of not less also PARTIALLY GRANTED the petition in G.R. Nos. 157870
than six (6) years and one (1) day shall undergo and 158633 by
a mandatory drug test; declaring Sec.36(c) and (d) of RA165 CONSTITUTIONAL,
but declaring its Sec. 36(f) UNCONSTITUTIONAL. The Court
(g) All candidates for public office whether thus permanently enjoined all the concerned agencies from
appointed or elected both in the national or local implementing Sec. 36(f) and (g) of RA 9165.
government shall undergo a mandatory drug
test. NO, paragraphs (c) and (d) of Sec. 36, RA 9165 are NOT
UNCONSTITUTIONAL; YES, paragraphs (f) thereof is
For one, the provisions constitute undue delegation of UNCONSTITUTIONAL.
legislative power when they give unbridled discretion to
schools and employers to determine the manner of drug As to paragraph (c), covering students of secondary and
testing. For another, the provisions trench in the equal tertiary schools
protection clause inasmuch as they can be used to harass a
student or an employee deemed undesirable. And for a third,

Page 56 of 186
Citing the U.S. cases of Vernonia School District 47J v. Acton mandated intrusion on the individual's privacy interest against
and Board of Education of Independent School District No. 92 the promotion of some compelling state interest. In the
of Pottawatomie County, et al. v. Earls, et al., the Court criminal context, reasonableness requires showing of
deduced and applied the following principles: (1) schools and probable cause to be personally determined by a judge. Given
their administrators stand in loco parentis with respect to their that the drug-testing policy for employees—and students for
students; (2) minor students have contextually fewer rights that matter—under RA 9165 is in the nature of administrative
than an adult, and are subject to the custody and supervision search needing what was referred to in Vernonia as “swift and
of their parents, guardians, and schools; (3) schools, acting in informal disciplinary procedures,” the probable-cause
loco parentis, have a duty to safeguard the health and well- standard is not required or even practicable. Be that as it may,
being of their students and may adopt such measures as may the review should focus on the reasonableness of the
reasonably be necessary to discharge such duty; and (4) challenged administrative search in question.
schools have the right to impose conditions on applicants for
admission that are fair, just, and non-discriminatory. The first factor to consider in the matter of reasonableness is
the nature of the privacy interest upon which the drug testing,
Guided by Vernonia, supra, and Board of Education, supra, which effects a search within the meaning of Sec. 2, Art. III of
the Court is of the view and so holds that the provisions of RA the Constitution, intrudes. In this case, the office or workplace
9165 requiring mandatory, random, and suspicionless drug serves as the backdrop for the analysis of the privacy
testing of students are constitutional. Indeed, it is within the expectation of the employees and the reasonableness of drug
prerogative of educational institutions to require, as a testing requirement. The employees' privacy interest in an
condition for admission, compliance with reasonable school office is to a large extent circumscribed by the company's work
rules and regulations and policies. To be sure, the right to policies, the collective bargaining agreement, if any, entered
enrol is not absolute; it is subject to fair, reasonable, and into by management and the bargaining unit, and the inherent
equitable requirements. right of the employer to maintain discipline and efficiency in
the workplace. Their privacy expectation in a regulated office
The Court is of the view and so holds that the provisions of RA environment is, in fine, reduced; and a degree of impingement
9165(c) requiring mandatory, random, and suspicionless drug upon such privacy has been upheld.
testing of students are constitutional. Indeed, it is within the
prerogative of educational institutions to require, as a Just as defining as the first factor is the character of the
condition for admission, compliance with reasonable school intrusion authorized by the challenged law. Reduced to a
rules and regulations and policies. To be sure, the right to question form, is the scope of the search or intrusion clearly
enroll is not absolute; it is subject to fair, reasonable, and set forth, or, as formulated in Ople v. Torres, is the enabling
equitable requirements. A random drug testing of students in law authorizing a search "narrowly drawn" or "narrowly
secondary and tertiary schools is not only acceptable, but may focused"?
even be necessary if the safety and interest of the student
population, doubtless a legitimate concern of the The poser should be answered in the affirmative. For one,
government, are to be promoted and protected. Sec. 36 of RA 9165 and its implementing rules and regulations
(IRR), as couched, contain provisions specifically directed
Just as in the case of secondary and tertiary level students, towards preventing a situation that would unduly embarrass
the mandatory but random drug test prescribed by Sec. 36 of the employees or place them under a humiliating experience.
RA 9165(d) for officers and employees of public and private While every officer and employee in a private establishment is
offices is justifiable, albeit not exactly for the same reason. The under the law deemed forewarned that he or she may be a
Court notes in this regard that petitioner SJS, other than possible subject of a drug test, nobody is really singled out in
saying that “subjecting almost everybody to drug testing, advance for drug testing. The goal is to discourage drug use
without probable cause, is unreasonable, an unwarranted by not telling in advance anyone when and who is to be tested.
intrusion of the individual right to privacy,” has failed to show And as may be observed, Sec. 36(d) of RA 9165 itself
how the mandatory, random, and suspicionless drug testing prescribes what, in Ople, is a narrowing ingredient by
under Sec. 36(c) and (d) of RA 9165 violates the right to providing that the employees concerned shall be subjected to
privacy and constitutes unlawful and/or unconsented search “random drug test as contained in the company’s work rules
under Art. III, Secs. 1 and 2 of the Constitution. Petitioner and regulations x x x for purposes of reducing the risk in the
Laserna’s lament is just as simplistic, sweeping, and work place.”
gratuitous and does not merit serious consideration.
For another, the random drug testing shall be undertaken
The essence of privacy is the right to be left alone. In context, under conditions calculated to protect as much as possible the
the right to privacy means the right to be free from employee's privacy and dignity. As to the mechanics of the
unwarranted exploitation of one’s person or from intrusion into test, the law specifies that the procedure shall employ two
one’s private activities in such a way as to cause humiliation testing methods, i.e., the screening test and the confirmatory
to a person’s ordinary sensibilities; and while there has been test, doubtless to ensure as much as possible the
general agreement as to the basic function of the guarantee trustworthiness of the results. But the more important
against unwarranted search, “translation of the abstract consideration lies in the fact that the test shall be conducted
prohibition against ‘unreasonable searches and seizures’ into by trained professionals in access-controlled laboratories
workable broad guidelines for the decision of particular cases monitored by the Department of Health (DOH) to safeguard
is a difficult task,” to borrow from C. Camara v. Municipal against results tampering and to ensure an accurate chain of
Court. Authorities are agreed though that the right to privacy custody. In addition, the IRR issued by the DOH provides that
yields to certain paramount rights of the public and defers to access to the drug results shall be on the “need to know”
the state’s exercise of police power. basis; that the “drug test result and the records shall be [kept]
confidential subject to the usual accepted practices to protect
As to paragraph (d), covering officers and employees of public the confidentiality of the test results.” Notably, RA 9165 does
and private offices not oblige the employer concerned to report to the prosecuting
agencies any information or evidence relating to the violation
As the warrantless clause of Sec. 2, Art III of the Constitution of the Comprehensive Dangerous Drugs Act received as a
is couched and as has been held, “reasonableness” is the result of the operation of the drug testing. All told, therefore,
touchstone of the validity of a government search or intrusion. the intrusion into the employees’ privacy, under RA 9165, is
And whether a search at issue hews to the reasonableness accompanied by proper safeguards, particularly against
standard is judged by the balancing of the government-

Page 57 of 186
embarrassing leakages of test results, and is relatively 12. JAIME D. DELA CRUZ vs. PEOPLE OF THE
minimal. PHILIPPINES (G.R. No. 200748; July 23, 2014)
SERENO, CJ
Taking into account the foregoing factors, i.e., the reduced
expectation of privacy on the part of the employees, the
compelling state concern likely to be met by the search, and Facts: Petitioner Jaime D. dela Cruz was charged with
the well-defined limits set forth in the law to properly guide violation of Section 15, Article II of Republic Act No. (R.A.)
authorities in the conduct of the random testing, we hold that 9165, by the Graft Investigation and Prosecution Officer of the
the challenged drug test requirement is, under the limited Office of the Ombudsman – Visayas.
context of the case, reasonable and, ergo, constitutional.
The prosecution reveals that the agents and special
Like their counterparts in the private sector, government investigators of the National Bureau of Investigation, Central
officials and employees also labor under reasonable Visayas Regional Office (NBI-CEVRO) or simply NBI,
supervision and restrictions imposed by the Civil Service law received a Complaint from Corazon Absin (Corazon) and
and other laws on public officers, all enacted to promote a high Charito Escobido (Charito). The complainants claimed that
standard of ethics in the public service. And if RA 9165 Ariel Escobido (Ariel), the live-in partner of Corazon and son
passes the norm of reasonableness for private employees, the of Charito, was picked up by several unknown male persons
more reason that it should pass the test for civil servants, who, believed to be police officers for allegedly selling drugs. In the
by constitutional command, are required to be accountable at police office, they met "James" who demanded from them
all times to the people and to serve them with utmost ₱100,000, later lowered to ₱40,000, in exchange for the
responsibility and efficiency. release of Ariel. After the meeting, the complainants
proceeded to the NBI-CEVRO to file a complaint and narrate
As to paragraph (f), covering persons charged before the the circumstances of the meeting to the authorities. While at
prosecutor’s office with a crime with an imposable penalty of the NBI-CEVRO, Charito even received calls supposedly from
imprisonment of not less than 6 years and 1 day "James" instructing her to bring the money as soon as
possible.
Paragraph (f) of RA 9165 was declared unconstitutional by the
Court. Unlike the situation covered by Sec. 36(c) and (d) of RA The special investigators at the NBI-CEVRO verified the text
9165, the Court finds no valid justification for mandatory drug messages received by the complainants. A team was
testing for persons accused of crimes. In the case of students, immediately formed to implement an entrapment operation,
the constitutional viability of the mandatory, random, and which took place inside a Jollibee branch at the corner of Gen.
suspicionless drug testing for students emanates primarily Maxilom and Gorordo Avenues, Cebu City. Petitioner was
from the waiver by the students of their right to privacy when later brought to the forensic laboratory of the NBI-CEVRO
they seek entry to the school, and from their voluntarily where forensic examination was done by forensic chemist
submitting their persons to the parental authority of school Rommel Paglinawan. Petitioner was required to submit his
authorities. In the case of private and public employees, the urine for drug testing. It later yielded a positive result for
constitutional soundness of the mandatory, random, and presence of dangerous drugs as indicated in the confirmatory
suspicionless drug testing proceeds from the reasonableness test result labeled as Toxicology (Dangerous Drugs) Report.
of the drug test policy and requirement.
The defense presented petitioner as the lone witness. He
The Court finds the situation entirely different in the case of denied the charges and testified that while eating at the said
persons charged before the public prosecutor’s office with Jollibee branch, he was arrested allegedly for extortion by NBI
criminal offenses punishable with six (6) years and one (1) day agents. When he was at the NBI Office, he was required to
imprisonment. The operative concepts in the mandatory drug extract urine for drug examination, but he refused saying he
testing are “randomness” and “suspicionless.” In the case of wanted it to be done by the (PNP) Crime Laboratory and not
persons charged with a crime before the prosecutor's office, a by the NBI.
mandatory drug testing can never be random or suspicionless.
The ideas of randomness and being suspicionless are The Regional Trial Court (RTC) found the accused guilty
antithetical to their being made defendants in a criminal beyond reasonable doubt of violating Section 15, Article II of
complaint. They are not randomly picked; neither are they R.A. 9165. The CA found the appeal devoid of merit and
beyond suspicion. When persons suspected of committing a affirmed the ruling of the RTC.
crime are charged, they are singled out and are impleaded
against their will. The persons thus charged, by the bare fact
of being haled before the prosecutor’s office and peaceably THE ISSUE: Whether or not the drug test conducted upon the
submitting themselves to drug testing, if that be the case, do petitioner is legal.
not necessarily consent to the procedure, let alone waive their
right to privacy. To impose mandatory drug testing on the
accused is a blatant attempt to harness a medical test as a OUR RULING: No. The drug test in Section 15 does not cover
tool for criminal prosecution, contrary to the stated objectives persons apprehended or arrested for any unlawful act, but
of RA 9165. Drug testing in this case would violate a person’s only for unlawful acts listed under Article II of R.A. 9165.
right to privacy guaranteed under Sec. 2, Art. III of the
Constitution. Worse still, the accused persons are veritably A person apprehended or arrested” cannot literally mean any
forced to incriminate themselves. person apprehended or arrested for any crime. The phrase
must be read in context and understood in consonance with
R.A. No. 9165. Section 15 comprehends persons arrested or
apprehended for unlawful acts listed under Article II of the law.
Hence, a drug test can be made upon persons who are
apprehended or arrested for, among others, the “importation,”
“sale, trading, administration, dispensation, delivery,
distribution and transportation,” “manufacture” and
“possession” of dangerous drugs and/or controlled precursors
and essential chemicals; possession thereof “during parties,
social gatherings or meetings”; being “employees and visitors
of a den, dive or resort”; “maintenance of a den, dive or resort”;

Page 58 of 186
“illegal chemical diversion of controlled precursors and warrant of arrest shall issue except upon probable cause to be
essential chemicals”; “manufacture or delivery” or determined personally by the judge after examination under
“possession” of equipment, instrument, apparatus, and other oath or affirmation of the complainant and the witnesses he
paraphernalia for dangerous drugs and/or controlled may produce, and particularly describing the place to be
precursors and essential chemicals; possession of dangerous searched and the persons or things to be seized. Section 17.
drugs “during parties, social gatherings or meetings”; No person shall be compelled to be a witness against himself.
“unnecessary” or “unlawful” prescription thereof; “cultivation or In the face of these constitutional guarantees, we cannot
culture of plants classified as dangerous drugs or are sources condone drug testing of all arrested persons regardless of the
thereof”; and “maintenance and keeping of original records of crime or offense for which the arrest is being made. Dela Cruz
transactions on dangerous drugs and/or controlled precursors vs. People, 730 SCRA 655, G.R. No. 200748 July 23, 2014
and essential chemicals.” To make the provision applicable to
all persons arrested or apprehended for any crime not listed
under Article II is tantamount to unduly expanding its meaning.
Note that accused appellant here was arrested in the alleged
act of extortion.

Making the phrase “a person apprehended or arrested” in


Section 15 applicable to all persons arrested or apprehended
for unlawful acts, not only under R.A. 9165 but for all other
crimes, is tantamount to a mandatory drug testing of all
persons apprehended or arrested for any crime. To
overextend the application of this provision would run counter
to our pronouncement in Social Justice Society v. Dangerous
Drugs Board and Philippine Drug Enforcement Agency, 570
SCRA 410 (2008), to wit: x x x [M]andatory drug testing can
never be random and suspicionless. The ideas of randomness
and being suspicionless are antithetical to their being made
defendants in a criminal complaint. They are not randomly
picked; neither are they beyond suspicion. When persons
suspected of committing a crime are charged, they are singled
out and are impleaded against their will. The persons thus
charged, by the bare fact of being haled before the
prosecutor’s office and peaceably submitting themselves to
drug testing, if that be the case, do not necessarily consent to
the procedure, let alone waive their right to privacy. To impose
mandatory drug testing on the accused is a blatant attempt to
harness a medical test as a tool for criminal prosecution,
contrary to the stated objectives of RA 6195. Drug testing in
this case would violate a person’s right to privacy guaranteed
under Sec. 2, Art. III of the Constitution. Worse still, the
accused persons are veritably forced to incriminate
themselves.

In the Gutang v. People, 335 SCRA 479 (2000) case, the


Court clarified that “what the Constitution prohibits is the use
of physical or moral compulsion to extort communication from
the accused, but not an inclusion of his body in evidence,
when it may be material.” The situation in Gutang was
categorized as falling among the exemptions under the
freedom from testimonial compulsion since what was sought
to be examined came from the body of the accused. The Court
said: This was a mechanical act the accused was made to
undergo which was not meant to unearth undisclosed facts but
to ascertain physical attributes determinable by simple
observation. In fact, the record shows that petitioner and his
co-accused were not compelled to give samples of their urine
but they in fact voluntarily gave the same when they were
requested to undergo a drug test. Assuming arguendo that the
urine samples taken from the petitioner are inadmissible in
evidence, we agree with the trial court that the record is replete
with other pieces of credible evidence including the testimonial
evidence of the prosecution which point to the culpability of
the petitioner for the crimes charged.

It is incontrovertible that petitioner refused to have his urine


extracted and tested for drugs. He also asked for a lawyer
prior to his urine test. He was adamant in exercising his rights,
but all of his efforts proved futile, because he was still
compelled to submit his urine for drug testing under those
circumstances. The pertinent provisions in Article III of the
Constitution are clear: Section 2. The right of the people to be
secure in their persons, houses, papers, and effects against
unreasonable searches and seizures of whatever nature and
for any purpose shall be inviolable, and no search warrant or

Page 59 of 186
13. KYLLO v. UNITED STATES (CERTIORARI TO THE where the eavesdropping device in question picked up only
UNITED STATES COURT OF APPEALS FOR THE sound waves that reached the exterior of the phone booth to
NINTH CIRCUIT; No. 99-8508. Argued February 20, which it was attached. Reversing that approach would leave
200l-Decided June 11,2001) the homeowner at the mercy of advancing technology-
including imaging technology that could discern all human
activity in the home. Also rejected is the Government's
Suspicious that marijuana was being grown in petitioner contention that the thermal imaging was constitutional
Kyllo's home in a triplex, agents used a thermal-imaging because it did not detect "intimate details." Such an approach
device to scan the triplex to determine if the amount of heat would be wrong in principle because, in the sanctity of the
emanating from it was consistent with the high-intensity lamps home, all details are intimate details. See, e. g., United States
typically used for indoor marijuana growth. The scan showed v. Karo, 468 U. S. 705; Dow Chemical, supra, at 238,
that Kyllo's garage roof and a side wall were relatively hot distinguished. It would also be impractical in application,
compared to the rest of his home and substantially warmer failing to provide a workable accommodation between law
than the neighboring units. Based in part on the thermal enforcement needs and Fourth Amendment interests. See
imaging, a Federal Magistrate Judge issued a warrant to Oliver v. United States, 466 U. S. 170, 181. Pp. 35-40.
search Kyllo's home, where the agents found marijuana
growing. Mter Kyllo was indicted on a federal drug charge, he (d) Since the imaging in this case was an unlawful search, it
unsuccessfully moved to suppress the evidence seized from will remain for the District Court to determine whether, without
his home and then entered a conditional guilty plea. The Ninth the evidence it provided, the search warrant was supported by
Circuit ultimately affirmed, upholding the thermal imaging on probable cause-and if not, whether there is any other basis for
the ground that Kyllo had shown no subjective expectation of supporting admission of that evidence. P. 40. 190 F.3d 1041,
privacy because he had made no attempt to conceal the heat reversed and remanded.
escaping from his home. Even if he had, ruled the court, there
was no objectively reasonable expectation of privacy because
the thermal imager did not expose any intimate details of
Kyllo's life, only amorphous hot spots on his home's exterior.
Held: Where, as here, the Government uses a device that is
not in general public use, to explore details of a private home
that would previously have been unknowable without physical
intrusion, the surveillance is a Fourth Amendment "search,"
and is presumptively unreasonable without a warrant. Pp. 31-
41.

(a) The question whether a warrantless search of a home is


reasonable and hence constitutional must be answered no in
most instances, but the antecedent question whether a Fourth
Amendment "search" has occurred is not so simple. This
Court has approved warrantless visual surveillance of a home,
see California v. Ciraolo, 476 U. S. 207, 213, ruling that visual
observation is no "search" at all, see Dow Chemical Co. v.
United States, 476 U. S. 227, 234-235, 239. In assessing
when a search is not a search, the Court has adapted a
principle first enunciated in Katz v. United States, 389 U. S.
347, 361: A "search" does not occur-even when its object is a
house explicitly protected by the Fourth Amendment-unless
the individual manifested a subjective expectation of privacy
in the searched object, and society is willing to recognize that
expectation as reasonable, see, e. g., California v. Ciraolo,
supra, at 211. Pp. 31-33.

(b) While it may be difficult to refine the Katz test in some


instances, in the case of the search of a home's interior-the
prototypical and hence most commonly litigated area of
protected privacy-there is a ready criterion, with roots deep in
the common law, of the minimal expectation of privacy that
exists, and that is acknowledged to be reasonable. To
withdraw protection of this minimum expectation would be to
permit police technology to erode the privacy guaranteed by
the Fourth Amendment. Thus, obtaining by sense-enhancing
technology any information regarding the home's interior that
could not otherwise have been obtained without physical
"intrusion into a constitutionally protected area," Silverman v.
United States, 365 U. S. 505, 512, constitutes a search-at
least where (as here) the technology in question is not in
general public use. This assures preservation of that degree
of privacy against government that existed when the Fourth
Amendment was adopted. Pp. 33-35.

(c) Based on this criterion, the information obtained by the


thermal imager in this case was the product of a search. The
Court rejects the Government's argument that the thermal
imaging must be upheld because it detected only heat
radiating from the home's external surface. Such a mechanical
interpretation of the Fourth Amendment was rejected in Katz,

Page 60 of 186
14. UNITED STATES v. JONES (Certiorari to the United a reasonable one—is forfeited because it was not
States Court of Appeals for the District of Columbia raised below. P. 12. 615 F. 3d 544, affirmed.
Circuit No. 10–1259. Argued November 8, 2011—
Decided January 23, 2012) Scalia, J., delivered the opinion of the Court, in which
Roberts, C. J., and Kennedy, Thomas, and Sotomayor, JJ.,
joined. Sotomayor, J., filed a concurring opinion. Alito, J., filed
The Government obtained a search warrant permitting it to an opinion concurring in the judgment, in which Ginsburg,
install a Global-Positioning-System (GPS) tracking device on Breyer, and Kagan, JJ., joined.
a vehicle registered to respondent Jones’s wife. The warrant
authorized installation in the District of Columbia and within 10
days, but agents installed the device on the 11th day and in
Maryland. The Government then tracked the vehicle’s
movements for 28 days. It subsequently secured an
indictment of Jones and others on drug trafficking conspiracy
charges. The District Court suppressed the GPS data
obtained while the vehicle was parked at Jones’s residence,
but held the remaining data admissible because Jones had no
reasonable expectation of privacy when the vehicle was on
public streets. Jones was convicted. The D. C. Circuit
reversed, concluding that admission of the evidence obtained
by warrantless use of the GPS device violated the Fourth
Amendment.

Held: The Government’s attachment of the GPS device to the


vehicle, and its use of that device to monitor the vehicle’s
movements, constitutes a search under the Fourth
Amendment. Pp. 3–12.

(a) The Fourth Amendment protects the “right of the


people to be secure in their persons, houses, papers,
and effects, against unreasonable searches and
seizures.” Here, the Government’s physical intrusion
on an “effect” for the purpose of obtaining information
constitutes a “search.” This type of encroachment on
an area enumerated in the Amendment would have
been considered a search within the meaning of the
Amendment at the time it was adopted. Pp. 3–4.

(b) This conclusion is consistent with this Court’s Fourth


Amendment jurisprudence, which until the latter half of
the 20th century was tied to common-law trespass.
Later cases, which have deviated from that exclusively
property-based approach, have applied the analysis of
Justice Harlan’s concurrence in Katz v. United States,
389 U. S. 347 , which said that the Fourth Amendment
protects a person’s “reasonable expectation of
privacy,” id., at 360. Here, the Court need not address
the Government’s contention that Jones had no
“reasonable expectation of privacy,” because Jones’s
Fourth Amendment rights do not rise or fall with the
Katz formulation. At bottom, the Court must “assur[e]
preservation of that degree of privacy against
government that existed when the Fourth Amendment
was adopted.” Kyllo v. United States, 533 U. S. 27 .
Katz did not repudiate the understanding that the
Fourth Amendment embodies a particular concern for
government trespass upon the areas it enumerates.
The Katz reasonable-expectation-of-privacy test has
been added to, but not substituted for, the common-
law trespassory test. See Alderman v. United States,
394 U. S. 165 ; Soldal v. Cook County, 506 U. S. 56 .
United States v. Knotts, 460 U. S. 276 , and United
States v. Karo, 468 U. S. 705 —post-Katz cases
rejecting Fourth Amendment challenges to “beepers,”
electronic tracking devices representing another form
of electronic monitoring—do not foreclose the
conclusion that a search occurred here. New York v.
Class, 475 U. S. 106 , and Oliver v. United States, 466
U. S. 170 , also do not support the Government’s
position. Pp. 4–12.

(c) The Government’s alternative argument—that if the


attachment and use of the device was a search, it was

Page 61 of 186
15. BRICCIO "Ricky" A. POLLO vs. CHAIRPERSON cannot assert any privacy right to a computer assigned to him.
KARINA CONSTANTINO-DAVID, DIRECTOR IV Even assuming that there was no such administrative policy,
RACQUEL DE GUZMAN BUENSALIDA, DIRECTOR the CSC was of the view that the search of petitioner’s
IV LYDIA A. CASTILLO, DIRECTOR III ENGELBERT computer successfully passed the test of reasonableness for
ANTHONY D. UNITE AND THE CIVIL SERVICE warrantless searches in the workplace as enunciated in the
COMMISSION (G.R. No. 181881 October 18, 2011; aforecited authorities. The CSC stressed that it pursued the
VILLARAMA, JR., J.) search in its capacity as government employer and that it was
undertaken in connection with an investigation involving work-
related misconduct, which exempts it from the warrant
FACTS: This case involves a search of office computer requirement under the Constitution. With the matter of
assigned to a government employee who was charged admissibility of the evidence having been resolved, the CSC
administratively and eventually dismissed from the service. then ruled that the totality of evidence adequately supports the
The employee’s personal files stored in the computer were charges of grave misconduct, dishonesty, conduct prejudicial
used by the government employer as evidence of misconduct. to the best interest of the service and violation of R.A. No.
6713 against the petitioner. These grave infractions justified
Petitioner is a former Supervising Personnel Specialist of the petitioner’s dismissal from the service with all its accessory
CSC Regional Office No. IV and also the Officer-in-Charge of penalties.
the Public Assistance and Liaison Division (PALD) under the
"Mamamayan Muna Hindi Mamaya Na" program of the CSC. Squarely raised by the petitioner is the legality of the search
On January 3, 2007 at around 2:30 p.m., an unsigned letter- conducted on his office computer and the copying of his
complaint addressed to respondent CSC Chairperson Karina personal files without his knowledge and consent, alleged as
Constantino-David which was marked "Confidential" and sent a transgression on his constitutional right to privacy.
through a courier service (LBC) from a certain "Alan San
Pascual" of Bagong Silang, Caloocan City, was received by Under the facts obtaining, the search conducted on
the Integrated Records Management Office (IRMO) at the petitioner’s computer was justified at its inception and scope.
CSC Central Office. Following office practice in which Considering the damaging nature of the accusation, the
documents marked "Confidential" are left unopened and Commission had to act fast, if only to arrest or limit any
instead sent to the addressee, the aforesaid letter was given possible adverse consequence or fall-out. Thus, on the same
directly to Chairperson David. date that the complaint was received, a search was forthwith
conducted involving the computer resources in the concerned
Chairperson David immediately formed a team of four regional office. That it was the computers that were subjected
personnel with background in information technology (IT), and to the search was justified since these furnished the easiest
issued a memo directing them to conduct an investigation and means for an employee to encode and store documents.
specifically "to back up all the files in the computers found in Indeed, the computers would be a likely starting point in
the Mamamayan Muna (PALD) and Legal divisions."4 After ferreting out incriminating evidence. Concomitantly, the
some briefing, the team proceeded at once to the CSC-ROIV ephemeral nature of computer files, that is, they could easily
office at Panay Avenue, Quezon City. Upon their arrival be destroyed at a click of a button, necessitated drastic and
thereat around 5:30 p.m., the team informed the officials of the immediate action. Pointedly, to impose the need to comply
CSC-ROIV, respondents Director IV Lydia Castillo (Director with the probable cause requirement would invariably defeat
Castillo) and Director III Engelbert Unite (Director Unite) of the purpose of the wok-related investigation.
Chairperson David’s directive.
Worthy to mention, too, is the fact that the Commission
It was found that most of the files in the 17 diskettes containing effected the warrantless search in an open and transparent
files copied from the computer assigned to and being used by manner. Officials and some employees of the regional office,
the petitioner, numbering about 40 to 42 documents, were who happened to be in the vicinity, were on hand to observe
draft pleadings or letters7 in connection with administrative the process until its completion. In addition, the respondent
cases in the CSC and other tribunals. On the basis of this himself was duly notified, through text messaging, of the
finding, Chairperson David issued the Show-Cause Order8 search and the concomitant retrieval of files from his
dated January 11, 2007, requiring the petitioner, who had computer.
gone on extended leave, to submit his explanation or counter-
affidavit within five days from notice. All in all, the Commission is convinced that the warrantless
search done on computer assigned to Pollo was not, in any
On July 24, 2007, the CSC issued Resolution No. 071420, way, vitiated with unconstitutionality. It was a reasonable
which found petitioner GUILTY of Dishonesty, Grave exercise of the managerial prerogative of the Commission as
Misconduct, Conduct Prejudicial to the Best Interest of the an employer aimed at ensuring its operational effectiveness
Service and Violation of Republic Act 6713. He is meted the and efficiency by going after the work-related misfeasance of
penalty of DISMISSAL FROM THE SERVICE with all its its employees. Consequently, the evidence derived from the
accessory penalties. questioned search are deemed admissible

Petitioner’s claim of violation of his constitutional right to


ISSUE: On the paramount issue of the legality of the search privacy must necessarily fail. His other argument invoking the
conducted on petitioner’s computer, the CSC noted the dearth privacy of communication and correspondence under Section
of jurisprudence relevant to the factual milieu of this case 3(1), Article III of the 1987 Constitution is also untenable
where the government as employer invades the private files considering the recognition accorded to certain legitimate
of an employee stored in the computer assigned to him for his intrusions into the privacy of employees in the government
official use, in the course of initial investigation of possible workplace under the aforecited authorities. We likewise find
misconduct committed by said employee and without the no merit in his contention that O’Connor and Simons are not
latter’s consent or participation. relevant because the present case does not involve a criminal
offense like child pornography. As already mentioned, the
search of petitioner’s computer was justified there being
HELD: The CSC held that petitioner has no reasonable reasonable ground for suspecting that the files stored therein
expectation of privacy with regard to the computer he was would yield incriminating evidence relevant to the investigation
using in the regional office in view of the CSC computer use being conducted by CSC as government employer of such
policy which unequivocally declared that a CSC employee misconduct subject of the anonymous complaint. This

Page 62 of 186
situation clearly falls under the exception to the warrantless CARPIO, J., Separate Concurring Opinion:
requirement in administrative searches defined in O’Connor.
Right to Privacy;
The above case is to be distinguished from the case at bar
because, unlike the former which involved a personal Any private use of a government property, like a government-
computer of a court employee, the computer from which the owned computer, is prohibited by law. Consequently, a
personal files of herein petitioner were retrieved is a government employee cannot expect any privacy when he
government-issued computer, hence government property the uses a government-owned computer because he knows he
use of which the CSC has absolute right to regulate and cannot use the computer for any private purpose. The CSC
monitor. Such relationship of the petitioner with the item regulation declaring a no-privacy expectation on the use of
seized (office computer) and other relevant factors and government-owned computers logically follows from the
circumstances under American Fourth Amendment statutory rule that government-owned property shall be used
jurisprudence, notably the existence of CSC MO 10, S. 2007 “solely” for a public purpose.
on Computer Use Policy, failed to establish that petitioner had
a reasonable expectation of privacy in the office computer
assigned to him.

The right to privacy has been accorded recognition in this


jurisdiction as a facet of the right protected by the guarantee
against unreasonable search and seizure under Section 2,
Article III of the 1987 Constitution, which provides: Sec. 2. The
right of the people to be secure in their persons, houses,
papers, and effects against unreasonable searches and
seizures of whatever nature and for any purpose shall be
inviolable, and no search warrant or warrant of arrest shall
issue except upon probable cause to be determined
personally by the judge after examination under oath or
affirmation of the complainant and the witnesses he may
produce, and particularly describing the place to be searched
and the persons or things to be seized.

The CSC in this case had implemented a policy that put its
employees on notice that they have no expectation of privacy
in anything they create, store, send or receive on the office
computers, and that the CSC may monitor the use of the
computer resources using both automated or human means.
This implies that on-the-spot inspections may be done to
ensure that the computer resources were used only for such
legitimate business purposes.

A search by a government employer of an employee’s office


is justified at inception when there are reasonable grounds for
suspecting that it will turn up evidence that the employee is
guilty of work-related misconduct. Thus, in the 2004 case
decided by the US Court of Appeals Eighth Circuit, it was held
that where a government agency’s computer use policy
prohibited electronic messages with pornographic content and
in addition expressly provided that employees do not have any
personal privacy rights regarding their use of the agency
information systems and technology, the government
employee had no legitimate expectation of privacy as to the
use and contents of his office computer, and therefore
evidence found during warrantless search of the computer
was admissible in prosecution for child pornography. In that
case, the defendant employee’s computer hard drive was first
remotely examined by a computer information technician after
his supervisor received complaints that he was inaccessible
and had copied and distributed non-work-related e-mail
messages throughout the office. When the supervisor
confirmed that defendant had used his computer to access the
prohibited websites, in contravention of the express policy of
the agency, his computer tower and floppy disks were taken
and examined. A formal administrative investigation ensued
and later search warrants were secured by the police
department. The initial remote search of the hard drive of
petitioner’s computer, as well as the subsequent warrantless
searches was held as valid under the O’Connor ruling that a
public employer can investigate work-related misconduct so
long as any search is justified at inception and is reasonably
related in scope to the circumstances that justified it in the first
place.

Page 63 of 186
16. JESSE U. LUCAS vs. JESUS S. LUCAS (G.R. No. have to face has been widely misunderstood and misapplied
190710 June 6, 2011; NACHURA, J.) in this case. A party is confronted by these so-called
procedural aspects during trial, when the parties have
presented their respective evidence. They are matters of
FACTS: Is a prima facie showing necessary before a court evidence that cannot be determined at this initial stage of the
can issue a DNA testing order? In this petition for review on proceedings, when only the petition to establish filiation has
certiorari, we address this question to guide the Bench and the been filed. The CA’s observation that petitioner failed to
Bar in dealing with a relatively new evidentiary tool. establish a prima facie case—the first procedural aspect in a
paternity case—is therefore misplaced. A prima facie case is
Petitioner, Jesse U. Lucas, filed a Petition to Establish built by a party’s evidence and not by mere allegations in the
Illegitimate Filiation (with Motion for the Submission of Parties initiatory pleading.
to DNA Testing)2 before the (RTC) Valenzuela City. Petitioner
narrated that, sometime in 1967, his mother, Elsie Uy (Elsie), Clearly then, it was also not the opportune time to discuss the
migrated to Manila from Davao and stayed with a certain "Ate lack of a prima facie case vis-à-vis the motion for DNA testing
Belen (Belen)" who worked in a prominent nightspot in Manila. since no evidence has, as yet, been presented by petitioner.
Elsie would oftentimes accompany Belen to work. On one More essentially, it is premature to discuss whether, under the
occasion, Elsie got acquainted with respondent, Jesus S. circumstances, a DNA testing order is warranted considering
Lucas, at Belen’s workplace, and an intimate relationship that no such order has yet been issued by the trial court. In
developed between the two. Elsie eventually got pregnant fact, the latter has just set the said case for hearing.
and, on March 11, 1969, she gave birth to petitioner, Jesse U.
Lucas. The name of petitioner’s father was not stated in At any rate, the CA’s view that it would be dangerous to allow
petitioner’s certificate of live birth. However, Elsie later on told a DNA testing without corroborative proof is well taken and
petitioner that his father is respondent. On August 1, 1969, deserves the Court’s attention. In light of this observation, we
petitioner was baptized at San Isidro Parish, Taft Avenue, find that there is a need to supplement the Rule on DNA
Pasay City. Respondent allegedly extended financial support Evidence to aid the courts in resolving motions for DNA testing
to Elsie and petitioner for a period of about two years. When order, particularly in paternity and other filiation cases. We,
the relationship of Elsie and respondent ended, Elsie refused thus, address the question of whether a prima facie showing
to accept respondent’s offer of support and decided to raise is necessary before a court can issue a DNA testing order.
petitioner on her own. While petitioner was growing up, Elsie
made several attempts to introduce petitioner to respondent, The Rule on DNA Evidence was enacted to guide the Bench
but all attempts were in vain. and the Bar for the introduction and use of DNA evidence in
the judicial system. It provides the "prescribed parameters on
Respondent was not served with a copy of the petition. the requisite elements for reliability and validity (i.e., the
Nonetheless, respondent learned of the petition to establish proper procedures, protocols, necessary laboratory reports,
filiation. His counsel therefore went to the trial court on August etc.), the possible sources of error, the available objections to
29, 2007 and obtained a copy of the petition. the admission of DNA test results as evidence as well as the
probative value of DNA evidence." It seeks "to ensure that the
The RTC held that the ruling on the grounds relied upon by evidence gathered, using various methods of DNA analysis, is
petitioner for filing the petition is premature considering that a utilized effectively and properly, [and] shall not be misused
full-blown trial has not yet taken place. The court also and/or abused and, more importantly, shall continue to ensure
dismissed respondent’s arguments that there is no basis for that DNA analysis serves justice and protects, rather than
the taking of DNA test, and that jurisprudence is still unsettled prejudice the public."
on the acceptability of DNA evidence. It noted that the new
Rule on DNA Evidence allows the conduct of DNA testing, Not surprisingly, Section 4 of the Rule on DNA Evidence
whether at the court’s instance or upon application of any merely provides for conditions that are aimed to safeguard the
person who has legal interest in the matter in litigation. accuracy and integrity of the DNA testing. Section 4 states:

Respondent filed a Motion for Reconsideration. On SEC. 4. Application for DNA Testing Order. – The
September 25, 2009, the CA decided the petition for certiorari appropriate court may, at any time, either motu
in favor of respondent. proprio or on application of any person who has a
legal interest in the matter in litigation, order a DNA
The CA remarked that petitioner filed the petition to establish testing. Such order shall issue after due hearing and
illegitimate filiation, specifically seeking a DNA testing order to notice to the parties upon a showing of the following:
abbreviate the proceedings
(a) A biological sample exists that is relevant to the
Finally, petitioner asserts that the motion for DNA testing case;
should not be a reason for the dismissal of the petition since it (b) The biological sample: (i) was not previously
is not a legal ground for the dismissal of cases. If the CA subjected to the type of DNA testing now requested;
entertained any doubt as to the propriety of DNA testing, it or (ii) was previously subjected to DNA testing, but
should have simply denied the motion. Petitioner points out the results may require confirmation for good
that Section 4 of the Rule on DNA Evidence does not require reasons;
that there must be a prior proof of filiation before DNA testing (c) The DNA testing uses a scientifically valid
can be ordered. He adds that the CA erroneously relied on the technique;
four significant procedural aspects of a paternity case, as (d) The DNA testing has the scientific potential to
enunciated in Herrera v. Alba. Petitioner avers that these produce new information that is relevant to the
procedural aspects are not applicable at this point of the proper resolution of the case; and
proceedings because they are matters of evidence that should (e) The existence of other factors, if any, which the
be taken up during the trial. court may consider as potentially affecting the
accuracy or integrity of the DNA testing.

HELD: The petition is meritorious. This Rule shall not preclude a DNA testing, without need of a
prior court order, at the behest of any party, including law
The statement in Herrera v. Alba that there are four significant enforcement agencies, before a suit or proceeding is
procedural aspects in a traditional paternity case which parties commenced.

Page 64 of 186
This does not mean, however, that a DNA testing order will be 17. MARYLAND v. KING (Certiorari to the court of
issued as a matter of right if, during the hearing, the said appeals of Maryland No. 12–207. Argued February
conditions are established. 26, 2013—Decided June 3, 2013)

In some states, to warrant the issuance of the DNA testing


order, there must be a show cause hearing wherein the After his 2009 arrest on first- and second-degree assault
applicant must first present sufficient evidence to establish a charges, respondent King was processed through a Wicomico
prima facie case or a reasonable possibility of paternity or County, Maryland, facility, where booking personnel used a
"good cause" for the holding of the test. 36 In these states, a cheek swab to take a DNA sample pursuant to the Maryland
court order for blood testing is considered a "search," which, DNA Collection Act (Act). The swab was matched to an
under their Constitutions (as in ours), must be preceded by a unsolved 2003 rape, and King was charged with that crime.
finding of probable cause in order to be valid. Hence, the He moved to suppress the DNA match, arguing that the Act
requirement of a prima facie case, or reasonable possibility, violated the Fourth Amendment, but the Circuit Court Judge
was imposed in civil actions as a counterpart of a finding of found the law constitutional. King was convicted of rape. The
probable cause. The Supreme Court of Louisiana eloquently Maryland Court of Appeals set aside the conviction, finding
explained — unconstitutional the portions of the Act authorizing DNA
Although a paternity action is civil, not criminal, the collection from felony arrestees.
constitutional prohibition against unreasonable
searches and seizures is still applicable, and a
proper showing of sufficient justification under the Held: When officers make an arrest supported by probable
particular factual circumstances of the case must be cause to hold for a serious offense and bring the suspect to
made before a court may order a compulsory blood the station to be detained in custody, taking and analyzing a
test. Courts in various jurisdictions have differed cheek swab of the arrestee’s DNA is, like fingerprinting and
regarding the kind of procedures which are required, photographing, a legitimate police booking procedure that is
but those jurisdictions have almost universally found reasonable under the Fourth Amendment. Pp. 3–28.
that a preliminary showing must be made before a
court can constitutionally order compulsory blood (a) DNA testing may “significantly improve both the criminal
testing in paternity cases. We agree, and find that, as justice system and police investigative practices,” District
a preliminary matter, before the court may issue an Attorney’s Office for Third Judicial Dist. v. Osborne, 557 U. S.
order for compulsory blood testing, the moving party 52 , by making it “possible to determine whether a biological
must show that there is a reasonable possibility of tissue matches a suspect with near certainty,” id., at 62.
paternity. As explained hereafter, in cases in which Maryland’s Act authorizes law enforcement authorities to
paternity is contested and a party to the action collect DNA samples from, as relevant here, persons charged
refuses to voluntarily undergo a blood test, a show with violent crimes, including first-degree assault. A sample
cause hearing must be held in which the court can may not be added to a database before an individual is
determine whether there is sufficient evidence to arraigned, and it must be destroyed if, e.g., he is not convicted.
establish a prima facie case which warrants issuance Only identity information may be added to the database. Here,
of a court order for blood testing. the officer collected a DNA sample using the common “buccal
swab” procedure, which is quick and painless, requires no
The same condition precedent should be applied in our “surgical intrusio[n] beneath the skin,” Winston v. Lee, 470 U.
jurisdiction to protect the putative father from mere S. 753 , and poses no threat to the arrestee’s “health or
harassment suits. Thus, during the hearing on the motion for safety,” id., at 763. Respondent’s identification as the rapist
DNA testing, the petitioner must present prima facie evidence resulted in part through the operation of the Combined DNA
or establish a reasonable possibility of paternity. Lucas vs. Index System (CODIS), which connects DNA laboratories at
Lucas, 650 SCRA 667, G.R. No. 190710 June 6, 2011 the local, state, and national level, and which standardizes the
points of comparison, i.e., loci, used in DNA analysis. Pp. 3–
7.

(b) The framework for deciding the issue presented is well


established. Using a buccal swab inside a person’s cheek to
obtain a DNA sample is a search under the Fourth
Amendment. And the fact that the intrusion is negligible is of
central relevance to determining whether the search is
reasonable, “the ultimate measure of the constitutionality of a
governmental search,” Vernonia School Dist. 47J v. Acton,
515 U. S. 646 . Because the need for a warrant is greatly
diminished here, where the arrestee was already in valid
police custody for a serious offense supported by probable
cause, the search is analyzed by reference to
“reasonableness, not individualized suspicion,” Samson v.
California, 547 U. S. 843 , n. 4, and reasonableness is
determined by weighing “the promotion of legitimate
governmental interests” against “the degree to which [the
search] intrudes upon an individual’s privacy,” Wyoming v.
Houghton, 526 U. S. 295 . Pp. 7–10.

(c) In this balance of reasonableness, great weight is given to


both the significant government interest at stake in the
identification of arrestees and DNA identification’s unmatched
potential to serve that interest. Pp. 10–23.

(1) The Act serves a well-established, legitimate


government interest: the need of law enforcement
officers in a safe and accurate way to process and

Page 65 of 186
identify persons and possessions taken into custody. efficacy of the search for its purpose of prompt
“[P]robable cause provides legal justification for identification, not the constitutionality of the search.
arresting a [suspect], and for a brief period of detention Rapid technical advances are also reducing DNA
to take the administrative steps incident to arrest,” processing times. Pp. 18–23.
Gerstein v. Pugh, 420 U. S. 103 –114; and the “validity
of the search of a person incident to a lawful arrest” is
settled, United States v. Robinson, 414 U. S. 218 . (d) The government interest is not outweighed by
respondent’s privacy interests. Pp. 23–28.
Individual suspicion is not necessary. The “routine (1) By comparison to the substantial government interest
administrative procedure[s] at a police station house and the unique effectiveness of DNA identification, the
incident to booking and jailing the suspect” have intrusion of a cheek swab to obtain a DNA sample is
different origins and different constitutional minimal. Reasonableness must be considered in the
justifications than, say, the search of a place not context of an individual’s legitimate privacy
incident to arrest, Illinois v. Lafayette, 462 U. S. 640 , expectations, which necessarily diminish when he is
which depends on the “fair probability that contraband taken into police custody. Bell, supra, at 557. Such
or evidence of a crime will be found in a particular searches thus differ from the so-called special needs
place,” Illinois v. Gates, 462 U. S. 213 . And when searches of, e.g., otherwise law-abiding motorists at
probable cause exists to remove an individual from the checkpoints. See Indianapolis v. Edmond, 531 U. S. 32
normal channels of society and hold him in legal . The reasonableness inquiry considers two other
custody, DNA identification plays a critical role in circumstances in which particularized suspicion is not
serving those interests. First, the government has an categorically required: “diminished expectations of
interest in properly identifying “who has been arrested privacy [and a] minimal intrusion.” Illinois v. McArthur,
and who is being tried.” Hiibel v. Sixth Judicial Dist. 531 U. S. 326 . An invasive surgery may raise privacy
Court of Nev., Humboldt Cty., 542 U. S. 177 . Criminal concerns weighty enough for the search to require a
history is critical to officers who are processing a warrant, notwithstanding the arrestee’s diminished
suspect for detention. They already seek identity privacy expectations, but a buccal swab, which
information through routine and accepted means: involves a brief and minimal intrusion with “virtually no
comparing booking photographs to sketch artists’ risk, trauma, or pain,” Schmerber v. California, 384 U.
depictions, showing mugshots to potential witnesses, S. 757 , does not increase the indignity already
and comparing fingerprints against electronic attendant to normal incidents of arrest. Pp. 23–26.
databases of known criminals and unsolved crimes.
The only difference between DNA analysis and (2) The processing of respondent’s DNA sample’s CODIS
fingerprint databases is the unparalleled accuracy DNA loci also did not intrude on his privacy in a way that
provides. DNA is another metric of identification used would make his DNA identification unconstitutional.
to connect the arrestee with his or her public persona, Those loci came from noncoding DNA parts that do not
as reflected in records of his or her actions that are reveal an arrestee’s genetic traits and are unlikely to
available to the police. Second, officers must ensure reveal any private medical information. Even if they
that the custody of an arrestee does not create could provide such information, they are not in fact
inordinate “risks for facility staff, for the existing tested for that end. Finally, the Act provides statutory
detainee population, and for a new detainee.” Florence protections to guard against such invasions of privacy.
v. Board of Chosen Freeholders of County of Pp. 26–28.
Burlington, 566 U. S. ___, ___. DNA allows officers to
know the type of person being detained. Third, “the
Government has a substantial interest in ensuring that 425 Md. 550, 42 A. 3d 549, reversed.
persons accused of crimes are available for trials.” Bell
v. Wolfish, 441 U. S. 520. An arrestee may be more
inclined to flee if he thinks that continued contact with
the criminal justice system may expose another serious
offense. Fourth, an arrestee’s past conduct is essential
to assessing the danger he poses to the public, which
will inform a court’s bail determination. Knowing that
the defendant is wanted for a previous violent crime
based on DNA identification may be especially
probative in this regard. Finally, in the interests of
justice, identifying an arrestee as the perpetrator of
some heinous crime may have the salutary effect of
freeing a person wrongfully imprisoned. Pp. 10–18.

(2) DNA identification is an important advance in the


techniques long used by law enforcement to serve
legitimate police concerns. Police routinely have used
scientific advancements as standard procedures for
identifying arrestees. Fingerprinting, perhaps the most
direct historical analogue to DNA technology, has, from
its advent, been viewed as a natural part of “the
administrative steps incident to arrest.” County of
Riverside v. McLaughlin, 500 U. S. 44 . However, DNA
identification is far superior. The additional intrusion
upon the arrestee’s privacy beyond that associated
with fingerprinting is not significant, and DNA
identification is markedly more accurate. It may not be
as fast as fingerprinting, but rapid fingerprint analysis is
itself of recent vintage, and the question of how long it
takes to process identifying information goes to the

Page 66 of 186
18. UNITED STATES v. VERDUGO-URQUIDEZ(1990) No. power. See, e. g., Balzac v. Porto Rico, 258 U.S. 298 . Indeed,
88-1353 the claim that extraterritorial aliens are entitled to rights under
the Fifth Amendment - which speaks in the relatively universal
Argued: November 7, 1989Decided: February 28, 1990 term of "person" - has been emphatically rejected. Johnson v.
Eisentrager, 339 U.S. 763, 784 . Pp. 268-269.

After the Government obtained an arrest warrant for


respondent - a Mexican citizen and resident believed to be a (e) Respondent's reliance on Reid, supra, is misplaced, since
leader of an organization that smuggles narcotics into this that case stands only for the proposition that United States
country - he was apprehended by Mexican police and citizens stationed abroad could invoke the protection of the
transported here, where he was arrested. Following his arrest, Fifth and Sixth Amendments. Similarly, those cases in which
Drug Enforcement Administration (DEA) agents, working with aliens have been determined to enjoy certain constitutional
Mexican officials, searched his Mexican residences and rights establish only that aliens receive such protections when
seized certain documents. The District Court granted his they have come within the territory of, and have developed
motion to suppress the evidence, concluding that the Fourth substantial connections with, this country. See, e. g., Plyler v.
Amendment - which protects "the people" against Doe, 457 U.S. 202, 212 . Respondent, however, is an alien
unreasonable searches and seizures - applied to the with no previous significant voluntary connection with the
searches, and that the DEA agents had failed to justify United States, and his legal but involuntary presence here
searching the premises without a warrant. The Court of does not indicate any substantial connection with this country.
Appeals affirmed. Citing Reid v. Covert, 354 U.S. 1 - which The Court of Appeals' reliance on INS v. Lopez-Mendoza,
held that American citizens tried abroad by United States supra, is also misplaced, since that case assumed that, but
military officials were entitled to Fifth and Sixth Amendment did not expressly address the question whether, the Fourth
protections - the court concluded that the Constitution Amendment applies to illegal aliens in the United States. Even
imposes substantive constraints on the Federal Government, assuming such aliens - who are in this country voluntarily and
even when it operates abroad. Relying on INS v. Lopez- presumably have accepted some societal obligations - would
Mendoza, 468 U.S. 1032 - where a majority assumed that be entitled to Fourth Amendment protections, their situation
illegal aliens in the United States have Fourth Amendment differs from that of respondent, who had no voluntary
rights - the court observed that it would be odd to acknowledge connection with this country that might place him among "the
that respondent was entitled to trial-related rights guaranteed people." This Court's decisions expressly according differing
by the Fifth and Sixth Amendments, but not to Fourth protection to aliens than to citizens also undermine
Amendment protection. respondent's claim that treating aliens differently under the
Fourth Amendment violates the equal protection component
of the Fifth Amendment. Pp. 269-273.
Held: The Fourth Amendment does not apply to the search
and seizure by United States agents of property owned by a
nonresident alien and located in a foreign country. Pp. 264- (f) The Court of Appeals' rule would have significant and
275. deleterious consequences for the United States in conducting
activities beyond its [494 U.S. 259, 261] borders. The rule
(a) If there were a constitutional violation in this case, it would apply not only to law enforcement operations abroad,
occurred solely in Mexico, since a Fourth Amendment but also to other foreign operations - such as Armed Forces
violation is fully accomplished at the time of an unreasonable actions - which might result in "searches and seizures." Under
governmental intrusion whether or not the evidence seized is the rule, aliens with no attachment to this country might bring
sought for use in a criminal trial. Thus, the Fourth Amendment actions for damages to remedy claimed violations of the
functions differently from the Fifth Amendment, whose Fourth Amendment in foreign countries or in international
privilege against self-incrimination is a fundamental trial right waters, and Members of the Executive and Legislative
of criminal defendants. P. 264. Branches would be plunged into a sea of uncertainty as to
what might be reasonable in the way of searches and seizures
conducted abroad. Any restrictions on searches and seizures
(b) The Fourth Amendment phrase "the people" seems to be incident to American action abroad must be imposed by the
a term of art used in select parts of the Constitution and political branches through diplomatic understanding, treaty, or
contrasts with the words "person" and "accused" used in legislation. Pp. 273-275.
Articles of the Fifth and Sixth Amendments regulating criminal
procedures. This suggests that "the people" [494 U.S. 259, 856 F.2d 1214, reversed.
260] refers to a class of persons who are part of a national
community or who have otherwise developed sufficient
connection with this country to be considered part of that
community. Pp. 264-266.

(c) The Fourth Amendment's drafting history shows that its


purpose was to protect the people of the United States against
arbitrary action by their own Government and not to restrain
the Federal Government's actions against aliens outside
United States territory. Nor is there any indication that the
Amendment was understood by the Framers' contemporaries
to apply to United States activities directed against aliens in
foreign territory or in international waters. Pp. 266-268.

(d) The view that every constitutional provision applies


wherever the Government exercises its power is contrary to
this Court's decisions in the Insular Cases, which held that not
all constitutional provisions apply to governmental activity
even in territories where the United States has sovereign

Page 67 of 186
2] We granted certiorari in order to consider the constitutional
E. PRIVACY OF COMMUNICATION AND questions thus presented. [Footnote 3]
CORRESPONDENCE
The petitioner has phrased those questions as follows:
"A. Whether a public telephone booth is a
1. Katz v. United States, 389 U.S. 347 (1967) No. 35 (389 constitutionally protected area so that evidence
U.S. 347 CERTIORARI TO THE UNITED STATES obtained by attaching an electronic listening
COURT OF APPEALS FOR THE NINTH CIRCUIT) recording device to the top of such a booth is
obtained in violation of the right to privacy of the user
Argued October 17, 1967 of the booth. "

Decided December 18, 1967 "B. Whether physical penetration of a constitutionally


protected area is necessary before a search and
seizure can be said to be violative of the Fourth
Syllabus: Petitioner was convicted under an indictment Amendment to the United States Constitution."
charging him with transmitting wagering information by
telephone across state lines in violation of 18 U.S.C. § 1084. We decline to adopt this formulation of the issues. In the first
Evidence of petitioner's end of the conversations, overheard place, the correct solution of Fourth Amendment problems is
by FBI agents who had attached an electronic listening and not necessarily promoted by incantation of the phrase
recording device to the outside of the telephone booth from "constitutionally protected area." Secondly, the Fourth
which the calls were made, was introduced at the trial. The Amendment cannot be translated into a general constitutional
Court of Appeals affirmed the conviction, finding that there "right to privacy." That Amendment protects individual privacy
was no Fourth Amendment violation, since there was "no against certain kinds of governmental intrusion, but its
physical entrance into the area occupied by" petitioner. protections go further, and often have nothing to do with
privacy at all. [Footnote 4] Other provisions of the Constitution
protect personal privacy from other forms of governmental
Held: invasion. [Footnote 5] But the protection of a person's general
right to privacy -- his right to be let alone by other people
1. The Government's eavesdropping activities violated the [Footnote 6] -- is, like the protection of his property and of his
privacy upon which petitioner justifiably relied while using very life, left largely to the law of the individual States.
the telephone booth, and thus constituted a "search and [Footnote 7]
seizure" within the meaning of the Fourth Amendment.
Pp. 389 U. S. 350-353. Because of the misleading way the issues have been
formulated, the parties have attached great significance to the
(a) The Fourth Amendment governs not only the seizure characterization of the telephone booth from which the
of tangible items, but extends as well to the recording petitioner placed his calls. The petitioner has strenuously
of oral statements. Silverman v. United States, 365 argued that the booth was a "constitutionally protected area."
U. S. 505, 365 U. S. 511. P. 389 U. S. 353. The Government has maintained with equal vigor that it was
not. [Footnote 8] But this effort to decide whether or not a
(b) Because the Fourth Amendment protects people, given "area," viewed in the abstract, is "constitutionally
rather than places, its reach cannot turn on the protected" deflects attention from the problem presented by
presence or absence of a physical intrusion into any this case. [Footnote 9] For the Fourth Amendment protects
given enclosure. The "trespass" doctrine of people, not places. What a person knowingly exposes to the
Olmstead v. United States, 277 U. S. 438, and public, even in his own home or office, is not a subject of
Goldman v. United States, 316 U. S. 129, is no longer Fourth Amendment protection. See Lewis v. United States,
controlling. Pp. 389 U. S. 351, 389 U. S. 353. 385 U. S. 206, 385 U. S. 210; United States v. Lee, 274 U. S.
559, 274 U. S. 563. But what he seeks to preserve as private,
even in an area accessible to the public, may be
2. Although the surveillance in this case may have been so constitutionally protected. See Rios v. United States, 364 U.
narrowly circumscribed that it could constitutionally have S. 253; Ex parte Jackson, 96 U. S. 727, 96 U. S. 733.
been authorized in advance, it was not in fact conducted
pursuant to the warrant procedure which is a The Government stresses the fact that the telephone booth
constitutional precondition of such electronic from which the petitioner made his calls was constructed
surveillance. Pp. 389 U. S. 354-359. partly of glass, so that he was as visible after he entered it as
he would have been if he had remained outside. But what he
369 F.2d 130, reversed. sought to exclude when he entered the booth was not the
intruding eye -- it was the uninvited ear. He did not shed his
MR. JUSTICE STEWART delivered the opinion of the Court. right to do so simply because he made his calls from a place
where he might be seen. No less than an individual in a
The petitioner was convicted in the District Court for the business office, [Footnote 10] in a friend's apartment,
Southern District of California under an eight-count indictment [Footnote 11] or in a taxicab, [Footnote 12] a person in a
charging him with transmitting wagering information by telephone booth may rely upon the protection of the Fourth
telephone from Los Angeles to Miami and Boston, in violation Amendment. One who occupies it, shuts the door behind him,
of a federal statute. [Footnote 1] At trial, the Government was and pays the toll that permits him to place a call is surely
permitted, over the petitioner's objection, to introduce entitled to assume that the words he utters into the
evidence of the petitioner's end of telephone conversations, mouthpiece will not be broadcast to the world. To read the
overheard by FBI agents who had attached an electronic Constitution more narrowly is to ignore the vital role that the
listening and recording device to the outside of the public public telephone has come to play in private communication.
telephone booth from which he had placed his calls. In
affirming his conviction, the Court of Appeals rejected the The Government contends, however, that the activities of its
contention that the recordings had been obtained in violation agents in this case should not be tested by Fourth Amendment
of the Fourth Amendment, because "[t]here was no physical requirements, for the surveillance technique they employed
entrance into the area occupied by [the petitioner]." [Footnote involved no physical penetration of the telephone booth from
which the petitioner placed his calls. It is true that the absence

Page 68 of 186
of such penetration was at one time thought to foreclose necessary under the circumstances." Id. at 388 U. S. 57.
further Fourth Amendment inquiry, Olmstead v. United States, [Footnote 16] Here, too, a similar judicial order could have
277 U. S. 438, 277 U. S. 457, 277 U. S. 464, 277 U. S. 466; accommodated "the legitimate needs of law enforcement"
Goldman v. United States, 316 U. S. 129, 316 U. S. 134-136, [Footnote 17] by authorizing the carefully limited use of
for that Amendment was thought to limit only searches and electronic surveillance.
seizures of tangible property. [Footnote 13] But "[t]he premise
that property interests control the right of the Government to The Government urges that, because its agents relied upon
search and seize has been discredited." Warden v. Hayden, the decisions in Olmstead and Goldman, and because they
387 U. S. 294, 387 U. S. 304. Thus, although a closely divided did no more here than they might properly have done with prior
Court supposed in Olmstead that surveillance without any judicial sanction, we should retroactively validate their
trespass and without the seizure of any material object fell conduct. That we cannot do. It is apparent that the agents in
outside the ambit of the Constitution, we have since departed this case acted with restraint. Yet the inescapable fact is that
from the narrow view on which that decision rested. Indeed, this restraint was imposed by the agents themselves, not by a
we have expressly held that the Fourth Amendment governs judicial officer. They were not required, before commencing
not only the seizure of tangible items, but extends as well to the search, to present their estimate of probable cause for
the recording of oral statements, overheard without any detached scrutiny by a neutral magistrate. They were not
"technical trespass under . . . local property law." Silverman v. compelled, during the conduct of the search itself, to observe
United States, 365 U. S. 505, 365 U. S. 511. Once this much precise limits established in advance by a specific court order.
is acknowledged, and once it is recognized that the Fourth Nor were they directed, after the search had been completed,
Amendment protects people -- and not simply "areas" -- to notify the authorizing magistrate in detail of all that had been
against unreasonable searches and seizures, it becomes seized. In the absence of such safeguards, this Court has
clear that the reach of that Amendment cannot turn upon the never sustained a search upon the sole ground that officers
presence or absence of a physical intrusion into any given reasonably expected to find evidence of a particular crime and
enclosure. voluntarily confined their activities to the least intrusive means
consistent with that end. Searches conducted without
We conclude that the underpinnings of Olmstead and warrants have been held unlawful "notwithstanding facts
Goldman have been so eroded by our subsequent decisions unquestionably showing probable cause," Agnello v. United
that the "trespass" doctrine there enunciated can no longer be States, 269 U. S. 20, 269 U. S. 33, for the Constitution requires
regarded as controlling. The Government's activities in "that the deliberate, impartial judgment of a judicial officer . . .
electronically listening to and recording the petitioner's words be interposed between the citizen and the police. . . ." Wong
violated the privacy upon which he justifiably relied while using Sun v. United States, 371 U. S. 471, 371 U. S. 481-482. "Over
the telephone booth, and thus constituted a "search and and again, this Court has emphasized that the mandate of the
seizure" within the meaning of the Fourth Amendment. The [Fourth] Amendment requires adherence to judicial
fact that the electronic device employed to achieve that end processes," United States v. Jeffers, 342 U. S. 48, 342 U. S.
did not happen to penetrate the wall of the booth can have no 51, and that searches conducted outside the judicial process,
constitutional significance. The question remaining for without prior approval by judge or magistrate, are per se
decision, then, is whether the search and seizure conducted unreasonable under the Fourth Amendment [Footnote 18] --
in this case complied with constitutional standards. In that subject only to a few specifically established and well
regard, the Government's position is that its agents acted in delineated exceptions. [Footnote 19]
an entirely defensible manner: they did not begin their
electronic surveillance until investigation of the petitioner's It is difficult to imagine how any of those exceptions could ever
activities had established a strong probability that he was apply to the sort of search and seizure involved in this case.
using the telephone in question to transmit gambling Even electronic surveillance substantially contemporaneous
information to persons in other States, in violation of federal with an individual's arrest could hardly be deemed an
law. Moreover, the surveillance was limited, both in scope and "incident" of that arrest. [Footnote 20] Nor could the use of
in duration, to the specific purpose of establishing the contents electronic surveillance without prior authorization be justified
of the petitioner's unlawful telephonic communications. The on grounds of "hot pursuit." [Footnote 21] And, of course, the
agents confined their surveillance to the brief periods during very nature of electronic surveillance precludes its use
which he used the telephone booth, [Footnote 14] and they pursuant to the suspect's consent. [Footnote 22]
took great care to overhear only the conversations of the
petitioner himself. [Footnote 15] The Government does not question these basic principles.
Rather, it urges the creation of a new exception to cover this
Accepting this account of the Government's actions as case. [Footnote 23] It argues that surveillance of a telephone
accurate, it is clear that this surveillance was so narrowly booth should be exempted from the usual requirement of
circumscribed that a duly authorized magistrate, properly advance authorization by a magistrate upon a showing of
notified of the need for such investigation, specifically probable cause. We cannot agree. Omission of such
informed of the basis on which it was to proceed, and clearly authorization "bypasses the safeguards provided by an
apprised of the precise intrusion it would entail, could objective predetermination of probable cause, and substitutes
constitutionally have authorized, with appropriate safeguards, instead the far less reliable procedure of an after-the-event
the very limited search and seizure that the Government justification for the . . . search, too likely to be subtly influenced
asserts, in fact, took place. Only last Term we sustained the by the familiar shortcomings of hindsight judgment."
validity of such an authorization, holding that, under
sufficiently "precise and discriminate circumstances," a Beck v. Ohio, 379 U. S. 89, 379 U. S. 96. And bypassing a
federal court may empower government agents to employ a neutral predetermination of the scope of a search leaves
concealed electronic device "for the narrow and particularized individuals secure from Fourth Amendment violations "only in
purpose of ascertaining the truth of the . . . allegations" of a the discretion of the police." Id. at 379 U. S. 97.
"detailed factual affidavit alleging the commission of a specific
criminal offense." Osborn v. United States, 385 U. S. 323, 385 These considerations do not vanish when the search in
U. S. 329-330. Discussing that holding, the Court in Berger v. question is transferred from the setting of a home, an office,
New York, 388 U. S. 41, said that "the order authorizing the or a hotel room to that of a telephone booth. Wherever a man
use of the electronic device" in Osborn "afforded similar may be, he is entitled to know that he will remain free from
protections to those . . . of conventional warrants authorizing unreasonable searches and seizures. The government agents
the seizure of tangible evidence." Through those protections, here ignored "the procedure of antecedent justification . . . that
"no greater invasion of privacy was permitted than was is central to the Fourth Amendment," [Footnote 24] a

Page 69 of 186
procedure that we hold to be a constitutional precondition of 2. CECILIA ZULUETA vs. COURT OF APPEALS and
the kind of electronic surveillance involved in this case. ALFREDO MARTIN (G.R. No. 107383; February 20, 1996)
Because the surveillance here failed to meet that condition,
and because it led to the petitioner's conviction, the judgment
must be reversed. FACTS: Cecilia Zulueta is the wife of private respondent
Alfredo Martin. On March 26, 1982, petitioner entered the
It is so ordered. clinic of her husband, a doctor of medicine, and in the
presence of her mother, a driver and private respondent's
secretary, forcibly opened the drawers and cabinet in her
husband's clinic and took 157 documents consisting of private
correspondence between Dr. Martin and his alleged
paramours, greetings cards, cancelled checks, diaries, Dr.
Martin's passport, and photographs. The documents and
papers were seized for use in evidence in a case for legal
separation and for disqualification from the practice of
medicine which petitioner had filed against her husband.

Dr. Martin brought this action below for recovery of the


documents and papers and for damages against petitioner.
The case was filed with the Regional Trial Court of Manila,
Branch X, which, after trial, rendered judgment for private
respondent, Dr. Alfredo Martin, declaring him "the
capital/exclusive owner of the properties described in
paragraph 3 of plaintiff's Complaint or those further described
in the Motion to Return and Suppress" and ordering Cecilia
Zulueta and any person acting in her behalf to a immediately
return the properties to Dr. Martin and to pay him damages.

In appealing from the decision of the Court of Appeals


affirming the trial court's decision, petitioner's only ground is
that in Alfredo Martin v. Alfonso Felix, Jr.,1 this Court ruled that
the documents and papers were admissible in evidence and,
therefore, their use by petitioner's attorney, Alfonso Felix did
not constitute malpractice or gross misconduct, For this
reason it is contended that the Court of Appeals erred in
affirming the decision of the trial court instead of dismissing
private respondent's complaint.

ISSUE: Were the documents indeed admissible in evidence?

HELD: No, the documents and papers in question are


inadmissible in evidence. The constitutional injunction
declaring "the privacy of communication and correspondence
[to be] inviolable" is no less applicable simply because it is the
wife (who thinks herself aggrieved by her husband's infidelity)
who is the party against whom the constitutional provision is
to be enforced. The only exception to the prohibition in the
Constitution is if there is a "lawful order [from a] court or when
public safety or order requires otherwise, as prescribed by
law.” Any violation of this provision renders the evidence
obtained inadmissible "for any purpose in any proceeding."

The intimacies between husband and wife do not justify any


one of them in breaking the drawers and cabinets of the other
and in ransacking them for any telltale evidence of marital
infidelity. A person, by contracting marriage, does not shed
his/her integrity or his right to privacy as an individual and the
constitutional protection is ever available to him or to her.

The law insures absolute freedom of communication between


the spouses by making it privileged. Neither husband nor wife
may testify for or against the other without the consent of the
affected spouse while the marriage subsists.6 Neither may be
examined without the consent of the other as to any
communication received in confidence by one from the other
during the marriage, save for specified exceptions.7 But one
thing is freedom of communication; quite another is a
compulsion for each one to share what one knows with the
other. And this has nothing to do with the duty of fidelity that
each owes to the other.

Page 70 of 186
The right to privacy as such is accorded recognition
3. BLAS F. OPLE vs. RUBEN D. TORRES, ALEXANDER independently of its identification with liberty; in itself,
AGUIRRE, HECTOR VILLANUEVA (G.R. No. 127685 it is fully deserving of constitutional protection. The
July 23, 1998) language of Prof. Emerson is particularly apt: "The
concept of limited government has always included
the idea that governmental powers stop short of
FACTS: Petitioner Ople prays that we invalidate certain intrusions into the personal life of the citizen.
Administrative Order No. 308 entitled "Adoption of a National This is indeed one of the basic distinctions between
Computerized Identification Reference System" on two absolute and limited government. Ultimate and
important constitutional grounds, viz: one, it is a usurpation of pervasive control of the individual, in all aspects of
the power of Congress to legislate, and two, it impermissibly his life, is the hallmark of the absolute state. In
intrudes on our citizenry's protected zone of privacy. We grant contrast, a system of limited government safeguards
the petition for the rights sought to be vindicated by the a private sector, which belongs to the individual,
petitioner need stronger barriers against further erosion. He firmly distinguishing it from the public sector, which
alleges that A.O. No. 308 establishes a system of identification the state can control. Protection of this private sector
that is all-encompassing in scope, affects the life and liberty of — protection, in other words, of the dignity and
every Filipino citizen and foreign resident, and more integrity of the individual — has become increasingly
particularly, violates their right to privacy. important as modern society has developed.

A.O. No. 308 was published in four newspapers of general Zones of privacy are likewise recognized and protected in our
circulation on January 22, 1997 and January 23, 1997. On laws. The Civil Code provides that "[e]very person shall
January 24, 1997, petitioner filed the instant petition against respect the dignity, personality, privacy and peace of mind of
respondents, then Executive Secretary Ruben Torres and the his neighbors and other persons" and punishes as actionable
heads of the government agencies, who as members of the torts several acts by a person of meddling and prying into the
Inter-Agency Coordinating Committee, are charged with the privacy of another. It also holds a public officer or employee or
implementation of A.O. No. 308. On April 8, 1997, we issued any private individual liable for damages for any violation of
a temporary restraining order enjoining its implementation. the rights and liberties of another person, and recognizes the
privacy of letters and other private communications. The
Revised Penal Code makes a crime the violation of secrets by
ISSUE: Is A.O. No. 308 not a mere administrative order but a an officer, the revelation of trade and industrial secrets, and
law and hence, beyond the power of the President to issue? trespass to dwelling. Invasion of privacy is an offense in
special laws like the Anti-Wiretapping Law, the Secrecy of
Bank Deposits Act and the Intellectual Property Code. The
HELD: A.O. No. 308 involves a subject that is not appropriate Rules of Court on privileged communication likewise
to be covered by an administrative order. recognize the privacy of certain information.

An administrative order is an ordinance issued by the A.O. No. 308 should also raise our antennas for a further look
President which relates to specific aspects in the will show that it does not state whether encoding of data is
administrative operation of government. It must be in harmony limited to biological information alone for identification
with the law and should be for the sole purpose of purposes. In fact, the Solicitor General claims that the
implementing the law and carrying out the legislative policy. adoption of the Identification Reference System will contribute
The Court rejects the argument that A.O. No. 308 implements to the "generation of population data for development
the legislative policy of the Administrative Code of 1987. The planning." This is an admission that the PRN will not be used
Code is a general law and "incorporates in a unified document solely for identification but the generation of other data with
the major structural, functional and procedural principles of remote relation to the avowed purposes of A.O. No. 308.
governance." and "embodies changes in administrative Clearly, the indefiniteness of A.O. No. 308 can give the
structure and procedures designed to serve the people." government the roving authority to store and retrieve
information for a purpose other than the identification of the
It cannot be simplistically argued that A.O. No. 308 merely individual through his PRN. The potential for misuse of the
implements the Administrative Code of 1987. It establishes for data to be gathered under A.O. No. 308 cannot be
the first time a National Computerized Identification Reference undarplayed as the dissenters do. Pursuant to said
System. Such a System requires a delicate adjustment of administrative order, an individual must present his PRN
various contending state policies — the primacy of national everytime he deals with a government agency to avail of basic
security, the extent of privacy interest against dossier- services and security. His transactions with the government
gathering by government, the choice of policies, etc. Indeed, agency will necessarily be recorded — whether it be in the
the dissent of Mr. Justice Mendoza states that the A.O. No. computer or in the documentary file of the agency. The
308 involves the all-important freedom of thought. As said individual's file may include his transactions for loan
administrative order redefines the parameters of some basic availments, income tax returns, statement of assets and
rights of our citizenry vis-a-vis the State as well as the line that liabilities, reimbursements for medication, hospitalization, etc.
separates the administrative power of the President to make The more frequent the use of the PRN, the better the chance
rules and the legislative power of Congress, it ought to be of building a huge formidable information base through the
evident that it deals with a subject that should be covered by electronic linkage of the files.
law.
It is plain and we hold that A.O. No. 308 falls short of assuring
Assuming, arguendo, that A.O. No. 308 need not be the that personal information which will be gathered about our
subject of a law, still it cannot pass constitutional muster as an people will only be processed for unequivocally specified
administrative legislation because facially it violates the right purposes. The lack of proper safeguards in this regard of A.O.
to privacy. The essence of privacy is the "right to be let alone." No. 308 may interfere with the individual's liberty of abode and
In the 1968 case of Morfe v. Mutuc, we adopted the Griswold travel by enabling authorities to track down his movement; it
ruling that there is a constitutional right to privacy. Speaking may also enable unscrupulous persons to access confidential
thru Mr. Justice, later Chief Justice, Enrique Fernando, we information and circumvent the right against self-incrimination;
held: it may pave the way for "fishing expeditions" by government
authorities and evade the right against unreasonable
searches and seizures. The possibilities of abuse and misuse

Page 71 of 186
of the PRN, biometrics and computer technology are 4. KILUSANG MAYO UNO, NATIONAL FEDERATION
accentuated when we consider that the individual lacks control OF LABOR UNIONS-KILUSANG MAYO UNO (NAFLU-
over what can be read or placed on his ID, much less verify KMU), et. al vs. DIRECTOR-GENERAL, NEDA and
the correctness of the data encoded. 62 They threaten the Secretary of DBM (G.R. No. 167798 April 19, 2006)
very abuses that the Bill of Rights seeks to prevent.

The Solicitor General urges us to validate A.O. No. 308's


FACTS: On April 13, 2005, President Gloria Macapagal
abridgment of the right of privacy by using the rational
Arroyo issued EO No. 420, reads –
relationship test. 75 He stressed that the purposes of A.O. No.
308 are: (1) to streamline and speed up the implementation of REQUIRING ALL GOVERNMENT AGENCIES AND
basic government services, (2) eradicate fraud by avoiding GOVERNMENT-OWNED AND CONTROLLED
duplication of services, and (3) generate population data for CORPORATIONS TO STREAMLINE AND
development planning. He cocludes that these purposes HARMONIZE THEIR IDENTIFICATION (ID)
justify the incursions into the right to privacy for the means are SYSTEMS, AND AUTHORIZING FOR SUCH
rationally related to the end. We are not impressed by the PURPOSE THE DIRECTOR-GENERAL,
argument. In Morfe v. Mutuc, we upheld the constitutionality NATIONAL ECONOMIC AND DEVELOPMENT
of R.A. 3019, the Anti-Graft and Corrupt Practices Act, as a AUTHORITY TO IMPLEMENT THE SAME, AND
valid police power measure. We declared that the law, in FOR OTHER PURPOSES
compelling a public officer to make an annual report disclosing
his assets and liabilities, his sources of income and expenses, directing all government agencies and government-owned
did not infringe on the individual's right to privacy. The law was and controlled corporations to adopt a uniform data co llection
enacted to promote morality in public administration by and format for their existing identification (ID) systems.
curtailing and minimizing the opportunities for official
corruption and maintaining a standard of honesty in the public The EO was assailed as unconstitutional based on the same
service. The same circumstances do not obtain in the case at grounds used in the earlier case of Ople vs. Torres. EO 420 is
bar. For one, R.A. 3019 is a statute, not an administrative a usurpation of legislative power by the President and that it is
order. Secondly, R.A. 3019 itself is sufficiently detailed. The an infringement on the citizen’s right to privacy and that the
law is clear on what practices were prohibited and penalized, implementation of the EO will use public funds not
and it was narrowly drawn to avoid abuses. IN the case at bar, appropriated by Congress for that purpose. EO 420 is vague
A.O. No. 308 may have been impelled by a worthy purpose, and without adequate safeguards or penalties for any violation
but, it cannot pass constitutional scrutiny for it is not narrowly of its provisions. Granting without conceding that the
drawn. President may issue EO 420, the Executive Order was issued
without public hearing.
In no uncertain terms, we also underscore that the right to
privacy does not bar all incursions into individual privacy. The
right is not intended to stifle scientific and technological
ISSUE:
advancements that enhance public service and the common
good. It merely requires that the law be narrowly focused and 1. WON EO 420 is a usurpation of legislative power by the
a compelling interest justify such intrusions. Intrusions into the President.
right must be accompanied by proper safeguards and well- 2. WON EO 420 infringes on the citizen’s right to privacy.
defined standards to prevent unconstitutional invasions. We
reiterate that any law or order that invades individual privacy
will be subjected by this Court to strict scrutiny.
RULING: No. EO 420 does not establish a national ID card
The right to privacy is one of the most threatened rights of man system. EO 420 does not compel all citizens to have an ID
living in a mass society. The threats emanate from various card. EO 420 applies only to government entities that under
sources — governments, journalists, employers, social existing laws are already collecting data and issuing ID cards
scientists, etc. In th case at bar, the threat comes from the as part of their governmental functions.
executive branch of government which by issuing A.O. No.
308 pressures the people to surrender their privacy by giving Section 2 of EO 420 provides, "Coverage. – All government
information about themselves on the pretext that it will agencies and government-owned and controlled corporations
facilitate delivery of basic services. Given the record-keeping issuing ID cards to their members or constituents shall be
power of the computer, only the indifferent fail to perceive the covered by this executive order." EO 420 applies only to
danger that A.O. No. 308 gives the government the power to government entities that issue ID cards as part of their
compile a devastating dossier against unsuspecting citizens. functions under existing laws. These government entities have
It is timely to take note of the well-worded warning of Kalvin, already been issuing ID cards even prior to EO 420. Examples
Jr., "the disturbing result could be that everyone will live of these government entities are the GSIS, SSS, Philhealth,
burdened by an unerasable record of his past and his Mayor’s Office, LTO, PRC, and similar government entities.
limitations. In a way, the threat is that because of its record- Section 1 of EO 420 directs these government entities to
keeping, the society will have lost its benign capacity to "adopt a unified multi-purpose ID system." Thus, all
forget." government entities that issue IDs as part of their functions
under existing laws are required to adopt a uniform data
collection and format for their IDs. The purposes of the uniform
ID data collection and ID format are to reduce costs, achieve
efficiency and reliability, insure compatibility, and provide
convenience to the people served by government entities.

At present, government entities like LTO require considerably


more data from applicants for identification purposes. EO 420
will reduce the data required to be collected and recorded in
the ID databases of the government entities. Government
entities cannot collect or record data, for identification
purposes, other than the 14 specific data. Various laws allow
several government entities to collect and record data for their

Page 72 of 186
ID systems, either expressly or impliedly by the nature of the 5. MARYNETTE R. GAMBOA vs. P/SSUPT. MARLOU C.
functions of these government entities. Under their existing ID CHAN, in his capacity as the PNP-Provincial Director
systems, some government entities collect and record more of Ilocos Norte and WILLIAM O. FANG, in his
data than what EO 420 allows. capacity as Chief, Intelligence Division, PNP
Provincial Office, Ilocos Norte (G.R. No. 193636; July
A unified ID system for all these government entities can be 24, 2012)
achieved in either of two ways. First, the heads of these
existing government entities can enter into a memorandum of
agreement making their systems uniform. If the government FACTS: At the time the present Petition was filed, petitioner
entities can individually adopt a format for their own ID Marynette R. Gamboa (Gamboa) was the Mayor of Dingras.
pursuant to their regular functions under existing laws, they
can also adopt by mutual agreement a uniform ID format, On 8 December 2009, former President Gloria Macapagal-
especially if the uniform format will result in substantial Arroyo issued Administrative Order No. 275 (A.O. 275),
savings, greater efficiency, and optimum compatibility. This is "Creating an Independent Commission to Address the Alleged
purely an administrative matter, and does not involve the Existence of Private Armies in the Country." The body, which
exercise of legislative power. Second, the President may by was later on referred to as the Zeñarosa Commission, was
executive or administrative order direct the government formed to investigate the existence of private army groups
entities under the Executive department to adopt a uniform ID (PAGs) in the country with a view to eliminating them before
data collection and format. Section 17, Article VII of the 1987 the 10 May 2010 elections and dismantling them permanently
Constitution provides that the "President shall have control of in the future. Upon the conclusion of its investigation, the
all executive departments, bureaus and offices." The same Zeñarosa Commission released and submitted to the Office of
Section also mandates the President to "ensure that the laws the President a confidential report entitled "A Journey
be faithfully executed." Certainly, under this constitutional Towards H.O.P.E.: The Independent Commission Against
power of control the President can direct all government Private Armies’ Report to the President" (the Report). Gamboa
entities, in the exercise of their functions under existing laws, alleged that the Philippine National Police in Ilocos Norte
to adopt a uniform ID data collection and ID format to achieve (PNP–Ilocos Norte) conducted a series of surveillance
savings, efficiency, reliability, compatibility, and convenience operations against her and her aides, and classified her as
to the public. someone who keeps a PAG. Purportedly without the benefit
of data verification, PNP–Ilocos Norte forwarded the
The Constitution also mandates the President to ensure that
information gathered on her to the Zeñarosa Commission,
the laws are faithfully executed. There are several laws
thereby causing her inclusion in the Report’s enumeration of
mandating government entities to reduce costs, increase
individuals maintaining PAGs.
efficiency, and in general, improve public services. The
adoption of a uniform ID data collection and format under EO
On 6 and 7 July 2010, ABS-CBN broadcasted on its evening
420 is designed to reduce costs, increase efficiency, and in
news program the portion of the Report naming Gamboa as
general, improve public services. Thus, in issuing EO 420, the
one of the politicians alleged to be maintaining a PAG.
President is simply performing the constitutional duty to
Gamboa averred that her association with a PAG also
ensure that the laws are faithfully executed. Clearly, EO 420
appeared on print media. Thus, she was publicly tagged as
is well within the constitutional power of the President to
someone who maintains a PAG on the basis of the unverified
promulgate. The President has not usurped legislative power
information that the PNP-Ilocos Norte gathered and forwarded
in issuing EO 420. EO 420 is an exercise of Executive power
to the Zeñarosa Commission. As a result, she claimed that her
– the President’s constitutional power of control over the
malicious or reckless inclusion in the enumeration of
Executive department.
personalities maintaining a PAG as published in the Report
What require legislation are three aspects of a government also made her, as well as her supporters and other people
maintained ID card system. First, when the implementation of identified with her, susceptible to harassment and police
an ID card system requires a special appropriation because surveillance operations.
there is no existing appropriation for such purpose. Second,
when the ID card system is compulsory on all branches of Contending that her right to privacy was violated and her
government, including the independent constitutional reputation maligned and destroyed, Gamboa filed a Petition
commissions, as well as compulsory on all citizens whether dated 9 July 2010 for the issuance of a writ of habeas data
they have a use for the ID card or not. Third, when the ID card against respondents in their capacities as officials of the PNP-
system requires the collection and recording of personal data Ilocos Norte. The case was docketed as Special Proc. No.
beyond what is routinely or usually required for such purpose, 14979 and was raffled to RTC Br. 13, which issued the
such that the citizen’s right to privacy is infringed. In the corresponding writ on 14 July 2010 after finding the Petition
present case, EO 420 does not require any special meritorious on its face. However, RTC dismissed the Petition
appropriation because the existing ID card systems of on the ground that Gamboa failed to prove through substantial
government entities covered by EO 420 have the proper evidence that the subject information originated from
appropriation or funding. EO 420 is not compulsory on all respondents, and that they forwarded this database to the
branches of government and is not compulsory on all citizens. Zeñarosa Commission without the benefit of prior verification.
EO 420 requires a very narrow and focused collection and The trial court also ruled that even before respondents
recording of personal data while safeguarding the assumed their official positions, information on her may have
confidentiality of such data. In fact, the data collected and already been acquired. The trial court categorically ruled that
recorded under EO 420 are far less than the data collected the inclusion of Gamboa in the list of persons maintaining
and recorded under the ID systems existing prior to EO 420. PAGs, as published in the Report, constituted a violation of
her right to privacy.
Petitioners have not shown how EO 420 will violate their right
to privacy. Petitioners cannot show such violation by a mere
facial examination of EO 420 because EO 420 narrowly draws ISSUE: Was there a violation of Gamboa’s right to privacy?
the data collection, recording and exhibition while prescribing
comprehensive safeguards. Ople v. Torres is not authority to
hold that EO 420 violates the right to privacy because in that HELD: This Court holds that Gamboa was able to sufficiently
case the assailed executive issuance, broadly drawn and establish that the data contained in the Report listing her as a
devoid of safeguards, was annulled solely on the ground that PAG coddler came from the PNP. Contrary to the ruling of the
the subject matter required legislation. trial court, however, the forwarding of information by the PNP

Page 73 of 186
to the Zeñarosa Commission was not an unlawful act that Emerson is particularly apt: "The concept of limited
violated or threatened her right to privacy in life, liberty or government has always included the idea that governmental
security. powers stop short of certain intrusions into the personal life of
the citizen. This is indeed one of the basic distinctions
The Constitution explicitly mandates the dismantling of private between absolute and limited government. Ultimate and
armies and other armed groups not recognized by the duly pervasive control of the individual, in all aspects of his life, is
constituted authority. It also provides for the establishment of the hallmark of the absolute state. In contrast, a system of
one police force that is national in scope and civilian in limited government, safeguards a private sector, which
character, and is controlled and administered by a national belongs to the individual, firmly distinguishing it from the public
police commission. Taking into account these constitutional sector, which the state can control. Protection of this private
fiats, it is clear that the issuance of A.O. 275 articulates a sector — protection, in other words, of the dignity and integrity
legitimate state aim, which is to investigate the existence of of the individual — has become increasingly important as
PAGs with the ultimate objective of dismantling them modern society has developed.
permanently.
The right to privacy is considered a fundamental right that
The PNP was rationally expected to forward and share must be protected from intrusion or constraint. However, in
intelligence regarding PAGs with the body specifically created Standard Chartered Bank v. Senate Committee on Banks, this
for the purpose of investigating the existence of these Court underscored that the right to privacy is not absolute, viz:
notorious groups. Moreover, the Zeñarosa Commission was
explicitly authorized to deputize the police force in the With respect to the right of privacy which petitioners
fulfillment of the former’s mandate, and thus had the power to claim respondent has violated, suffice it to state that
request assistance from the latter. Following the privacy is not an absolute right. While it is true that
pronouncements of the ECHR in Leander, the fact that the Section 21, Article VI of the Constitution, guarantees
PNP released information to the Zeñarosa Commission respect for the rights of persons affected by the
without prior communication to Gamboa and without affording legislative investigation, not every invocation of the
her the opportunity to refute the same cannot be interpreted right to privacy should be allowed to thwart a
as a violation or threat to her right to privacy since that act is legitimate congressional inquiry. In Sabio v. Gordon,
an inherent and crucial component of intelligence-gathering we have held that the right of the people to access
and investigation. Additionally, Gamboa herself admitted that information on matters of public concern generally
the PNP had a validation system, which was used to update prevails over the right to privacy of ordinary financial
information on individuals associated with PAGs and to transactions. In that case, we declared that the right
ensure that the data mirrored the situation on the field. Thus, to privacy is not absolute where there is an overriding
safeguards were put in place to make sure that the information compelling state interest. Employing the rational
collected maintained its integrity and accuracy. basis relationship test, as laid down in Morfe v.
Mutuc, there is no infringement of the individual’s
However, to accord the right to privacy with the kind of right to privacy as the requirement to disclosure
protection established in existing law and jurisprudence, this information is for a valid purpose, in this case, to
Court nonetheless deems it necessary to caution these ensure that the government agencies involved in
investigating entities that information-sharing must observe regulating banking transactions adequately protect
strict confidentiality. Intelligence gathered must be released the public who invest in foreign securities. Suffice it
exclusively to the authorities empowered to receive the to state that this purpose constitutes a reason
relevant information. After all, inherent to the right to privacy is compelling enough to proceed with the assailed
the freedom from "unwarranted exploitation of one’s person or legislative investigation.
from intrusion into one’s private activities in such a way as to
cause humiliation to a person’s ordinary sensibilities."

The right to privacy, as an inherent concept of liberty, has long The writ of habeas data is an independent and summary
been recognized as a constitutional right. This Court, in Morfe remedy designed to protect the image, privacy, honor,
v. Mutuc, thus enunciated: information, and freedom of information of an individual, and
to provide a forum to enforce one’s right to the truth and to
The due process question touching on an alleged informational privacy. It seeks to protect a person’s right to
deprivation of liberty as thus resolved goes a long control information regarding oneself, particularly in instances
way in disposing of the objections raised by plaintiff in which such information is being collected through unlawful
that the provision on the periodical submission of a means in order to achieve unlawful ends. It must be
sworn statement of assets and liabilities is violative emphasized that in order for the privilege of the writ to be
of the constitutional right to privacy. There is much to granted, there must exist a nexus between the right to privacy
be said for this view of Justice Douglas: "Liberty in on the one hand, and the right to life, liberty or security on the
the constitutional sense must mean more than other.
freedom from unlawful governmental restraint; it
must include privacy as well, if it is to be a repository The notion of informational privacy is still developing in
of freedom. The right to be let alone is indeed the Philippine law and jurisprudence. Considering that even the
beginning of all freedom." As a matter of fact, this Latin American habeas data, on which our own Rule on the
right to be let alone is, to quote from Mr. Justice Writ of Habeas Data is rooted, finds its origins from the
Brandeis "the most comprehensive of rights and the European tradition of data protection, this Court can be guided
right most valued by civilized men." by cases on the protection of personal data decided by the
European Court of Human Rights (ECHR). The ECHR ruled
The concept of liberty would be emasculated if it that the storage in the secret police register of information
does not likewise compel respect for his personality relating to the private life of Leander, coupled with the refusal
as a unique individual whose claim to privacy and to allow him the opportunity to refute the same, amounted to
interference demands respect. an interference in his right to respect for private life. However,
the ECHR held that the interference was justified on the
The right to privacy as such is accorded recognition following grounds: (a) the personnel control system had a
independently of its identification with liberty; in itself, it is fully legitimate aim, which was the protection of national security,
deserving of constitutional protection. The language of Prof. and (b) the Personnel Control Ordinance gave the citizens

Page 74 of 186
adequate indication as to the scope and the manner of 6. RHONDA AVE S. VIVARES and SPS. MARGARITA and
exercising discretion in the collection, recording and release DAVID SUZARA vs ST. THERESA’S COLLEGE, MYLENE
of information by the authorities. RHEZA T. ESCUDERO, and JOHN DOES (G.R. No.
202666 September 29, 2014)

TOPIC: right to informational privacy, writ of habeas data

PONENTE: Velasco, Jr.

PREFATORY: The individual’s desire for privacy is never


absolute, since participation in society is an equally powerful
desire. Thus each individual is continually engaged in a
personal adjustment process in which he balances the desire
for privacy with the desire for disclosure and communication
of himself to others, in light of the environmental conditions
and social norms set by the society in which he lives. – Alan
Westin, Privacy and Freedom (1967)

FACTS: Julia and Julienne, both minors, were graduating high


school students at St. Theresa’s College (STC), Cebu City.
Sometime in January 2012, while changing into their
swimsuits for a beach party they were about to attend, Julia
and Julienne, along with several others, took digital pictures
of themselves clad only in their undergarments. These
pictures were then uploaded by Angela on her Facebook
profile.

At STC, Mylene Escudero, a computer teacher at STC’s high


school department, learned from her students that some
seniors at STC posted pictures online, depicting themselves
from the waist up, dressed only in brassieres. Escudero then
asked her students if they knew who the girls in the photos
are. In turn, they readily identified Julia and Julienne, among
others.

Using STC’s computers, Escudero’s students logged in to


their respective personal Facebook accounts and showed her
photos of the identified students, which include: (a) Julia and
Julienne drinking hard liquor and smoking cigarettes inside a
bar; and (b) Julia and Julienne along the streets of Cebu
wearing articles of clothing that show virtually the entirety of
their black brassieres.

Also, Escudero’s students claimed that there were times when


access to or the availability of the identified students’ photos
was not confined to the girls’ Facebook friends, but were, in
fact, viewable by any Facebook user.
Investigation ensued. Then Julia, Julienne and other students
involved were barred from joining the commencement
exercises.

Petitioners, who are the respective parents of the minors, filed


a Petition for the Issuance of a Writ of Habeas Data. RTC
dismissed the petition for habeas data on the following
grounds:
1. Petitioners failed to prove the existence of an
actual or threatened violation of the minors’ right to
privacy, one of the preconditions for the issuance of
the writ of habeas data.
2. The photos, having been uploaded on Facebook
without restrictions as to who may view them, lost
their privacy in some way.
3. STC gathered the photographs through legal
means and for a legal purpose, that is, the
implementation of the school’s policies and rules on
discipline.

ISSUE: Whether or not there was indeed an actual or


threatened violation of the right to privacy in the life, liberty, or
security of the minors involved in this case. (Is there a right to
informational privacy in online social network activities of its
users?)

Page 75 of 186
HELD: (Note that you can skip the preliminary discussions Right to informational privacy
and check the ruling at the latter part) Right to informational privacy is the right of individuals to
control information about themselves. Several
Nature of Writ of Habeas Data commentators regarding privacy and social networking sites,
It is a remedy available to any person whose right to privacy however, all agree that given the millions of OSN users, “in
in life, liberty or security is violated or threatened by an this Social Networking environment, privacy is no longer
unlawful act or omission of a public official or employee, or of grounded in reasonable expectations, but rather in some
a private individual or entity engaged in the gathering, theoretical protocol better known as wishful thinking.” So the
collecting or storing of data or information regarding the underlying question now is: Up to what extent is the right to
person, family, home and correspondence of the aggrieved privacy protected in OSNs?
party.
Facebook Privacy Tools
It is an independent and summary remedy designed to protect To address concerns about privacy, but without defeating its
the image, privacy, honor, information, and freedom of purpose, Facebook was armed with different privacy tools
information of an individual, and to provide a forum to enforce designed to regulate the accessibility of a user’s profile as well
one’s right to the truth and to informational privacy. It seeks to as information uploaded by the user. In H v. W, the South
protect a person’s right to control information regarding Gauteng High Court recognized this ability of the users to
oneself, particularly in instances in which such information is “customize their privacy settings,” but did so with this caveat:
being collected through unlawful means in order to achieve “Facebook states in its policies that, although it makes every
unlawful ends. effort to protect a user’s information, these privacy settings are
In developing the writ of habeas data, the Court aimed to not foolproof.”
protect an individual’s right to informational privacy, among For instance, a Facebook user can regulate the visibility and
others. A comparative law scholar has, in fact, defined habeas accessibility of digital images (photos), posted on his or her
data as “a procedure designed to safeguard individual personal bulletin or “wall,” except for the user’s profile picture
freedom from abuse in the information age.” and ID, by selecting his or her desired privacy setting:
1. Public – the default setting; every Facebook user can view
Issuance of writ of habeas data; requirements the photo;
1. The existence of a person’s right to informational privacy 2. Friends of Friends – only the user’s Facebook friends and
2. An actual or threatened violation of the right to privacy in their friends can view the photo;
life, liberty or security of the victim (proven by at least 3. Friends – only the user’s Facebook friends can view the
substantial evidence) photo;
4. Custom – the photo is made visible only to particular friends
Note that the writ will not issue on the basis merely of an and/or networks of the Facebook user; and
alleged unauthorized access to information about a person. 5. Only Me – the digital image can be viewed only by the user.

The writ of habeas data is not only confined to cases of The foregoing are privacy tools, available to Facebook users,
extralegal killings and enforced disappearances designed to set up barriers to broaden or limit the visibility of
The writ of habeas data can be availed of as an independent his or her specific profile content, statuses, and photos,
remedy to enforce one’s right to privacy, more specifically the among others, from another user’s point of view. In other
right to informational privacy. The remedies against the words, Facebook extends its users an avenue to make the
violation of such right can include the updating, rectification, availability of their Facebook activities reflect their choice
suppression or destruction of the database or information or as to “when and to what extent to disclose facts about
files in possession or in control of respondents. Clearly then, themselves – and to put others in the position of receiving
the privilege of the Writ of Habeas Data may also be availed such confidences.”
of in cases outside of extralegal killings and enforced
disappearances.
LONE ISSUE:
Meaning of “engaged” in the gathering, collecting or NONE. The Supreme Court held that STC did not violate
storing of data or information petitioners’ daughters’ right to privacy as the subject digital
Habeas data is a protection against unlawful acts or omissions photos were viewable either by the minors’ Facebook friends,
of public officials and of private individuals or entities engaged or by the public at large.
in gathering, collecting, or storing data about the aggrieved
party and his or her correspondences, or about his or her Without any evidence to corroborate the minors’ statement
family. Such individual or entity need not be in the business of that the images were visible only to the five of them, and
collecting or storing data. without their challenging Escudero’s claim that the other
students were able to view the photos, their statements are,
To “engage” in something is different from undertaking a at best, self-serving, thus deserving scant consideration.
business endeavour. To “engage” means “to do or take
part in something.” It does not necessarily mean that the It is well to note that not one of petitioners disputed Escudero’s
activity must be done in pursuit of a business. What sworn account that her students, who are the minors’
matters is that the person or entity must be gathering, Facebook “friends,” showed her the photos using their own
collecting or storing said data or information about the Facebook accounts. This only goes to show that no special
aggrieved party or his or her family. Whether such undertaking means to be able to view the allegedly private posts were ever
carries the element of regularity, as when one pursues a resorted to by Escudero’s students, and that it is reasonable
business, and is in the nature of a personal endeavour, for any to assume, therefore, that the photos were, in reality, viewable
other reason or even for no reason at all, is immaterial and either by (1) their Facebook friends, or (2) by the public at
such will not prevent the writ from getting to said person or large.
entity.
Considering that the default setting for Facebook posts is
As such, the writ of habeas data may be issued against a “Public,” it can be surmised that the photographs in question
school like STC. were viewable to everyone on Facebook, absent any proof
that petitioners’ children positively limited the disclosure of the
photograph. If such were the case, they cannot invoke the
protection attached to the right to informational privacy.

Page 76 of 186
US v. Gines-Perez: A person who places a photograph on the 7. LEE v. P/SUPT. NERI A. ILAGAN (G.R. No. 203254,
Internet precisely intends to forsake and renounce all privacy October 08, 2014)
rights to such imagery, particularly under circumstances such
as here, where the Defendant did not employ protective FACTS: In his Petition for Issuance of the Writ of Habeas
measures or devices that would have controlled access to the Data, Ilagan alleged that he and petitioner Dr. Joy Margate
Web page or the photograph itself. Lee were former common law partners. Sometime in July
2011, he visited Lee at the latter’s condominium, Ilagan
United States v. Maxwell: The more open the method of noticed that his digital camera was missing. Lee confronted
transmission is, the less privacy one can reasonably expect. Ilagan at the latter’s office regarding a purported sex video she
Messages sent to the public at large in the chat room or e-mail discovered from the aforesaid camera involving Ilagan and
that is forwarded from correspondent to correspondent loses another woman.
any semblance of privacy.
Ilagan denied the video and demanded Lee to return the
The Honorable Supreme Court continued and held that setting camera, but to no avail. During the confrontation, Ilagan
a post’s or profile detail’s privacy to “Friends” is no assurance allegedly slammed Lee’s head against a wall inside his office
that it can no longer be viewed by another user who is not and walked away.
Facebook friends with the source of the content. The user’s
own Facebook friend can share said content or tag his or her Subsequently, Lee utilized the said video as evidence in filing
own Facebook friend thereto, regardless of whether the user various complaints against Ilagan, namely: (a) a criminal
tagged by the latter is Facebook friends or not with the former. complaint for violation of R.A. 9262; and (b) an administrative
Also, when the post is shared or when a person is tagged, the complaint for grave misconduct before the NAPOLCOM.
respective Facebook friends of the person who shared the
post or who was tagged can view the post, the privacy setting Ilagan claimed that Lee’s acts of reproducing the subject video
of which was set at “Friends.” Thus, it is suggested, that a and threatening to distribute the same to the upper echelons
profile, or even a post, with visibility set at “Friends Only” of the NAPOLCOM and uploading it to the internet violated not
cannot easily, more so automatically, be said to be “very only his right to life, liberty, security, and privacy but also that
private,” contrary to petitioners’ argument. of the other woman, and thus, the issuance of a writ of habeas
data in his favor is warranted.
No privacy invasion by STC; fault lies with the friends of
minors The RTC granted the privilege of the writ of habeas data in
Respondent STC can hardly be taken to task for the perceived Ilagan’s favor, and ordered the implementing officer to turn-
privacy invasion since it was the minors’ Facebook friends over copies of the subject video to him, and enjoined Lee from
who showed the pictures to Tigol. Respondents were mere further reproducing the same.
recipients of what were posted. They did not resort to any
unlawful means of gathering the information as it was Dissatisfied, Lee filed this petition.
voluntarily given to them by persons who had legitimate
access to the said posts. Clearly, the fault, if any, lies with the
friends of the minors. Curiously enough, however, neither the ISSUE: Whether or not the RTC correctly extended the
minors nor their parents imputed any violation of privacy privilege of the writ of habeas data in favor of Ilagan.
against the students who showed the images to Escudero.

Different scenario of setting is set on “Me Only” or RULING: The petition is meritorious.
“Custom”
Had it been proved that the access to the pictures posted were The Rule on the Writ of Habeas Data was conceived as a
limited to the original uploader, through the “Me Only” privacy response, given the lack of effective and available remedies,
setting, or that the user’s contact list has been screened to to address the extraordinary rise in the number of killings and
limit access to a select few, through the “Custom” setting, the enforced disappearances. It was conceptualized as a judicial
result may have been different, for in such instances, the remedy enforcing the right to privacy, most especially the right
intention to limit access to the particular post, instead of being to informational privacy of individuals, which is defined as “the
broadcasted to the public at large or all the user’s friends en right to control the collection, maintenance, use, and
masse, becomes more manifest and palpable. dissemination of data about oneself.”

As defined in Section 1 of the Habeas Data Rule, the writ of


habeas data now stands as “a remedy available to any person
whose right to privacy in life, liberty or security is violated or
threatened by an unlawful act or omission of a public official
or employee, or of a private individual or entity engaged in the
gathering, collecting or storing of data or information regarding
the person, family, home, and correspondence of the
aggrieved party.”

Thus, in order to support a petition for the issuance of such


writ, Section 6 of the Habeas Data Rule essentially requires
that the petition sufficiently alleges, among others, “[t]he
manner the right to privacy is violated or threatened and how
it affects the right to life, liberty or security of the aggrieved
party.” Corollarily, the allegations in the petition must be
supported by substantial evidence showing an actual or
threatened violation of the right to privacy in life, liberty or
security of the victim.

In this relation, it bears pointing out that the writ of habeas data
will not issue to protect purely property or commercial

Page 77 of 186
concerns nor when the grounds invoked in support of the F. PRIVACY OF COMMUNICATION AND
petitions therefor are vague and doubtful. CORRESPONDENCE

In this case, the Court finds that Ilagan was not able to
sufficiently allege that his right to privacy in life, liberty or 1. Francisco Chavez vs. Raul M. Gonzales and NTC
security was or would be violated through the supposed (G.R. No. 168338 | February 15, 2008)
reproduction and threatened dissemination of the subject sex
video. Facts: As a consequence of the public release of copies of the
“Hello Garci” compact disc audiotapes involving a
As the rules and existing jurisprudence on the matter evoke, wiretapped mobile phone conversation between then-
alleging and eventually proving the nexus between one’s President Gloria Arroyo and Comelec Commissioner Virgilio
privacy right to the cogent rights to life, liberty or security are Garcillano, respondent DOJ Secretary Gonzales warned
crucial in habeas data cases, so much so that a failure on reporters that those who had copies of the CD and those
either account certainly renders a habeas data petition broadcasting or publishing its contents could be held liable
dismissible, as in this case. under the Anti-Wiretapping Act. He also stated that persons
Hence, due to the insufficiency of the allegations as well as possessing or airing said tapes were committing a continuing
the glaring absence of substantial evidence, the Court finds it offense, subject to arrest by anybody. Finally, he stated that
proper to reverse the RTC Decision and dismiss the habeas he had ordered the NBI to go after media organizations “found
data petition. to have caused the spread, the playing and the printing of the
contents of a tape.” Meanwhile, respondent NTC warned TV
and radio stations that their broadcast/airing of such false
information and/or willful misrepresentation shall be a just
cause for the suspension, revocation and/or cancellation of
the licenses or authorizations issued to the said media
establishments. Petitioner Chavez filed a petition under Rule
65 against respondents Secretary Gonzales and the NTC
directly with the Supreme Court.

Issues: (1) Will a purported violation of law such as the Anti-


Wiretapping Law justify straitjacketing the exercise of freedom
of speech and of the press?

(2) Did the mere press statements of respondents DOJ


Secretary and the NTC constitute a form of content-based
prior restraint that has transgressed the Constitution?

Held: (1) No, a purported violation of law such as the Anti-


Wiretapping Law will not justify straitjacketing the exercise of
freedom of speech and of the press. A governmental action
that restricts freedom of speech or of the press based on
content is given the strictest scrutiny, with
the government having the burden of overcoming the
presumed unconstitutionality by the clear and present danger
rule. This rule applies equally to all kinds of
media, including broadcast media. Respondents, who have
the burden to show that these acts do not abridge freedom of
speech and of the press, failed to hurdle the clear and present
danger test. For this failure of the respondents alone to offer
proof to satisfy the clear and present danger test, the Court
has no option but to uphold the exercise of free speech and
free press. There is no showing that the feared violation of the
anti-wiretapping law clearly endangers the national security of
the State.

(2) Yes, the mere press statements of respondents


DOJ Secretary and the NTC constituted a form of content-
based prior restraint that has transgressed the Constitution. It
is not decisive that the press statements made by respondents
were not reduced in or followed up with formal orders or
circulars. It is sufficient that the press statements were made
by respondents while in the exercise of their official functions.
Any act done, such as a speech uttered, for and on behalf of
the government in an official capacity is covered by the rule
on prior restraint. The concept of an “act” does not limit itself
to acts already converted to a formal order or official
circular. Otherwise, the non formalization of an act into an
official order or circular will result in the easy circumvention of
the prohibition on prior restraint.

Page 78 of 186
2. DISINI v. SECRETARY OF JUSTICE (G.R. No. of Representatives, except to the extent reserved to the
203335) people by the provision on initiative and referendum.

FACTS: These consolidated petitions seek to declare several


provisions of R.A. 10175, known as The Cybercrime
Prevention Act of 2012 unconstitutional and void. This case
holds a handful of petitions seeking the removal of different
sections deemed to be infringing on privacy rights and more.

For this instance, the focus is the creation of the “Cybercrime


Investigation and Coordinating Center” which, in the same
breath, promulgates powers and functions to the agents of the
said center. “Petitioners mainly contend that Congress
invalidly delegated its power when it gave the Cybercrime
Investigation and Coordinating Center (CICC) the power to
formulate a national cybersecurity plan without any sufficient
standards or parameters for it to follow.”

“Cybersecurity refers to the collection of tools, policies, risk


management approaches, actions, training, best practices,
assurance and technologies that can be used to protect cyber
environment and organization and user’s assets. This
definition serves as the parameters within which CICC should
work in formulating the cybersecurity plan.”

ISSUE: Whether or not the CICC is constitutional for it to


operate under delegated powers by Congress

HELD: YES. “In order to determine whether there is undue


delegation of legislative power, the Court has adopted two
tests: the completeness test and the sufficient standard test.
Under the first test, the law must be complete in all its terms
and conditions when it leaves the legislature such that when it
reaches the delegate, the only thing he will have to do is to
enforce The second test mandates adequate guidelines or
limitations in the law to determine the boundaries of the
delegate’s authority and prevent the delegation from running
riot.

Here, the cybercrime law is complete in itself when it directed


the CICC to formulate and implement a national cybersecurity
plan. Also, contrary to the position of the petitioners, the law
gave sufficient standards for the CICC to follow when it
provided a definition of cybersecurity.”

IMPORTANT LAWS/STATUTES/PROVISIONS/SECTIONS
R.A. 10175

Sec. 24. Cybercrime Investigation and Coordinating Center.–


There is hereby created, within thirty (30) days from the
effectivity of this Act, an inter-agency body to be known as the
Cybercrime Investigation and Coordinating Center (CICC),
under the administrative supervision of the Office of the
President, for policy coordination among concerned agencies
and for the formulation and enforcement of the national
cybersecurity plan.

Sec. 26. Powers and Functions.– The CICC shall have the
following powers and functions:

(a) To formulate a national cybersecurity plan and extend


immediate assistance of real time commission of cybercrime
offenses through a computer emergency response team
(CERT); x x x.

Art. 1, Sec. 6 Philippine Constitution

Sec 1. The legislative power shall be vested in the Congress


of the Philippines which shall consist of a Senate and a House

Page 79 of 186
3. MTRCB vs ABS CBN (G.R. No. 155282; January 17, 4. Lester Packingham v. North Carolina, No. 15-1194,
2005) June 19, 2017

FACTS: On October 15, 1991, at 10:45 in the evening, FACTS: Lester Packingham was convicted of taking “indecent
respondent ABS-CBN aired "Prosti-tuition," an episode of the liberties” with a minor in 2002, as a 21-year-old college
television (TV) program "The Inside Story" produced and student. Per North Carolina law, he was sentenced to a
hosted by respondent Legarda. It depicted female students standard 10-12 month imprisonment, followed by a 24-month
moonlighting as prostitutes to enable them to pay for their supervised release. Aside from being told to “remain away
tuition fees. In the course of the program, student prostitutes, from” the minor, his conviction entailed no special stipulations.
pimps, customers, and some faculty members were Packingham was arrested in 2010 after authorities came
interviewed. The Philippine Women’s University (PWU) was across a post on his Facebook profile, thanking God for having
named as the school of some of the students involved and the a parking ticket dismissed. He was arrested for violating North
facade of PWU Building at Taft Avenue, Manila conspicuously Carolina’s laws regarding convicted sex offenders, which
served as the background of the episode. The showing of "The barred the offender’s access to social media websites.
Inside Story" caused uproar in the PWU community. Dr.
Leticia P. de Guzman, Chancellor and Trustee of the PWU, In his defense, Packingham argued that the law violated his
and the PWU Parents and Teachers Association filed letter- First Amendment rights. He was convicted in trial court, which
complaints3 with petitioner MTRCB. Acting on the letter- found that the state had a weighty interest in keeping sexual
complaints, the MTRCB Legal Counsel initiated a formal predators off of social media websites for the “protection of
complaint with the MTRCB Investigating Committee, alleging minors.” The North Carolina Court of Appeals reversed and
among others, that respondents (1) did not submit "The Inside held that the social media website provision of the law was
Story" to petitioner for its review and (2) exhibited the same unconstitutional. The North Carolina Supreme Court reversed
without its permission, thus, violating Section 74 of and held that the law was constitutional by finding that the law
Presidential Decree (P.D.) No. 19865 and Section 3,6 Chapter was a “limitation on conduct” and not a restriction of free
III and Section 7,7 Chapter IV of the MTRCB Rules and speech. The court found that the state had a sufficient interest
Regulations respondents explained that the "The Inside Story" in “forestalling the illicit lurking and contact” of registered sex
is a "public affairs program, news documentary and socio- offenders and their potential future victims.
political editorial," the airing of which is protected by the
constitutional provision on freedom of expression and of the
press. Accordingly, petitioner has no power, authority and ISSUE: Does the North Carolina Law violate the First
jurisdiction to impose any form of prior restraint upon Amendment (Right to Freedom of Speech)?
respondents

HELD: YES. In order to be valid under the First Amendment,


ISSUE: Whether or not the “inside story” is protected by the a content-neutral regulation of speech must be narrowly
constitutional provision on freedom of expression and of the tailored to serve a significant government interest. In other
press words, the law cannot burden substantially more speech than
necessary to advance the government’s legitimate interest. In
this case, although the government has a legitimate interest in
HELD: Respondents claim that the showing of "The Inside protecting children from abuse, this law too broadly restricted
Story" is protected by the constitutional provision on freedom access to all sorts of websites. Even if it were limited only to
of speech and of the press. However, there has been no social media websites, the law would still unconstitutionally
declaration at all by the framers of the Constitution that restrict speech because of the vast number of functions that
freedom of expression and of the press has a preferred status. social media websites perform in the modern world. First
If this Court, in Iglesia ni Cristo, did not exempt religious Amendment jurisprudence has never allowed for such a broad
programs from the jurisdiction and review power of petitioner regulation of speech, and similarly broad restrictions have
MTRCB, with more reason, there is no justification to exempt been struck down. However, a state could accomplish the
therefrom "The Inside Story" which, according to respondents, same goal by enacting a more narrowly written statute.
is protected by the constitutional provision on freedom of
expression and of the press, a freedom bearing no preferred Justice Samuel A. Alito, Jr., concurring in the judgment,
status. The only exceptions from the MTRCB’s power of argued that the majority opinion erred in equating the entire
review are those expressly mentioned in Section 7 of P. D. No. internet with a traditionally public forum instead of recognizing
1986, such as (1) television programs imprinted or exhibited the importance of allowing states to regulate certain types of
by the Philippine Government and/or its departments and websites. The government certainly has a compelling interest
agencies, and (2) newsreels. in protecting children from potential sexual predation, and the
internet is a place that allows sexual offenders to contact
children in ways that they might not otherwise be able, so the
government should be able to regulate sex offenders’ use of
the internet to an extent. However, the North Carolina law at
issue here went too far because it encompassed websites that
were unlikely to facilitate a sex crime against a child. Because
the North Carolina law prohibited more speech than
necessary to further the government’s significant interest, it
violated the First Amendment.

Page 80 of 186
5. Osmeña v. COMELEC, G.R. No. 132231, March 31, circulation in every province or city: Provided,
1998 however, That in the absence of said newspaper,
publication shall be done in any other magazine or
periodical in said province or city, which shall be
FACTS: Petitioners Emilio Osmeña is candidate for President known as “Comelec Space” wherein candidates can
of the Philippines, while petitioner Pablo Garcia is governor of announce their candidacy. Said space shall be
Cebu Province, seeking reelection. They contend that events allocated, free of charge, equally and impartially by
after the ruling in National Press Club v. COMELEC “have the Commission among all candidates within the
called into question the validity of the very premises of that area in which the newspaper is circulated.
decision.” They seek a reexamination of the validity of Sec.
11(b) of R.A. No. 6646 (Electoral Reforms Law of 1987) which SEC. 92. Comelec time.—The Commission shall
prohibits mass media from selling or giving free of charge print procure radio and television time to be known as
space or air time for campaign or other political purposes, “Comelec Time” which shall be allocated equally and
except to COMELEC. impartially among the candidates within the area of
coverage of all radio and television stations. For this
Petitioners claim that experience in the last five years since purpose, the franchise of all radio broadcasting and
the decision shown the “undesirable effects” of the law television stations are hereby amended so as to
because “the ban on political advertising has not only failed to provide radio or television time, free of charge, during
level the playing field, but actually worked to the grave the period of the campaign.
disadvantage of the poor candidates” by depriving them of a
medium which they can afford to pay for while their more In this case, there is no total ban on political ads, much less
affluent rivals can always resort to other means of reaching restriction on the content of the speech. Given the fact that
voters like airplanes, boats, rallies, parades, and handbills. No print space and air time can be controlled or dominated by rich
empirical data have been presented by petitioners to back up candidates to the disadvantage of poor candidates, there is a
their claim, however. substantial or legitimate governmental interest justifying
exercise of the regulatory power of the COMELEC under Art.
Unable to show the “experience” and “subsequent events” IX-C, §4 of the Constitution, which provides:
which they claim invalidate the major premise of said decision,
petitioners now say “there is no need for ‘empirical data’ to The commission may, during the election period, supervise or
determine whether the political ad ban offends the regulate the enjoyment or utilization of all franchises or
Constitution or not.” Instead they make arguments from which permits for the operation of transportation and other public
it is clear that their disagreement is with the opinion of the utilities, media of communication or information, all grants,
Court on the constitutionality of §11(b) of R.A. No. 6646 and special privileges, or concessions granted by the Government
that what they seek is a reargument on the same issue already or any subdivision, agency, or instrumentality thereof,
decided in that case. including any government-owned or controlled corporation or
its subsidiary. Such supervision or regulation shall aim to
ensure equal opportunity, time, and space, and the right to
ISSUE: Does Sec. 11(b) of RA 6646 violate the right to reply, including reasonable, equal rates therefor, for public
freedom of expression of political ads? information campaigns and forums among candidates in
connection with the objective of holding free, orderly, honest,
peaceful, and credible elections.
HELD: NO. The term political “ad ban,” when used to describe
§11(b) of RA 6646, is misleading, for even as §11(b) prohibits In short, the law only prohibits the sale or donation of print
the sale or donation of print space and air time to political space and air time to candidates but require the COMELEC
candidates, it mandates the COMELEC to procure and itself instead to procure space and time in the mass media for
allocate to the candidates space and time in the media. There allocation, free of charge, to the candidates. In effect, during
is no suppression of political ads but only a regulation of the the election period, the COMELEC takes over the advertising
time and manner of advertising. The law’s concern is not with page of newspapers or the commercial time of radio and TV
the message or content of the ad but with ensuring media stations and allocates these to the candidates. The main
equality between candidates with “deep pockets,” and those purpose of §11(b) is regulatory. Any restriction on speech is
with less resources. only incidental, and it is no more than is necessary to achieve
its purpose of promoting equality of opportunity in the use of
Thus, §11(b) states: “Prohibited Forms of Election mass media for political advertising. The restriction on speech,
Propaganda.—In addition to the forms of election propaganda as pointed out in NPC, is limited both as to time and as to
prohibited in Section 85 of Batas Pambansa Blg. 881, it shall scope.
be unlawful:
What petitioners seem to miss is that the prohibition against
(b) for any newspapers, radio broadcasting or paid or sponsored political advertising is only half of the
television station, or other mass media, or any regulatory framework, the other half being the mandate of the
person making use of the mass media to sell or to COMELEC to procure print space and air time so that these
give free of charge print space or air time for can be allocated free of charge to the candidates.
campaign or other political purposes except to the
Commission as provided under Sections 90 and 92 It is finally argued by the petitioners that COMELEC Space
of Batas Pambansa Blg. 881. Any mass media and COMELEC Time are ineffectual. It is claimed that people
columnist, commentator, announcer or personality hardly read or watch or listen to them. Again, this is a factual
who is a candidate for any elective public office shall assertion without any empirical basis to support it. To be sure,
take a leave of absence from his work as such during this Court did not hold in PPI v. COMELEC that it should not
the campaign period. procure newspaper space for allocation to candidates. What it
ruled is that the COMELEC cannot procure print space without
On the other hand, the Omnibus Election Code provisions paying just compensation. Whether by its manifestation the
referred to in §11(b) read: COMELEC meant it is not going to buy print space or only that
it will not require newspapers to donate free of charge print
SEC. 90. Comelec space.—The Commission shall space is not clear from the manifestation. It is to be presumed
procure space in at least one newspaper of general that the COMELEC, in accordance with its mandate under

Page 81 of 186
§11(b)of R.A. No. 6646 and §90 of the Omnibus Election 6. Social Weather Stations v. COMELEC (G.R. No.
Code, will procure print space for allocation to candidates, 147571, May 5, 2001)
paying just compensation to newspapers providing print
space.
FACTS: Petitioner Social Weather Stations is a social
In any event, the validity of a law cannot be made to depend research institution conducting surveys in various fields,
on the faithful compliance of those charged with its including economics, politics, demography, and social
enforcement but by appropriate constitutional provisions. development, and thereafter processing, analyzing, and
There is a remedy for such lapse if it should happen. In publicly reporting the results thereof. On the other hand,
addition, there is the COMELEC Time during which petitioner Kamahalan Publishing Corporation publishes the
candidates may advertise themselves. Resolution No. 2983-A Manila Standard, a newspaper of general circulation, which
of the COMELEC provides: features newsworthy items of information including election
surveys. Petitioners brought this action for prohibition to enjoin
SEC. 2. Grant of “Comelec Time.”—Every radio broadcasting the COMELEC from enforcing §5.4 of R.A. No. 9006 (Fair
and television station operating under franchise shall grant the Election Act), which provides:
Commission, upon payment of just compensation, at least
thirty (30) minutes of prime time daily, to be known as Surveys affecting national candidates shall not be published
“Comelec Time,” effective February 10, 1998 for candidates fifteen (15) days before an election and surveys affecting local
for President, Vice-President and Senators, and effective candidates shall not be published seven (7) days before an
March 27, 1998, for candidates for local elective offices, until election.
May 9, 1998.
The term “election surveys” is defined in §5.1 of the law as
In Adiong v. COMELEC, the Test for Content-Neutral follows: “Election surveys refer to the measurement of
Restrictions was quoted by the Court from the decision of the opinions and perceptions of the voters as regards a
U.S. Supreme Court sustaining a Los Angeles City ordinance candidate’s popularity, qualifications, platforms or a matter of
which prohibited the posting of campaign signs on public public discussion in relation to the election, including voters’
property: preference for candidates or publicly discussed issues during
the campaign period.” To implement §5.4, Resolution 3636,
A government regulation is sufficiently justified if it is within the §24(h), COMELEC enjoins these Surveys affecting national
constitutional power of the Government, if it furthers an candidates shall not be published 15 days before an election
important or substantial governmental interest; if the and surveys affecting local candidates shall not be published
governmental interest is unrelated to the suppression of free 7 days before an election.
expression; and if the incident restriction on alleged First
Amendment freedoms is no greater than is essential to the Petitioner SWS states that it wishes to conduct an election
furtherance of that interest. survey throughout the period of the elections both at the
national and local levels and release to the media the results
It is an appropriate test for restrictions on speech which, like of such survey as well as publish them directly. Petitioner
§11(b), are content-neutral. Unlike content-based restrictions, Kamahalan Publishing Corporation, on the other hand, states
they are not imposed because of the content of the speech. that it intends to publish election survey results up to the last
For this reason, content-neutral restrictions are tests day of the elections on May 14, 2001.
demanding standards. For example, a rule such as that
involved in Sanidad v. COMELEC, prohibiting columnists, Petitioners argue that the restriction on the publication of
commentators, and announcers from campaigning either for election survey results constitutes a prior restraint on the
or against an issue in a plebiscite must have a compelling exercise of freedom of speech without any clear and present
reason to support it, or it will not pass muster under strict danger to justify such restraint. They claim that SWS and other
scrutiny. These restrictions, it will be seen, are censorial and pollsters conducted and published the results of surveys prior
therefore they bear a heavy presumption of constitutional to the 1992, 1995, and 1998 elections up to as close as two
invalidity. In addition, they will be tested for possible days before the election day without causing confusion among
overbreadth and vagueness. It is apparent that these the voters and that there is neither empirical nor historical
doctrines have no application to content-neutral regulations evidence to support the conclusion that there is an immediate
which, like §11(b), are not concerned with the content of the and inevitable danger to the voting process posed by election
speech. These regulations need only a substantial surveys. They point out that no similar restriction is imposed
governmental interest to support them. A deferential standard on politicians from explaining their opinion or on newspapers
of review will suffice to test their validity. Petition is or broadcast media from writing and publishing articles
DISMISSED. concerning political issues up to the day of the election.
Consequently, they contend that there is no reason for
ordinary voters to be denied access to the results of election
surveys which are relatively objective.

Respondent Commission on Elections justifies the restrictions


in §5.4 of R.A. No. 9006 as necessary to prevent the
manipulation and corruption of the electoral process by
unscrupulous and erroneous surveys just before the election.
It contends that (1) the prohibition on the publication of
election survey results during the period proscribed by law
bears a rational connection to the objective of the law, i.e., the
prevention of the debasement of the electoral process
resulting from manipulated surveys, bandwagon effect, and
absence of reply; (2) it narrowly tailored to meet the “evils”
sought to be prevented; and (3) the impairment of freedom of
expression is minimal, the restriction being limited both in
duration, i.e., the last 15 days before the national election and
the last 7 days before a local election, and in scope as it does
not prohibit election survey results but only require timeliness.

Page 82 of 186
Respondent claims that in National Press Club v. COMELEC, COMELEC and Osmeña v. COMELEC. For the ban imposed
a total ban on political advertisements, with candidates being by R.A. No. 6646, §11(b) is not only authorized by a specific
merely allocated broadcast time during the so-called constitutional provision, but it also provided an alternative so
COMELEC space or COMELEC hour, was upheld by this that, as this Court pointed out in Osmeña, there was actually
Court. In contrast, according to respondent, it states that the no ban but only a substitution of media advertisements by the
prohibition in §5.4 of R.A. No. 9006 is much more limited. COMELEC space and COMELEC hour.

Second. Even if the governmental interest sought to be


ISSUE: Is §5.4 of R.A. No. 9006 unconstitutional for violating promoted is unrelated to the suppression of speech and the
the right to freedom of speech? resulting restriction of free expression is only incidental, §5.4
nonetheless fails to meet fourth criterion of the O’Brien test.
As already stated, §5.4 aims at the prevention of last-minute
HELD: YES. §5.4 lays a prior restraint on freedom of speech, pressure on voters, the creation of bandwagon effect,
expression, and the press by prohibiting the publication of “junking” of weak or “losing” candidates, and resort to the form
election survey results affecting candidates within the of election cheating called “dagdag-bawas.” Praiseworthy as
prescribed periods. Because of the preferred status of the these aims of the regulation might be, they cannot be attained
constitutional rights of speech, expression, and the press, at the sacrifice of the fundamental right of expression, when
such a measure is vitiated by a presumption of invalidity. such aim can be more narrowly pursued by punishing unlawful
Indeed, “any system of prior restraints of expression bearing acts, rather than speech because of apprehension that such
a presumption against its constitutional validity. . . . The speech creates the danger of such evils. Thus, under the
Government ‘carries a heavy burden of showing justification Administrative Code of 1987, the COMELEC is given the
for the enforcement of such restraint.’ The test employed to power:
determine the constitutional validity of §5.4 is enunciated in
United States v. O’Brien: To stop any illegal activity, or confiscate, tear down, and stop
any unlawful, libelous, misleading or false election
[A] government regulation is sufficiently justified [1] if it is propaganda, after due notice and hearing.
within the constitutional power of the Government; [2] if it
furthers an important or substantial governmental interest; [3] This is surely a less restrictive means than the prohibition
if the governmental interest is unrelated to the suppression of contained in §5.4. Pursuant to this power of the COMELEC, it
free expression; and [4] if the incidental restriction on alleged can confiscate bogus survey results calculated to mislead
First Amendment freedoms [of speech, expression and press] voters. Candidates can have their own surveys conducted. No
is no greater than is essential to the furtherance of that right of reply can be invoked by others. No principle of equality
interest. Under this test, even if a law furthers an important or is involved. As for the purpose of the law to prevent
substantial governmental interest, it should be invalidated if bandwagon effects, it is doubtful whether the Government can
such governmental interest is “not unrelated to the deal with this natural-enough tendency of some voters.
suppression of free expression.” Moreover, even if the
purpose is unrelated to the suppression of free speech, the To summarize then, §5.4 is invalid because (1) it imposes a
law should nevertheless be invalidated if the restriction on prior restraint on the freedom of expression, (2) it is a direct
freedom of expression is greater than is necessary to achieve and total suppression of a category of expression even though
the governmental purpose in question. such suppression is only for a limited period, and (3) the
governmental interest sought to be promoted can be achieved
First. Sec. 5.4 fails to meet third criterion of the O’Brien test by means other than the suppression of freedom of
because the causal connection of expression to the asserted expression. Hence, petition for prohibition is GRANTED and
governmental interest makes such interest “not unrelated to §5.4 of R.A. No. 9006 and §24(h) of COMELEC Resolution
the suppression of free expression.” By prohibiting the 3636 are declared unconstitutional.
publication of election survey results because of the possibility
that such publication might undermine the integrity of the
election, §5.4 actually suppresses a whole class of
expression, while allowing the expression of opinion
concerning the same subject matter by newspaper
columnists, radio and TV commentators, armchair theorists,
and other opinion makers. In effect, §5.4 shows a bias for a
particular subject matter by preferring personal opinion to
statistical results. The inhibition of speech should be upheld
only if the expression falls within one of the few unprotected
categories dealt with in Chaplinsky v. New Hampshire, thus:

These include the lewd and obscene, the profane, the


libelous, and the insulting or ‘fighting’ words—those which by
their very utterance inflict injury or tend to incite an immediate
breach of the peace.

Contrary to the claim of the Solicitor General, the prohibition


imposed by §5.4 cannot be justified on the ground that it is
only for a limited period and is only incidental. The prohibition
may be for a limited time, but the curtailment of the right of
expression is direct, absolute, and substantial. It constitutes a
total suppression of a category of speech and is not made less
so because it is only for a period of 15 days immediately
before a national election and 7 days immediately before a
local election.

This sufficiently distinguishes §5.4 from R.A. No. 6646, §ll(b),


which this Court found to be valid in National Press Club v.

Page 83 of 186
7. Social Weather Stations v. COMELEC (G.R. No. The petitioners are of the position that Resolution No. 9674, in
208062, April 7, 2015) requiring the submission of information on subscribers, is in
excess of what the Fair Elections Act requires.

FACTS: COMELEC Resolution No. 9674 directed Social


Weather Stations and Pulse Asia to submit to COMELEC the ISSUE: Is Resolution No. 9674 invalid for requiring the
names of all commissioners and payors of all surveys disclosure of the names of “subscribers” of election surveys
published from February 12, 2013 to April 23, 2013, including as curtailment of the rights of petitioners to free speech?
those of their “subscribers.” As recounted by SWS and Pulse
Asia, on February 15 to February 17, 2013, SWS conducted
a pre-election survey on voters’ preferences for senatorial HELD: NO. The names of those who commission or pay for
candidates. Thereafter, it published its findings. The following election surveys, including subscribers of survey firms, must
question was asked in the survey: be disclosed pursuant to Section 5.2(a) of the Fair Elections
Act. This requirement is a valid regulation in the exercise of
Kung ang eleksyon ay gaganapin ngayon, sino ang police power and effects the constitutional policy of
pinakamalamang ninyong iboboto bilang mga SENADOR ng “guaranteeing equal access to opportunities for public
PILIPINAS? Narito ang listahan ng mga kandidato. Paki- service.” Section 5.2(a)’s requirement of disclosing
shade o itiman po ang naaangkop na oval katabi ng pangalan subscribers neither curtails petitioners’ free speech rights nor
ng mga taong pinakamalamang ninyong iboboto. Maaari po violates the constitutional proscription against the impairment
kayong pumili ng hanggang labindalawang (12) kandidato. of contracts.

Rep. Tobias Tiangco, Secretary-General of the United Section 5.2(a) of the Fair Elections Act, read in a manner
Nationalist Alliance (UNA), wrote Atty. Esmeralda Ladra, consistent not only with its text but also with the purpose for
Director of COMELEC’s Law Department asking COMELEC which it, along with the Fair Elections Act, was adopted,
to “compel SWS to either comply with the directive in the Fair sustains COMELEC’s position. Republic Act No. 9006 was
Elections Act and COMELEC Resolution No. 9615 and give adopted with the end in mind of “guaranteeing or ensuring
the names or identities of the subscribers who paid for the pre- equal opportunity for public service” and to this end, stipulates
election survey conducted from February 15 to February 17, mechanisms for the “supervision or regulation of the
2013, or be liable for the violation thereof, an act constitutive enjoyment or utilization of all franchises or permits for the
of an election offense.” Tiangco recounted that he wrote to operation of media of communication or information.”
SWS requesting that he be furnished the identity of persons
who paid for the pre-election survey. SWS supposedly replied The Fair Elections Act provides means to realize the policy
to Tiangco, “furnishing him with some particulars about the articulated in Article II, Section 26 of the Constitution to
survey but without disclosing the identity of the persons who “guarantee equal access to opportunities for public service.”
commissioned or subscribed to the survey.” Article II, Section 26 models an understanding of Philippine
political and electoral reality as it sums up an aversion to the
Acting on Tiangco’s letter and on the COMELEC Law perpetuation of political power through electoral contests
Department’s recommendation, the COMELEC En Banc skewed in favor of those with resources to dominate the
issued the Order setting the matter for hearing and directed deliberative space in any media.
SWS and Pulse Asia to submit its Comment within three (3)
days of receipt. During the hearing, Chairman Sixto Brillantes Moreover, the constitutional desire to “guarantee equal
stated that the proceeding was merely a clarificatory hearing access to opportunities for public service” is the same intent
and not a formal hearing or an investigation. On April 23, 2013, that animates the Constitution’s investiture in COMELEC of
COMELEC issued the assailed Resolution No. 9674. As basis the power to “supervise or regulate the enjoyment or utilization
for Resolution No. 9674, COMELEC cited Article IX-C, Section of all franchises or permits for the operation of transportation
2(1) of the 1987 Constitution and Sections 5.1 to 5.3 of and other public utilities, media of communication or
Republic Act No. 9006, otherwise known as the Fair Elections information, all grants, special privileges, or concessions
Act, as implemented by COMELEC Resolution No. 9615. granted by the Government or any subdivision, agency, or
instrumentality thereof, including any GOCC or its subsidiary.”
SWS and Pulse Asia alleged that following the issuance of
Resolution No. 9674, they have not been furnished copies of The Fair Elections Act also governs published surveys during
Resolution No. 9674. They also articulated their view that elections. Section 5.1 defines election surveys as “the
Resolution No. 9674 was tainted with irregularities, having measurement of opinions and perceptions of the voters as
been issued ultra vires and in violation of the non-impairment regards a candidate’s popularity, qualifications, platforms or a
of contracts clause of the Constitution. Thus, they requested matter of public discussion in relation to the election, including
that COMELEC defer or hold in abeyance Resolution No. voters’ preference for candidates or publicly discussed issues
9674’s enforcement. during the campaign period.” Sections 5.2 and 5.3 provide
regulations that facilitate transparency with respect to election
COMELEC Law Department issued a Notice to SWS and surveys. Section 5.2 enumerates the information that a person
Pulse Asia directing it to furnish COMELEC with a list of the publishing an election survey must publish along with the
names of all commissioners, subscribers, and payors of survey itself.
surveys published and failure to comply with the Notice shall
constitute an election offense punishable under the Omnibus The inclusion of election surveys in the list of items regulated
Election Code. COMELEC then issued a Subpoena notifying by the Fair Elections Act is a recognition that election surveys
SWS and Pulse Asia that a Complaint “for violation of Section are not a mere descriptive aggregation of data. Publishing
264, pars. 1 and 2 of the Omnibus Election Code in relation to surveys are a means to shape the preference of voters, inform
R.A. No. 9006” was filed against them. SWS and Pulse Asia the strategy of campaign machineries, and ultimately, affect
maintained that before receiving the Subpoena, they were the outcome of elections. Election surveys have a similar
never informed that a criminal case had been filed against nature as election propaganda. They are expensive, normally
them. They added that they were never furnished copies of paid for by those interested in the outcome of elections, and
the relevant criminal Complaint. Hence, the case. have tremendous consequences on election results.

The bandwagon effect in election surveys published during


election periods create the “politics of expectations.” Voters

Page 84 of 186
act in accordance with what is perceived to be an existing or Petitioners have been complying with this without incident
emerging state of affairs with respect to how candidates are since the Fair Elections Act was enacted in 2001. After more
faring. Surveys can warp existing public opinion and can mold than a decade of compliance, it is odd for petitioners to
public opinion. Published election surveys offer valuable suddenly assail the disclosure requirement as unduly
insight into public opinion not just because they represent it burdensome or onerous.
but more so because they also tend to make it. Appreciating
this tendency to both entrench and marginalize is of relevance However, it is evident that Resolution No. 9674 was
in the context of Philippine political reality. This is the same promulgated in violation of the period set by the Fair Elections
reality that our policymakers, primarily the framers of the Act. Petitioners were also not served a copy of Resolution No.
Constitution, have seen fit to address. 9674 with which it was asked to comply. They were neither
shown nor served copies of the criminal Complaint subject of
To reiterate, the inclusion of published election surveys in a E.O. Case No. 13-222. Petitioners’ right to due process was,
statute that regulates election propaganda and other means thus, violated.
through which candidates may shape voter preferences is
itself telling of the recognition that published election surveys,
too, may influence voter preferences. This inclusion is
similarly telling of a recognition that, left unregulated, election
surveys can undermine the purposes of ensuring “fair”
elections. It is necessary that the Fair Elections Act be
appreciated for what it is: a mechanism for ensuring equality.
The Fair Elections Act is a means to effect the “necessary
condition” to a genuine democratic dialogue, to realizing a
deliberative democracy.

What is involved here is petitioners’ freedom of speech and of


expression, that is, to publish their findings. More specifically,
what is involved here is their right to political speech, that
which “refers to speech ‘both intended and received as a
contribution to public deliberation about some issue,’ ‘fostering
informed and civic-minded deliberation.’ The nature of the
speech involved, as well as the Fair Elections Act’s purpose
of ensuring political equality, calls into operation the equality-
based approach to weighing liberty to express vis-à-vis
equality of opportunities.

Regulation of speech in the context of electoral campaigns


made by persons who are not candidates or who do not speak
as members of a political party which are, taken as a whole,
principally advocacies of a social issue that the public must
consider during elections is unconstitutional. Such regulation
is inconsistent with the guarantee of according the fullest
possible range of opinions coming from the electorate
including those that can catalyze candid, uninhibited, and
robust debate in the criteria for the choice of a candidate.

Concededly, what are involved here are not election


propaganda per se. Election surveys, on their face, do not
state or allude to preferred candidates. Election surveys
become unambiguous only when viewed in relation to the end
for which they are employed. To those whose end is to get a
candidate elected, election surveys, when limited to their own
private consumption, are a means to formulate strategy. When
published, however, the tendency to shape voter preferences
comes into play. In this respect, published election surveys
partake of the nature of election propaganda. It is then
declarative speech in the context of an electoral campaign
properly subject to regulation. Hence, Section 5.2 of the Fair
Elections Act’s regulation of published surveys.

While it does regulate expression (i.e., petitioners’ publication


of election surveys), it does not go so far as to suppress
desired expression. There is neither prohibition nor
censorship specifically aimed at election surveys. The
freedom to publish election surveys remains. All Resolution
No. 9674 does is articulate a regulation as regards the manner
of publication, that is, that the disclosure of those who
commissioned and/or paid for, including those subscribed to,
published election surveys must be made.

In any case, the requirement of disclosing subscribers is


neither unduly burdensome nor onerous. Prior to the
promulgation of Resolution No. 9674, survey firms are already
understood to be bound by the requirement to disclose those
who commission or pay for published election surveys.

Page 85 of 186
8. GMA Network v. COMELEC (G.R. No. 205357, this would give life to the constitutional objective to equalize
September 2, 2014) access to media during elections. It sees this as a more
effective way of levelling the playing field between
candidates/political parties with enormous resources and
FACTS: The heart of the controversy revolves upon the those without much. Moreover, the COMELEC’s issuance of
proper interpretation of the limitation on the number of minutes the assailed Resolution is pursuant to Section 4, Article IX(C)
that candidates may use for television and radio of the Constitution which vests on the COMELEC the power
advertisements, as provided in Section 6 of R.A. No. 9006, to supervise and regulate, during election periods,
(Fair Election Act): transportation and other public utilities, as well as mass
media.
Sec. 6. Equal Access to Media Time and Space.—All
registered parties and bona fide candidates shall have equal Respondent also sees no prior restraint in the provisions
access to media time and space. The following guidelines may requiring notice to the COMELEC for appearances or
be amplified on by the COMELEC: guestings of candidates in bona fide news broadcasts. It
points out that the fact that notice may be given 24 hours after
6.2 (a) Each bona fide candidate or registered political party first broadcast only proves that the mechanism is for
for a nationally elective office shall be entitled to not more than monitoring purposes only, not for censorship. Further,
one hundred twenty (120) minutes of television advertisement respondent argues, that for there to be prior restraint, official
and one hundred eighty (180) minutes of radio advertisement governmental restrictions on the press or other forms of
whether by purchase or donation. expression must be done in advance of actual publication or
dissemination. Moreover, petitioners are only required to
(b) Each bona fide candidate or registered political party for a inform the COMELEC of candidates’/parties’ guestings, but
locally elective office shall be entitled to not more than sixty there is no regulation as to the content of the news or the
(60) minutes of television advertisement and ninety (90) expressions in news interviews or news documentaries.
minutes of radio advertisement whether by purchase or Respondent then emphasized that the Supreme Court has
donation. held that freedom of speech and the press may be limited in
light of the duty of the COMELEC to ensure equal access to
For this purpose, the COMELEC shall require any broadcast opportunities for public service.
station or entity to submit to the COMELEC a copy of its
broadcast logs and certificates of performance for the review
and verification of the frequency, date, time and duration of ISSUE: Does Section 9(a) of COMELEC Resolution No. 9615
advertisements broadcast for any candidate or political party. violate the petitioners’ right of speech?

During the previous elections of May 14, 2007 and May 10,
2010, COMELEC issued Resolutions implementing and HELD: YES. The authority of the COMELEC to impose airtime
interpreting Section 6 of R.A. No. 9006, regarding airtime limits directly flows from the Fair Election Act—120 minutes of
limitations, to mean that a candidate is entitled to the stated television advertisement and 180 minutes for radio
number of minutes “per station.” For the May 2013 elections, advertisement. For the 2013 elections, the COMELEC,
however, respondent COMELEC promulgated Resolution No. through Resolution No. 9615, chose to aggregate the total
9615 changing the interpretation of said candidates’ and broadcast time among the different broadcast media, thus:
political parties’ airtime limitation for political campaigns or
advertisements from a “per station” basis, to a “total Section 9. Requirements and/or Limitations on the Use of
aggregate” basis. Election Propaganda through Mass Media.—All parties and
bona fide candidates shall have equal access to media time
Petitioners ABS-CBN, ABC, GMA, MBC, NBN, and RMN, and space for their election propaganda during the campaign
owners/operators of radio and television networks in the period subject to the following requirements and/or limitations:
Philippines, and petitioner Kapisanan ng mga Brodkaster ng
Pilipinas (KBP), the national organization of broadcasting a. Broadcast Election Propaganda
companies in the Philippines representing operators of radio
and television stations and said stations themselves, sent their The duration of an airtime that a candidate, or party may use
respective letters to the COMELEC questioning the provisions for their broadcast advertisements or election propaganda
of the aforementioned Resolution, thus, the COMELEC held shall be, as follows: In cases where two or more candidates or
public hearings. Thereafter, respondent issued Resolution No. parties whose names, initials, images, brands, logos,
9631 amending provisions of Resolution No. 9615. insignias, color motifs, symbols, or forms of graphical
Nevertheless, petitioners still found the provisions representations are displayed, exhibited, used, or mentioned
objectionable and oppressive. together in the broadcast election propaganda or
advertisements, the length of time during which they appear
Petitioners posit that Section 9(a) of the assailed Resolution or are being mentioned or promoted will be counted against
provides for a very restrictive aggregate airtime limit and a the airtime limits allotted for the said candidates or parties and
vague meaning for a proper computation of “aggregate total” the cost of the said advertisement will likewise be considered
airtime, and violates the equal protection guarantee, thereby as their expenditures, regardless of whoever paid for the
defeating the intent and purpose of R.A. No. 9006. Petitioners advertisements or to whom the said advertisements were
contend that Section 9(a), which imposes a notice donated.
requirement, is vague and infringes on the constitutionally
protected freedom of speech, of the press and of expression, From the foregoing, it appears that COMELEC did not have
and on the right of people to be informed on matters of public any other basis for coming up with a new manner of
concern Also, Section 9(a) is a cruel and oppressive determining allowable time limits except its own idea as to
regulation as it imposes an unreasonable and almost what should be the maximum number of minutes based on its
impossible burden on broadcast mass media of monitoring a exercise of discretion as to how to level the playing field.
candidate’s or political party’s aggregate airtime, otherwise, it
may incur administrative and criminal liability. COMELEC is duty bound to come up with reasonable basis
for changing the interpretation and implementation of the
Respondent maintains that the per candidate rule or total airtime limits It could not simply adopt measures or regulations
aggregate airtime limit is in accordance with R.A. No. 9006 as just because it feels that it is the right thing to do, insofar as it

Page 86 of 186
might be concerned. It does have discretion, but such drastically curtailing the ability of a candidate to effectively
discretion is something that must be exercised within the reach out to the electorate would unjustifiably curtail his
bounds and intent of the law. The COMELEC is not free to freedom to speak as a means of connecting with the people.
simply change the rules especially if it has consistently
interpreted a legal provision in a particular manner in the past. It should be understandable that when an administrative rule
If ever it has to change the rules, the same must be properly is merely interpretative in nature, its applicability needs
explained with sufficient basis. nothing further than its bare issuance for it gives no real
consequence more than what the law itself has already
What the COMELEC came up with does not measure up to prescribed. When, upon the other hand, the administrative
that level of requirement and accountability which elevates rule goes beyond merely providing for the means that can
administrative rules to the level of respectability and facilitate or render least cumbersome the implementation of
acceptability. Those governed by administrative regulations the law but substantially adds to or increases the burden of
are entitled to a reasonable and rational basis for any changes those governed, it behooves the agency to accord at least to
in those rules by which they are supposed to live by, especially those directly affected a chance to be heard, and thereafter to
if there is a radical departure from the previous ones. The be duly informed, before that new issuance is given the force
COMELEC went beyond the authority granted it by the law in and effect of law.
adopting “aggregate” basis in the determination of allowable
airtime. The law, on its face, does not justify a conclusion that However, the legal duty of monitoring lies with the Comelec.
the maximum allowable airtime should be based on the totality Broadcast stations are merely required to submit certain
of possible broadcast in all television or radio stations. documents to aid the Comelec in ensuring that candidates are
not sold airtime in excess of the allowed limits. Further,
Section 9(a) of COMELEC Resolution No. 9615, with its pursuant to Resolution No. 9631, the respondent revised the
adoption of the “aggregate-based” airtime limits unreasonably third paragraph of Section 9(a).
restricts the guaranteed freedom of speech and of the press.
Political speech is one of the most important expressions
protected by the Fundamental Law. In regard to limitations on
political speech relative to other state interests, an American
case observed:

A restriction on the amount of money a person or group can


spend on political communication during a campaign
necessarily reduces the quantity of expression by restricting
the number of issues discussed, the depth of their exploration,
and the size of the audience reached. This is because virtually
every means of communicating ideas in today’s mass society
requires the expenditure of money. The electorate’s
increasing dependence on television, radio, and other mass
media for news and information has made these expensive
modes of communication indispensable instruments of
effective political speech.

The assailed rule on “aggregate-based” airtime limits is


unreasonable and arbitrary as it unduly restricts and
constrains the ability of candidates and political parties to
reach out and communicate with the people. Here, the
adverted reason for imposing the “aggregate-based” airtime
limits — leveling the playing field — does not constitute a
compelling state interest which would justify such a substantial
restriction on the freedom of candidates and political parties
to communicate their ideas, philosophies, platforms and
programs of government. And, this is specially so in the
absence of a clear-cut basis for the imposition of such a
prohibitive measure. In this particular instance, what the
COMELEC has done is analogous to letting a bird fly after one
has clipped its wings.

It is also particularly unreasonable and whimsical to adopt the


aggregate-based time limits on broadcast time when we
consider that the Philippines is not only composed of so many
islands. There are also a lot of languages and dialects spoken
among the citizens across the country. Accordingly, for a
national candidate to really reach out to as many of the
electorates as possible, then it might also be necessary that
he conveys his message through his advertisements in
languages and dialects that the people may more readily
understand and relate to. To add all of these airtimes in
different dialects would greatly hamper the ability of such
candidate to express himself — a form of suppression of his
political speech.

Respondent itself states that “television is arguably the most


cost-effective medium of dissemination. Even a slight increase
in television exposure can significantly boost a candidate’s
popularity, name recall and electability.” If that be so, then

Page 87 of 186
9. Diocese of Bacolod vs. COMELEC, 747 SCRA SCRA One of the arguments of the respondents is that the tarpaulin
1 (2015) Ponente: Justice Leonen is an election propaganda subject to regulation by COMELEC
pursuant to its mandate under Article IX-C, Section 4 of the
Constitution. Hence, the respondents claim that the issuances
FACTS: This is a case centered on whether the COMELEC ordering its removal for being oversized are valid and
has the competence to limit expressions made by citizens – constitutional.
who are not candidates – during elections.
On February 21, 2013, petitioners posted 2 tarpaulins within a
private compound housing the San Sebastian Cathedral of ISSUES:
Bacolod. Each tarpaulin was approximately 6 feet by 10 feet 1. Whether or not the COMELEC has legal basis to
in size. They were posted on the front walls of the cathedral regulate expressions made by private citizens (No)
within the public view. The first tarpaulin contains the message 2. Whether or not the petitioner’s fundamental right to
“IBASURA RH Law”. The second tarpaulin is the subject of the freedom of speech was violated by the issuance of
present case. This second tarpaulin contains the heading notice and letter for the removal of the tarpaulin (Yes)
“Conscience Vote” and lists candidates as either “(Anti-RH) 3. Whether or not large tarpaulins are part of expression
Team Buhay” with a check mark, or “Pro-RH) Team Patay” protected under Art. III, Sec. 4 of the Constitution (Yes)
with an “X” mark. Basically, the electoral candidates were 4. Whether or not the tarpaulin is an election propaganda,
classified according to their vote on the adoption of RH Law. being petitioner’s way of endorsing candidates who
voted against the RH Law (No)
TEAM BUHAY TEAM PATAY 5. Whether or not the COMELEC Law Department’s notice
Estrada, JV Angara, Juan Edgardo and letter removing petitioner’s tarpaulin valid and
Honasan, Gregorio Casiño, Teddy constitutional (No)
Magsaysay, Mitos Cayetano, Alan Peter
Pimentel, Koko Enrile, Jackie RULING:
Trillanes, Antonio Escudero, Francis
Villar, Cynthia Hontiveros, Risa PROCEDURAL ISSUES – Rule 65 is proper. This case may
Party List Buhay Legarda, Loren be heard and ruled by the Supreme Court.
Party List Ang Pamilya Party List Gabriela
Respondent assails that the notice and letter are not final
Party List Akbayan
orders of the COMELEC En Banc in the exercise of its
Party List Bayan Muna
adjudicatory powers, reviewable via Rule 64 of the ROC.
Party List Anak Pawis
The Court ruled that in the present case, petitioners are not
candidates seeking for public office. Their petition is filed to
During oral arguments, respondents conceded that the assert their fundamental right to expression.
tarpaulin was neither sponsored nor paid for by any candidate.
Petitioners also conceded that the tarpaulin contains names Furthermore, all these cases cited by respondents pertained
of candidates for the 2013 elections, but not of politicians who to COMELEC’s exercise of its adjudicatory or quasi-judicial
helped in the passage of the RH Law but were not candidates power. This case pertains to acts of COMELEC in the
for that election. implementation of its regulatory powers. When it issued the
notice and letter, the COMELEC was allegedly enforcing
On February 22, 2013, respondent Atty. Mavil V. Majarucon election laws.
as Election Officer of Bacolod City issued a notice to Remove
Campaign Materials within 3 days from receipt “for being It is clear that the subject matter of the controversy is the effect
oversized” addressed to Rev. Bishop Vicente M. Navarra. of COMELEC’s notice and letter on free speech. Certainly, a
COMELEC Resolution No. 9165 provides for the size breach of the fundamental right of expression by COMELEC
requirement of 2 feet by 3 feet. is a gave abuse of discretion. Thus, the constitutionality of the
notice and letter coming from COMELEC is within this court’s
On February 25, 2013, petitioners replied requesting that 1) power to review.
petitioner Bishop be given a definite ruling by the COMELEC
Law Department regarding the tarpaulin, and that 2) pending
this opinion and the availment of legal remedies, the tarpaulin
be allowed to remain. SUBSTANTIVE ISSUES –
COMELEC Law Department issued a letter ordering the 1. No. The Supreme Court ruled that COMELEC had no
immediate removal of the tarpaulin; otherwise, it will be legal basis to regulate expressions made by private
constrained to file an election offense against petitioners. citizens. Provisions [in the Constitution], laws and
Concerned about an imminent threat of prosecution for their jurisprudence pertain to candidates and political
exercise of free speech, petitioners initiated this case. They parties. Petitioners are not candidates. Neither do they
question the respondent’s notice and letter. They also prayed belong to any political party. COMELEC does not have
for: the authority to regulate the enjoyment of the preferred
1) the grant of their petition, right to freedom of expression exercised by a non-
2) temporary restraining order (TRO) and/or writ of preliminary candidate in this case.
injunction be issued restraining respondents from further
proceeding in enforcing their orders for the removal of the Respondents considered the tarpaulin as a campaign material
Team Patay tarpaulin and that in their issuances. The provisions asserted by the
3) after notice and hearing, a decision be rendered declaring respondents regulating the posting of campaign materials only
the questioned orders of respondents as unconstitutional and apply to candidates and political parties, and petitioners are
void, and permanently restraining respondents from enforcing neither of the two.
them or any other similar order.
Furthermore, the tarpaulin was not paid for by any candidate
Trial Court: Issued a TRO enjoining respondents from or political party. There was no allegation that petitioners
enforcing the assailed notice and letter. coordinated with any of the persons named in the tarpaulin

Page 88 of 186
regarding its posting. On the other hand, petitioners posted may respond to the claims by . . . either accepting the speech
the tarpaulin as part of their advocacy against the RH Law. act’s claims or opposing them with criticism or requests for
In this case, the tarpaulin contains speech on a matter of justification."
public concern, that is, a statement of either appreciation or
criticism on votes made in the passing of the RH law. Thus, Speech is not limited to vocal communication. "[C]onduct is
petitioners invoke their right to freedom of expression. treated as a form of speech sometimes referred to as
‘symbolic speech[,]’" such that "‘when ‘speech’ and
‘nonspeech’ elements are combined in the same course of
2. Yes. The Court ruled that: conduct,’ the ‘communicative element’ of the conduct may be
‘sufficient to bring into play the [right to freedom of
Fundamental to the consideration of this issue is Article III, expression].’"
Section 4 of the Constitution:
Section 4. No law shall be passed abridging the The right to freedom of expression, thus, applies to the entire
freedom of speech, of expression, or of the press, or continuum of speech from utterances made to conduct
the right of the people peaceably to assemble and enacted, and even to inaction itself as a symbolic manner of
petition the government for redress of grievances. communication.

No law. . . Freedom of speech includes the right to be silent. Aptly has it


been said that the Bill of Rights that guarantees to the
While it is true that the present petition assails not a law but individual the liberty to utter what is in his mind also
an opinion by the COMELEC Law Department, this court has guarantees to him the liberty not to utter what is not in his
applied Article III, Section 4 of the Constitution even to mind. The salute is a symbolic manner of communication that
governmental acts. conveys its message as clearly as the written or spoken word.
As a valid form of expression, it cannot be compelled any more
Our Constitution has also explicitly included the freedom of than it can be prohibited in the face of valid religious objections
expression, separate and in addition to the freedom of speech like those raised in this petition. To impose it on the petitioners
and of the press provided in the US Constitution. The word is to deny them the right not to speak when their religion bids
"expression" was added in the 1987 Constitution by them to be silent. This coercion of conscience has no place in
Commissioner Brocka for having a wider scope: the free society.

MR. BROCKA: This is a very minor amendment, Mr. The democratic system provides for the accommodation of
Presiding Officer. On Section 9, page 2, line 29, it diverse ideas, including the unconventional and even the
says: "No law shall be passed abridging the freedom bizarre or eccentric. The will of the majority prevails, but it
of speech." I would like to recommend to the cannot regiment thought by prescribing the recitation by rote
Committee the change of the word "speech" to of its opinions or proscribing the assertion of unorthodox or
EXPRESSION; or if not, add the words AND unpopular views as in this case. The conscientious objections
EXPRESSION after the word "speech," because it is of the petitioners, no less than the impatience of those who
more expansive, it has a wider scope, and it would disagree with them, are protected by the Constitution. The
refer to means of expression other than speech. State cannot make the individual speak when the soul within
rebels.
THE PRESIDING OFFICER (Mr.Bengzon): What
does the Committee say? Even before freedom "of expression" was included in Article
III, Section 4 of the present Constitution, this court has applied
FR. BERNAS: "Expression" is more broad than its precedent version to expressions other than verbal
speech. We accept it. utterances.

MR. BROCKA: Thank you.


3. Yes. The Court ruled that size does matter. The form of
THE PRESIDING OFFICER (Mr.Bengzon): Is it expression is just as important as the information
accepted? conveyed that it forms part of the expression.

FR. BERNAS: Yes. The court ruled that size matters because:
A) It enhances efficiency in communication.
THE PRESIDING OFFICER (Mr.Bengzon): Is there B) The size of tarpaulin may underscore the importance of the
any objection? (Silence) The Chair hears none; the message to the reader
amendment is approved. C) Lager spaces allow for more messages. Larger spaces
may translate to more opportunities to amplify and argue
FR. BERNAS: So, that provision will now read: "No points which the speakers might want to communicate.
law shall be passed abridging the freedom of speech,
expression or of the press . . . ." Speech may be said These points become more salient when it is the electorate,
to be inextricably linked to freedom itself as "[t]he not the candidates or the political parties, that speaks. Too
right to think is the beginning of freedom, and speech often, the terms of public discussion during elections are
must be protected from the government because framed and kept hostage by brief and catchy but meaningless
speech is the beginning of thought." sound bites extolling the character of the candidate. Worse,
elections sideline political arguments and privilege the
endorsement by celebrities. Rather than provide obstacles to
Communication and Speech their speech, government should in fact encourage it.
Between the candidates and the electorate, the latter have the
Communication is an essential outcome of protected speech. better incentive to demand discussion of the more important
Communication exists when "(1) a speaker, seeking to signal issues. Between the candidates and the electorate, the former
others, uses conventional actions because he or she have better incentives to avoid difficult political standpoints
reasonably believes that such actions will be taken by the and instead focus on appearances and empty promises.
audience in the manner intended; and (2) the audience so
takes the actions." "[I]n communicative action[,] the hearer

Page 89 of 186
Large tarpaulins, therefore, are not analogous to time and suffer and what not. But the subject of this autonomy
place. They are fundamentally part of expression protected is never the contingent, private individual as that
under Article III, Section 4 of the Constitution. which he actually is or happens to be; it is rather the
individual as a human being who is capable of being
free with the others. And the problem of making
4. No. The Supreme Court held that the tarpaulin is not an possible such a harmony between every individual
election propaganda. liberty and the other is not that of finding a
compromise between competitors, or between
“The term "political advertisement" or "election propaganda" freedom and law, between general and individual
refers to any matter broadcasted, published, printed, interest, common and private welfare in an
displayed or exhibited, in any medium, which contain the established society, but of creating the society in
name, image, logo, brand, insignia, color motif, initials, and which man is no longer enslaved by institutions
other symbol or graphic representation that is capable of being which vitiate self-determination from the beginning.
associated with a candidate or party, and is intended to draw In other words, freedom is still to be created even for
the attention of the public or a segment thereof to promote or the freest of the existing societies.277 (Emphasis in
oppose, directly or indirectly, the election of the said candidate the original)
or candidates to a public office.” It is clear that this paragraph
suggests that personal opinions are not included, while Marcuse suggests that the democratic argument — with all
sponsored messages are covered. opinions presented to and deliberated by the people —
While the tarpaulin may influence the success or failure of the "implies a necessary condition, namely, that the people must
named candidates and political parties, this does not be capable of deliberating and choosing on the basis of
necessarily mean it is election propaganda. The tarpaulin was knowledge, that they must have access to authentic
not paid for or posted "in return for consideration" by any information, and that, on this basis, their evaluation must be
candidate, political party, or party-list group. the result of autonomous thought."278 He submits that
"[d]ifferent opinions and ‘philosophies’ can no longer compete
Long Note: peacefully for adherence and persuasion on rational grounds:
the ‘marketplace of ideas’ is organized and delimited by those
Speech and equality: who determine the national and the individual interest."279 A
slant toward left manifests from his belief that "there is a
Some considerations We first establish that there are two ‘natural right’ of resistance for oppressed and overpowered
paradigms of free speech that separate at the point of giving minorities to use extralegal means if the legal ones have
priority to equality vis-à-vis liberty. proved to be inadequate."280 Marcuse, thus, stands for an
equality that breaks away and transcends from established
In an equality-based approach, "politically disadvantaged hierarchies, power structures, and indoctrinations. The
speech prevails over regulation[,] but regulation promoting tolerance of libertarian society he refers to as "repressive
political equality prevails over speech." This view allows the tolerance."
government leeway to redistribute or equalize ‘speaking
power,’ such as protecting, even implicitly subsidizing,
unpopular or dissenting voices often systematically subdued Long Note #2:
within society’s ideological ladder. This view acknowledges
that there are dominant political actors who, through authority, There says that there are two views, right?
power, resources, identity, or status, have capabilities that
may drown out the messages of others. This is especially true The first one is: “we have acknowledged the Constitution’s
in a developing or emerging economy that is part of the guarantee for more substantive expressive freedoms that take
majoritarian world like ours. equality of opportunities into consideration during elections.”
The other view is: that considerations of equality of
opportunity or equality in the ability of citizens as speakers
The question of libertarian tolerance: should not have a bearing in free speech doctrine. Under this
view, "members of the public are trusted to make their own
This balance between equality and the ability to express so as individual evaluations of speech, and government is forbidden
to find one’s authentic self or to participate in the self to intervene for paternalistic or redistributive reasons . . .
determination of one’s communities is not new only to law. It [thus,] ideas are best left to a freely competitive ideological
has always been a philosophical problematique. market." This is consistent with the libertarian suspicion on
the use of viewpoint as well as content to evaluate the
In his seminal work, Repressive Tolerance, philosopher and constitutional validity or invalidity of speech.
social theorist Herbert Marcuse recognized how
institutionalized inequality exists as a background limitation,
rendering freedoms exercised within such limitation as merely When private speech amounts to election paraphernalia
"protect[ing] the already established machinery of
discrimination." In his view, any improvement "in the normal The traditional view has been to tolerate the viewpoint of the
course of events" within an unequal society, without speaker and the content of his or her expression. This view,
subversion, only strengthens existing interests of those in thus, restricts laws or regulation that allows public officials to
power and control. make judgments of the value of such viewpoint or message
content. This should still be the principal approach.
In other words, abstract guarantees of fundamental rights like However, the requirements of the Constitution regarding
freedom of expression may become meaningless if not taken equality in opportunity must provide limits to some expression
in a real context. This tendency to tackle rights in the abstract during electoral campaigns.
compromises liberties. In his words: Thus clearly, regulation of speech in the context of electoral
campaigns made by candidates or the members of their
Liberty is self-determination, autonomy—this is political parties or their political parties may be regulated as to
almost a tautology, but a tautology which results from time, place, and manner. This is the effect of our rulings in
a whole series of synthetic judgments. It stipulates Osmeña v. COMELEC and National Press Club v.
the ability to determine one’s own life: to be able to COMELEC.
determine what to do and what not to do, what to

Page 90 of 186
Regulation of speech in the context of electoral campaigns primordial right by our Constitution. The expression in the
made by persons who are not candidates or who do not speak medium chosen by petitioners deserves our protection.
as members of a political party which are, taken as a whole,
principally advocacies of a social issue that the public must Petition GRANTED.
consider during elections is unconstitutional. Such regulation
is inconsistent with the guarantee of according the fullest
possible range of opinions coming from the electorate
including those that can catalyze candid, uninhibited, and
robust debate in the criteria for the choice of a candidate.

This does not mean that there cannot be a specie of speech


by a private citizen which will not amount to an election
paraphernalia to be validly regulated by law.

Regulation of election paraphernalia will still be


constitutionally valid if it reaches into speech of persons who
are not candidates or who do not speak as members of a
political party if they are not candidates, only if what is
regulated is declarative speech that, taken as a whole, has for
its principal object the endorsement of a candidate only. The
regulation (a) should be provided by law, (b) reasonable, (c)
narrowly tailored to meet the objective of enhancing the
opportunity of all candidates to be heard and considering the
primacy of the guarantee of free expression, and (d)
demonstrably the least restrictive means to achieve that
object. The regulation must only be with respect to the time,
place, and manner of the rendition of the message. In no
situation may the speech be prohibited or censored on the
basis of its content. For this purpose, it will not matter whether
the speech is made with or on private property.

This is not the situation, however, in this case for two reasons.
First, as discussed, the principal message in the twin
tarpaulins of petitioners consists of a social advocacy.
Second, as pointed out in the concurring opinion of Justice
Antonio Carpio, the present law — Section 3.3 of Republic Act
No. 9006 and Section 6(c) of COMELEC Resolution No. 9615
— if applied to this case, will not pass the test of reasonability.
A fixed size for election posters or tarpaulins without any
relation to the distance from the intended average audience
will be arbitrary. At certain distances, posters measuring 2 by
3 feet could no longer be read by the general public and,
hence, would render speech meaningless. It will amount to the
abridgement of speech with political consequences.

5. No. The Court ruled that the COMELEC Law


Department’s notice and letter removing petitioner’s
tarpaulin is not valid and is unconstitutional.

COMELEC’s general role includes a mandate to ensure equal


opportunities and reduce spending among candidates and
their registered political parties. It is not to regulate or limit the
speech of the electorate as it strives to participate in the
electoral exercise.

Embedded in the tarpaulin, however, are opinions expressed


by petitioners. It is a specie of expression protected by our
fundamental law. It is an expression designed to invite
attention, cause debate, and hopefully, persuade. It may be
motivated by the interpretation of petitioners of their
ecclesiastical duty, but their parishioner’s actions will have
very real secular consequences. Certainly, provocative
messages do matter for the elections.

What is involved in this case is the most sacred of speech


forms: expression by the electorate that tends to rouse the
public to debate contemporary issues. This is not speech by
candidates or political parties to entice votes. It is a portion of
the electorate telling candidates the conditions for their
election. It is the substantive content of the right to suffrage.
This. is a form of speech hopeful of a quality of democracy that
we should all deserve. It is protected as a fundamental and

Page 91 of 186
10. 1-United Transport Koalisyon (1-Utak) vs. the right to free speech of the private owners of PUVs and
COMELEC, 755 SCRA 441 Ponente: Justice Reyes transport terminals. The petitioner then requested the
COMELEC to reconsider the implementation of the assailed
provisions and allow private owners of PUVs and transport
FACTS: On February 12, 2001, Republic Act (R.A.) No. 9006, terminals to post election campaign materials on their vehicles
otherwise known as the "Fair Elections Act", was passed. and transport terminals.
Section 9 thereof provides:
On February 5, 2013, the COMELEC en banc issued Minute
Resolution No. 13-0214,5 which denied the petitioner's
Sec. 9. Posting of Campaign Materials. - The request to reconsider the implementation of Section 7(g) items
COMELEC may authorize political parties and party- (5) and (6), in relation to Section 7(f), of Resolution No. 9615.
list groups to erect common poster areas for their
candidates in not more than ten (10) public places The COMELEC en banc opined that Under the Constitution
such as plazas, markets, barangay centers and the itself, Section 6, Article XII, the use of property bears a social
like, wherein candidates can post, display or exhibit function and all economic agents shall contribute to the
election propaganda: Provided that the size of the common good; and there is no higher Common good than that
poster areas shall not exceed twelve (12) by sixteen as espoused in RA. 9006 – the equalization of opportunities
(16) feet or its equivalent. for all candidates for political office during elections – a policy
which Res. No. 9615 merely implements.
Independent candidates with no political parties may
likewise be authorized to erect common poster areas The exact purpose for placing political advertisements on a
in not more than ten (10) public places, the size of PUV or in transport terminals is exactly because it is public
which shall not exceed four (4) by six (6) feet or its and can be seen by all; and although it is true that private
equivalent. vehicles ply the same route as public vehicles, the exposure
of a [PUV] servicing the general, riding public is much more
Candidates may post any lawful propaganda compared to private vehicles. Categorizing PUVs and
material in private places with the consent of the transport terminals as 'public places' under Section 7 (f) of
owner thereof, and in public places or property which Reso. No. 9615 is therefore logical. The same reasoning for
shall be allocated equitably and impartially among limiting political advertisements in print media, in radio, and in
the candidates. television therefore holds true for political advertisements in
PUVs and transport terminals.
On January 15, 2013, the COMELEC promulgated Resolution
No. 9615, which provided for the rules implementing R.A. No. Petitioner’s argument: The petitioner maintains that Section
9006 in connection with the May 13, 2013 national and local 7(g) items (5) and (6), in relation to Section 7(f), of Resolution
elections and subsequent elections. Section 7 thereof, which No. 9615 violate the right to free speech of the owners of
enumerates the prohibited forms of election propaganda, PUVs and transport terminals; that the prohibition curtails their
pertinently provides: ideas of who should be voted by the public. Further, assuming
that substantial public interest exists in the said prohibition
SEC. 7. Prohibited Forms of Election Propaganda. - imposed under Resolution No. 9615, the petitioner claims that
During the campaign period, it is unlawful: the curtailment of the right to free speech of the owners of
PUVs and transport terminals is much greater than is
xxxx necessary to achieve the desired governmental purpose, i.e.,
ensuring equality of opportunity to all candidates in elective
(f) To post, display or exhibit any election campaign office.
or propaganda material outside of authorized
common poster areas, in public places, or in private Respondent’s argument: COMELEC points out that PUVs and
properties without the consent of the owner thereof. private transport terminals hold a captive audience - the
commuters, who have no choice but be subjected to the blare
(g) Public places referred to in the previous of political propaganda. Thus, the COMELEC avers, it is within
subsection (f) include any of the following: its constitutional authority to prevent privately-owned PUVs
xxxx and transport terminals from concurrently serving campaign
materials to the captive audience that they transport.
5. Public utility vehicles such as buses, jeepneys,
trains, taxi cabs, ferries, pedicabs and tricycles, The COMELEC further claims that Resolution No. 9615 is a
whether motorized or not; valid content-neutral regulation and, thus, does not impinge
on the constitutional right to freedom of speech. It avers that
6. Within the premises of public transport terminals, such the assailed regulation is within the constitutional power of the
as bus terminals, airports, seaports, docks, piers, train COMELEC pursuant to Section 4, Article IX-C of the
stations, and the like. Constitution. The COMELEC alleges that the regulation
simply aims to ensure equal campaign opportunity, time, and
The violation of items [5 and 6] under subsection (g) shall space for all candidates - an important and substantial
be a cause for the revocation of the public utility franchise governmental interest, which is totally unrelated to the
and will make the owner and/or operator of the suppression of free expression; that any restriction on free
transportation service and/or terminal liable for an election speech is merely incidental and is no greater than is essential
offense under Section 9 of Republic Act No. 9006 as to the furtherance of the said governmental interest.
implemented by Section 18 (n) of these Rules.

In its letter dated January 30, 2013, the petitioner, through its ISSUE: Whether or not Section 7(g) items (5) and (6), in
president, Melencio F. Vargas, sought clarification from the relation to Section 7(f), of Resolution No. 9615 which prohibits
COMELEC as regards the application of Resolution No. 9615, the posting of any election campaign or propaganda material
particularly Section 7(g) items (5) and (6), in relation to Section in PUVs and public transport terminals are constitutional.
7(f), vis-a-vis privately owned public utility vehicles (PUVs)
and transport terminals. The petitioner explained that the
prohibition stated in the aforementioned provisions impedes

Page 92 of 186
RULING: The Supreme Court held that the said provisions of The COMELEC may only regulate the franchise or permit to
Resolution No. 9615 are null and void for being repugnant to operate and not the ownership per se of PUVs and transport
Sections 1 and 4, Article III of the 1987 Constitution. terminals.

Section 7(g) items (5) and (6), in relation to Section 7(f), of In the instant case, the Court further delineates the
Resolution No. 9615 are prior restraints on speech constitutional grant of supervisory and regulatory powers to
the COMELEC during an election period. As worded, Section
Free speech may be identified with the liberty to discuss 4, Article IX-C of the Constitution only grants COMELEC
publicly and truthfully any matter of public concern without supervisory and regulatory powers over the enjoyment or
prior restraint or censorship and subsequent punishment. utilization “of all franchises or permits for the operation,” inter
Prior restraint refers to official governmental restrictions on the alia, of transportation and other public utilities. The
press or other forms of expression in advance of actual COMELEC’s constitutionally delegated powers of supervision
publication or dissemination. Freedom from prior restraint is and regulation do not extend to the ownership per se of PUVs
largely freedom from government censorship of publications, and transport terminals, but only to the franchise or permit to
whatever the form of censorship, and regardless of whether it operate the same.
is wielded by the executive, legislative or judicial branch of the
government. Any system of prior restraints of expression Section 7(g) items (5) and (6) of Resolution No. 9615 are not
comes to this Court bearing a heavy presumption against its within the constitutionally delegated power of the COMELEC
validity. to supervise or regulate the franchise or permit to operate of
transportation utilities. The posting of election campaign
Section 7(g) items (5) and (6), in relation to Section 7(f), of material on vehicles used for public transport or on transport
Resolution No. 9615 unduly infringe on the fundamental right terminals is not only a form of political expression, but also an
of the people to freedom of speech. Central to the prohibition act of ownership – it has nothing to do with the franchise or
is the freedom of individuals, i.e., the owners of PUVs and permit to operate the PUV or transport terminal.
private transport terminals, to express their preference,
through the posting of election campaign material in their Section 7(g) items (5) and (6) of Resolution No. 9615 are not
property, and convince others to agree with them. justified under the captive-audience doctrine.

Pursuant to the assailed provisions of Resolution No. 9615, The captive-audience doctrine states that when a listener
posting an election campaign material during an election cannot, as a practical matter, escape from intrusive speech,
period in PUVs and transport terminals carries with it the the speech can be restricted. The “captive-audience” doctrine
penalty of revocation of the public utility franchise and shall recognizes that a listener has a right not to be exposed to an
make the owner thereof liable for an election offense. unwanted message in circumstances in which the
The prohibition constitutes a clear prior restraint on the right communication cannot be avoided.
to free expression of the owners of PUVs and transport
terminals. As a result of the prohibition, owners of PUVs and A regulation based on the captive-audience doctrine is in the
transport terminals are forcefully and effectively inhibited from guise of censorship, which undertakes selectively to shield the
expressing their preferences under the pain of indictment for public from some kinds of speech on the ground that they are
an election offense and the revocation of their franchise or more offensive than others. Such selective restrictions have
permit to operate. been upheld only when the speaker intrudes on the privacy of
the home or the degree of captivity makes it either impossible
The assailed prohibition on posting election campaign or impractical for the unwilling viewer or auditor to avoid
materials is an invalid content-neutral regulation repugnant to exposure.
the free speech clause.
Thus, a government regulation based on the captive-audience
A content-neutral regulation, i.e., which is merely concerned doctrine may not be justified if the supposed “captive
with the incidents of the speech, or one that merely controls audience” may avoid exposure to the otherwise intrusive
the time, place or manner, and under well-defined standards, speech. The prohibition under Section 7(g) items (5) and (6)
is constitutionally permissible, even if it restricts the right to of Resolution No. 9615 is not justified under the captive-
free speech, provided that the following requisites concur: audience doctrine; the commuters are not forced or compelled
1. The government regulation is within the to read the election campaign materials posted on PUVs and
constitutional power of the Government; transport terminals. Nor are they incapable of declining to
2. It furthers an important or substantial governmental receive the messages contained in the posted election
interest; campaign materials since they may simply avert their eyes if
3. The governmental interest is unrelated to the they find the same unbearably intrusive.
suppression of free expression; and
4. The incidental restriction on freedom of expression is Prohibiting owners of PUVs and transport terminals from
no greater than is essential to the furtherance of that interest. posting election campaign materials violates the equal
Section 7(g) items (5) and (6) of Resolution No. 9615 are protection clause.
content-neutral regulations since they merely control the place
where election campaign materials may be posted. However, Section 7(g) items (5) and (6) of Resolution No. 9615 do not
the prohibition is still repugnant to the free speech clause as it only run afoul of the free speech clause, but also of the equal
fails to satisfy all of the requisites for a valid content-neutral protection clause. One of the basic principles on which this
regulation. government was founded is that of the equality of right, which
is embodied in Section 1, Article III of the 1987 Constitution.
Section 7(g) items (5) and (6), in relation to Section 7(f), of "Equal protection requires that all persons or things similarly
Resolution No. 9615, are not within the constitutionally situated should be treated alike, both as to rights conferred
delegated power of the COMELEC under Section 4, Article IX- and responsibilities imposed. Similar subjects, in other words,
C of the Constitution. Also, there is absolutely no necessity to should not be treated differently, so as to give undue favor to
restrict the right to free speech of the owners of PUVs and some and unjustly discriminate against others.”
transport terminals.
In order that there can be valid classification so that a
discriminatory governmental act may pass the constitutional
norm of equal protection, it is necessary that the four

Page 93 of 186
requisites of valid classification be complied with, namely: (1) 11. United States vs. Alvarez, 567 U.S. (No. 11-210, 28
it must be based upon substantial distinctions; (2) it must be June 2012)
germane to the purposes of the law; (3) it must not be limited
to existing conditions only; and (4) it must apply equally to all
members of the class.46 FACTS: In 2007, Xavier Alvarez, an elected member of a
water district board in California, identified himself at a public
It is conceded that the classification under Section 7(g) items meeting as a retired U.S. Marine who had been wounded in
(5) and (6) of Resolution No. 9615 is not limited to existing combat many times and had received the Congressional
conditions and applies equally to the members of the Medal of Honor.
purported class. However, the classification remains
constitutionally impermissible since it is not based on "I'm a retired Marine of 25 years. I retired in the year 2001,"
substantial distinction and is not germane to the purpose of Mr. Alvarez said at a public meeting of the board. "Back in
the law. 1987, I was awarded the Congressional Medal of Honor. I got
wounded many times by the same guy."

NOTE: None of Alvarez's claims was true. He never served in the


On a final note, it bears stressing that the freedom to advertise Marine Corps or any branch of the military, was never
one's political candidacy is clearly a significant part of our wounded in combat, and has never received a medal of any
freedom of expression. A restriction on this freedom without kind, including the nation's highest military award – the Medal
rhyme or reason is a violation of the most valuable feature of of Honor. Alvarez had previously boasted, untruly, that he
the democratic way of life. played hockey for the Detroit Red Wings and that he once
married a starlet from Mexico.

After FBI agents obtained a tape recording of the meeting,


federal prosecutors charged Alvarez with two counts of
violating the Stolen Valor Act (Act punishing lying about
receiving military medals or honors).

Alvarez's lawyer argued that the Stolen Valor Act was invalid
under the First Amendment and, therefore, the case should be
dismissed.

The trial court rejected this argument. Alvarez was tried and
convicted in the United States District Court for the Central
District of California. He was sentenced to probation for three
years and ordered to pay a $5,000 fine. He was the first
person convicted under the Stolen Valor Act.

Alvarez appealed the First Amendment issue, claiming that


the Stolen Valor law violated the First Amendment’s
guarantee of the right to free speech and, therefore, his
conviction was unlawful. A three-judge panel of the United
States Court of Appeals for the Ninth Circuit agreed with
Alvarez and reversed his conviction, declaring the Stolen
Valor Act unconstitutional in a vote of 2-to-1.

The government appealed the case to the Supreme Court of


the United States, which agreed to hear it. After the Court
agreed to hear the case, the United States Court of Appeals
for the Tenth Circuit, ruling in a different case, declared the
Stolen Valor Act constitutional in a vote of 2-to-1.

Oral arguments were heard on February 22, 2012.

ISSUE: Whether or not the Stolen Valor Act, which makes it a


crime to lie about receiving military medals or honors, violates
the First Amendment’s guarantee of the right to free speech

RULING: Yes. The Supreme Court held that the Stolen


Valor’s Act’s prohibition against making false statements of
having been awarded a military medal violated the First
Amendment. However, 6 justices in the majority could not
agree on a single rationale for the decision.

JUSTICE KENNEDY, joined by THE CHIEF JUSTICE,


JUSTICE GINSBURG, and JUSTICE SOTOMAYOR,
concluded that the Act infringes upon speech protected by the
First Amendment. Pp. 3–18.

(a) The Constitution “demands that content-based


restrictions on speech be presumed invalid . . . and
that the Government bear the burden of showing

Page 94 of 186
their constitutionality.” Ashcroft v. American Civil 12. Reno vs. American Civil Liberties Union, 521 U.S.
Liberties Union, 542 U. S. 656, 660. 844 (1997)

Content-based restrictions on speech have been permitted


only for a few historic categories of speech, including FACTS: Several litigants challenged the constitutionality of
incitement, obscenity, defamation, speech integral to criminal two provisions in the 1996 Communications Decency Act.
conduct, so-called “fighting words,” child pornography, fraud, Intended to protect minors from unsuitable internet material,
true threats, and speech presenting some grave and imminent the Act criminalized the intentional transmission of “obscene
threat the Government has the power to prevent. Absent from or indecent” messages as well as the transmission of
these few categories is any general exception for false information which depicts or describes “sexual or excretory
statements. The Government argues that cases such as activities or organs” in a manner deemed “offensive” by
Hustler Magazine, Inc., v. Falwell, 485 U. S. 46, 52, support community standards. After being enjoined by a District Court
its claim that false statements have no value and hence no from enforcing the above provisions, except for the one
First Amendment protection. But all the Government’s concerning obscenity and its inherent protection against child
quotations derive from cases discussing defamation, fraud, or pornography, Attorney General Janet Reno appealed directly
some other legally cognizable harm associated with a false to the Supreme Court as provided for by the Act’s special
statement. review provisions.

In those decisions the falsity of the speech at issue was not


irrelevant to the Court’s analysis, but neither was it ISSUE: Whether certain provisions of the 1996
determinative. These prior decisions have not confronted a Communications Decency Act violate the First and Fifth
measure, like the Stolen Valor Act, that targets falsity and Amendments by being overly broad and vague in their
nothing more. definitions of the types of internet communications which they
criminalized.
Even when considering some instances of defamation or
fraud, the Court has instructed that falsity alone may not
suffice to bring the speech outside the First Amendment; the RULING: Yes. The Court held that the Act violated the First
statement must be a knowing and reckless falsehood. Amendment because its regulations amounted to a content-
based blanket restriction of free speech. The Act failed to
clearly define “indecent” communications, limit its restrictions
to particular times or individuals (by showing that it would not
impact adults), provide supportive statements from an
authority on the unique nature of internet communications, or
conclusively demonstrate that the transmission of “offensive”
material is devoid of any social value. The Court added that
since the First Amendment distinguishes between “indecent”
and “obscene” sexual expressions, protecting only the former,
the Act could be saved from facial overbreadth challenges if it
dropped the words “or indecent” from its text. The Court
refused to address any Fifth Amendment issues.

Page 95 of 186
Upon the facts, and under applicable laws and principles, the
13. Social Weather Stations, Inc. vs. Asuncion, 288 complaint fails to make a prima facie showing of the charges
SCRA xi (1993) made therein, and must perforce be as it is thereby,
DISMISSED.
FACTS: Published under the by-line of one Marichu
Villanueva and titled “Judiciary worse than PNP”, an item in
that June 17, 1993 issue of the Manila Standard, a
metropolitan daily, reported that the results of the latest
opinion polls conducted by the Ateneo Social Weather Station,
as Social Weather Stations, Inc. (SWS) is also known, showed
the Judiciary to have an even lower satisfaction rating than the
PNP. The item went on to state that the President and his
Cabinet had been briefed on the results of the survey by the
Professors Mahar Mangahas and Felipe Miranda of the SWS,
and that Malacanang had expressed concern over the
Judiciary’s law standing. Press Secretary Jesus Sison was
also quoted as saying that this was “most puzzling”, although
he could not recall the exact rating, noting only that the PNP
had “a better image than the judiciary”.

Said report appears to have prompted Judge Maximiano C.


Asuncion, presiding judge of Branch 104 of the RTC at
Quezon City, motu proprio to initiate on the same date of June
17, 1993 proceedings ordering the President of the SWS to
“explain why you should not be held in contempt or distributing
to the general public without prior permission from any court
your findings that the people have more confidence with the
police than with judges thereby tending directly or indirectly to
degrade the administration of justice.”

On June 21, 1993, Prof. Mahar Mangahas through Atty.


Antonio M. Abad, Jr. submitted his comment and explanation
that it was not true that the SWS distributed to the general
public the alleged survey. Said survey was privately given to
Pres. Ramos and the Cabinet and was not intended for
publication nor for public consumption and that if ever it
reaches the media, he had not authorized anyone to do so.
The hearing was scheduled on June 23, 1993, after
which Judge Asuncion promulgated an Order dated July 2,
1993, finding Professor Mangahas’ explanation satisfactory
and dismissing the contempt charge against him.

After three weeks or so, or more precisely on July 26, 1993,


Professor Managahas addressed a a letter to the Chief Justice
intended “as a formal complaint against Honorable Maximiano
C. Asuncion for grave abuse of authority and gross ignorance
of the law, in connection with his issuance of an Order dated
June 17, 1993.

ISSUE: Whether the Order dated June 17, 1993 is violative of


the constitutional guaranteed of freedom of speech and
freedom from prior restraint.

RULING: No.

What was clearly implicit in the newspaper report about the


results of the SWS poll – in the words of Judge Asuncion, “that
the people have more confidence with the police than with the
judges” – in light of this fact, of which judicial notice is taken,
that said report came out at a time when there already was
widespread publicity adverse to the judiciary, there can be no
doubt of its clear tendency to degrade the administration of
justice.

Thus, Judge Asuncion can hardly be faulted for what, at a


minimum, he must have felt duty-bound to do in the
circumstances. No question of prior restraint or violation of the
guarantee of free speech arises here, what he did being, in
essence, merely to initiate an inquiry into the source and basis
of the derogatory news report. And he forthwith abated the
proceedings upon receiving an explanation he deemed
satisfactory.

Page 96 of 186
14. RE: LETTER OF THE UP LAW FACULTY ENTITLED citizens of a democracy, and an exercise of academic
"RESTORING INTEGRITY” (A.M. No. 10-10-4-SC. freedom.
March 8, 2011)

ISSUE: Does the Show Cause Resolution deny respondents


FACTS: On 28 April 2010, the decision of the case Vinuya v their freedom of expression?
Executive Secretary was promulgated with Justice Mariano
del Castillo as its ponente. Motion for reconsideration was
filed by the petitioner’s counsel on various grounds but most HELD: No.
notably on the ground that not only did the ponente of the case
plagiarised at least 3 books and articles in discussing the It is respondents’ collective claim that the Court, with the
principles of jus cogens and erga omnes, but have also twisted issuance of the Show Cause Resolution, has interfered with
such quotations making it appear contrary to the intent of the respondents’ constitutionally mandated right to free speech
original works. The authors and their purportedly plagiarized and expression. It appears that the underlying assumption
articles are: 1) Evan J Criddle and Evan Fox-Decent’s A behind respondents’ assertion is the misconception that this
Fiduciary Theory of Jus Cogens published in 2009 in the Yale Court is denying them the right to criticize the Court’s
Journal of International Law; 2) Christian J. Tams’ Enforcing decisions and actions, and that this Court seeks to "silence"
Erga Omnes Obligations in Internation Law published by the respondent law professors’ dissenting view on what they
Cambridge University Press in 2005; and 3) Mark Ellis’ characterize as a "legitimate public issue."
Breaking the Silence: On Rape as an International Crime
published in the Case Western Reserve Journal of Internation This is far from the truth. A reading of the Show Cause
Law in 2006. Thereafter, news regarding the plagiarism by Resolution will plainly show that it was neither the fact that
the Supreme Court spread over the media and the original respondents had criticized a decision of the Court nor that they
authors wrote letters to the Chief Justice expressing had charged one of its members of plagiarism that motivated
discontent by the questioned act of Justice del Castillo. the said Resolution. It was the manner of the criticism and the
contumacious language by which respondents, who are not
On 27 July 2010, the UP College of Law faculty members gave parties nor counsels in the Vinuya case, have expressed their
their opinion on the matter of plagiarism by issuing an article opinion in favor of the petitioners in the said pending case for
titled “Restoring Integrity: A statement by the Faculty of the the "proper disposition" and consideration of the Court that
University of the Philippines College of Law on the Allegations gave rise to said Resolution. The Show Cause Resolution
of Plagiarism and Misrepresentation in the Supreme Court” painstakingly enumerated the statements that the Court
signed overall 37 faculty members. In said article, the faculty considered excessive and uncalled for under the
expressly gave their dismay saying that the court had the circumstances surrounding the issuance, publication, and
hopes of relief from those “comfort women” during the war later submission to this Court of the UP Law faculty’s
“crushed by a singularly reprehensible act of dishonesty and Restoring Integrity Statement. To reiterate, it was not the
misrepresentation by the Highest Court of the Land.” circumstance that respondents expressed a belief that Justice
Del Castillo was guilty of plagiarism but rather their expression
In the article, it was stated that plagiarism, as appropriation
of that belief as "not only as an established fact, but a truth"
and misrepresentation of another person’s work as one’s own,
when it was "[o]f public knowledge [that there was] an ongoing
is considered as “dishonesty, pure and simple.” Hence, it was
investigation precisely to determine the truth of such
argued that since the decision in the Vinuya case form part of
allegations." It was also pointed out in the Show Cause
the Philippine judicial system, the Court, in fine, is allowing
Resolution that there was a pending motion for
dishonesty to be promulgated. Furthermore, the plagiarism
reconsideration of the Vinuya decision. The Show Cause
and misrepresentation in the Vinuya case undermines the
Resolution made no objections to the portions of the Restoring
judicial system of our country and is a dirt on the honor and
Integrity Statement that respondents claimed to be
dignity of the Supreme Court, the article sought for the
"constructive" but only asked respondents to explain those
resignation of Associate Justice Mariano del Castillo.
portions of the said Statement that by no stretch of the
In response to the said article, the Court issued a resolution imagination could be considered as fair or constructive.
stating that the remarks and choice of words used were such
To be sure, the Show Cause Resolution itself recognized
a great insult to the members of the Court and a threat to the
respondents’ freedom of expression when it stated that:
independence of the judiciary, a clear violation of Canons 1,
11 and 13 and the Rules 1.02 and 11.05 of the Code of While most agree that the right to criticize the
Professional Responsibility. Thereafter, the Court ordered the judiciary is critical to maintaining a free and
signatories to show cause on why they should not be democratic society, there is also a general
disciplined as members of the Bar for such alleged violations. consensus that healthy criticism only goes so far.
Many types of criticism leveled at the judiciary cross
In fulfillment of the directive by the Court, the signatories
the line to become harmful and irresponsible attacks.
passed a Common Compliance stating therein that their
These potentially devastating attacks and unjust
intention in issuing the article in question “was not to malign
criticism can threaten the independence of the
the Court but rather to defend its integrity and credibility and
judiciary. The court must "insist on being permitted to
to ensure continued confidence in the legal system” by the
proceed to the disposition of its business in an
words used therein as “focusing on constructive action.” Also,
orderly manner, free from outside interference
it was alleged that the respondents are correct in seeking
obstructive of its functions and tending to embarrass
responsibility from Justice del Castillo for he, indeed,
the administration of justice."
committed plagiarism thus, rectifying their issuance of the
article. Furthermore, the respondents argued that the article The Court could hardly perceive any reasonable
in question is a valid exercise of the freedom of expression as purpose for the faculty’s less than objective
comments except to discredit the April 28, 2010

Page 97 of 186
Decision in the Vinuya case and undermine the Nevertheless, such a right is not without limit. For, as this
Court’s honesty, integrity and competence in Court warned in Almacen:
addressing the motion for its reconsideration. As if
the case on the comfort women’s claims is not But it is a cardinal condition of all such criticism that
controversial enough, the UP Law faculty would fan it shall be bona fide, and shall not spill over the walls
the flames and invite resentment against a resolution of decency and propriety. A wide chasm exists
that would not reverse the said decision. This runs between fair criticism, on the one hand, and abuse
contrary to their obligation as law professors and and slander of courts and the judges thereof, on the
officers of the Court to be the first to uphold the other. Intemperate and unfair criticism is a gross
dignity and authority of this Court, to which they owe violation of the duty of respect to courts. It is such a
fidelity according to the oath they have taken as misconduct, that subjects a lawyer to disciplinary
attorneys, and not to promote distrust in the action.
administration of justice.
Elsewise stated, the right to criticize, which is guaranteed by
In re: Atty. Vicente Raul Almacen, cited in the Common the freedom of speech and of expression in the Bill of Rights
Compliance and the Vasquez Compliance, was an instance of the Constitution, must be exercised responsibly, for every
where the Court indefinitely suspended a member of the Bar right carries with it a corresponding obligation. Freedom is not
for filing and releasing to the press a "Petition to Surrender freedom from responsibility, but freedom with responsibility.
Lawyer’s Certificate of Title" in protest of what he claimed was
Applying by analogy the Court’s past treatment of the "free
a great injustice to his client committed by the Supreme Court.
speech" defense in other bar discipline cases, academic
It is true that in Almacen the Court extensively discussed
freedom cannot be successfully invoked by respondents in
foreign jurisprudence on the principle that a lawyer, just like
this case. The implicit ruling in the jurisprudence discussed
any citizen, has the right to criticize and comment upon
above is that the constitutional right to freedom of expression
actuations of public officers, including judicial authority.
of members of the Bar may be circumscribed by their ethical
However, the real doctrine in Almacen is that such criticism of
duties as lawyers to give due respect to the courts and to
the courts, whether done in court or outside of it, must conform
uphold the public’s faith in the legal profession and the justice
to standards of fairness and propriety. This case engaged in
system. To our mind, the reason that freedom of expression
an even more extensive discussion of the legal authorities
may be so delimited in the case of lawyers applies with greater
sustaining this view. To quote from that decision:
force to the academic freedom of law professors.
But it is the cardinal condition of all such criticism that
It would do well for the Court to remind respondents that, in
it shall be bona fide, and shall not spill over the walls
view of the broad definition in Cayetano v. Monsod, lawyers
of decency and propriety. A wide chasm exists
when they teach law are considered engaged in the practice
between fair criticism, on the one hand, and abuse
of law. Unlike professors in other disciplines and more than
and slander of courts and the judges thereof, on the
lawyers who do not teach law, respondents are bound by their
other. Intemperate and unfair criticism is a gross
oath to uphold the ethical standards of the legal profession.
violation of the duty of respect to courts. It is such a
Thus, their actions as law professors must be measured
misconduct that subjects a lawyer to disciplinary
against the same canons of professional responsibility
action.
applicable to acts of members of the Bar as the fact of their
For, membership in the Bar imposes upon a person being law professors is inextricably entwined with the fact that
obligations and duties which are not mere flux and they are lawyers.
ferment. His investiture into the legal profession
Even if the Court was willing to accept respondents’
places upon his shoulders no burden more basic,
proposition in the Common Compliance that their issuance of
more exacting and more imperative than that of
the Statement was in keeping with their duty to "participate in
respectful behavior toward the courts. He vows
the development of the legal system by initiating or supporting
solemnly to conduct himself "with all good fidelity x x
efforts in law reform and in the improvement of the
x to the courts;" and the Rules of Court constantly
administration of justice" under Canon 4 of the Code of
remind him "to observe and maintain the respect due
Professional Responsibility, we cannot agree that they have
to courts of justice and judicial officer.
fulfilled that same duty in keeping with the demands of Canons
It does not, however, follow that just because a lawyer is an 1, 11 and 13 to give due respect to legal processes and the
officer of the court, he cannot criticize the courts. That is his courts, and to avoid conduct that tends to influence the courts.
right as a citizen, and it is even his duty as an officer of the Members of the Bar cannot be selective regarding which
court to avail of such right. Thus, in In Re: Almacen, this Court canons to abide by given particular situations. With more
explicitly declared: reason that law professors are not allowed this indulgence,
since they are expected to provide their students exemplars
Hence, as a citizen and as officer of the court, a of the Code of Professional Responsibility as a whole and not
lawyer is expected not only to exercise the right, but just their preferred portions thereof.
also to consider it his duty to avail of such right. No
law may abridge this right. Nor is he "professionally
answerable to a scrutiny into the official conduct of
the judges, which would not expose him to legal
animadversion as a citizen." (Case of Austin, 28 Am
Dec. 657, 665).

xxxx

Page 98 of 186
15. PHILIP SIGFRID A. FORTUN vs. PRIMA JESUSA B. disbarment complaint relate to petitioners supposed actions
QUINSAYAS, MA. GEMMA OQUENDO et al. (G.R. No. involving the Maguindanao Massacre case.
194578; February 13, 2013)
Since the disbarment complaint is a matter of public interest,
legitimate media had a right to publish such fact under
freedom of the press. The Court also recognizes that
FACTS: On 23 November 2009, a convoy of seven vehicles respondent media groups and personalities merely acted on a
carrying the relatives of then Maguindanao vice-mayor news lead they received when they reported the filing of the
Esmael "Toto" Mangudadatu, as well as lawyers and disbarment complaint.
journalists, was on their way to the Commission on Elections
office in Shariff Aguak to file Mangudadatu’s Certificate of The distribution by Atty. Quinsayas to the media of the
Candidacy when they were accosted by a group of about 100 disbarment complaint, by itself, is not sufficient to absolve the
armed men at a checkpoint in Sitio Malating, Ampatuan town. media from responsibility for violating the confidentiality rule.
The gruesome aftermath of the hostage-taking was later However, since petitioner is a public figure or has become a
discovered and shocked the world. The hostages were public figure because he is representing a matter of public
systematically killed by shooting them at close range with concern, and because the event itself that led to the filing of
automatic weapons, and their bodies and vehicles were the disbarment case against petitioner is a matter of public
dumped in mass graves and covered with the use of a concern, the media has the right to report the filing of the
backhoe. These gruesome killings became known as the disbarment case as legitimate news.
Maguindanao Massacre. A total of 57 victims were killed, 30
of them journalists. Subsequently, criminal cases for Murder
were filed and raffled to the Regional Trial Court of Quezon
Defenses and Court’s answer:
City. Petitioner is the counsel for Datu Andal Ampatuan, Jr.
(Ampatuan, Jr.), the principal accused in the murder cases. GMA Network’s defense is that it has no newspaper or any
publication where the article could be printed; it did not
In November 2010, Atty. Quinsayas, et al. filed a disbarment
broadcast the disbarment complaint in its television station;
complaint against petitioner before this Court, docketed as Bar
and that the publication was already completed when Atty.
Matter No. A.C. 8827. The disbarment case is still pending.
Quinsayas distributed copies of the disbarment complaint to
Petitioner alleged that on 22 November 2010, GMA News TV
the media. – Online posting, however, is already publication
internet website posted an article, written by Dedace, entitled
considering that it was done on GMA Network’s online news
"Mangudadatu, others seek disbarment of Ampatuan lawyer".
website.
On even date, Inquirer.net, the website of PDI, also published
an article, written by Torres, which according to petitioner also PDI averred that it only shares its contents with Inquirer.net
stated details of the disbarment case. Petitioner further through a syndication. PDI attached a photocopy of the
alleged that on 23 November 2010, PhilStar published an syndication page stating that "[d]ue to syndication agreements
article, written by Punay, which gave details of the disbarment between PDI and Inquirer.net, some articles published in PDI
allegations. Further, petitioner alleged that on 23 November may not appear in Inquirer.net." – PDI was not able to fully
2010, Channel 23 aired on national television a program establish that it has a separate personality from Inquirer.net.
entitled "ANC Presents: Crying for Justice: the Maguindanao
Massacre." Drilon, the program’s host, asked questions and ABS-CBN alleged that SNN is its subsidiary and although they
allowed Atty. Quinsayas to discuss the disbarment case have interlocking directors, SNN has its own juridical
against petitioner, including its principal points. personality separate from its parent company. ABS-CBN
alleged that SNN controls the line-up of shows of ANC. – Court
Petitioner alleged that the public circulation of the disbarment agrees. A subsidiary has an independent and separate
complaint against him exposed this Court and its investigators juridical personality distinct from that of its parent company
to outside influence and public interference. Petitioner alleged and that any suit against the the latter does not bind the former
that opinion writers wrote about and commented on the and vice-versa.
disbarment complaint which opened his professional and
personal reputation to attack. He alleged that the purpose of Respondent Ressa alleged that she was on terminal leave
respondents in publishing the disbarment complaint was to when the program about the Maguindanao Massacre was
malign his personal and professional reputation. aired on ANC and that she had no hand in its production. –
Ressa’s defense was supported by a certification from the
Human Resource Account Head of ABS-CBN.
ISSUE: whether respondents violated the confidentiality rule Basically, the defense of respondents Dedace, Torres, Drilon,
in disbarment proceedings, warranting a finding of guilt for and Punay was that the disbarment complaint was published
indirect contempt of court. without any comment, in good faith and without malice; that
petitioner is a public figure; that the Maguindanao Massacre is
a matter of public interest; and that there was no conspiracy
HELD: The Court recognizes that "publications which are on their part in publishing the disbarment complaint. They also
privileged for reasons of public policy are protected by the argued that the news reports were part of privileged
constitutional guaranty of freedom of speech."22 As a general communication.
rule, disbarment proceedings are confidential in nature until
their final resolution and the final decision of this Court. In this
case, however, the filing of a disbarment complaint against Section 18, Rule 139-B of the Rules of Court provides:
petitioner is itself a matter of public concern considering that it
arose from the Maguindanao Massacre case. The interest of Section 18. Confidentiality. - Proceedings against
the public is not on petitioner himself but primarily on his attorneys shall be private and confidential. However,
involvement and participation as defense counsel in the the final order of the Supreme Court shall be
Maguindanao Massacre case. Indeed, the allegations in the published like its decisions in other cases.

Page 99 of 186
The Court explained the purpose of the rule, as follows: 16. PHARMACEUTICAL and HEALTH CARE
ASSOCIATION of the PHILIPPINES, Petitioner vs.
x x x. The purpose of the rule is not only to enable HEALTH SECRETARY FRANCISCO T. DUQUE III
this Court to make its investigations free from any et.al
extraneous influence or interference, but also to
protect the personal and professional reputation of FACTS: Before the Court is a petition for certiorari under Rule
attorneys and judges from the baseless charges of 65 of the Rules of Court, seeking to nullify Administrative
disgruntled, vindictive, and irresponsible clients and Order (A.O.) No. 2006-0012 entitled, Revised Implementing
litigants; it is also to deter the press from publishing Rules and Regulations of Executive Order No. 51, Otherwise
administrative cases or portions thereto without Known as The Milk Code, Relevant International Agreements,
authority. Penalizing Violations Thereof, and for Other Purposes
(RIRR).Petitioner posits that the RIRR is not valid as it
Petitioner also failed to substantiate his claim that respondent contains provisions that are not constitutional and go beyond
media groups and personalities acted in bad faith and that the law it is supposed to implement. Executive Order No. 51
they conspired with one another in their postings and (Milk Code) was issued by President Corazon Aquino on
publications of the filing of a disbarment complaint against October 28, 1986 by virtue of the legislative powers granted
him. Respondent media groups and personalities reported the to the president under the Freedom Constitution. One of the
filing of the disbarment complaint without any comments or preambular clauses of the Milk Code states that the law seeks
remarks but merely as it was – a news item. Petitioner failed to give effect to Article 11 of the International Code of
to prove that respondent media groups and personalities Marketing of Breastmilk Substitutes (ICMBS), a code adopted
acted with malicious intent. Respondent media groups and by the World Health Assembly (WHA) in 1981. From 1982 to
personalities made a fair and true news report and appeared 2006, the WHA adopted several Resolutions to the effect that
to have acted in good faith in publishing and posting the details breastfeeding should be supported, promoted and protected,
of the disbarment complaint. In the televised broadcast of the hence, it should be ensured that nutrition and health claims
commemoration of the Maguindanao Massacre over ANC, the are not permitted for breastmilk substitutes.
disbarment case was briefly discussed but petitioner was not
named. There was also no proof that respondent media In 1990, the Philippines ratified the International Convention
groups and personalities posted and published the news to on the Rights of the Child. Article 24 of said instrument
influence this Court on its action on the disbarment case or to provides that State Parties should take appropriate measures
deliberately destroy petitioner’s reputation. to diminish infant and child mortality, and ensure that all
segments of society, specially parents and children, are
informed of the advantages of breastfeeding. On May 15,
2006, the DOH issued herein assailed RIRR which was to take
effect on July 7, 2006.

ISSUE: Whether or not respondents officers of the DOH acted


without or in excess of jurisdiction, or with grave abuse of
discretion amounting to lack or excess of jurisdiction, and in
violation of the provisions of the Constitution in promulgating
the RIRR

HELD: When it comes to information regarding nutrition of


infants and young children, however, the Milk Code
specifically delegated to the Ministry of Health (hereinafter
referred to as DOH) the power to ensure that there is
adequate, consistent and objective information on
breastfeeding and use of breastmilk substitutes, supplements
and related products; and the power to control such
information. Further, DOH is authorized by the Milk Code to
control the content of any information on breastmilk vis-à-vis
breastmilk substitutes, supplement and related products The
DOH is also authorized to control the purpose of the
information and to whom such information may be
disseminated under Sections 6 through 9 of the Milk Code to
ensure that the information that would reach pregnant women,
mothers of infants, and health professionals and workers in
the health care system is restricted to scientific and factual
matters and shall not imply or create a belief that bottle-
feeding is equivalent or superior to breastfeeding.

It bears emphasis, however, that the DOH's power under the


Milk Code to control information regarding breastmilk vis-a-vis
breastmilk substitutes is not absolute as the power to control
does not encompass the power to absolutely prohibit the
advertising, marketing, and promotion of breastmilk
substitutes.

Page 100 of 186


Sections 13 on "total effect" and 26 of Rule VII of the RIRR and 10 of the Milk Code, by which the IAC shall screen
contain some labeling requirements, specifically: a) that there advertising, promotional, or other marketing materials. It is
be a statement that there is no substitute to breastmilk; and b) pursuant to such responsibility that the DOH correctly
that there be a statement that powdered infant formula may provided for Section 13 in the RIRR which reads as follows:
contain pathogenic microorganisms and must be prepared
and used appropriately. Section 16 of the RIRR prohibits all SECTION 13. "Total Effect" - Promotion of products
health and nutrition claims for products within the scope of the within the scope of this Code must be objective and
Milk Code, such as claims of increased emotional and should not equate or make the product appear to be
intellectual abilities of the infant and young child. These as good or equal to breastmilk or breastfeeding in the
requirements and limitations are consistent with the provisions advertising concept. It must not in any case
of Section 8 of the Milk Code and Section 10(d) which bars the undermine breastmilk or breastfeeding. The "total
use on containers and labels of the terms "humanized," effect" should not directly or indirectly suggest that
"maternalized," or similar terms. These provisions of the Milk buying their product would produce better
Code expressly forbid information that would imply or create a individuals, or resulting in greater love, intelligence,
belief that there is any milk product equivalent to breastmilk or ability, harmony or in any manner bring better health
which is humanized or maternalized, as such information to the baby or other such exaggerated and
would be inconsistent with the superiority of breastfeeding. unsubstantiated claim.

Such standards bind the IAC in formulating its rules and


regulations on advertising, promotion, and marketing.
Section 11 of the RIRR, to wit: Through that single provision, the DOH exercises control over
the information content of advertising, promotional and
SECTION 11. Prohibition – No advertising, marketing materials on breastmilk vis-a-vis breastmilk
promotions, sponsorships, or marketing materials substitutes, supplements and other related products. It also
and activities for breastmilk substitutes intended for sets a viable standard against which the IAC may screen such
infants and young children up to twenty-four (24) materials before they are made public.
months, shall be allowed, because they tend to
convey or give subliminal messages or impressions Contrary to petitioner's claim, Section 22 of the RIRR does not
that undermine breastmilk and breastfeeding or prohibit the giving of information to health professionals on
otherwise exaggerate breastmilk substitutes and/or scientific and factual matters. What it prohibits is the
replacements, as well as related products covered involvement of the manufacturer and distributor of the
within the scope of this Code. products covered by the Code in activities for the promotion,
education and production of Information, Education and
prohibits advertising, promotions, sponsorships or marketing Communication (IEC) materials regarding breastfeeding that
materials and activities for breastmilk substitutes in line with are intended for women and children. Said provision cannot
the RIRR’s declaration of principle under Section 4(f), to wit: be construed to encompass even the dissemination of
information to health professionals, as restricted by the Milk
SECTION 4. Declaration of Principles –
Code.
xxxx

(f) Advertising, promotions, or sponsorships of infant


formula, breastmilk substitutes and other related
products are prohibited.

The DOH, through its co-respondents, evidently arrogated to


itself not only the regulatory authority given to the IAC but also
imposed absolute prohibition on advertising, promotion, and
marketing.

Yet, oddly enough, Section 12 of the RIRR reiterated the


requirement of the Milk Code in Section 6 thereof for prior
approval by IAC of all advertising, marketing and promotional
materials prior to dissemination. Even respondents, through
the OSG, acknowledged the authority of IAC, and repeatedly
insisted, during the oral arguments on June 19, 2007, that the
prohibition under Section 11 is not actually operational.

Sections 11 and 4(f) of the RIRR are clearly violative of the


Milk Code.

However, although it is the IAC which is authorized to


promulgate rules and regulations for the approval or rejection
of advertising, promotional, or other marketing materials under
Section 12(a) of the Milk Code, said provision must be related
to Section 6 thereof which in turn provides that the rules and
regulations must be "pursuant to the applicable standards
provided for in this Code." Said standards are set forth in
Sections 5(b), 8(b), and 10 of the Code.

Thus, the DOH has the significant responsibility to translate


into operational terms the standards set forth in Sections 5, 8,

Page 101 of 186


17. Pleasant Grove City, Utah et al. vs. Summum 555 US 18. John Walker, III, et al. vs. Texas Division, Sons of
_ (2009) Confederate Veterans, Inc., et al. 576 US _ (2015)

FACTS: Summum, a religious organization, sent a letter to the FACTS: In August 2009, the Texas division of the Sons of
mayor of Pleasant Grove, Utah asking to place a monument Confederate Veterans (Texas SCV), a non-profit organization
in one of the city's parks. Although the park already housed a that works to preserve the memory and reputation of soldiers
monument to the Ten Commandments, the mayor denied who fought for the confederacy in the Civil War, applied to
Summum's request because the monument did not "directly have a new specialty license plate issued by the Texas
relate to the history of Pleasant Grove." Summum filed suit Department of Motor Vehicles (TDMV). The proposed license
against the city in federal court citing, among other things, a plate had two confederate flags on it: one in the organization's
violation of its First Amendment free speech rights. The U.S. logo, and one faintly making up the background of the plate.
District Court for the District of Utah denied Summum's The TDMV had a policy stating that it "may refuse to create a
request for a preliminary injunction. new specialty license plate if the design might be offensive to
any member of the public." The board in charge of approving
The U.S. Court of Appeals for the Tenth Circuit reversed the new specialty plates received multiple negative comments
district court and granted Summum's injunction request. The from the public regarding this plate and ultimately voted to
Tenth Circuit held that the park was in fact a "public" forum, deny Texas SCV's application.
not a non-public forum as the district court had held.
Furthermore, Summum demonstrated that it would suffer Texas SCV sued in federal district court claiming their First
irreparable harm if the injunction were to be denied, and the and Fourteenth Amendment rights were violated. The TDMV
interests of the city did not outweigh this potential harm. The argued that the Free Speech Clause did not apply in this case
injunction, according to the court, was also not against the because license plates are a form of government speech;
public interest. therefore, they were within their rights to choose which
messages and views they wanted to express on the plates.
The district court disagreed and held that the plates were
private, non-governmental speech, and that the TDMV's
ISSUE: Does a city's refusal to place a religious organization's
denial was a reasonable, content-based restriction of speech
monument in a public park violate that organization's First
in a non-public forum. The United States Court of Appeals for
Amendment free speech rights when the park already
the Fifth Circuit reversed and held that TDMV's denial was a
contains a monument from a different religious group?
form of viewpoint discrimination that "discriminated against
Texas SCV's view that the Confederate flag is a symbol of
sacrifice, independence, and Southern heritage."
HELD: No. The Supreme Court reversed the Tenth Circuit
holding that the placement of a monument in a public park is
a form of government speech and therefore not subject to
ISSUE:
scrutiny under the Free Speech Clause of the First
Amendment. With Justice Samuel A. Alito writing for the 1. Do specialty license plates constitute government speech
majority and joined by Chief Justice John G. Roberts and that is immune from any requirement of viewpoint neutrality?
Justices John Paul Stevens, Antonin G. Scalia, Anthony M. (YES)
Kennedy, Clarence Thomas, Ruth Bader Ginsburg, and
Stephen G. Breyer, the Court reasoned that since Pleasant 2. Does preventing the confederate flag from appearing on
Grove City had retained final authority over which monuments license plates constitute viewpoint discrimination? (NO)
were displayed, the monuments represented an expression of
the city's viewpoints and thus government speech.

Justice Stevens, joined by Justice Ginsburg, wrote a separate HELD: Justice Stephen G. Breyer delivered the opinion of the
concurring opinion that largely embraced the majority's 5-4 majority. The Court held that the government choosing the
reasoning. Justice Scalia, joined by Justice Thomas, also content of its speech is not unconstitutional viewpoint
wrote a separate concurring opinion. Agreeing with the Court's discrimination because that expression is the product of the
reasoning, he also noted that there were likely no violations of democratic electoral process. Based on the analysis from
the Establishment Clause of the First Amendment on the part Pleasant Grove City v. Summum, Texas’s specialty license
of Pleasant Grove City. He argued that displays of the Ten plate is an example of such government speech (as opposed
Commandments had been construed by the Court as "having to a forum open for private expression) because Texas and
an undeniable historical meaning" and thus did not attempt to other states have long used license plates to convey
establish a religion. Justice Breyer also wrote a separate messages. Moreover, the public associates license plates with
concurring opinion in which he noted that "government the State. Finally, Texas maintains direct control over the
speech" should be considered a rule of thumb and not a rigid messages on its specialty plates from design to final approval.
category. He stated that sometimes the Court should ask
Justice Samuel A. Alito, Jr., wrote a dissent in which he argued
"whether a government's actions burdens speech
that, with over 350 varieties of specialty plates, an observer
disproportionately in light of the action's tendency to further a
would think that the plates were the expression of the
legitimate government objective." Justice Souter also wrote
individual drivers, not Texas. Because the specialty license
separately, concurring in the judgment, but warning that public
plates are a limited public forum for private expression, Texas
monuments should not be considered government speech
rejecting the confederate flag design because it might be
categorically.
offensive is unconstitutional viewpoint discrimination. Chief
Justice John G. Roberts, Jr., Justice Antonin Scalia, and
Justice Anthony M. Kennedy joined in the dissent.

Page 102 of 186


19. Bayan et.al. vs. Ermita The first point to mark is that the right to peaceably assemble
and petition for redress of grievances is, together with freedom
of speech, of expression, and of the press, a right that enjoys
primacy in the realm of constitutional protection. For these
Facts: Petitioners come in three groups – Bayan, et al, Jess
rights constitute the very basis of a functional democratic
del Prado, et al, , Kilusang Mayo Uno (KMU), et al, KMU, et
polity, without which all the other rights would be meaningless
al.,
and unprotected
The rally was scheduled to proceed along España Avenue in
Rights to peaceful assembly to petition the government for a
front of the University of Santo Tomas and going towards
redress of grievances and, for that matter, to organize or form
Mendiola bridge. Police officers blocked them along Morayta
associations for purposes not contrary to law, as well as to
Street and prevented them from proceeding further. They
engage in peaceful concerted activities. These rights are
were then forcibly dispersed, causing injuries on one of them.
guaranteed by no less than the Constitution, particularly
Three other rallyists were arrested
Sections 4 and 8 of the Bill of Rights, Section 2(5) of Article
in the case of Bayan, et al allege that they are citizens and IX, and Section 3 of Article XIII. Jurisprudence abounds with
taxpayers of the Philippines and that their rights as hallowed pronouncements defending and promoting the
organizations and individuals were violated when the rally they people’s exercise of these rights
participated in on October 6, 2005 was violently dispersed by
It is very clear, therefore, that B.P. No. 880 is not an absolute
policemen implementing Batas Pambansa (B.P.) No. 880
ban of public assemblies but a restriction that simply regulates
All petitioners assail Batas Pambansa No. 880, some of them the time, place and manner of the assemblies, it as a "content-
in toto and others only Sections 4, 5, 6, 12, 13(a), and 14(a), neutral" regulation of the time, place, and manner of holding
as well as the policy of CPR, "Calibrated Preemptive public assemblies
Response". They seek to stop violent dispersals of rallies
A fair and impartial reading of B.P. No. 880 thus readily shows
under the "no permit, no rally" policy and the CPR policy
that it refers to all kinds of public assemblies22 that would use
recently announced.
public places. The reference to "lawful cause" does not make
Bayan et al argued that B.P. No. 880 requires a permit before it content-based because assemblies really have to be for
one can stage a public assembly regardless of the presence lawful causes, otherwise they would not be "peaceable" and
or absence of a clear and present danger. It also curtails the entitled to protection. Neither are the words "opinion,"
choice of venue and is thus repugnant to the freedom of "protesting" and "influencing" in the definition of public
expression clause as the time and place of a public assembly assembly content based, since they can refer to any subject.
form part of the message for which the expression is sought. The words "petitioning the government for redress of
Furthermore, it is not content-neutral as it does not apply to grievances" come from the wording of the Constitution, so its
mass actions in support of the government. The words "lawful use cannot be avoided. Finally, maximum tolerance is for the
cause," "opinion," "protesting or influencing" suggest the protection and benefit of all rallyists and is independent of the
exposition of some cause not espoused by the government. content of the expressions in the rally.
Also, the phrase "maximum tolerance" shows that the law
Furthermore, the permit can only be denied on the ground of
applies to assemblies against the government because they
clear and present danger to public order, public safety, public
are being tolerated. As a content-based legislation, it cannot
convenience, public morals or public health.
pass the strict scrutiny test.
The so-called calibrated preemptive response policy has no
place in our legal firmament and must be struck down as a
Issue: Whether or not the implementation of B.P. No. 880 darkness that shrouds freedom. It merely confuses our people
volated their rights as organizations and individuals when the and is used by some police agents to justify abuses. On the
rally they participated in on October 6, 2005 other hand, B.P. No. 880 cannot be condemned as
unconstitutional; it does not curtail or unduly restrict freedoms;
it merely regulates the use of public places as to the time,
place and manner of assemblies. Far from being insidious,
Held: Petitioners’ standing cannot be seriously challenged.
"maximum tolerance" is for the benefit of rallyists, not the
Their right as citizens to engage in peaceful assembly and
government. The delegation to the mayors of the power to
exercise the right of petition, as guaranteed by the
issue rally "permits" is valid because it is subject to the
Constitution, is directly affected by B.P. No. 880 which
constitutionally-sound "clear and present danger" standard.
requires a permit for all who would publicly assemble in the
nation’s streets and parks. They have, in fact, purposely In this Decision, the Court goes even one step further in
engaged in public assemblies without the required permits to safeguarding liberty by giving local governments a deadline of
press their claim that no such permit can be validly required 30 days within which to designate specific freedom parks as
without violating the Constitutional guarantee. Respondents, provided under B.P. No. 880. If, after that period, no such
on the other hand, have challenged such action as contrary to parks are so identified in accordance with Section 15 of the
law and dispersed the public assemblies held without the law, all public parks and plazas of the municipality or city
permit. concerned shall in effect be deemed freedom parks; no prior
permit of whatever kind shall be required to hold an assembly
Sec. 4 Art. III Section 4 of Article III of the Constitution
therein. The only requirement will be written notices to the
Sec. 4. No law shall be passed abridging the freedom police and the mayor’s office to allow proper coordination and
of speech, of expression, or of the press, or the right orderly activities.
of the people peaceably to assemble and petition the
government for redress of grievances

Page 103 of 186


20. INTEGRATED BAR OF THE PHILIPPINES In Bayan, Karapatan, Kilusang Magbubukid ng Pilipinas
represented by its National President, Jose Anselmo (KMP) v. Ermita, the Court reiterated:
I. Cadiz, H. HARRY L. ROQUE, and JOEL RUIZ
BUTUYAN vs. HONORABLE MANILA MAYOR JOSE x x x Freedom of assembly connotes the right of the
"LITO" ATIENZA (G.R. No. 175241; February 24, people to meet peaceably for consultation and
2010) discussion of matters of public concern. It is entitled
to be accorded the utmost deference and respect. It
is not to be limited, much less denied, except on a
showing, as is the case with freedom of expression,
FACTS: On June 15, 2006, the IBP, through its then National of a clear and present danger of a substantive evil
President Jose Anselmo Cadiz (Cadiz), filed with the Office of that the state has a right to prevent. Even prior to the
the City Mayor of Manila a letter application4 for a permit to 1935 Constitution, Justice Malcolm had occasion to
rally at the foot of Mendiola Bridge on June 22, 2006 from 2:30 stress that it is a necessary consequence of our
p.m. to 5:30 p.m. to be participated in by IBP officers and republican institutions and complements the right of
members, law students and multi-sectoral organizations. free speech. To paraphrase the opinion of Justice
Respondent issued a permit dated June 16, 2006 allowing the Rutledge, speaking for the majority of the American
IBP to stage a rally on given date but indicated therein Plaza Supreme Court in Thomas v. Collins, it was not by
Miranda as the venue, instead of Mendiola Bridge, which accident or coincidence that the rights to freedom of
permit the IBP received on June 19, 2006. speech and of the press were coupled in a single
guarantee with the rights of the people peaceably to
Aggrieved, petitioners filed on June 21, 2006 before the Court
assemble and to petition the government for redress
of Appeals a petition for certiorari docketed as CA-G.R. SP
of grievances. All these rights, while not identical, are
No. 94949. The petition having been unresolved within 24
inseparable. In every case, therefore, where there is
hours from its filing, petitioners filed before this Court on June
a limitation placed on the exercise of this right, the
22, 2006 a petition for certiorari docketed as G.R. No. 172951
judiciary is called upon to examine the effects of the
which assailed the appellate court’s inaction or refusal to
challenged governmental actuation. The sole
resolve the petition within the period provided under the Public
justification for a limitation on the exercise of this
Assembly Act of 1985.
right, so fundamental to the maintenance of
The Court, by Resolutions of July 26, 2006, August 30, 2006 democratic institutions, is the danger, of a character
and November 20, 2006, respectively, denied the petition for both grave and imminent, of a serious evil to public
being moot and academic, denied the relief that the petition safety, public morals, public health, or any other
be heard on the merits in view of the pendency of CA-G.R. SP legitimate public interest.
No. 94949, and denied the motion for reconsideration.
It is true that the licensing official, here respondent Mayor, is
The rally pushed through on June 22, 2006 at Mendiola not devoid of discretion in determining whether or not a permit
Bridge, after Cadiz discussed with P/Supt. Arturo Paglinawan would be granted. It is not, however, unfettered discretion.
whose contingent from the Manila Police District (MPD) earlier While prudence requires that there be a realistic appraisal not
barred petitioners from proceeding thereto. Petitioners allege of what may possibly occur but of what may probably occur,
that the participants voluntarily dispersed after the peaceful given all the relevant circumstances, still the assumption –
conduct of the program. The MPD thereupon instituted on especially so where the assembly is scheduled for a specific
June 26, 2006 a criminal action, docketed as I.S. No. 06I- public place – is that the permit must be for the assembly
12501, against Cadiz for violating the Public Assembly Act in being held there. The exercise of such a right, in the language
staging a rally at a venue not indicated in the permit, to which of Justice Roberts, speaking for the American Supreme Court,
charge Cadiz filed a Counter-Affidavit of August 3, 2006. is not to be "abridged on the plea that it may be exercised in
some other place.

ISSUE: whether the appellate court erred in holding that the


modification of the venue in IBP’s rally permit does not
constitute grave abuse of discretion.

HELD: Yes, in modifying the permit outright, respondent


gravely abused his discretion when he did not immediately
inform the IBP who should have been heard first on the matter
of his perceived imminent and grave danger of a substantive
evil that may warrant the changing of the venue. The
opportunity to be heard precedes the action on the permit,
since the applicant may directly go to court after an
unfavorable action on the permit.

Respondent failed to indicate how he had arrived at modifying


the terms of the permit against the standard of a clear and
present danger test which, it bears repeating, is an
indispensable condition to such modification. Nothing in the
issued permit adverts to an imminent and grave danger of a
substantive evil, which "blank" denial or modification would,
when granted imprimatur as the appellate court would have it,
render illusory any judicial scrutiny thereof.

Page 104 of 186


G. FREEDOM OF RELIGION The rationale of the rule is summed up in the familiar
saying, "Strong fences make good neighbors." The
idea is to delineate the boundaries between the two
1. RENATO V. PERALTA, PETITIONER, VS. PHILIPPINE institutions and, thus, avoid encroachments by one
POSTAL CORPORATION (PHILPOST) (G.R. No. against the other because of a misunderstanding of
223395, December 04, 2018) the limits of their respective exclusive jurisdictions.
The demarcation line calls on the entities to "render
therefore unto Caesar the things that are Caesar's
FACTS: Philippine Postal Corporation (PhilPost) issued a and unto God the things that are God's."
stamp commemorating Iglesia ni Cristo's (INC's) Centennial
Celebration. The design of the stamp showed a photo of INC In upholding the issuance of the Thirty-third International
founder, the late Felix Y. Manalo (Manalo) with the Eucharistic Congress commemorative stamp, this Court in
designation on the left side containing the words "Felix Y. Aglipay v. Ruiz recognized how religion is integrated in the
Manalo, 1886-1963 First Executive Minister of Iglesia ni Filipino way of life:
Cristo", with the Central Temple of the religious group at the
background. The more important question raised refers to the
alleged violation of the Constitution by the
On June 16, 2014, petitioner Renato V. Peralta (petitioner) respondent in issuing and selling postage stamps
filed a complaint for injunction with the Regional Trial Court commemorative of the Thirty-third International
(RTC), Br. 33 of Manila, assailing the constitutionality of the Eucharistic Congress. It is alleged that this action of
printing, issuance and distribution of the INC commemorative the respondent is violative of the provisions of
centennial stamps, allegedly paid for by respondent PhilPost section 13, subsection 3, Article VI, of the
using public funds. In his complaint, petitioner alleged that the Constitution of the Philippines, which provides as
printing and issuance of the INC commemorative stamp follows:
involved disbursement of public funds, and violated. Section
29(2) of Article VI of the 1987 Constitution. He argued that "No public money or property shall ever be
respondents' act of releasing the said stamps was appropriated, applied, or used, directly or indirectly,
unconstitutional because it was tantamount to sponsorship of for the use, benefit, or support of any sect, church,
a religious activity; it violated the separation of the Church and denomination, secretarian institution, or system of
the State; and the non-establishment of religion clause. religion, or for the use, benefit, or support of any
priest, preacher, minister, or other religious teacher
Respondents filed their Answer, maintaining that no public or dignitary as such, except when such priest,
funds were disbursed in the printing of the INC preacher, minister, or dignitary is assigned to the
commemorative stamps. They alleged that there was a armed forces or to any penal institution, orphanage,
Memorandum of Agreement (MOA) dated May 7, 2014 or leprosarium."
executed between PhilPost and INC, where it was provided
that the costs of printing will be borne by INC. They claimed Religious freedom, however, as a constitutional
that the proceeds of the sale of the stamps will not redound to mandate is not inhibition of profound reverence for
the sole benefit of INC. The printing, according to them, is part religion and is not denial of its influence in human
of PhilPost's philatelic products, which will promote tourism in affairs. Religion as a profession of faith to an active
the country because it will attract people from all over the power that binds and elevates man to his Creator is
world. recognized. And, in so far as it instills into the minds
the purest principles of morality, its influence is
On July 25, 2014, the RTC issued an Order, denying deeply felt and highly appreciated. When the Filipino
petitioner's application for the issuance of a preliminary people, in the preamble of their Constitution,
injunction and dismissing the action. It ruled that it was not a implored "the aid of Divine Providence, in order to
taxpayer's suit and that it did not violate Section 29 (2), Article establish a government that shall embody their
VI of the 1987 Philippine Constitution. Petitioner appealed the ideals, conserve and develop the patrimony of the
RTC's decision with the CA, but the same was denied in its nation, promote the general welfare, and secure to
July 24, 2015 decision. themselves and their posterity the blessings of
independence under a regime of justice, liberty and
democracy," they thereby manifested their intense
religious nature and placed unfaltering reliance upon
ISSUE: Was the selling of the postage stamps
Him who guides the destinies of men and nations.
commemorating the INC’s centennial celebration
The elevating influence of religion in human society
constitutional?
is recognized here as elsewhere. In fact, certain
general concessions are indiscriminately accorded
to religious sects and denominations.
HELD: There is no quibbling that as to the 50,000 stamps
ordered, printed and issued to INC, the same did not violate
the Constitutional prohibitions separating State matters from
Religious Profession and Worship
religion.
The right to religious profession and worship has a two-fold
The non-establishment of religion clause is not equivalent to
aspect, viz., freedom to believe and freedom to act on one's
indifference to religion
beliefs. The first is absolute as long as the belief is confined
True, fundamental to the resolution of this case is the policy of within the realm of thought. The second is subject to regulation
the State on the inviolability of the principle of separation of where the belief is translated into external acts that affect the
the church and the state. Justice Isagani Cruz explained the public welfare.
rationale of this principle in this wise:
(1) Freedom to Believe

Page 105 of 186


The individual is free to believe (or disbelieve) as he pleases originated from the case of Lemon vs. Kurtzman. In that case,
concerning the hereafter. He may indulge his own theories the Court used a three-pronged test to adjudge whether the
about life and death; worship any god he chooses, or none at assailed governmental act violated the First Amendment, as
all; embrace or reject any religion; acknowledge the divinity of follows:
God or of any being that appeals to his reverence; recognize
or deny the immortality of his soul - in fact, cherish any 1. The statute must have a secular legislative
religious conviction as he and he alone sees fit. However purpose;
absurd his beliefs may be to others, even if they be hostile and
2. Its principal or primary effect must be one that
heretical to the majority, he has full freedom to believe as he
neither advances nor inhibits religion; and,
pleases. He may not be required to prove his beliefs. He may
not be punished for his inability to do so. Religion, after all, is 3. The statute must not foster "an excessive
a matter of faith. Men may believe what they cannot prove. government entanglement with religion."
Every one has a right to his beliefs and he may not be called
to account because he cannot prove what he believes. It is plain, that the costs for the printing and issuance of the
aforesaid 50,000 stamps were all paid for by INC. Any
(2) Freedom to Act on One's Beliefs perceived use of government property, machines or
otherwise, is de minimis and certainly do not amount to a
But where the individual externalizes his beliefs in acts or
sponsorship of a specific religion. Also, We see no violation of
omissions that affect the public, his freedom to do so becomes
the Constitutional prohibition on establishment of religion,
subject to the authority of the State. As great as this liberty
insofar as the remaining 1,150,000 pieces of stamps printed
may be, religious freedom, like all the other rights guaranteed
and distributed by PhilPost. First, there is no law mandating
in the Constitution, can be enjoyed only with a proper regard
anyone to avail of the INC commemorative stamps, nor is
for the rights of others. It is error to think that the mere
there any law purporting to require anyone to adopt the INC's
invocation of religious freedom will stalemate the State and
teachings. Arguably, while then President Aquino issued
render it impotent in protecting the general welfare. The
Proclamation No. 815, s. 2014, authorizing the issuance of the
inherent police power can be exercised to prevent religious
INC commemorative stamp, the same did not contain any
practices inimical to society. And this is true even if such
legal mandate endorsing or requiring people to conform to the
practices are pursued out of sincere religious conviction and
INC's teachings.
not merely for the purpose of evading the reasonable
requirements or prohibitions of the law. As to the use of the government's machinery in printing and
distribution of the 1.2 million stamps, this Court does not find
By adopting the above constitutional provisions on religion,
that the same amounted to sponsorship of INC as a religion
the Filipinos manifested their adherence to the benevolent
considering that the same is no different from other stamps
neutrality approach in interpreting the religion clauses, an
issued by PhilPost acknowledging persons and events of
approach that looks further than the secular purposes of
significance to the country.
government action and examines the effect of these actions
on religious exercise. Benevolent neutrality recognizes the In the same vein, We do not find that there was illegal
religious nature of the Filipino people and the elevating disbursement of funds under Section 29(2) of Article VI of the
influence of religion in society; at the same time, it Constitution. The application of this prohibition towards
acknowledges that government must pursue its secular goals. government acts was already clarified by the Court in Re:
In pursuing these goals, however, government might adopt Letter of Tony Q. Valenciana, Holding Of Religious Rituals At
laws or actions of general applicability which inadvertently The Hall Of Justice Building In Quezon City:
burden religious exercise. Benevolent neutrality gives room
for accommodation of these religious exercises as required by Section 29 (2), Article VI of the 1987 Constitution
the Free Exercise Clause. provides, "No public money or property shall be
appropriated, applied, paid, or employed, directly or
We adopt the benevolent neutrality approach not only indirectly, for the use, benefit, or support of any sect,
because of its merits as discussed above, but more church, denomination, sectarian institution, or
importantly, because our constitutional history and system of religion, or of any priest, preacher,
interpretation indubitably show that benevolent neutrality is minister, or other religious teacher, or dignitary as
the launching pad from which the Court should take off in such, except when such priest, preacher, minister, or
interpreting religion clause cases. dignitary is assigned to the armed forces, or to any
penal institution, or government orphanage or
Where the Court has been asked to determine whether there
leprosarium."
has been an undue encroachment of this Constitutionally
forged "wall", this Court has adopted a stance of "benevolent The word "apply" means "to use or employ for a
neutrality". Rightfully so, for this incorporates the particular purpose." "Appropriate" means "to
Constitutional principle of separation of the Church and the prescribe a particular use for particular moneys or to
State while recognizing the people's right to express their designate or destine a fund or property for a distinct
belief or non-belief of a Supreme Being. use, or for the payment of a particular demand."
Under the principle of noscitur a sociis, where a
Even in the U. S., whose jurisprudence are of persuasive
particular word or phrase is ambiguous in itself or is
weight in this jurisdiction, it can be gleaned that the religious
equally susceptible of various meanings, its correct
nature of certain governmental acts does not automatically
construction may be made clear and specific by
result in striking them as unconstitutional for violation of the
considering the company of words in which it is found
non-establishment clause, particularly if the act involves
with or with which it is associated. This is because a
constitutionally protected form of exercise of religious
word or phrase in a statute is always used in
freedom. The "Lemon test", which has been extensively
association with other words or phrases, and its
applied by the U. S. Supreme Court in issues involving the
meaning may, thus, be modified or restricted by the
determination of non-establishment of religion clause
latter. The particular words, clauses and phrases

Page 106 of 186


should not be studied as detached and isolated 2. ESTRADA VS. ESCRITOR A.M. No. P-02-1651
expressions, but the whole and every part of the August 4, 2003 J. PUNO
statute must be considered in fixing the meaning of
any of its parts and in order to produce a harmonious
whole. A statute must be so construed as to FACTS: In a sworn letter-complaint, Estrada wrote to Judge
harmonize and give effect to all its provisions Caoibes, Jr. requesting for an investigation of rumors that
whenever possible. Escritor, court interpreter in said court, is living with a man not
her husband. They allegedly have a child of 18 to 20 years
Thus, the words "pay" and "employ" should be old. Estrada is not personally related either to Escritor or her
understood to mean that what is prohibited is the use partner and is a resident not of Las Piñas City but of Bacoor,
of public money or property for the sole purpose of Cavite. Nevertheless, he filed the charge against Escritor as
benefiting or supporting any church. The prohibition he believes that she is committing an immoral act that
tarnishes the image of the court, thus she should not be
contemplates a scenario where the appropriation is
allowed to remain employed therein as it might appear that the
primarily intended for the furtherance of a particular court condones her act.
church.
Escritor testified that when she entered the judiciary in 1999,
It has also been held that the aforecited constitutional she was already a widow, her husband having died in 1998.
provision "does not inhibit the use of public property She admitted that she has been living with Luciano Quilapio,
for religious purposes when the religious character of Jr. without the benefit of marriage for 20 years and that they
such use is merely incidental to a temporary use have a son. But as a member of the religious sect known as
which is available indiscriminately to the public in the Jehovah’s Witnesses and the Watch Tower and Bible
general." Tract Society, their conjugal arrangement is in conformity with
their religious beliefs and it does not consider it immoral. In
fact, after ten years of living together, she executed a
“Declaration of Pledging Faithfulness”. Quilapio also executed
Adopting the stance of benevolent neutrality, this Court deems a similar pledge on the same day. At the time Escritor
the design of the INC commemorative stamp constitutionally executed her pledge, her husband was still alive but living with
permissible. As correctly held by the CA, there is an intrinsic another woman. Quilapio was likewise married at that time,
historical value in the fact that Felix Y Manalo is a Filipino and but had been separated in fact from his wife.
that the INC is a Filipino institution. Thus, this Court sees no Estrada argued, through counsel, that the Declaration of
religious overtones surrounding the commemorative stamps, Pledging Faithfulness recognizes the supremacy of the
as insisted upon by the petitioner. the design depicted in the “proper public authorities” such that she bound herself “to
INC commemorative stamp is merely a recognition of the seek means to . . . legalize their union.” Thus, even assuming
continuous existence of a group that is strictly Filipino. As arguendo that the declaration is valid and binding in her
compared to major religious groups established in the country, congregation, it is binding only to her co-members in the
Felix Y. Manalo, and the INC, are not plain religious symbols, congregation and serves only the internal purpose of
but also a representation of a group that is distinctly unique displaying to the rest of the congregation that she and her
to the Philippines. To the mind of this Court, the use of the mate are a respectable and morally upright couple. Their
religious belief and practice, however, cannot override the
facade of the Church and the image of Felix Y. Manalo is
norms of conduct required by law for government employees.
nothing more than an acknowledgment of a historical
To rule otherwise would create a dangerous precedent as
milestone. It does not endorse, establish or disparage other those who cannot legalize their live-in relationship can simply
religious groups and even non-believers, especially join the Jehovah’s Witnesses congregation and use their
considering the fact that PhilPost also print stamps with religion as a defense against legal liability.
symbols which can arguably be connected to religion.

ISSUE: WoN Escritor’s right to religious freedom should carve


out an exception from the prevailing jurisprudence on illicit
relations for which government employees are held
administratively liable. (WoN Escritor should be found guilty of
the administrative charge of “gross and immoral conduct.”)

HELD: Yes. The Philippine constitution’s religion clauses


prescribe the benevolent neutrality approach not the strict
neutrality approach. Benevolent neutrality recognizes that
government must pursue its secular goals and interests but at
the same time strives to uphold religious liberty to the greatest
extent possible within flexible constitutional limits. Thus,
although the morality contemplated by laws is secular,
benevolent neutrality could allow for accommodation of
morality based on religion, provided it does not offend
compelling state interests.
With benevolent neutrality as a framework, the Court cannot
simply reject respondent’s plea of religious freedom without
even subjecting it to the “compelling state interest” test that
would balance her freedom with the paramount interests of the
state.

Page 107 of 186


Application of Benevolent Neutrality and the Compelling proscribes government from questioning a person’s
State Interest Test to the Case at Bar. beliefs or imposing penalties or disabilities based solely
on those beliefs—it extends to both beliefs and unbeliefs.
Escritor’s claim of religious freedom (the possibility that the i. e. religious speech and expressive religious conduct o
claim to religious freedom would warrant carving out an BELIEF-ACTION TEST which allows absolute protection
exception from the Civil Service Law) versus government’s to belief but not to action.
showing of compelling interest. o DELIBERATE-INADVERTENT DISTINCTION -
In applying the test, the first inquiry is whether respondent’s the distinction between deliberate state
right to religious freedom has been burdened. There is no interference of religious exercise for religious
doubt that choosing between keeping her employment and reasons which was plainly unconstitutional and
abandoning her religious belief and practice and family on the government’s inadvertent interference with religion
one hand, and giving up her employment and keeping her in pursuing some secular objective.
religious practice and family on the other hand, puts a burden o BALANCING TEST - a two-part balancing test of
on her free exercise of religion. The burden on respondent in validity where the first step was for plaintiff to show
the case at bar is even greater as the price she has to pay for that the regulation placed a real burden on his
her employment is not only her religious precept but also her religious exercise. Next, the burden would be
family which, by the Declaration Pledging Faithfulness, stands upheld only if the state showed that it was pursuing
“honorable before God and men.” an overriding secular goal by the means which
imposed the least burden on religious practices.
The second step is to ascertain respondent’s sincerity in her o COMPELLING STATE INTEREST TEST –
religious belief. Respondent appears to be sincere in her stresses that the state interest is not merely any
religious belief and practice and is not merely using the colorable state interest, but must be paramount
“Declaration of Pledging Faithfulness” to avoid punishment for and compelling to override the free exercise claim.
immorality. She did not secure the Declaration only after o EXEMPTION DOCTRINE - Under the exemption
entering the judiciary where the moral standards are strict and doctrine when general laws conflict with scruples
defined, much less only after an administrative case for of conscience, exemptions ought to be granted
immorality was filed against her. The Declaration was issued unless some “compelling state interest”
to her by her congregation after ten years of living together intervenes.
with her partner, Quilapio, and ten years before she entered
the judiciary. Ministers from her congregation testified on the  NON ESTABLISHMENT CLAUSE creates “a wall of
authenticity of the Jehovah’s Witnesses’ practice of securing separation between church and state”. o NEUTRALITY
a Declaration and their doctrinal or scriptural basis for such a AND BENEVOLENT NEUTRALITY STANDARDS - Two
practice. As the ministers testified, the Declaration is not main standards used by the U.S. Supreme Court in
whimsically issued to avoid legal punishment for illicit conduct deciding religion clause cases—separation (in the form of
but to make the “union” of their members under respondent’s strict separation or the tamer version of strict neutrality or
circumstances “honorable before God and men.” It is also separation) and benevolent neutrality or accommodation.
worthy of notice that the Report and Recommendation of the o The STRICT SEPARATIONIST VIEW holds that
investigating judge annexed letters of the OCA to the the “wall of separation” is meant to protect the
respondent regarding her request to be exempt from attending state from the church.
the flag ceremony after Circular No. 62-2001 was issued o Under BENEVOLENT NEUTRALITY, which gives
requiring attendance in the flag ceremony. The OCA’s letters room for accommodation, the wall of separation is
were not submitted by respondent as evidence but annexed meant to protect the church from the state.
by the investigating judge in explaining that he was caught in o An ACCOMMODATIONIST holds that it is good
a dilemma whether to find respondent guilty of immorality public policy, and sometimes constitutionally
because the Court Administrator and Deputy Court required, for the state to make conscious and
Administrator had different positions regarding respondent’s deliberate efforts to avoid interference with
request for exemption from the flag ceremony on the ground religious freedom.
of the Jehovah’s Witnesses’ contrary belief and practice. o A STRICT NEUTRALITY ADHERENT believes
Respondent’s request for exemption from the flag ceremony that it is good public policy, and also
shows her sincerity in practicing the Jehovah’s Witnesses’ constitutionally required, for the government to
beliefs and not using them merely to escape punishment. She avoid religion-specific policy even at the cost of
is a practicing member of the Jehovah’s Witnesses and the inhibiting religious exercise.
Jehovah ministers testified that she is a member in good
standing.  PHILIPPINE RELIGION CLAUSES CASE LAW A close
GUYS THE CASE WAS REMANDED TO THE OCA KASI DI scrutiny of the Philippine cases involving the religion
NAGPARTICIPATE SI SOLGEN. SO INORDER SYA TO clauses would reveal that while U.S. jurisprudence on
INTERVENE AND TO SHOW THE STATES’ COMPELLING religion clauses flows into two main streams of
REASON…. In the final decision on the case, Estrada v. interpretation—separation and benevolent neutrality—
Escritor, 492 SCRA 1 (2006), the Court then went on to allow the well-spring of Philippine jurisprudence on this subject
a court interpreter to continue with her conjugal arrangement is for the most part benevolent neutrality which gives
with a married man not her husband, since the same was room for accommodation. o The Philippine Supreme
sanctioned by her religion and there was no compelling state Court has adopted a posture of not invalidating a law
interest that would have warranted overriding her religion- offensive to religious freedom, but carving out an
grounded practice – “we find that in this particular case and exception or upholding an exception to accommodate
under these distinct circumstances, respondent Escritor’s religious exercise where it is justified.
conjugal arrangement cannot be penalized as she has made o How the tension between the Non Establishment
out a case for exemption from the law based on her Clause and the Free Exercise Clause will be
fundamental right to freedom of religion.” resolved is a question for determination in the
actual cases that come to the Court—the two
clauses should be balanced against each other.
[RELEVANT DEFINITONS] o Although our constitutional history and
interpretation mandate benevolent neutrality, it
 FREE EXERCISE CLAUSE accords absolute protection does not mean that the Court ought to grant
to individual religious convictions and beliefs and exemptions every time a free exercise claim

Page 108 of 186


comes before it, but that the Court will not look with 3. Ang Ladlad LGBT Party v. COMELEC (G.R. No.
hostility or act indifferently towards religious beliefs 190582, April 8, 2010)
and practices and that it will strive to accommodate
them when it can within flexible constitutional
limits; it does mean that the Court will not simply FACTS: Ang Ladlad is an organization composed of men and
dismiss a claim under the Free Exercise Clause. women who identify themselves as lesbians, gays, bisexuals,
o When the law speaks of “immorality” in the Civil or trans-gendered individuals (LGBTs). Incorporated in 2003,
Service Law or “immoral” in the Code of Ang Ladlad first applied for registration with the COMELEC in
Professional Responsibility for lawyers, or “public 2006. The application for accreditation was denied on the
morals” in the Revised Penal Code, or “morals” in ground that the organization had no substantial membership
the New Civil Code, or “moral character” in the base. On August 17, 2009, Ang Ladlad again filed a Petition
Constitution, the distinction between public and for registration with the COMELEC. Before the COMELEC,
secular morality on the one hand, and religious petitioner argued that the LGBT community is a marginalized
morality, on the other, should be kept in mind—the and under-represented sector that is particularly
morality referred to in the law is public and disadvantaged because of their sexual orientation and gender
necessarily secular, not religious. identity; that LGBTs are victims of exclusion, discrimination,
and violence; that because of negative societal attitudes,
LGBTs are constrained to hide their sexual orientation; and
that Ang Ladlad complied with the 8-point guidelines
enunciated by this Court in Ang Bagong Bayani-OFW Labor
Party v. Commission on Elections. Ang Ladlad laid out its
national membership base consisting of individual members
and organizational supporters, and outlined its platform of
governance. On November 11, 2009, after admitting the
petitioner’s evidence, the COMELEC (Second Division)
dismissed the Petition on moral grounds –

a marginalized and under-represented sector that is


particularly disadvantaged because of their sexual
orientation and gender identity.
and proceeded to define sexual orientation as that
which:
x x x refers to a person’s capacity for profound
emotional, affectional and sexual attraction to, and
intimate and sexual rela tions with, individuals of a
different gender, of the same gender, or more than
one gender.
This definition of the LGBT sector makes it crystal
clear that petitioner tolerates immorality which
offends religious beliefs.

The ANG LADLAD apparently advocates sexual


immorality as indicated in the Petition’s par. 6F:
‘Consensual partnerships or relationships by gays
and lesbians who are already of age’. It is further
indicated in par. 24 of the Petition which waves for
the record: ‘In 2007, Men Having Sex with Men or
MSMs in the Philippines were estimated as 670,000
(Genesis 19 is the history of Sodom and Gomorrah).
Laws are deemed incorporated in every contract,
permit, license, relationship, or accreditation. Hence,
pertinent provisions of the Civil Code and the
Revised Penal Code are deemed part of the
requirement to be complied with for accreditation.

ANG LADLAD collides with Article 695 of the Civil


Code which defines nuisance as ‘Any act, omission,
establishment, business, condition of property, or
anything else which x x x (3) shocks, defies; or
disregards decency or morality x x x

It also collides with Article 1306 of the Civil Code:


‘The contracting parties may establish such
stipulations, clauses, terms and conditions as they
may deem convenient, provided they are not
contrary to law, morals, good customs, public order
or public policy. Art 1409 of the Civil Code provides
that ‘Contracts whose cause, object or purpose is
contrary to law, morals, good customs, public order
or public policy’ are inexistent and void from the
beginning.

When Ang Ladlad sought reconsideration,9 three


commissioners voted to overturn the First Assailed Resolution

Page 109 of 186


(Commissioners Gregorio Y. Larrazabal, Rene V. Sarmiento, constitutional guarantees against the establishment of
and Armando Velasco), while three commissioners voted to religion. The OSG concurred with Ang Ladlad’s petition and
deny Ang Ladlad’s Motion for Reconsideration argued that the COMELEC erred in denying petitioner’s
(Commissioners Nicodemo T. Ferrer, Lucenito N. Tagle, and application for registration since there was no basis for
Elias R. Yusoph). The COMELEC Chairman, breaking the tie COMELEC’s allegations of immorality. It also opined that
and speaking for the majority in his Separate Opinion, upheld LGBTs have their own special interests and concerns which
the First Assailed Resolution – should have been recognized by the COMELEC as a separate
classification.
Ladlad is applying for accreditation as a sectoral
party in the party-list system. Even assuming that it In its Comment, the COMELEC reiterated that petitioner does
has properly proven its under-representation and not have a concrete and genuine national political agenda to
marginalization, it cannot be said that Ladlad’s benefit the nation and that the petition was validly dismissed
expressed sexual orientations per se would benefit on moral grounds. It also argued for the first time that the
the nation as a whole. LGBT sector is not among the sectors enumerated by the
Constitution and RA 7941
Section 2 of the party-list law unequivocally states
that the purpose of the party-list system of electing
congressional representatives is to enable Filipino ISSUE: Whether Ang Ladlad party-list should be given
citizens belonging to marginalized and under- accreditation
represented sectors, organizations and parties, and
who lack well-defined political constituencies but who
could contribute to the formulation and enactment of RULING: Yes as they have sufficiently demonstrated its
appropriate legislation that will benefit the nation as compliance with the legal requirements for accreditation.
a whole, to become members of the House of Respondent mistakenly opines that our ruling in Ang Bagong
Representatives. If entry into the party-list system Bayani stands for the proposition that only those sectors
would depend only on the ability of an organization specifically enumerated in the law or related to said sectors
to represent its constituencies, then all (labor, peasant, fisherfolk, urban poor, indigenous cultural
representative organizations would have found communities, elderly, handicapped, women, youth, veterans,
themselves into the party-list race. But that is not the overseas workers, and professionals) may be registered
intention of the framers of the law. The party-list under the party-list system. As we explicitly ruled in Ang
system is not a tool to advocate tolerance and Bagong Bayani-OFW Labor Party v. Commission on
acceptance of misunderstood persons or groups of Elections, “the enumeration of marginalized and under-
persons. Rather, the party-list system is a tool for the represented sectors is not exclusive.” The crucial element is
realization of aspirations of marginalized individuals not whether a sector is specifically enumerated, but whether a
whose interests are also the nation’s—only that their particular organization complies with the requirements of the
interests have not been brought to the attention of Constitution and RA 7941.
the nation because of their under representation.
Until the time comes when Ladlad is able to justify The Court finds that there has been no misrepresentation. A
that having mixed sexual orientations and cursory perusal of Ang Ladlad’sinitial petition shows that it
transgender identities is beneficial to the nation, its never claimed to exist in each province of the Philippines.
application for accreditation under the party-list Rather, petitioner alleged that the LGBT community in the
system will remain just that. Philippines was estimated to constitute at least 670,000
persons; that it had 16,100 affiliates and members around the
Even if society’s understanding, tolerance, and country, and 4,044 members in its electronic discussion
acceptance of LGBT’s is elevated, there can be no group. Ang Ladladalso represented itself to be “a national
denying that Ladlad constituencies are still males LGBT umbrella organization with affiliates around the
and females, and they will remain either male or Philippines composed of the following LGBT networks XXX”
female protected by the same Bill of Rights that Since the COMELEC only searched for the names ANG
applies to all citizens alike. LADLAD LGBT or LADLAD LGBT, it is no surprise that they
found that petitioner had no presence in any of these regions.
What are being adopted as moral parameters and In fact, if COMELEC’s findings are to be believed, petitioner
precepts are generally accepted public morals. They does not even exist in Quezon City, which is registered as Ang
are possibly religious-based, but as a society, the Ladlad’s principal place of business.
Philippines cannot ignore its more than 500 years of
Muslim and Christian upbringing, such that some
moral precepts espoused by said religions have
Our Constitution provides in Article III, Section 5 that “[n]o law
sipped [sic] into society and these are not publicly
shall be made respecting an establishment of religion, or
accepted moral norms.
prohibiting the free exercise thereof.” At bottom, what our non-
establishment clause calls for is “government neutrality in
But above morality and social norms, they have
religious matters.” Clearly, “governmental reliance on religious
become part of the law of the land. Article 201 of the
justification is inconsistent with this policy of neutrality.” We
Revised Penal Code imposes the penalty of prision
thus find that it was grave violation of the non-establishment
mayor upon “Those who shall publicly expound or
clause for the COMELEC to utilize the Bible and the Koran to
proclaim doctrines openly contrary to public morals.”
justify the exclusion of Ang Ladlad. The government must act
It penalizes “immoral doctrines, obscene
for secular purposes and in ways that have primarily secular
publications and exhibition and indecent shows.”
effects. As we held in Estrada v. Escritor:
“Ang Ladlad” apparently falls under these legal
provisions. if government relies upon religious beliefs in
formulating public policies and morals, the resulting
On January 4, 2010, Ang Ladlad filed this Petition, praying that policies and morals would require conformity to what
the Court annul the Assailed Resolutions and direct the some might regard as religious programs or agenda.
COMELEC to grant Ang Ladlad’s application for accreditation. The non-believers would therefore be compelled to
Ang Ladlad argued that the denial of accreditation, insofar as conform to a standard of conduct buttressed by a
it justified the exclusion by using religious dogma, violated the religious belief.

Page 110 of 186


government action, including its proscription of homosexual behavior. Indeed, even if we were to assume that
immorality as expressed in criminal law like public opinion is as the COMELEC describes it, the asserted
concubinage, must have a secular purpose. That is, state interest here—that is, moral disapproval of an unpopular
the government proscribes this conduct because it is minority—is not a legitimate state interest that is sufficient to
“detrimental (or dangerous) to those conditions upon satisfy rational basis review under the equal protection clause.
which depend the existence and progress of human The COMELEC’s differentiation, and its unsubstantiated claim
society” and not because the conduct is proscribed that Ang Ladlad cannot contribute to the formulation of
by the beliefs of one religion or the other. legislation that would benefit the nation, furthers no legitimate
state interest other than disapproval of or dislike for a
disfavored group.
Respondent suggests that although the moral condemnation
of homosexuality and homosexual conduct may be religion-
based, it has long been transplanted into generally accepted Under our system of laws, every group has the right to
public morals. We are not blind to the fact that, through the promote its agenda and attempt to persuade society of the
years, homosexual conduct, and perhaps homosexuals validity of its position through normal democratic means. It is
themselves, have borne the brunt of societal disapproval. It is in the public square that deeply held convictions and differing
not difficult to imagine the reasons behind this censure— opinions should be distilled and deliberated upon. Freedom of
religious beliefs, convictions about the preservation of expression constitutes one of the essential foundations of a
marriage, family, and procreation, even dislike or distrust of democratic society, and this freedom applies not only to those
homosexuals themselves and their perceived lifestyle. that are favorably received but also to those that offend,
Nonetheless, we recall that the Philippines has not seen fit to shock, or disturb. Any restriction imposed in this sphere must
criminalize homosexual conduct. Evidently, therefore, these be proportionate to the legitimate aim pursued. Absent any
“generally accepted public morals” have not been convincingly compelling state interest, it is not for the COMELEC or this
transplanted into the realm of law. The Assailed Resolutions Court to impose its views on the populace. Otherwise stated,
have not identified any specific overt immoral act performed the COMELEC is certainly not free to interfere with speech for
by Ang Ladlad. Even the OSG agrees that “there should have no better reason than promoting an approved message or
been a finding by the COMELEC that the group’s members discouraging a disfavored one. We do not doubt that a number
have committed or are committing immoral acts.” Respondent of our citizens may believe that homosexual conduct is
has failed to explain what societal ills are sought to be distasteful, offensive, or even defiant. They are entitled to hold
prevented, or why special protection is required for the youth. and express that view. On the other hand, LGBTs and their
Neither has the COMELEC condescended to justify its supporters, in all likelihood, believe with equal fervor that
position that petitioner’s admission into the party-list system relationships between individuals of the same sex are morally
would be so harmful as to irreparably damage the moral fabric equivalent to heterosexual relationships. They, too, are
of society. entitled to hold and express that view. However, as far as this
Court is concerned, our democracy precludes using the
religious or moral views of one part of the community to
exclude from consideration the values of other members of the
We also find the COMELEC’s reference to purported
community. The OSG argues that since there has been
violations of our penal and civil laws flimsy, at best;
neither prior restraint nor subsequent punishment imposed on
disingenuous, at worst. Article 694 of the Civil Code defines a
Ang Ladlad, and its members have not been deprived of their
nuisance as “any act, omission, establishment, condition of
right to voluntarily associate, then there has been no
property, or anything else which shocks, defies, or disregards
restriction on their freedom of expression or association. The
decency or morality,” the remedies for which are a prosecution
OSG fails to recall that petitioner has, in fact, established its
under the Revised Penal Code or any local ordinance, a civil
qualifications to participate in the party-list system, and—as
action, or abatement without judicial proceedings. A violation
advanced by the OSG itself—the moral objection offered by
of Article 201 of the Revised Penal Code, on the other hand,
the COMELEC was not a limitation imposed by law.
requires proof beyond reasonable doubt to support a criminal
conviction. It hardly needs to be emphasized that mere
allegation of violation of laws is not proof, and a mere blanket
invocation of public morals cannot replace the institution of The Court explicitly recognize the principle of non-
civil or criminal proceedings and a judicial determination of discrimination as it relates to the right to electoral participation,
liability or culpability. enunciated in the UDHR and the ICCPR. The principle of non-
discrimination is laid out in Article 26 of the ICCPR, as follows:

All persons are equal before the law and are entitled
Despite the absolutism of Article III, Section 1 of our without any discrimination to the equal protection of
Constitution, which provides “nor shall any person be denied the law. In this respect, the law shall prohibit any
equal protection of the laws,” courts have never interpreted discrimination and guarantee to all persons equal
the provision as an absolute prohibition on classification. and effective protection against discrimination on any
“Equality,” said Aristotle, “consists in the same treatment of ground such as race, colour, sex, language, religion,
similar persons.” The equal protection clause guarantees that political or other opinion, national or social origin,
no person or class of persons shall be deprived of the same property, birth or other status.
protection of laws which is enjoyed by other persons or other
classes in the same place and in like circumstances. Recent In this context, the principle of non-discrimination requires that
jurisprudence has affirmed that if a law neither burdens a laws of general application relating to elections be applied
fundamental right nor targets a suspect class, we will uphold equally to all persons, regardless of sexual orientation.
the classification as long as it bears a rational relationship to Although sexual orientation is not specifically enumerated as
some legitimate government end. The COMELEC posits that a status or ratio for discrimination in Article 26 of the ICCPR,
the majority of the Philippine population considers the ICCPR Human Rights Committee has opined that the
homosexual conduct as immoral and unacceptable, and this reference to “sex” in Article 26 should be construed to include
constitutes sufficient reason to disqualify the petitioner. “sexual orientation.” As stated by the CHR in its Comment-in-
Unfortunately for the respondent, the Philippine electorate has Intervention, the scope of the right to electoral participation is
expressed no such belief. No law exists to criminalize elaborated by the Human Rights Committee in its General
homosexual behavior or expressions or parties about

Page 111 of 186


Comment No. 25 (Participation in Public Affairs and the Right 4. RE: LETTER OF TONY Q. V ALENCIANO, HOLDING
to Vote) as follows: OF RELIGIOUS RITUALS AT THE HALL OF JUSTICE
BUILDING IN QUEZON CITY (A.M. No. 10-4-19-SC)
“1. Article 25 of the Covenant recognizes and
protects the right of every citizen to take part in the
conduct of public affairs, the right to vote and to be
elected and the right to have access to public FACTS: This controversy originated from a series of letters,
service. Whatever form of constitution or government written by Tony Q. Valenciano (Valenciano) and addressed to
is in force, the Covenant requires States to adopt then Chief Justice Reynato S. Puno (Chief Justice Puno).
such legislative and other measures as may be
necessary to ensure that citizens have an effective In his first Letter, Valenciano reported that the basement of the
opportunity to enjoy the rights it protects. Article 25 Hall of Justice of Quezon City (QC) had been converted into
lies at the core of democratic government based on a Roman Catholic Chapel, complete with offertory table,
the consent of the people and in conformity with the images of Catholic religious icons, a canopy, an electric organ,
principles of the Covenant. and a projector. He believed that such practice violated the
We stress, however, that although this Court stands willing to constitutional provision on the separation of Church and State
assume the responsibility of giving effect to the Philippines’ and the constitutional prohibition against the appropriation of
international law obligations, the blanket invocation of public money or property for the benefit of a sect, church,
international law is not the panacea for all social ills. We refer denomination, or any other system of religion.
now to the petitioner’s invocation of the Yogyakarta Principles
(the Application of International Human Rights Law In Relation Valenciano further averred that the holding of masses at the
to Sexual Orientation and Gender Identity), which petitioner basement of the QC Hall of Justice showed that it tended to
declares to reflect binding principles of international law. At favor Catholic litigants; that the rehearsals of the choir caused
this time, we are not prepared to declare that these great disturbance to other employees; that the public could no
Yogyakarta Principles contain norms that are obligatory on the longer use the basement as resting place; that the employees
Philippines. There are declarations and obligations outlined in and litigants of the Public Attorney's Office (PAO), RTC, Legal
said Principles which are not reflective of the current state of Library, Philippine Mediation Center, and Records Section of
international law, and do not find basis in any of the sources the Office of the Clerk of Court (OCC) could not attend to their
of international law enumerated under Article 38(1) of the
personal necessities such as going to the lavatories because
Statute of the International Court of Justice.
they could not traverse the basement between 12:00 o'clock
Using even the most liberal of lenses, these Yogyakarta noontime and 1: 15 o'clock in the afternoon; that the court
Principles, consisting of a declaration formulated by various employees became hostile toward each other as they vied for
international law professors, are—at best—de lege ferenda— the right to read the epistle; and that the water supply in the
and do not constitute binding obligations on the Philippines. entire building was cut off during the mass because the
Indeed, so much of contemporary international law is generator was turned off to ensure silence.
characterized by the “soft law” nomenclature, i.e., international
law is full of principles that promote international cooperation, Valenciano wrote another letter, praying that rules be
harmony, and respect for human rights, most of which amount promulgated by the Court to put a stop to the holding of
to no more than well-meaning desires, without the support of Catholic masses, or any other religious rituals, at the QC Hall
either State practice or opinio juri. of Justice and in all other halls of justice in the country.

The Court noted the letter of Valenciano and referred the


matter to the Office of the Court Administrator (OCA) for
evaluation, report and recommendation.

In its Memorandum, the OCA believed that the practical


inconveniences cited by Valenciano were unfounded. It, thus,
recommended that his letter-complaints be dismissed for lack
of merit.

ISSUE: WON the holding of masses at the basement of the


Quezon City Hall of Justice violates the constitutional principle
of separation of church and state as well as the constitutional
prohibition against appropriation of public money or property
for the benefit of any sect, church, denomination, sectarian
institution, or system of religion. NO

HELD: Rituals in the Halls of Justice does not Amount to a


Union of Church and State

The State still recognizes the inherent right of the people to


have some form of belief system, whether such may be belief
in a Supreme Being, a certain way of life, or even an outright
rejection of religion.

Thus, the right to believe or not to believe has again been


enshrined in Section 5, Article III of the 1987 Constitution:

Page 112 of 186


Section 5. xxx. The free exercise and enjoyment of initiative as they are there on their own free will and volition,
religious profession and worship, without without any coercion from the judges or administrative
discrimination or preference, shall forever be officers. Third, no government funds are being spent because
allowed. xxx. the lightings and airconditioning continue to be operational
even if there are no religious rituals there. Fourth, the
"The right to religious profession and worship has a basement has neither been converted into a Roman Catholic
two-fold aspect - freedom to believe and freedom to chapel nor has it been permanently appropriated for the
act on one's beliefs. The first is absolute as long as exclusive use of its faithful. Fifth, the allowance of the masses
the belief is confined within the realm of thought. The has not prejudiced other religions.
second is subject to regulation where the belief is
translated into external acts that affect the public No Appropriation of Public Money or Property for the Benefit
welfare." of any Church

Allowing religion to flourish is not contrary to the principle of Here, the basement of the QC Hall of Justice is not
separation of Church and State. In fact, these two principles appropriated, applied or employed for the sole purpose of
are in perfect harmony with each other. supporting the Roman Catholics.

Clearly, allowing the citizens to practice their religion is not Further, it has not been converted into a Roman Catholic
equivalent to a fusion of Church and State. chapel for the exclusive use of its faithful contrary to the claim
of Valenciana. Judge Maceren reported that the basement is
also being used as a public waiting area for most of the day
and a meeting place for different employee organizations. The
No Compelling State Interest
use of the area for holding masses is limited to lunch break
Religious freedom, however, is not absolute. It cannot have its period from twelve (12) o'clock to one (1) o'clock in the
way if there is a compelling state interest. To successfully afternoon. Further, Judge Sagun, Jr. related that masses run
invoke compelling state interest, it must be demonstrated that for just a little over thirty (30) minutes. It is, therefore, clear that
the masses in the QC Hall of Justice unduly disrupt the no undue religious bias is being committed when the subject
delivery of public services or affect the judges and employees basement is allowed to be temporarily used by the Catholics
in the performance of their official functions. to celebrate mass, as the same area can be used by other
groups of people and for other purposes.49 Thus, the
As reported by the Executive Judges of Quezon City, the basement of the QC Hall of Justice has remained to be a
masses were being conducted only during noon breaks and public property devoted for public use because the holding of
were not disruptive of public services. The court proceedings Catholic masses therein is a mere incidental consequence of
were not being distracted or interrupted and that the its primary purpose.
performance of the judiciary employees were not being
adversely affected. Moreover, no Civil Service rules were
being violated. As there has been no detrimental effect on the
Conclusion
public service or prejudice to the State, there is simply no state
interest compelling enough to prohibit the exercise of religious Directing the Executive Judges of the RTC and MeTC to
freedom in the halls of justice. regulate and closely monitor the holding of masses and other
religious practices within the courts does not promote
excessive collaboration between courts and various religions.
Accommodation, Not Establishment of Religion On the contrary, this is necessary to ensure that there would
be no excessive entanglement.
In order to give life to the constitutional right of freedom of
religion, the State adopts a policy of accommodation. To disallow the holding of religious rituals within halls of justice
Accommodation is a recognition of the reality that some would set a dangerous precedent and commence a domino
governmental measures may not be imposed on a certain effect. Strict separation, rather than benevolent
portion of the population for the reason that these measures neutrality/accommodation, would be the norm. Thus, the
are contrary to their religious beliefs. As long as it can be establishment of Shari'a courts, the National Commission for
shown that the exercise of the right does not impair the public Muslim Filipinos, and the exception of Muslims from the
welfare, the attempt of the State to regulate or prohibit such provisions of the RPC relative to the crime of bigamy would all
right would be an unconstitutional encroachment. be rendered nugatory because of strict separation. The
exception of members of Iglesia ni Cristo from joining a union
Establishment entails a positive action on the part of the State. or the non-compulsion recognized in favor of members of the
Accommodation, on the other hand, is passive. In the former, Jehovah's Witnesses from doing certain gestures during the
the State becomes involved through the use of government flag ceremony, will all go down the drain simply because we
resources with the primary intention of setting up a state insist on strict separation.
religion. In the latter, the State, without being entangled,
merely gives consideration to its citizens who want to freely That the holding of masses at the basement of the QC Hall of
exercise their religion. Justice may offend non-Catholics is no reason to proscribe it.
Our Constitution ensures and mandates an unconditional
Guided by the foregoing, it is our considered view that the tolerance, without regard to whether those who seek to
holding of Catholic masses at the basement of the QC Hall of profess their faith belong to the majority or to the minority. It is
Justice is not a case of establishment, but merely emphatic in saying that "the free exercise and enjoyment of
accommodation. First, there is no law, ordinance or circular religious profession and worship shall be without
issued by any duly constitutive authorities expressly discrimination or preference." Otherwise, accommodation or
mandating that judiciary employees attend the Catholic tolerance would just be mere lip service.
masses at the basement. Second, when judiciary employees
attend the masses to profess their faith, it is at their own

Page 113 of 186


5. DENMARK S. V ALMORES vs. DR. CRISTINA respondent to perform the act required; such duty need not be
ACHACOSO, in her capacity as Dean of the College absolutely expressed, so long as it is clear.49 In this regard, a
of Medicine, and DR. GIOVANNI CABILDO, Faculty duty is considered ministerial where an officer is required to
of the Mindanao State University (G.R. No. 217453) perform an act not requiring the exercise of official discretion
or judgment in a given state of facts.50 Conversely, if the law
imposes a duty upon a public officer and gives him the right to
decide how or when the duty shall be performed, such duty is
FACTS: Petitioner Denmark S. Valmores (Valmores) is a
discretionary.
member of the Seventh-day Adventist Church, whose
fundamental beliefs include the strict observance of the MSU is an HEI created by legislative charter under Republic
Sabbath as a sacred day. As such, petitioner Valmores joins Act No. 1387, as amended, and was established "to better
the faithful in worshipping and resting on Saturday, the implement the policy of the Government in the intensification
seventh day of the week, and refrains from non-religious of the education of the Filipino youth, especially among the
undertakings from sunset of Friday to sunset of Saturday. Muslims and others belonging to the national minorities."52
Thus, respondents herein, as faculty members of MSU, fall
Prior to the instant controversy, petitioner Valmores was
under the policy-making authority of the CHED and therefore
enrolled as a first-year student at the MSU-College of
bound to observe the issuances promulgated by the latter.
Medicine. Some of petitioner Valmores' classes and
examinations were moved from weekdays to Saturdays. In At once, a plain reading of the memorandum reveals the
one instance, petitioner Valmores was unable to take his ministerial nature of the duty imposed upon HEIs. Its policy is
Risto-Pathology laboratory examination held on a Saturday. crystal clear: a student's religious obligations takes
Respondent Cabildo was his professor for the said subject. precedence over his academic responsibilities, consonant
Despite his request for exemption, no accommodation was with the constitutional guarantee of free exercise and
given by either of the respondents. As a result, petitioner enjoyment of religious worship. Accordingly, the CHED
Valmores received a failing grade of 5 for that particular imposed a positive duty on all HEIs to exempt students, as
module and was considered ineligible to retake the exam. well as faculty members, from academic activities in case
such activities interfere with their religious obligations.
Thereafter, several pastors and officers of the Seventh-day
Adventist Church sent a letter to respondent Achacoso, Although the said memorandum contains the phrase "within
requesting for a possible audience with the members of the the bounds of school rules and regulations," the same relates
MSU school board. In addition, the church, through Pastor only to the requirement of remedial work, which, based on the
Hanani P. Nietes, issued a Certification in connection with language used, is merely optional on the part of the HEI.
petitioner Valmores' request for exemption. Neither can such phrase be said to have conferred discretion
as the use of the words "shall be enjoined" and "strict
Despite the foregoing communications, petitioner Valmores'
compliance" denote a mandatory duty on the part of the HEI
requests fell on deaf ears.
to excuse its students upon submission of the certification
Petitioner Valmores elevated the matter before the CHED. In prescribed in the same memorandum.
an Indorsement, the CHED Regional Office referred the
Clearly, under the 2010 CHED Memorandum, HEIs do not
matter directly to the President of MSU as well as respondent
possess absolute discretion to grant or deny requests for
Achacoso and requested that the office be advised of the
exemption of affected students. Instead, the memorandum
action thus taken.
only imposes minimum standards should HEIs decide to
Dr. Muslim, President of MSU, instructed respondent require remedial work, i.e., that the same is within the bounds
Achacoso to enforce the 2010 CHED Memorandum. Despite of school rules and regulations and that the grades of the
the foregoing correspondence, petitioner Valmores' request students will not be affected.
still went unheeded.
As a condition for exemption, the 2010 CHED Memorandum
simply requires the submission of "a certification or proof of
attendance/participation duly signed by their pastor, priest,
ISSUE: WON mandamus lies to compel respondents to minister or religious leader for periods of absence from
enforce the 2010 CHED Memorandum in the case of petitioner classes, work or school activities."
Valmores. YES
The certification submitted to the private respondents needs
little or no interpretation: petitioner Valmores, as a bona fide
member of the Seventh-day Adventist Church, is expected to
HELD: In 2010, the CHED institutionalized the framework for
miss "all his Sabbath x x x classes [and] exams" due to his
operationalizing Section 5, Article Ill of the 1987 Constitution
observance of the Sabbath day as a day of worship. There is
vis-a-vis the academic freedom of higher education
nothing in the 2010 CHED Memorandum that prohibits the
institutions (HEis ), pursuant to its statutory power to formulate
certification from being issued before the period of absence
policies, priorities, and programs on higher education in both
from class. Even then, the Certification dated September 15,
public and private HEIs.
2014 is broad enough to cover both past and future Sabbath
In the 2010 CHED Memorandum, the CHED laid down days for which petitioner Valmores would be absent.
guidelines for the exemption of teachers, personnel, and
For these reasons, the Court finds that respondents were duty
students from participating in school or related activities due
bound to enforce the 2010 CHED Memorandum insofar as it
to compliance with religious obligations.
requires the exemption of petitioner Valmores from academic
Mandamus is employed to compel the performance of a responsibilities that conflict with the schedule of his Saturday
ministerial duty by a tribunal, board, officer, or person.48 Case worship. Their failure to do so is therefore correctible by
law requires that the petitioner should have a right to the thing mandamus.
demanded and that it must be the imperative duty of the

Page 114 of 186


Without more, respondents' bare arguments crumble against H. LIBERTY OF ABODE AND FREEDOM OF MOVEMENT
constitutional standards. As discussed above, the Bill of
Rights guarantees citizens the freedom to act on their
individual beliefs and proscribes government intervention
1. B/GEN. (RET.) FRANCISCO V. GUDANI AND LT.
unless necessary to protect its citizens from injury or when
COL. ALEXANDER F. BALUTAN vs. LT./GEN.
public safety, peace, comfort, or convenience requires it.
GENEROSO S. SENGA CORONA, AS CHIEF OF
Thus, as faculty members of the MSU-College of Medicine,
STAFF OF THE CARPIO-MORALES, ARMED
respondents herein were duty-bound to protect and preserve
FORCES OF THE CALLEJO, SR., PHILIPPINES, COL.
petitioner Valmores' religious freedom.
GILBERTO AZCUNA, JOSE C. ROA AS THE PRE-
While in some cases the Court has sustained government TRIAL TINGA, INVESTIGATING OFFICER, THE
regulation of religious rights, the Court fails to see in the CHICO-NAZARIO, PROVOST MARSHALL GENERAL
present case how public order and safety will be served by the GARCIA, and OF THE ARMED FORCES OF THE
denial of petitioner Valmores' request for exemption. Neither PHILIPPINES AND THE GENERAL COURT-MARTIAL
is there any showing that petitioner Valmores' absence from (G.R. No. 170165 August 15, 2006)
Saturday classes would be injurious to the rights of others.
Precisely, the 2010 CHED Memorandum was issued to
address such conflicts and prescribes the action to be taken FACTS: The petitioners are high-ranking officers of the Armed
by HEIs should such circumstance arise. Forces of the Philippines (AFP). Both petitioners, Brigadier
General Francisco Gudani (Gen. Gudani) and Lieutenant
What is certain, as gathered from the foregoing, is that
Colonel Alexander Balutan (Col. Balutan), belonged to the
respondents' concerted refusal to accommodate petitioner
Philippine Marines.
Valmores rests mainly on extralegal grounds, which cannot,
by no stretch of legal verbiage, defeat the latter's Senator Rodolfo Biazon (Sen. Biazon) invited several senior
constitutionally-enshrined rights. That petitioner Valmores is officers of the AFP to appear at a public hearing before the
being made by respondents to choose between honoring his Senate Committee. The hearing was scheduled after topics
religious obligations and finishing his education is a patent concerning the conduct of the 2004 elections emerged in the
infringement of his religious freedoms. As the final bulwark of public eye, particularly allegations of massive cheating and
fundamental rights, this Court will not allow such violation to the surfacing of copies of an audio excerpt purportedly of a
perpetuate any further. phone conversation between President Gloria Macapagal
Arroyo and an official of the Commission on Elections
(COMELEC) widely reputed as then COMELEC
Commissioner Virgilio Garcillano. At the time of the 2004
elections, Gen. Gudani had been designated as commander,
and Col. Balutan a member, of "Joint Task Force Ranao" by
the AFP Southern Command. "Joint Task Force Ranao" was
tasked with the maintenance of peace and order during the
2004 elections in the provinces of Lanao del Norte and Lanao
del Sur.

Gen. Gudani, Col. Balutan, and AFP Chief of Staff Lieutenant


General Generoso Senga (Gen. Senga) were among the
several AFP officers who received a letter invitation from Sen.
Biazon to attend the hearing.

The Office of the Chief of Staff of the AFP issued a


Memorandum. Noting that Gen. Gudani and Col. Balutan had
been invited to attend the Senate Committee hearing, the
Memorandum directed the two officers to attend the hearing.
Conformably, Gen. Gudani and Col. Balutan filed their
respective requests for travel authority addressed to the PMA
Superintendent.

Gen. Senga sent a letter to Sen. Biazon, informing the senator


that "no approval has been granted by the President to any
AFP officer to appear" before the hearing scheduled on that
day. Nonetheless, both Gen. Gudani and Col. Balutan were
present as the hearing started, and they both testified as to
the conduct of the 2004 elections.

Gen. Senga issued a statement which noted that the two had
appeared before the Senate Committee "in spite of the fact
that a guidance has been given that a Presidential approval
should be sought prior to such an appearance;" that such
directive was "in keeping with the time[-]honored principle of
the Chain of Command;" and that the two officers "disobeyed
a legal order, in violation of A[rticles of] W[ar] 65 (Willfully
Disobeying Superior Officer), hence they will be subjected to
General Court Martial proceedings x x x" Both Gen. Gudani

Page 115 of 186


and Col. Balutan were likewise relieved of their assignments before they could travel to Manila to attend the Senate
then. Hearing.

Subsequently, petitioners were charged of violating Articles of It is clear that the basic position of petitioners impinges on
War 65 and 97. these fundamental principles we have discussed. They seek
to be exempted from military justice for having traveled to the
It was from these premises that the present petition for Senate to testify before the Senate Committee against the
certiorari and prohibition was filed. express orders of Gen. Senga, the AFP Chief of Staff. If
petitioners’ position is affirmed, a considerable exception
would be carved from the unimpeachable right of military
ISSUE: WON the petitioners' right to travel and movement can officers to restrict the speech and movement of their juniors.
be restricted by the President (Commander-in-Chief). YES The ruinous consequences to the chain of command and
military discipline simply cannot warrant the Court’s
imprimatur on petitioner’s position.
HELD: The vitality of the tenet that the President is the
commander-in-chief of the Armed Forces is most crucial to the
democratic way of life, to civilian supremacy over the military,
and to the general stability of our representative system of
government. The Constitution reposes final authority, control
and supervision of the AFP to the President, a civilian who is
not a member of the armed forces, and whose duties as
commander-in-chief represent only a part of the organic duties
imposed upon the office, the other functions being clearly civil
in nature. Civilian supremacy over the military also
countermands the notion that the military may bypass civilian
authorities, such as civil courts, on matters such as conducting
warrantless searches and seizures.

The commander-in-chief provision in the Constitution is


denominated as Section 18, Article VII, which begins with the
simple declaration that "[t]he President shall be the
Commander-in-Chief of all armed forces of the Philippines x x
x"37 Outside explicit constitutional limitations, such as those
found in Section 5, Article XVI, the commander-in-chief clause
vests on the President, as commander-in-chief, absolute
authority over the persons and actions of the members of the
armed forces. Such authority includes the ability of the
President to restrict the travel, movement and speech of
military officers, activities which may otherwise be sanctioned
under civilian law.

Of possibly less gravitas, but of equal importance, is the


principle that mobility of travel is another necessary restriction
on members of the military. A soldier cannot leave his/her post
without the consent of the commanding officer. The reasons
are self-evident. The commanding officer has to be aware at
all times of the location of the troops under command, so as
to be able to appropriately respond to any exigencies. For the
same reason, commanding officers have to be able to restrict
the movement or travel of their soldiers, if in their judgment,
their presence at place of call of duty is necessary. At times,
this may lead to unsentimental, painful consequences, such
as a soldier being denied permission to witness the birth of his
first-born, or to attend the funeral of a parent. Yet again,
military life calls for considerable personal sacrifices during
the period of conscription, wherein the higher duty is not to self
but to country.

Indeed, the military practice is to require a soldier to obtain


permission from the commanding officer before he/she may
leave his destination. A soldier who goes from the properly
appointed place of duty or absents from his/her command,
guard, quarters, station, or camp without proper leave is
subject to punishment by court-martial. It is even clear from
the record that petitioners had actually requested for travel
authority from the PMA in Baguio City to Manila, to attend the
Senate Hearing. Even petitioners are well aware that it was
necessary for them to obtain permission from their superiors

Page 116 of 186


2. LEAVE DIVISION, OFFICE OF ADMINISTRATIVE The exercise of one’s right to travel or the freedom to move
SERVICES-Office of the Court Administrator (OCA), from one place to another, as assured by the Constitution, is
vs. WILMA SALVACION P. HEUSDENS, Clerk IV not absolute. There are constitutional, statutory and inherent
Municipal Trial Court in Cities, Tagum City (A.M. No. limitations regulating the right to travel. Section 6 itself
P-11-2927 ; December 13, 2011 [Formerly A.M. OCA provides that "neither shall the right to travel be impaired
IPI No. 10-3532-P]) except in the interest of national security, public safety or
public health, as may be provided by law."

Inherent limitations on the right to travel are those that


FACTS: This case stemmed from the leave application for naturally emanate from the source. These are very basic and
foreign travel sent through mail by Wilma Salvacion P. are built-in with the power.
Heusdens (respondent), Staff Clerk IV of the Municipal Trial
Court in Cities, Tagum City, Davao del Norte. Supreme Court has administrative supervision over all courts
and the personnel thereof.
Records disclose that on July 10, 2009, the Employees Leave
Division, Office of Administrative Services, Office of the Court With respect to the power of the Court, Section 5 (6), Article
Administrator (OCA), received respondent’s leave application VIII of the 1987 Constitution provides that the "Supreme Court
for foreign travel from September 11, 2009 to October 11, shall have administrative supervision over all courts and the
2009. Respondent left for abroad without waiting for the result personnel thereof." This provision empowers the Court to
of her application. It turned out that no travel authority was oversee all matters relating to the effective supervision and
issued in her favor because she was not cleared of all her management of all courts and personnel under it. Recognizing
accountabilities as evidenced by the Supreme Court this mandate, Memorandum Circular No. 26 of the Office of
Certificate of Clearance. Respondent reported back to work the President, dated July 31, 1986, considers the Supreme
on October 19, 2009. Court exempt and with authority to promulgate its own rules
and regulations on foreign travels. Thus, the Court came out
The OCA, in its Memorandum dated November 26, 2009, with OCA Circular No. 49-2003 (B).
recommended the disapproval of respondent’s leave
application. It further advised that respondent be directed to Where a person joins the Judiciary or the government in
make a written explanation of her failure to secure authority to general, he or she swears to faithfully adhere to, and abide
travel abroad in violation of OCA Circular No. 49-2003. Chief with, the law and the corresponding office rules and
Justice Reynato S. Puno approved the OCA recommendation. regulations. These rules and regulations, to which one
submits himself or herself, have been issued to guide the
Respondent admitted having travelled overseas without the government officers and employees in the efficient
required travel authority. She explained that it was not her performance of their obligations. When one becomes a public
intention to violate the rules as she, in fact, mailed her leave servant, he or she assumes certain duties with their
application which was approved by her superior, Judge Arlene concomitant responsibilities and gives up some rights like the
Lirag-Palabrica. She honestly believed that her leave absolute right to travel so that public service would not be
application would be eventually approved by the Court. prejudiced.
The OCA found respondent to have violated OCA Circular No. As earlier stated, with respect to members and employees of
49-2003 for failing to secure the approval of her application for the Judiciary, the Court issued OCA Circular No. 49-2003 to
travel authority. regulate their foreign travel in an unofficial capacity. Such
regulation is necessary for the orderly administration of
Hence, the OCA recommended that the administrative
justice. If judges and court personnel can go on leave and
complaint be re-docketed as a regular administrative matter
travel abroad at will and without restrictions or regulations,
and that respondent be deemed guilty for violation of OCA
there could be a disruption in the administration of justice. A
Circular No. 49-2003 and be reprimanded with a warning that
situation where the employees go on mass leave and travel
a repetition of the same or similar offense in the future would
together, despite the fact that their invaluable services are
be dealt with more severely.
urgently needed, could possibly arise. For said reason,
members and employees of the Judiciary cannot just invoke
and demand their right to travel.
ISSUE: WON the requirements under OCA Circular No. 49-
2003 on vacation leave to be spent abroad unduly restricts a To permit such unrestricted freedom can result in disorder, if
citizen’s right to travel guaranteed by Section 6, Article III of not chaos, in the Judiciary and the society as well. In a
the 1987 Constitution. situation where there is a delay in the dispensation of justice,
litigants can get disappointed and disheartened. If their
expectations are frustrated, they may take the law into their
own hands which results in public disorder undermining public
HELD: NO.
safety. In this limited sense, it can even be considered that the
Section 6 of Article III of the 1987 Constitition reads: restriction or regulation of a court personnel’s right to travel is
a concern for public safety, one of the exceptions to the non-
Sec. 6. The liberty of abode and of changing the impairment of one’s constitutional right to travel.
same within the limits prescribed by law shall not be
impaired except upon lawful order of the court. Given the exacting standard expected from each individual
Neither shall the right to travel be impaired except in called upon to serve in the Judiciary, it is imperative that every
the interest of national security, public safety, or court employee comply with the travel notification and
public health, as may be provided by law. [Emphases authority requirements as mandated by OCA Circular No. 49-
supplied] 2003. A court employee who plans to travel abroad must file
his leave application prior to his intended date of travel with
sufficient time allotted for his application to be processed and

Page 117 of 186


approved first by the Court. He cannot leave the country SCSLA’s conformity, such requirement has no legal basis.
without his application being approved, much less assume The OCA does not have jurisdiction to require such clearance
that his leave application would be favorably acted upon. In because that would be tantamount to making the Court a
the case at bench, respondent should have exercised collecting agent of the SCSLA which is a private association.
prudence and asked for the status of her leave application
before leaving for abroad. Indeed, the OCA has no right to deny a court employee’s
constitutional right to travel just to enforce collection of the
WHEREFORE, respondent Wilma Salvacion P. Heusdens, SCSLA’s loans to its members. There is no law prohibiting a
Clerk IV Municipal Trial Court in Cities, Tagum City, is hereby person from traveling abroad just because he has an existing
ADMONISHED for traveling abroad without any travel debt or financial obligation. Requiring the court employee
authority in violation of OCA Circular No. 49-2003, with a clearance from the SCSLA is no different from requiring the
WARNING that a repetition of the same or similar offense court employee to secure a clearance from his or her creditor
would be dealt with more severely. banks before he or she can travel abroad. That would unduly
restrict a citizen’s right to travel which is guaranteed by
Section 6, Article III of the 1987 Constitution.
NOTE: DISSENTING OPINION CARPIO, J.: Although the constitutional right to travel is not absolute, it can
only be restricted in the interest of national security, public
(1) Respondent’s leave application for travel abroad was
safety, or public health, as may be provided by law.
received by the OCA on 10 July 2009, or two months before
her intended leave from 11 September 2009 to 11 October The SCSLA clearance is not required by any law before a
2009. However, it was only on 26 November 2009, or after court employee can travel abroad. The SCSLA clearance is
respondent’s intended leave, that the OCA issued a not even specifically required under OCA Circular No. 49-
memorandum recommending disapproval of her leave 2003. Clearly, respondent has submitted to the OCA all the
application. Furthermore, it was only in a letter dated 6 requirements for her leave application two months prior to her
January 2010 that the OCA informed respondent of the intended leave. Thus, respondent’s leave application was
disapproval of her leave application. Clearly, the OCA’s letter deemed approved as of 15 July 2009 pursuant to Section 49,
dated 6 January 2010 disapproving the leave application Rule XVI of the Omnibus Rules on Leave and the Omnibus
came too late. Although OCA Circular No. 49-2003 does not Rules Implementing Book V of EO 292.
provide for the time frame within which to act on the leave
application, it is understood that it should be prior to the
applicant’s intended leave. The requirement that the leave
application be submitted to the OCA at least two weeks before
the intended leave for travel is to give sufficient time for its
approval or disapproval before the intended leave.

Under the Omnibus Rules Implementing Book V of EO 292, a


leave application should be acted upon within five (5) working
days after its receipt, otherwise the leave application is
deemed approved.

In this case, where the leave application was received by the


OCA two months before the intended leave but was only acted
upon after the intended leave. Thus, respondent’s leave of
absence was deemed approved as of 15 July 2009 pursuant
to Section 49, Rule XVI of the Omnibus Rules on Leave.

(2) The majority states that although respondent submitted her


leave application for foreign travel, she failed to comply with
the clearance and accountability requirements because she
"failed to secure clearance from the Supreme Court Savings
and Loan Association (SCSLA) where she had an outstanding
loan." Thus, since OCA Circular No. 49-2003 specifically
provides that "no action shall be taken on requests for travel
authority with incomplete requirements," the majority
rationalizes that respondent should have expected that her
leave application would be disapproved.

I disagree with the majority’s view that clearance from the


SCSLA is required before a court employee can exercise his
or her constitutional right to travel abroad. The SCSLA is a
private association with private funds, even if some of its
investors are Supreme Court officials. The OCA has no power
to enforce the collection of loans extended by a private lender,
under pain of denying a constitutional right of a citizen if he
does not secure clearance from the private lender. Although
OCA Circular No. 49-2003 provides that "clearance as to
money and property accountability" is one of the requirements
to be submitted, this refers to accountability to the
government, not to a private company like the SCSLA. Even
if the OCA’s Certificate of Clearance Form requires the

Page 118 of 186


3. OFFICE OF ADMINISTRATIVE SERVICES-OFFICE with OCA Circular No. 49-2003. He promised not to commit
OF THE COURT ADMINISTRATOR vs. JUDGE the same infraction again. He further requested for
IGNACIO B. MACARINE, Municipal Circuit Trial reconsideration of the OCA’s intended action to deduct his
Court, Gen. Luna, Surigao del Norte (A.M. No. MTJ- salary corresponding to the seven (7) days that he was
10-1770; July 18, 2012 (Formerly A.M. OCA IPI No. absent, instead of charging his absences to his leave credits.
10-2255-MTJ)
OCA found the respondent guilty of violation of OCA Circular
No. 49-2003 for traveling out of the country without filing the
necessary application for leave and without first securing a
FACTS: The Office of the Court Administrator (OCA) filed the travel authority from the Court.
present administrative case against Judge Ignacio B.
Macarine (respondent) for violation of OCA Circular No. 49-
2003.
ISSUE: WON OCA Circular No. 49-2003 violates Section 6 of
OCA Circular No. 49-2003 requires that all foreign travels of Article III of the 1987 Constitution. NO
judges and court personnel, regardless of the number of days,
must be with prior permission from the Court. A travel authority
must be secured from the OCA Judges must submit the
HELD: True, the right to travel is guaranteed by the
following requirements:
Constitution.1âwphi1 However, the exercise of such right is
(1.) application or letter-request addressed to the Court not absolute. Section 6, Article III of the 1987 Constitution
Administrator stating the purpose of the travel abroad; allows restrictions on one’s right to travel provided that such
restriction is in the interest of national security, public safety or
(2.) application for leave covering the period of the travel public health as may be provided by law. This, however,
abroad, favorably recommended by the Executive Judge; and should by no means be construed as limiting the Court’s
inherent power of administrative supervision over lower
(3.) certification from the Statistics Division, Court courts. OCA Circular No. 49-2003 does not restrict but merely
Management Office, OCA as to the condition of the docket. regulates, by providing guidelines to be complied by judges
and court personnel, before they can go on leave to travel
abroad. To "restrict" is to restrain or prohibit a person from
The complete requirements should be submitted to and doing something; to "regulate" is to govern or direct according
received by the OCA at least two weeks before the intended to rule.
time of travel. No action shall be taken on requests for travel
To ensure management of court dockets and to avoid
authority with incomplete requirements.
disruption in the administration of justice, OCA Circular No.
Judges and personnel who shall leave the country without 49-2003 requires a judge who wishes to travel abroad to
travel authority issued by the OCA shall be subject to submit, together with his application for leave of absence duly
disciplinary action. recommended for approval by his Executive Judge, a
certification from the Statistics Division, Court Management
On August 13, 2009, the respondent wrote then Court Office of the OCA, as to the condition of his docket, based on
Administrator, now Associate Justice Jose Portugal Perez, his Certificate of Service for the month immediately preceding
requesting for authority to travel to Hongkong with his family the date of his intended travel, that he has decided and
for the period of September 10 - 14, 2009 where he would resolved all cases or incidents within three (3) months from
celebrate his 65th birthday. The respondent stated that his date of submission, pursuant to Section 15(1) and (2), Article
travel abroad shall be charged to his annual forced leave. VIII of the 1987 Constitution.
However, he did not submit the corresponding application for
leave. For his failure to submit the complete requirements, his For traveling abroad without having been officially allowed by
request for authority to travel remained unacted upon. The the Court, the respondent is guilty of violation of OCA Circular
respondent proceeded with his travel abroad without the No. 49-2003.
required travel authority from the OCA.
The Court had in several instances refrained from imposing
On January 28, 2010, the respondent was informed by the the actual penalties in the presence of mitigating facts, such
OCA that his leave of absence for the period of September 9- as the employee’s length of service, acknowledgement of his
15, 2009 had been disapproved and his travel considered or her infractions and feelings of remorse for the same,
unauthorized by the Court. advanced age, family circumstances, and other humanitarian
and equitable considerations.
In his letter-explanation, the respondent narrated that his
daughter, a nurse working in New Jersey, USA, gave him a In the present case, the respondent regretted his failure to
trip to Hongkong as a gift for his 65th birthday. In the first week comply with the requirements of OCA Circular No. 49-2003.
of September 2009, he received a call from his daughter that He acknowledged his mistake and promised not to commit the
she had already booked him, together with his wife and two same infraction in the future.
sons, in a hotel in Hongkong from September 13 to 15, 2009.
We consider the outlined circumstances as mitigating.
They flew in to Manila from Surigao City on September 9,
Following judicial precedents, the respondent deserves some
2009, intending to prepare the necessary papers for his
degree of leniency in imposing upon him the appropriate
authority to travel at the Supreme Court the following day.
penalty.
However, sensing time constraint and thinking of the futility of
completing the requirements before their scheduled flight, he WHEREFORE, respondent Judge Ignacio B. Macarine,
opted not to immediately complete the requirements and Municipal Circuit Trial Court, Gen. Luna, Surigao del Norte, is
simply went ahead with their travel abroad. He thought of hereby given the ADMONITION that he acted irresponsibly
submitting his compliance upon his return to Manila. He when he opted not to immediately secure a travel authority
acknowledged his mistake and regretted his failure to comply and is saved only from the full force that his violation carries

Page 119 of 186


by the attendant mitigating circumstances. He is also 4. Marcos v. Manglapus 177 SCRA 669, G.R. No. 88211
WARNED that the commission of a similar violation in the (1989)
future will merit a more severe penalty.

FACTS: Former President Ferdinand E. Marcos was deposed


from the presidency via the non-violent “people power”
revolution and was forced into exile. In his stead, Corazon C.
NOTE: DISSENTING AND CONCURRING OPINION Aquino was declared President of the Republic under a
SERENO, J.: revolutionary government.
I agree with the imposition of a penalty on Judge Macarine for Marcos, in his deathbed, has signified his wish to return to the
his failure to (a) file an application for leave and (b) submit a Philippines to die. But President Corazon Aquino, considering
report on the conditions of the docket pending in his sala prior the dire consequences to the nation of his return at a time
to his travel abroad. However, I do not agree that he should when the stability of government is threatened from various
be penalized for his failure to request a travel authority from directions and the economy is just beginning to rise and move
the OCA. forward, has stood firmly on the decision to bar the return of
Marcos and his family.
The policy of the Court requiring judges and court personnel
to secure a travel authority must be re-examined. Aquino barred Marcos from returning due to possible threats
& following supervening events listed as follows:
Requiring judges and court personnel prior submission of a
request for travel authority impairs their right to travel, a • The failed Manila Hotel coup in 1986 led by political
constitutional right that cannot be unduly curtailed. During the leaders of Mr. Marcos
approved leave of absence of a judge or court personnel, he
or she should be accorded the liberty to travel within the • the takeover of television station Channel 7 by rebel
country or abroad, as any other citizen, without this Court troops led by Col. Canlas with the support of "Marcos
imposing a requirement to secure prior permission therefor. loyalists"

Thus, Judge Macarine should not be held administratively • the unsuccessful plot of the Marcos spouses to
liable for his failure to secure a permit to travel prior to his surreptitiously return from Hawii with mercenaries
intended departure, as such action would amount to an aboard an aircraft chartered by a Lebanese arms
unjustified restriction to his constitutional right to travel. dealer Manila Bulletin, January 30, 1987
However, on account of his failure to file (a) an application for
leave and (b) a report on his caseload prior to his travel • On August 28, 1987, Col. Gregorio Honasan, one
abroad, I agree that he should be admonished. of the major players in the February Revolution, led
a failed coup that left scores of people, both
combatants and civilians, dead signifying a split in
the ranks of the military establishment that
threatened civilian supremacy over military and
brought to the fore the realization that civilian
government could be at the mercy of a fractious
military

• communist insurgency and the seccessionist


movement in Mindanao to the extent that the
communists have set up a parallel government of
their own on the areas they effectively control while
the separatist are virtually free to move about in
armed bands

• The accumulated foreign debt, and

• the plunder of the nation attributed to Mr. Marcos


and his cronies left the economy devastated

Marcos filed for a petition of mandamus and prohibition to


order the respondents to issue them their travel documents
and prevent the implementation of President Aquino’s
decision to bar Marcos from returning in the Philippines.
Petitioner questions Aquino’s power to bar his return in the
country. He also questioned the claim of the President that the
decision was made in the interest of national security, public
safety and health. Petitioner also claimed that the President
acted outside her jurisdiction.

According to the Marcoses, such act deprives them of their


right to life, liberty, property without due process and equal
protection of the laws. They also said that it deprives them of
their right to travel which according to Section 6, Article 3 of
the constitution, may only be impaired by a court order.

Page 120 of 186


Petitioners assert that the right of the Marcoses to return to national interest and welfare and decided to bar their
the Philippines is guaranteed under the following provisions of return.
the Bill of Rights, to wit:

Section 1. No person shall be deprived of life, liberty,


or property without due process of law, nor shall any HELD: No to both issues. Petition dismissed.
person be denied the equal protection of the laws.

Section 6. The liberty of abode and of changing the


1. The rights Marcoses are invoking are not absolute. They’re
same within the limits prescribed by law shall not be
flexible depending on the circumstances. The request of the
impaired except upon lawful order of the court.
Marcoses to be allowed to return to the Philippines cannot be
Neither shall the right to travel be impaired except in
considered in the light solely of the constitutional provisions
the interest of national security, public safety, or
guaranteeing liberty of abode and the right to travel, subject to
public health, as may be provided by law.
certain exceptions, or of case law which clearly never
Furthermore, they contend that the President is without power contemplated situations even remotely similar to the present
to impair the liberty of abode of the Marcoses because only a one. It must be treated as a matter that is appropriately
court may do so “within the limits prescribed by law.” Nor may addressed to those residual unstated powers of the President
the President impair their right to travel because no law has which are implicit in and correlative to the paramount duty
authorized her to do so. They advance the view that before residing in that office to safeguard and protect general welfare.
the right to travel may be impaired by any authority or agency In that context, such request or demand should submit to the
of the government, there must be legislation to that effect. exercise of a broader discretion on the part of the President to
determine whether it must be granted or denied.
The petitioners further assert that under international law, the
right of Mr. Marcos and his family to return to the Philippines It must be emphasized that the individual right involved is not
is guaranteed. the right to travel from the Philippines to other countries or
within the Philippines. These are what the right to travel would
normally connote. Essentially, the right involved is the right to
return to one’s country, a totally distinct right under
The Universal Declaration of Human Rights provides: international law, independent from although related to the
right to travel. Thus, the Universal Declaration of Humans
Article 13. (1) Everyone has the right to freedom of
Rights and the International Covenant on Civil and Political
movement and residence within the borders of each
Rights treat the right to freedom of movement and abode
state.
within the territory of a state, the right to leave a country, and
(2) Everyone has the right to leave any country, the right to enter one’s country as separate and distinct rights.
including his own, and to return to his country. The Declaration speaks of the “right to freedom of movement
and residence within the borders of each state” Art. 13(l)
Likewise, the International Covenant on Civil and Political separately from the “right to leave any country, including his
Rights, which had been ratified by the Philippines, provides: own, and to return to his country.” Art. 13(2). On the other
hand, the Covenant guarantees the “right to liberty of
Article 12
movement and freedom to choose his residence” Art. 12(l)
1) Everyone lawfully within the territory of a State and the right to “be free to leave any country, including his
shall, within that territory, have the right to liberty of own.” Art. 12(2) which rights may be restricted by such laws
movement and freedom to choose his residence. as “are necessary to protect national security, public order,
public health or morals or enter qqqs own country” of which
2) Everyone shall be free to leave any country, one cannot be “arbitrarily deprived.” Art. 12(4). It would
including his own. therefore be inappropriate to construe the limitations to the
right to return to one’s country in the same context as those
3) The above-mentioned rights shall not be subject pertaining to the liberty of abode and the right to travel.
to any restrictions except those which are provided
by law, are necessary to protect national security, The right to return to one’s country is not among the rights
public order (order public), public health or morals or specifically guaranteed in the Bill of Rights, which treats only
the rights and freedoms of others, and are consistent of the liberty of abode and the right to travel, but it is our well-
with the other rights recognized in the present considered view that the right to return may be considered, as
Covenant. a generally accepted principle of international law and, under
our Constitution, is part of the law of the land Art. II, Sec. 2 of
4) No one shall be arbitrarily deprived of the right to the Constitution. However, it is distinct and separate from the
enter his own country. right to travel and enjoys a different protection under the
International Covenant of Civil and Political Rights, i.e.,
against being “arbitrarily deprived” thereof Art. 12 (4).
ISSUES:
Although the 1987 Constitution imposes limitations on the
1. Whether or not the President has the power under the exercise of specific powers of the President, it maintains intact
Constitution, to bar the Marcoses from returning to the what is traditionally considered as within the scope of
Philippines. “executive power.” Corollarily, the powers of the President
2. Whether or not the President acted arbitrarily or with cannot be said to be limited only to the specific powers
grave abuse of discretion amounting to lack or excess of enumerated in the Constitution. In other words, executive
jurisdiction when she determined that the return of the power is more than the sum of specific powers so enumerated.
Marcose’s to the Philippines poses a serious threat to

Page 121 of 186


To the President, the problem is one of balancing the general is of common knowledge and is easily within the ambit of
welfare and the common good against the exercise of rights judicial notice.
of certain individuals. The power involved is the President’s
residual power to protect the general welfare of the people. It
is founded on the duty of the President, as steward of the
2. The question for the court to determine is whether or not
people.
there exist factual basis for the President to conclude that it
The Constitution declares among the guiding principles that was in the national interest to bar the return of the Marcoses
“the prime duty of the Government is to serve and protect the in the Philippines. It is proven that there are factual bases in
people” and that “the maintenance of peace and order, the her decision. The supervening events that happened before
protection of life, liberty, and property, and the promotion of her decision are factual. The President must take preemptive
the general welfare are essential for the enjoyment by all the measures for the self-preservation of the country & protection
people of the blessings of democracy.” Art. II, Secs. 4 and 5. of the people. She has to uphold the Constitution.

More particularly, this case calls for the exercise of the


President’s powers as protector of the peace. The power of
Fernan, Concurring
the President to keep the peace is not limited merely to
exercising the commander-in-chief powers in times of • The president’s power is not fixed. Limits would depend on
emergency or to leading the State against external and the imperatives of events and not on abstract theories of law.
internal threats to its existence. The President is not only We are undergoing a critical time and the current problem can
clothed with extraordinary powers in times of emergency, but only be answerable by the President.
is also tasked with attending to the day-to-day problems of
maintaining peace and order and ensuring domestic tranquility • Threat is real. Return of the Marcoses would pose a clear &
in times when no foreign foe appears on the horizon. Wide present danger. Thus, it’s the executive’s responsibility &
discretion, within the bounds of law, in fulfilling presidential obligation to prevent a grave & serious threat to its safety from
duties in times of peace is not in any way diminished by the arising.
relative want of an emergency specified in the commander-in-
• We can’t sacrifice public peace, order, safety & our political
chief provision. For in making the President commander-in-
& economic gains to give in to Marcos’ wish to die in the
chief the enumeration of powers that follow cannot be said to
country. Compassion must give way to the other state
exclude the President’s exercising as Commander-in- Chief
interests.
powers short of the calling of the armed forces, or suspending
the privilege of the writ of habeas corpus or declaring martial
law, in order to keep the peace, and maintain public order and
security. Cruz, Dissenting

The Court cannot close its eyes to present realities and • As a citizen of this country, it is Marcos’ right to return, live &
pretend that the country is not besieged from within by a well- die in his own country. It is a right guaranteed by the Consti to
organized communist insurgency, a separatist movement in all individuals, whether patriot, homesick, prodigal, tyrant, etc.
Mindanao, rightist conspiracies to grab power, urban
terrorism, the murder with impunity of military men, police • Military representatives failed to show that Marcos’ return
officers and civilian officials, to mention only a few. The would pose a threat to national security. Fears were mere
documented history of the efforts of the Marcose’s and their conjectures.
followers to destabilize the country, as earlier narrated in this
• Residual powers – but the executive’s powers were outlined
ponencia bolsters the conclusion that the return of the
to limit her powers & not expand.
Marcoses at this time would only exacerbate and intensify the
violence directed against the State and instigate more chaos.

The State, acting through the Government, is not precluded Paras, Dissenting
from taking pre- emptive action against threats to its existence
if, though still nascent they are perceived as apt to become • AFP has failed to prove danger which would allow State to
serious and direct. Protection of the people is the essence of impair Marcos’ right to return to the Philippines. .
the duty of government. The preservation of the State the
• Family can be put under house arrest & in the event that one
fruition of the people’s sovereignty is an obligation in the
dies, he/she should be buried w/in 10 days.
highest order. The President, sworn to preserve and defend
the Constitution and to see the faithful execution the laws, • Untenable that without a legislation, right to travel is absolute
cannot shirk from that responsibility. & state is powerless to restrict it. It’s w/in police power of the
state to restrict this right if national security, public
We cannot also lose sight of the fact that the country is only
safety/health demands that such be restricted. It can’t be
now beginning to recover from the hardships brought about by
the plunder of the economy attributed to the Marcoses and absolute & unlimited all the time. It can’t be arbitrary &
irrational.
their close associates and relatives, many of whom are still
here in the Philippines in a position to destabilize the country, • No proof that Marcos’ return would endanger national
while the Government has barely scratched the surface, so to security or public safety. Fears are speculative & military
speak, in its efforts to recover the enormous wealth stashed admits that it’s under control. Filipinos would know how to
away by the Marcoses in foreign jurisdictions. Then, We handle Marcos’ return.
cannot ignore the continually increasing burden imposed on
the economy by the excessive foreign borrowing during the
Marcos regime, which stifles and stagnates development and
is one of the root causes of widespread poverty and all its
attendant ills. The resulting precarious state of our economy

Page 122 of 186


Padilla, Dissenting 4.1 Marcos vs Manglapus (G.R. No. 88211, October 27,
1989)
Sarmiento, Dissenting

President’s determination that Marcos’ return would threaten


national security should be agreed upon by the court. Such FACTS: In its decision dated September 15, 1989, the Court
threat must be clear & present. by a vote of eight to seven, dismissed the petition, after finding
that the President did not act arbitrarily or with grave abuse of
discretion in determining that the return of former President
Marcos and his family pose a threat to national interest and
welfare and in prohibiting their return to the Philippines. On
September 28, 1989, Marcos died in Honolulu, Hawaii.

President Corazon Aquino issued a statement saying that in


the interest of the safety of those who will take the death of
Marcos in widely and passionately conflicting ways, and for
the tranquility and order of the state and society, she did not
allow the remains of Marcos to be brought back in the
Philippines.

A motion for Reconsideration was filed by the petitioners


raising the following arguments:

Barring their return would deny them their inherent right as


citizens to return to their country of birth and all other rights
guaranteed by the Constitution to all Filipinos. The President
has no power to bar a Filipino from his own country; if she has,
she had exercised it arbitrarily. There is no basis for barring
the return of the family of former President Marcos.

ISSUE: Whether or not the motion for reconsideration that the


Marcoses be allowed to return in the Philippines be granted.

HELD: No. The Marcoses were not allowed to return. Motion


for Reconsideration denied because of lack of merit.

Petitioners failed to show any compelling reason to warrant


reconsideration. Factual scenario during the time Court
rendered its decision has not changed. The threats to the
government, to which the return of the Marcoses has been
viewed to provide a catalytic effect, have not been shown to
have ceased. Imelda Marcos also called President Aquino
“illegal” claiming that it is Ferdinand Marcos who is the legal
president.

President has unstated residual powers implied from grant of


executive power. Enumerations are merely for specifying
principal articles implied in the definition; leaving the rest to
flow from general grant that power, interpreted in conformity
with other parts of the Constitution (Hamilton). Executive
unlike Congress can exercise power from sources not
enumerates so long as not forbidden by constitutional text
(Myers vs. US). This does not amount to dictatorship.
Amendment No. 6 expressly granted Marcos power of
legislation whereas 1987 Constitution granted Aquino with
implied powers. It is within Aquino’s power to protect &
promote interest & welfare of the people. She bound to comply
w/ that duty and there is no proof that she acted arbitrarily

Page 123 of 186


5. SAMAHAN NG MGA PROGRESIBONG KABATAAN (b) fail to pass the strict scrutiny test, for not being
(SPARK) vs QUEZON CITY, AS REPRESENTED BY narrowly tailored and for employing means that bear
MAYOR HERBERT BAUTISTA, CITY OF MANILA, AS no reasonable relation to their purpose.
REPRESENTED BY MAYOR JOSEPH ESTRADA, AND
NAVOTAS CITY, AS REPRESENTED BY MAYOR JOHN Furthermore, petitioners claim that Section 4 of Manila
REY TIANGCO (G.R. No. 225442, August 08, 2017) Ordinance contravenes Section RA 9344 given that the cited
curfew provision imposes on minors the penalties of
imprisonment, reprimand, and admonition.

FACTS: Following the campaign of President Rodrigo Roa Lastly, there is no compelling State interest to impose curfews
Duterte to implement a nationwide curfew for minors, several contrary to the parents' prerogative to impose them in the
local governments in Metro Manila started to strictly exercise of their natural and primary right in the rearing of the
implement their curfew ordinances on minors through police youth. In this regard, they suggest massive street lighting
operations which were publicly known as part of "Oplan Rody." programs, installation of CCTVs (closed-circuit televisions) in
public streets, and regular visible patrols by law enforcers as
Among those local governments that implemented curfew other viable means of protecting children and preventing
ordinances were respondents: crimes at night. They further opine that the government can
impose more reasonable sanctions, i.e., mandatory parental
(a) Navotas City - "Nagtatakda ng 'Curfew' ng mga
counseling and education seminars informing the parents of
Kabataan na Wala Pang Labing Walong (18) Taong
the reasons behind the curfew, and that imprisonment is too
Gulang sa Bayan ng Navotas, Kalakhang Maynila,";
harsh a penalty for parents who allowed their children to be
(b) City of Manila - "An Ordinance Declaring the out during curfew hours.
Hours from 10:00 P.M. to 4:00A.M. of the Following
Day as 'Barangay Curfew Hours' for Children and
Youths Below Eighteen (18) Years of Age; ISSUE: whether or not the Curfew Ordinances are
Prescribing Penalties Therefor; and for Other unconstitutional.
Purposes"; and

(c) Quezon City - "An Ordinance Setting for a


Disciplinary Hours in Quezon City for Minors from HELD The petition is partly granted.
10:00 P.M. to 5:00A.M., Providing Penalties for
Parent/Guardian, for Violation Thereof and for Other
Purposes".
A. Propriety of the Petition for Certiorari and Prohibition
Petitioners, spearheaded by SPARK - an association of young
A direct invocation of this Court's jurisdiction is allowed when
adults and minors for the protection of the rights and welfare
there are special and important reasons therefor, clearly and
of the youth and minors - filed this present petition, arguing
especially set out in the petition." This Court is tasked to
that the Curfew Ordinances are unconstitutional because
resolve "the issue of constitutionality of a law or regulation at
they:
the first instance if it is of paramount importance and
(a) result in arbitrary and discriminatory enforcement, immediately affects the social, economic, and moral well-
and thus, fall under the void for vagueness doctrine; being of the people," as in this case. Hence, petitioners' direct
resort to the Court is justified.
(b) suffer from overbreadth by proscribing or
impairing legitimate activities of minors during curfew Requisites of Judicial Review. - (a) there must be an actual
hours; case or controversy calling for the exercise of judicial power;
(b) the person challenging the act must have the standing to
(c) deprive minors of the right to liberty and the right question the validity of the subject act or issuance; (c) the
to travel without substantive due process; and question of constitutionality must be raised at the earliest
opportunity; and (d) the issue of constitutionality must be the
(d) deprive parents of their natural and primary right very lis mota of the case." In this case, respondents assail the
in rearing the youth without substantive due process. existence of the first two (2) requisites.
Petitioners posit that the Curfew Ordinances encourage
arbitrary and discriminatory enforcement as there are no
detailed standards on how law enforcers should apprehend Actual Case or Controversy.
and properly determine the age of the alleged curfew violators
since law enforcer's apprehension depends only on his This Court finds that there exists an actual justiciable
physical assessment, and, thus, subjective. controversy in this case given the evident clash of the parties'
legal claims, particularly on whether the Curfew Ordinances
Likewise, although the curfew ordinances exempt working impair the minors' and parents' constitutional rights, and
students or students with evening class, they contend that the whether the Manila Ordinance goes against the provisions of
lists of exemptions do not cover the range and breadth of RA 9344.
legitimate activities or reasons as to why minors would be out
at night, and, hence, proscribe or impair the legitimate
activities of minors during curfew hours.
Legal Standing.
Petitioners likewise proffer that the Curfew Ordinances:
Among the five (5) individual petitioners, only Clarissa Joyce
(a) are unconstitutional as they deprive minors of the Villegas has legal standing to raise the issue affecting the
right to liberty and the right to travel without minor's right to travel, because: (a) she was still a minor at the
substantive due process; and time the petition was filed before this Court, and, hence, a

Page 124 of 186


proper subject of the Curfew Ordinances; and (b) as alleged, Note that RA 10630 (the law that amended RA 9344) repeals
she travels from Manila to Quezon City at night after school all ordinances inconsistent with statutory law. Pursuant to
and is, thus, in imminent danger of apprehension by virtue of Section 57-A of RA 9344, as amended by RA 10630, minors
the Curfew Ordinances. On the other hand, petitioners Joanne caught in violation of curfew ordinances are children at risk
Rose Sace Lim, John Arvin Navarro Buenaagua, Ronel and, therefore, covered by its provisions.
Baccutan, and Mark Leo Delos Reyes admitted in the petition
that they are all of legal age, and therefore, beyond the
ordinances' coverage. Thus, they are not proper subjects of
B. Petitioners submit that the Curfew Ordinances are
the Curfew Ordinances, hence has no legal standing to raise
unconstitutional because they deprive parents of their natural
the issue, for which they could base any direct injury as a
and primary right in the rearing of the youth without
consequence thereof.
substantive due process.
As for SPARK, it is an unincorporated association and,
Petitioners' stance cannot be sustained.
consequently, has no legal personality to bring an action in
court. Even assuming that it has the capacity to sue, SPARK Section 12, Article II of the 1987 Constitution articulates the
still has no standing as it failed to allege that it was authorized State's policy relative to the rights of parents in the rearing of
by its members who were affected by the Curfew Ordinances, their children. Parents are not only given the privilege of
i.e., the minors, to file this case on their behalf. exercising their authority over their children; they are equally
obliged to exercise this authority conscientiously.
Hence, except Clarissa, petitioners do not have the required
personal interest in the controversy. However, Clarissa has While parents have the primary role in child-rearing, the State
standing only on the issue of the alleged violation of the may act to promote legitimate interests." Thus, "in cases in
minors' right to travel, but not on the alleged violation of the which harm to the physical or mental health of the child or to
parents' right. public safety, peace, order, or welfare is demonstrated, these
legitimate state interests may override the parents' qualified
Accordingly, this case is of overarching significance to the
right to control the upbringing of their children."
public, which, therefore, impels a relaxation of procedural
rules, including, among others, the standing requirement. As parens patriae, the State has the inherent right and duty to
aid parents in the moral development of their children, and,
In particular, petitioners submit that the Curfew Ordinances
thus, assumes a supporting role for parents to fulfill their
are void for vagueness. They claim that the lack of procedural
parental obligations.
guidelines in these issuances led to the questioning of
petitioners Ronel and Mark Leo, even though they were At this juncture, it should be emphasized that the Curfew
already of legal age. They maintain that the enforcing Ordinances apply only when the minors are not - whether
authorities apprehended the suspected curfew offenders actually or constructively - accompanied by their parents.
based only on their physical appearances and, thus, acted Parents are effectively given unfettered authority over their
arbitrarily. Meanwhile, although they conceded that the children's conduct during curfew hours when they are able to
Quezon City Ordinance requires enforcers to determine the supervise them. The Curfew Ordinances only amount to a
age of the child, they submit that nowhere does the said minimal - albeit reasonable - infringement upon a parent's right
ordinance require the law enforcers to ask for proof or to bring up his or her child.
identification of the child to show his age.

A statute or act suffers from the defect of vagueness when it


lacks comprehensible standards that men of common C. Petitioners further assail the constitutionality of the Curfew
intelligence must necessarily guess at its meaning and differ Ordinances based on the minors' right to travel. They claim
as to its application. that the liberty to travel is a fundamental right, which,
therefore, necessitates the application of the strict scrutiny.
It is repugnant to the Constitution in two (2) respects: (1) it
violates due process for failure to accord persons, especially At the outset, the Court rejects petitioners' invocation of the
the parties targeted by it, fair notice of the conduct to avoid; overbreadth doctrine, considering that petitioners have not
and (2) it leaves law enforcers unbridled discretion in carrying claimed any transgression of their rights to free speech or any
out its provisions and becomes an arbitrary flexing of the inhibition of speech-related conduct. This Court explained that
Government muscle." "the application of the overbreadth doctrine is limited to a facial
kind of challenge and, owing to the given rationale of a facial
They do not assert any confusion as to what conduct the challenge, applicable only to free speech cases.
subject ordinances prohibit or not prohibit but only point to the
ordinances' lack of enforcement guidelines This Court finds it improper to undertake an overbreadth
analysis in this case, there being no claimed curtailment of
As above-mentioned, petitioners fail to point out any free speech. On the contrary, however, this Court finds proper
ambiguous standard in any of the provisions of the Curfew to examine the assailed regulations under the strict scrutiny
Ordinances, but rather, lament the lack of detail on how the test.
age of a suspected minor would be determined. Thus, without
any correlation to any vague legal provision, the Curfew The right to travel is recognized and guaranteed as a
Ordinances cannot be stricken down under the void for fundamental right under Section 6, Article III of the 1987
vagueness doctrine. Constitution, to wit:

While it is true that the Curfew Ordinances do not explicitly Section 6. The liberty of abode and of changing the
state these parameters, law enforcement agents are still same within the limits prescribed by law shall not be
bound to follow the prescribed measures found in statutory impaired except upon lawful order of the court.
law when implementing ordinances. Neither shall the right to travel be impaired except in

Page 125 of 186


the interest of national security, public safety, or potential harm, and to insulate them against criminal pressure
public health, as may be provided by law. and influences which may even include themselves.

Jurisprudence provides that this right refers to the right to The local governments of Quezon City and Manila presented
move freely from the Philippines to other countries or within statistical data in their respective pleadings showing the
the Philippines. It is a right embraced within the general alarming prevalence of crimes involving juveniles, either as
concept of liberty (such as to exercise the right to education, victims or perpetrators, in their respective localities. Based on
free expression, assembly, association, and religion.) these findings, their city councils found it necessary to enact
curfew ordinances pursuant to their police power under the
As the 1987 Constitution itself reads, the State may impose general welfare clause.
limitations on the exercise of this right, provided that they: (1)
serve the interest of national security, public safety, or public
health; and (2) are provided by law.
b. Least Restrictive Means/ Narrowly Drawn.
The stated purposes of the Curfew Ordinances, specifically
the promotion of juvenile safety and prevention of juvenile After a thorough evaluation of the ordinances' respective
crime, inarguably serve the interest of public safety. The provisions, this Court finds that only the Quezon City
restriction on the minor's movement and activities within the Ordinance meets the above-discussed requirement, while the
confines of their residences and their immediate vicinity during Manila and Navotas Ordinances do not.
the curfew period is perceived to reduce the probability of the
The Manila Ordinance cites only four (4) exemptions, namely:
minor becoming victims of or getting involved in crimes and
criminal activities. (a) Minors accompanied by their parents, family
members of legal age, or guardian;
As to the second requirement, i.e., that the limitation "be
provided by law," our legal system is replete with laws (b) Those running lawful errands such as buying of
emphasizing the State's duty to afford special protection to medicines, using of telecommunication facilities for
children, particularly relevant to this case is Article 139 of PD emergency purposes and the like;
603, which explicitly authorizes local government units,
through their city or municipal councils, to set curfew hours for (c) Night school students and those who, by virtue of
children. their employment, are required in the streets or
outside their residence after 10:00 p.m.; and
The restrictions set by the Curfew Ordinances that apply
solely to minors are likewise constitutionally permissible. In (d) Those working at night.
this relation, this Court recognizes that minors do possess and
The Navotas Ordinance, to wit:
enjoy constitutional rights, but the exercise of these rights is
not co-extensive as those of adults. (a) Minors with night classes;
The US Supreme Court identified three (3) justifications for the (b) Those working at night;
differential treatment of the minors' constitutional rights. These
are: first, the peculiar vulnerability of children; second, their (c) Those who attended a school or church activity,
inability to make critical decisions in an informed and mature in coordination with a specific barangay office;
manner; and third, the importance of the parental role in child
rearing: (d) Those traveling towards home during the curfew
hours;
Three (3) tests of judicial scrutiny to determine the
reasonableness of classifications (e) Those running errands under the supervision of
their parents, guardians, or persons of legal age
1. Strict scrutiny test applies when a classification either (i) having authority over them
interferes with the exercise of fundamental rights, including
the basic liberties guaranteed under the Constitution, or (ii) (f) Those involved in accidents, calamities, and the
burdens suspect classes. like, and

2. Intermediate scrutiny test applies when a classification does (g) during these specific occasions: Christmas eve,
not involve suspect classes or fundamental rights, but requires Christmas day, New Year's eve, New Year's day, the
heightened scrutiny, such as in classifications based on night before the barangay fiesta, the day of the fiesta,
gender and legitimacy. All Saints' and All Souls' Day, Holy Thursday, Good
Friday, Black Saturday, and Easter Sunday.
3. Rational basis test applies to all other subjects not covered
by the first two tests. This Court observes that these two ordinances are not
narrowly drawn in that their exceptions are inadequate and
Considering that the right to travel is a fundamental right under therefore, run the risk of overly restricting the minors'
the Constitution, the strict scrutiny test is the applicable test. fundamental freedoms.

First, although it allows minors to engage in school or church


activities, it hinders them from engaging in legitimate non-
a. Compelling State Interest. school or non-church activities in the streets or going to and
from such activities; thus, their freedom of association is
Respondents have sufficiently established that the ultimate
effectively curtailed.
objective of the Curfew Ordinances is to keep unsupervised
minors during the late hours of night time off of public areas, Second, although the Navotas Ordinance does not impose the
so as to reduce - if not totally eliminate - their exposure to curfew during Christmas Eve and Christmas day, it effectively
prohibits minors from attending traditional religious activities

Page 126 of 186


(such as simbang gabi) at night without accompanying adults, The sanction of admonition imposed by the City of Manila is
Third, the Navotas Ordinance does not accommodate likewise consistent with Sections 57 and 57-A of RA 9344 as
avenues for minors to engage in political rallies or attend city it is merely a formal way of giving warnings and expressing
council meetings to voice out their concerns in line with their disapproval to the minor's misdemeanor.
right to peaceably assemble and to free expression.
In other words, the disciplinary measures of community-based
In sum, the Manila and Navotas Ordinances should be programs and admonition are clearly not penalties - as they
completely stricken down since their exceptions, which are are not punitive in nature - and are generally less intrusive on
essentially determinative of the scope and breadth of the the rights and conduct of the minor. To be clear, their
curfew regulations, are inadequate to ensure protection of the objectives are to formally inform and educate the minor, and
above-mentioned fundamental rights. for the latter to understand, what actions must be avoided so
as to aid him in his future conduct.
On the other hand, the list of exceptions under the Quezon
City Ordinance is more narrowly drawn to sufficiently protect Fines and/or imprisonment, on the other hand, undeniably
the minors' rights of association, free exercise of religion, constitute penalties - as provided in our various criminal and
travel, to peaceably assemble, and of free expression. administrative laws and jurisprudence - that Section 57-A of
RA 9344, as amended, evidently prohibits.
Specifically, the inclusion of items (b) and (g) in the list of
exceptions guarantees the protection of these aforementioned In sum, while the Court finds that all three Curfew Ordinances
rights. These items uphold the right of association by enabling have passed the first prong of the strict scrutiny test - that is,
minors to attend both official and extra-curricular activities not that the State has sufficiently shown a compelling interest to
only of their school or church but also of other legitimate promote juvenile safety and prevent juvenile crime in the
organizations. The rights to peaceably assemble and of free concerned localities, only the Quezon City Ordinance has
expression are also covered by these items given that the passed the second prong of the strict scrutiny test, as it is the
minors' attendance in the official activities of civic or religious only issuance out of the three which provides for the least
organizations are allowed during the curfew hours. Unlike in restrictive means to achieve this interest.
the Navotas Ordinance, the right to the free exercise of religion
is sufficiently safeguarded in the Quezon City Ordinance In particular, the Quezon City Ordinance provides for
adequate exceptions that enable minors to freely exercise
Meanwhile, the Manila Ordinance imposed various sanctions their fundamental rights during the prescribed curfew hours,
to the minor based on the age and frequency of violations, to and therefore, narrowly drawn to achieve the State's purpose.
wit: Section 4 (a) of the said ordinance, i.e., "those accompanied
by their parents or guardian", has also been construed to
SEC. 4. Sanctions and Penalties for Violation. Any include parental permission as a constructive form of
child or youth violating this ordinance shall be accompaniment and hence, an allowable exception to the
sanctioned/punished as follows: curfew measure; the manner of enforcement, however, is left
to the discretion of the local government unit. In fine, the
(a) If the offender is Fifteen (15) years of age and
Manila and Navotas Ordinances are declared unconstitutional
below, the sanction shall consist of a REPRIMAND
and thus, null and void, while the Quezon City Ordinance is
for the youth offender and ADMONITION to the
declared as constitutional and thus, valid in accordance with
offender's parent, guardian or person exercising
this Decision.
parental authority.

(b) If the offender is Fifteen (15) years of age and


under Eighteen (18) years of age, the
sanction/penalty shall be: For the FIRST OFFENSE,
Reprimand and Admonition; For the SECOND
OFFENSE, Reprimand and Admonition, and a
warning about the legal impostitions in case of a third
and subsequent violation; and For the THIRD AND
SUBSEQUENT OFFENSES, Imprisonment of one
(1) day to ten (10) days, or a Fine of TWO
THOUSAND PESOS (Php2,000.00), or both at the
discretion of the Court, PROVIDED, That the
complaint shall be filed by the Punong Barangay with
the office of the City Prosecutor.

Thus springs the question of whether local governments could


validly impose on minors these sanctions - i.e., (a) community
service; (b) reprimand and admonition; (c) fine; and (d)
imprisonment. Pertinently, Sections 57 and 57-A of RA 9344,
as amended, prohibit the imposition of penalties on minors for
status offenses such as curfew violations, viz.: what they
prohibit is the imposition of penalties on minors for violations
of these regulations.

In this regard, requiring the minor to perform community


service is a valid form of intervention program that a local
government (such as Navotas City in this case) could
appropriately adopt in an ordinance to promote the welfare of
minors.

Page 127 of 186


6. EFRAIM C. GENUINO, ERWIN F. GENUINO and Also, on November 8, 2011, De Lima issued an Order,
SHERYL G. SEE, et.al, vs. HON. LEILA M. DE LIMA, in denying GMA's application for an ADO because based on the
her capacity as Secretary of Justice, and RI CARDO V. medical condition of Secretary Ona, (1) there appears to be
PARAS III, in his capacity as Chief State Counsel, no urgent and immediate medical emergency situation for
CRISTINO L. NAGUIAT, JR. and the BUREAU OF Applicant to seek medical treatment abroad; (2) Applicant lists
IMMIGRATION several countries as her destination, some of which were not
for purposes of medical consultation, but for attending
conferences; (3) while the Applicant's undertaking is to return
to the Philippines upon the completion of her medical
FACTS: On March 19, 1998, then DOJ Secretary Silvestre H.
treatment, this means that her return will always depend on
Bello III issued DOJ Circular No. 17, prescribing rules and
said treatment, which, based on her presentation of her
regulations governing the issuance of Hold Departure Orders
condition, could last indefinitely; (4) Applicant has chosen for
(HDO). The said issuance was intended to restrain the
her destination five (5) countries, namely, Singapore,
indiscriminate issuance of HDOs which impinge on the
Germany, Austria, Spain and Italy, with which the Philippines
people's right to travel.
has no existing extradition treaty.
On April 23, 2007, former DOJ Secretary Raul M. Gonzalez
SC Issued TRO
issued DOJ Circular No. 18, prescribing rules and regulations
governing the issuance and implementation of watchlist The Court likewise resolved to issue a TRO in the
orders. In particular, it provides for the power of the DOJ consolidated petitions, enjoining the respondents from
Secretary to issue a Watchlist Order (WLO) against persons enforcing or implementing DOJ Circular No. 41 and WLOs
with criminal cases pending preliminary investigation or subject to the following conditions, to wit: (i) The petitioners
petition for review before the DOJ. Further, it states that the shall post a cash bond of Two Million Pesos (P2,000,000.00)
DOJ Secretary may issue an Allow Departure Order (ADO) to payable to this Court within five (5) days from notice hereof.
a person subject of a WLO who intends to leave the country Failure to post the bond within the aforesaid period will result
for some exceptional reasons. On May 25, 2010, then Acting in the automatic lifting of the temporary restraining order; (ii)
DOJ Secretary Alberto C. Agra issued the assailed DOJ The petitioners shall appoint a legal representative common
Circular No. 41, consolidating DOJ Circular Nos. 17 and 18, to both of them who will receive subpoena, orders and other
which will govern the issuance and implementation of HDOs, legal processes on their behalf during their absence. The
WLOs, and ADOs. petitioners shall submit the name of the legal representative,
also within five (5) days from notice hereof; and (iii) If there is
After the expiration of GMA's term as President of the
a Philippine embassy or consulate in the place where they will
Republic of the Philippines and her subsequent election as
be traveling, the petitioners shall inform said embassy or
Pampanga representative, criminal complaints were filed
consulate by personal appearance or by phone of their
against her before the DOJ.
whereabouts at all times;
Following the filing of criminal complaints, De Lima issued
On the very day of the issuance of the TRO, the petitioners
DOJ WLO No. 2011-573 against GMA and Miguel Arroyo on
tendered their compliance with the conditions set forth in the
October 27, 2011, with a validity period of 60 days, or until
Resolution.
December 26, 2011, unless sooner terminated or otherwise
extended. At around 8:00 p.m. on the same day, the petitioners
proceeded to the Ninoy Aquino International Airport (NAIA),
In three separate letters dated October 20, 2011, October 21,
with an aide-de-camp and a private nurse, to take their flights
2011, and October 24, 2011, GMA requested for the issuance
to Singapore. However, the BI officials at NAIA refused to
of an ADO, pursuant to Section 7 of DOJ Circular No. 41, so
process their travel documents which ultimately resulted to
that she may be able to seek medical attention from medical
them not being able to join their flights.
specialists abroad for her hypoparathyroidism and metabolic
bone mineral disorder. She mentioned six different countries On November 17, 2011, GMA, through counsel, filed an
where she intends to undergo consultations and treatments: Urgent Motion for Respondents to Cease and Desist from
United States of America, Germany, Singapore, Italy, Spain Preventing Petitioner GMA from Leaving the Country. She
and Austria. She likewise undertook to return to the strongly emphasized that the TRO issued by the Court was
Philippines, once her treatment abroad is completed, and immediately executory and that openly defying the same is
participate in the proceedings before the DOJ. tantamount to gross disobedience and resistance to a lawful
order of the Court.
To determine whether GMA's condition necessitates medical
attention abroad, the Medical Abstract prepared by Dr. Mirasol Respondent’s Contentions
was referred to then Secretary of the Department of Health,
Dr. Enrique Ona (Dr. Ona) for his expert opinion as the chief On November 16, 2011, the respondents filed a Consolidated
government physician. Dr. Ona visited GMA at her residence Urgent Motion for Reconsideration and/or to Lift TRO, praying
in La Vista Subdivision, Quezon City. After the visit, Dr. Ona that the Court reconsider and set aside the TRO issued in the
noted that "Mrs. Arroyo is recuperating reasonably well after consolidated petitions until they are duly heard on the merits.
having undergone a series of three major operations.
While the petitioners anchor their right in esse on the right to
GMA Filed for TRO travel under Section 6, Article III of the 1987 Constitution, the
said right is not absolute. One of the limitations on the right to
On November 8, 2011, before the resolution of her application travel is DOJ Circular No. 41, which was issued pursuant to
for ADO, GMA filed the present Petition for Certiorari and the rule-making powers of the DOJ in order to keep individuals
Prohibition under Rule 65 of the Rules of Court with Prayer for under preliminary investigation within the jurisdiction of the
the Issuance of a TRO and/or Writ of Preliminary Injunction, Philippine criminal justice system.
to annul and set aside DOJ Circular No. 41 and WLOs issued
against her for allegedly being unconstitutional.

Page 128 of 186


On November 18, 2011, the Court issued a Resolution, to which the delegate must conform in the performance of his
requiring De Lima to show cause why she should not be functions.
disciplinarily dealt with or held in contempt of court for failure
to comply with the TRO. She was likewise ordered to A painstaking examination of the provisions being relied upon
immediately comply with the TRO by allowing the petitioners by the former DOJ Secretary will disclose that they do not
to leave the country. At the same time, the Court denied the particularly vest the DOJ the authority to issue DOJ Circular
Consolidated Urgent Motion for Reconsideration and/or to Lift No. 41 which effectively restricts the right to travel through the
TRO dated November 16, 2011 filed by the Office of the issuance of WLOs and HDOs. Sections 1 and 3, Book IV, Title
Solicitor General. III, Chapter 1 of E.O. No. 292 reads:

Petitioner’s Contention Section 1. Declaration of Policy.- It is the declared


policy of the State to provide the government with a
Petitioner opined that DOI Circular No. 41 was validly issued principal law agency which shall be both its legal
pursuant to the agency's rule-making powers provided in counsel and prosecution arm; administer the criminal
Sections 1 and 3, Book IV, Title III Chapter 1 of E.O. No. 292 justice system in accordance with the accepted
and Section 50, Chapter 11, Book IV of the mentioned Code. processes thereof consisting in the investigation of
the crimes, prosecution of offenders and
administration of the correctional system; implement
the laws on the admission and stay of aliens,
ISSUE NO. 1
citizenship, land titling system, and settlement of land
Whether or not the DOJ has the power to issue a Hold problems involving small landowners and member of
Departure Order indigenous cultural minorities; and provide free legal
services to indigent members of the society.
HELD NO. 1
Section 3. Powers and Functions. - to accomplish its mandate,
NO. the Department shall have the following powers and functions:
Guided by the foregoing disquisition, the Court is in quandary (1) Act as principal law agency of the government
of identifying the authority from which the DOJ believed its and as legal counsel and representative thereof,
power to restrain the right to travel emanates. To begin with, whenever so required;
there is no law particularly providing for the authority of the
secretary of justice to curtail the exercise of the right to travel, (2) investigate the commission of crimes, prosecute
in the interest of national security, public safety or public offenders and administer the probation and
health. As it is, the only ground of the former DOJ Secretary correction system;
in restraining the petitioners, at that time, was the pendency
A plain reading of the foregoing provisions shows that they are
of the preliminary investigation of the Joint DOJ-COMELEC
mere general provisions designed to lay down the purposes
Preliminary Investigation Committee on the complaint for
of the enactment and the broad enumeration of the powers
electoral sabotage against them.
and functions of the DOJ. In no way can they be interpreted
To be clear, DOJ Circular No. 41 is not a law. It is not a as a grant of power to curtail a fundamental right as the
legislative enactment which underwent the scrutiny and language of the provision itself does not lend to that stretched
concurrence of lawmakers, and submitted to the President for construction.
approval. It is a mere administrative issuance apparently
In the same way, Section 3 does not authorize the DOJ to
designed to carry out the provisions of an enabling law which
issue WLOs and HDOs to restrict the constitutional right to
the former DOJ Secretary believed to be Executive Order
travel. There is even no mention of the exigencies stated in
(E.O.) No. 292, otherwise known as the "Administrative Code
the Constitution that will justify the impairment. The provision
of 1987."
simply grants the DOJ the power to investigate the
Indeed, administrative agencies possess quasi-legislative or commission of crimes and prosecute offenders, which are
rule-making powers, among others. It is the "power to make basically the functions of the agency. However, it does not
rules and regulations which results in delegated legislation carry with it the power to indiscriminately devise all means it
that is within the confines of the granting statute and the deems proper in performing its functions without regard to
doctrine of non-delegability and separability of powers." In the constitutionally-protected rights.
exercise of this power, the rules and regulations that
The DOJ cannot also rely on Section 50, Chapter 1 l, Book JV
administrative agencies promulgate should be within the
of E.O. No. 292, which simply provides for the types of
scope of the statutory authority granted by the legislature to
issuances that administrative agencies, in general, may issue.
the administrative agency. It is required that the regulation be
It does not speak of any authority or power but rather a mere
germane to the objects and purposes of the law, and be not in
clarification on the nature of the issuances that may be issued
contradiction to, but in conformity with, the standards
by a secretary or head of agency.
prescribed by law. They must conform to and be consistent
with the provisions of the enabling statute in order for such rule In the same manner, Section 7, Chapter 2, Title III, Book IV of
or regulation to be valid. E.O. 292 cited in the memorandum of the former DOJ
Secretary cannot justify the restriction on the right to travel in
It is, however, important to stress that before there can even
DOJ Circular No. 41. It is indisputable that the secretaries of
be a valid administrative issuance, there must first be a
government agencies have the power to promulgate rules and
showing that the delegation of legislative power is itself valid.
regulations that will aid performance in the performance of
It is valid only if there is a law that (a) is complete in itself,
their functions. This is adjunct to the power of administrative
setting forth the policy therein to be executed, carried out, or
agencies to execute laws and does not require the authority
implemented by the delegate: and (b) fixes a standard the
of a law. This is, however, different from the delegated
limits of which are sufficiently determinate and determinable
legislative power to promulgate rules of government agencies.

Page 129 of 186


The questioned circular does not come under the inherent That there is a risk of flight does not authorize the DOJ to take
power of the executive department to adopt rules and the situation upon itself and draft an administrative issuance
regulations as clearly the issuance of HDO and WLO is not to keep the individual within the Philippine jurisdiction so that
the DOJ's business. As such, it is a compulsory requirement he may not be able to evade criminal prosecution and
that there be an existing law, complete and sufficient in itself, consequent liability. It is an arrogation of power it does not
conferring the expressed authority to the concerned agency to have; it is a usurpation of function that properly belongs to the
promulgate rules. legislature.

Consistent with the foregoing, there must be an enabling law Without a law to justify its action, the issuance of DOJ Circular
from which DOJ Circular No. 41 must derive its life. No. 41 is an unauthorized act of the DOJ of empowering itself
Unfortunately, all of the supposed statutory authorities relied under the pretext of dire exigency or urgent necessity. This
upon by the DOJ did not pass the completeness test and action runs afoul the separation of powers between the three
sufficient standard test. The DOJ miserably failed to establish branches of the government and cannot be upheld. Even the
the existence of the enabling law that will justify the issuance Supreme Court, in the exercise of its power to promulgate
of the questioned circular. rules is limited in that the same shall not diminish, increase, or
modify substantive rights. This should have cautioned the
Petitioner Contends DOJ, which is only one of the many agencies of the executive
branch, to be more scrutinizing in its actions especially when
The DOJ stresses the necessity of the restraint imposed in
they affect substantive rights, like the right to travel.
DOJ Circular No. 41 in that to allow the petitioners, who are
under preliminary investigation, to exercise an untrammeled
right to travel, especially when the risk of flight is distinctly high
will surely impede the efficient and effective operation of the ISSUE NO 3
justice system. The absence of the petitioners, it asseverates,
would mean that the farthest criminal proceeding they could Whether or not the DOJ’s Power to issue Hold Departure
go would be the filing of the criminal information since they Order is pursuant to its police power.
cannot be arraigned in absentia.
HELD NO 3

NO.
ISSUE NO 2
The DOJ's reliance on the police power of the state cannot
Whether or not allowing the accused the right to travel when also be countenanced. Police power pertains to the "state
the risk of flight is extremely high will surely impede the authority to enact legislation that may interfere with personal
efficient and effective operation of the justice system. liberty or property in order to promote the general welfare." "It
may be said to be that inherent and plenary power in the State
HELD NO. 2 which enables it to prohibit all things hurtful to the comfort,
safety, and welfare of society." Verily, the exercise of this
NO. power is primarily lodged with the legislature but may be
wielded by the President and administrative boards, as well as
It bears emphasizing that the conduct of a preliminary
the lawmaking bodies on all municipal levels, including the
investigation is an implement of due process which essentially
barangay, by virtue of a valid delegation of power. It bears
benefits the accused as it accords an opportunity for the
noting, however, that police power may only be validly
presentation of his side with regard to the accusation. The
exercised if (a) the interests of the public generally, as
accused may, however, opt to waive his presence in the
distinguished from those of a particular class, require the
preliminary investigation. In any case, whether the accused
interference of the State, and (b) the means employed are
responds to a subpoena, the investigating prosecutor shall
reasonably necessary to the attainment of the object sought
resolve the complaint within 10 days after the filing of the
to be accomplished and not unduly oppressive upon
same.
individuals.
The point is that in the conduct of a preliminary investigation,
On its own, the DOJ cannot wield police power since the
the presence of the accused is not necessary for the
authority pertains to Congress. Even if it claims to be
prosecutor to discharge his investigatory duties. If the accused
exercising the same as the alter ego of the President, it must
chooses to waive his presence or fails to submit countervailing
first establish the presence of a definite legislative enactment
evidence, that is his own lookout. Ultimately, he shall be bound
evidencing the delegation of power from its principal. This, the
by the determination of the prosecutor on the presence of
DOJ failed to do. There is likewise no showing that the
probable cause and he cannot claim denial of due process.
curtailment of the right to travel imposed by DOJ Circular No.
The DOJ therefore cannot justify the restraint in the liberty of
41 was reasonably necessary in order for it to perform its
movement imposed by DOJ Circular No. 41 on the ground that
investigatory duties.
it is necessary to ensure presence and attendance in the
preliminary investigation of the complaints.

There is also no authority of law granting it the power to ISSUE NO. 4


compel the attendance of the subjects of a preliminary
investigation, pursuant to its investigatory powers under E.O. Whether or not the DOJ Circular violates due process
No. 292. Its investigatory power is simply inquisitorial and,
unfortunately, not broad enough to embrace the imposition of HELD NO. 4
restraint on the liberty of movement.
YES.

Page 130 of 186


Apart from lack of legal basis, DOJ Circular No. 41 also suffers that can prevent the Secretary of Justice to prevent anyone
from other serious infirmities that render it invalid. The from leaving the country under the guise of national security,
apparent vagueness of the circular as to the distinction public safety or public health.
between a HDO and WLO is violative of the due process
clause. An act that is vague "violates due process for failure Petitioner’s Contention
to accord persons, especially the parties targeted by it, fair
The DOJ argues that Section 6, Article III of the Constitution
notice of the conduct to avoid and leaves law enforcers
is not an exclusive enumeration of the instances wherein the
unbridled discretion in carrying out its provisions and becomes
right to travel may be validly impaired. It cites that this Court
an arbitrary flexing of the Government muscle." Here, the
has its own administrative issuances restricting travel of its
distinction is significant as it will inform the respondents of the
employees and that even lower courts may issue HDO even
grounds, effects and the measures they may take to contest
on grounds outside of what is stated in the Constitution.
the issuance against them. Verily, there must be a standard
by which a HDO or WLO may be issued, particularly against
those whose cases are still under preliminary investigation,
since at that stage there is yet no criminal information against ISSUE NO. 6
them which could have warranted the restraint.
Whether or not the enumeration of the instances wherein the
Further, a reading of the introductory provisions of DOJ right to travel may be validly impaired is exclusive.
Circular No. shows that it emanates from the DOJ's
HELD NO. 6
assumption of powers that is not actually conferred to it.
YES.

ISSUE NO. 5 It bears reiterating that the power to issue HDO is inherent to
the courts. The courts may issue a HDO against an accused
Whether or not a Watch List Order is constitutional in a criminal case so that he may be dealt with in accordance
with law. It does not require legislative conferment or
HELD NO 5 constitutional recognition; it co-exists with the grant of judicial
power. The inherent powers of the courts are essential in
NO.
upholding its integrity and largely beneficial in keeping the
It is apparent in Section 7 of the same circular that the subject people's faith in the institution by ensuring that it has the power
of a HDO or WLO cannot leave the country unless he obtains and the means to enforce its jurisdiction.
an ADO. The said section reads as follows:
As regards the power of the courts to regulate foreign travels,
Section 7. Allow Departure Order (ADO)- Any person by virtue of its administrative supervision over all courts and
subject of HDO/WLO issued pursuant to this Circular personnel that this Court came out with OCA Circular No. 49-
who intends, for some exceptional reasons, to leave 2003, which provided for the guidelines that must be observed
the country may, upon application under oath with by employees of the judiciary seeking to travel abroad.
the Secretary of Justice, be issued an ADO. Specifically, they are required to secure a leave of absence
for the purpose of foreign travel from this Court through the
The ADO may be issued upon submission of the following Chief Justice and the Chairmen of the Divisions, or from the
requirements: Office of the Court Administrator, as the case maybe. This is
"to ensure management of court dockets and to avoid
(a) Affidavit stating clearly the purpose, inclusive disruption in the administration of justice.
period of the date of travel, and containing an
undertaking to immediately report to the DOJ upon OCA Circular No. 49-2003 is therefore not a restriction, but
return; and more properly, a regulation of the employee's leave for
purpose of foreign travel which is necessary for the orderly
(b) Authority to travel or travel clearance from the administration of justice. To "restrict" is to restrain or prohibit
court or appropriate government office where the a person from doing something; to "regulate" is to govern or
case upon which the issued HDO/WLO was based is direct according to rule. This regulation comes as a necessary
pending, or from the investigating prosecutor in consequence of the individual's employment in the judiciary,
charge or the subject case. as part and parcel of his contract in joining the institution. For,
if the members of the judiciary are at liberty to go on leave any
By requiring an ADO before the subject of a HDO or WLO is
time, the dispensation of justice will be seriously hampered.
allowed to leave the country, the only plausible conclusion that
Short of key personnel, the courts cannot properly function in
can be made is that its mere issuance operates as a restraint
the midst of the intricacies in the administration of justice. At
on the right to travel. To make it even more difficult, the
any rate, the concerned employee is not prevented from
individual will need to cite an exceptional reason to justify the
pursuing his travel plans without complying with OCA Circular
granting of an ADO.
No. 49-2003 but he must be ready to suffer the consequences
The WLO also does not bear a significant distinction from a of his non-compliance.
HDO, thereby giving the impression that they are one and the
WHEREFORE, in view of the foregoing disquisition,
same or, at the very least, complementary such that whatever
Department of Justice Circular No. 41 is hereby declared
is not covered in Section 1, which pertains to the issuance of
UNCONSTITUTIONAL. All issuances which were released
HDO, can conveniently fall under Section 2, which calls for the
pursuant thereto are hereby declared NULL and VOID.
issuance of WLO. In any case, there is an identical provision
in DOJ Circular No. 41 which authorizes the Secretary of
Justice to issue a HDO or WLO against anyone, motu proprio,
in the interest of national security, public safety or public
health. With this all-encompassing provision, there is nothing

Page 131 of 186


I. RIGHT TO INFORMATION • the subdivision of the subject lot by the Director of Lands;
and

• the Land Management Bureau’s facilitation of the distribution


1. NAGKAKAISANG MARALITA NG SITIO MASIGASIG, and sale of the subject lot to its bona fide occupants.
INC. vs MILITARY SHRINE SERVICES - PHILIPPINE
VETERANS AFFAIRS OFFICE, DEPARTMENT OF Petitioner (WBLOAI) filed a Petition-in-Intervention
NATIONAL DEFENSE (G.R. No. 187587; June 5, 2013) substantially praying for the same reliefs as those prayed for
by NMSMI with regard to the area the former then occupied.

COSLAP issued a Resolution granting the Petition and


FACTS: By virtue of Proclamation 423, Former President declaring the portions of land in question alienable and
Carlos P. Garcia reserved parcels of land in the Municipalities disposable, with Associate Commissioner Lina Aguilar-
of Pasig, Taguig, Paranaque, Province of Rizal and Pasay General dissenting.
City for military reservation. The military reservation, then
known as Fort William McKinley, was later on renamed Fort COSLAP ruled that the handwritten addendum of President
Andres Bonifacio (Fort Bonifacio). Marcos was an integral part of Proclamation No. 2476, and
was therefore, controlling. The intention of the President could
President Ferdinand E. Marcos issued Proclamation No. 208, not be defeated by the negligence or inadvertence of others.
amending Proclamation No. 423, which excluded a certain Further, considering that Proclamation No. 2476 was done
area of Fort Bonifacio and reserved it for a national shrine. The while the former President was exercising legislative powers,
excluded area is now known as Libingan ng mga Bayani, it could not be amended, repealed or superseded, by a mere
which is under the administration of herein respondent MSS- executive enactment. Thus, Proclamation No. 172 could not
PVAO. have superseded much less displaced Proclamation No.
2476, as the latter was issued on October 16, 1987 when
Again, on 7 January 1986, President Marcos issued
President Aquino’s legislative power had ceased.
Proclamation No. 2476, further amending Proclamation No.
423, which excluded barangays Lower Bicutan, Upper Bicutan Herein respondent MSS-PVAO filed a Motion for
and Signal Village from the operation of Proclamation No. 423 Reconsideration, which was denied by the COSLAP in a
and declared it open for disposition under the provisions of Resolution dated 24 January 2007.
Republic Act Nos. (R.A.) 274 and 730.
MSS-PVAO filed a Petition with the Court of Appeals seeking
At the bottom of Proclamation No. 2476, President Marcos to reverse the COSLAP Resolutions dated 1 September 2006
made a handwritten addendum which includes Western and 24 January 2007.
Bicutan for the disposition of the area. The proclamation was
published in the Official Gazette without the handwritten Thus, on 29 April 2009, the then Court of Appeals First
addendum Division rendered the assailed Decision granting MSS-
PVAO’s Petition.
President Corazon C. Aquino issued Proclamation No. 172
which substantially reiterated Proclamation No. 2476, as Both NMSMI and WBLOAI appealed the said Decision by
published, but this time excluded Lots 1 and 2 of Western filing their respective Petitions for Review with this Court under
Bicutan from the operation of Proclamation No. 423 and Rule 45 of the Rules of Court.
declared the said lots open for disposition under the provisions
of R.A. 274 and 730.

Memorandum Order No. 119, implementing Proclamation No. ISSUE: WON the Court of Appeals erred in ruling that the
172, was issued on the same day. subject lots were not alienable and disposable by virtue of
Proclamation No. 2476 on the ground that the handwritten
Through the years, informal settlers increased and occupied addendum of President Marcos was not included in the
some areas of Fort Bonifacio including portions of the Libingan publication of the said law.
ng mga Bayani. Thus, Brigadier General Fredelito Bautista
issued General Order No. 1323 creating TFB, primarily to
prevent further unauthorized occupation and to cause the HELD: NO.
demolition of illegal structures at Fort Bonifacio. Demolition of
illegal structures existed to prevent the area from the Applying the foregoing ruling in Tanada vs Tuvera to the
increasing number of informal settlers. instant case, this Court cannot rely on a handwritten note that
was not part of Proclamation No. 2476 as published. The
Members of petitioner Nagkakaisang Maralita ng Sitio
handwritten addendum of President Marcos did not have the
Masigasig, Inc. (NMSMI) and Western Bicutan Lot Owners
force and effect law since it was not included in the publication.
Association, Inc. (WBLOAI) filed for a Petition with
We agree that the publication must be in full or it is no
Commission on Settlement of Land Problems (COSLAP)
publication at all since its purpose is to inform the public of the
praying for the reclassification of the areas they are occupying
contents of the laws.
as is already alienable and disposable, where it was docketed
as COSLAP Case No. 99-434. Furthermore, under Section 24, Chapter 6, Book I of the
Administrative Code, "the publication of any law, resolution or
Specifically, the Petition prayed for the following:
other official documents in the Official Gazette shall be prima
• the reclassification of the areas they occupied, covering Lot facie evidence of its authority." Thus, whether or not President
3 of SWO-13-000-298 of Western Bicutan, from public land to Marcos intended to include Western Bicutan is not only
alienable and disposable land pursuant to Proclamation No. irrelevant but speculative. Simply put, the courts may not
2476; speculate as to the probable intent of the legislature apart from
the words appearing in the law.

Page 132 of 186


This Court cannot rule that a word appears in the law when, 2. In Re: Production of Court Records and Documents
evidently, there is none. In Pagpalain Haulers, Inc. v. Hon. and the Attendance of Court officials and employees as
Trajano, we ruled that ' This does not mean, however, that witnesses under the subpoenas of February 10, 2012
courts can create law. The courts exist for inter"under Article and the various letters for the Impeachment Prosecution
8 of the Civil Code, 'judicial decisions applying or interpreting Panel dated January 19 and 25, 2012
the laws or the Constitution shall form a part of the legal
system of the Philippines. This does not mean, however, that
courts can create law. The courts exist for interpreting the law,
FACTS: During the impeachment proceedings against Chief
not for enacting it. To allow otherwise would be violative of the
Justice Corona, the prosecution Panel manifested in a
principle of separation of powers, inasmuch as the sole
COMPLIANCE that it would present about 100 witnesses
function of our courts is to apply or interpret the laws,
which included Justices of the Supreme Court, and Court
particularly where gaps or lacunae exist or where ambiguities
officials and employees who will testify on matters internal to
becloud issues, but it will not arrogate unto itself the task of
the Court and almost a thousand documents
legislating." The remedy sought in these Petitions is not
judicial interpretation, but another legislation that would In separate letters to the Supreme Court in January, members
amend the law ‘to include petitioners' lots in the of the prosecution panel requested the following:
reclassification.
- the rollo and certified true copies of the agenda and
WHEREFORE, in view of the foregoing, the instant petitions deliberations of Flight Attendants and Stewards Association of
are hereby DENIED for lack of merit. The assailed Decision of the Philippines (FASAP) vs. Philippine Airlines Inc.;
the Court of Appeals in CA-G.R. CV No. 97925 dated 29 April
2009 is AFFIRMED in toto. Accordingly, this Court's status - the rollo of Navarro vs. Ermita;
quo order dated 17 June 2009 is hereby LIFTED. Likewise, all
- the rollo of Ma. Merceditas N. Gutierrez vs. The House of
pending motions to cite respondent in contempt is DENIED,
Representatives Committee on Justice; and
having been rendered moot. No costs.
- the rollo of League of Cities vs. Comelec.

The Impeachment Court, on the other hand, issued a


subpoena ad testificandum et duces tecum dated February 9,
2012 directing the attendance of Clerk of Court Enriqueta
Vidal and Deputy Clerk of Court Felipa Anama, and the
production of documents involving FASAP vs. PAL, such as:

- records/logbook of the Raffle Committee showing the


assignment of the FASAP case; and

- letters of PAL counsel Estelito Mendoza addressed to the


Clerk of Court dated Sept. 13, 2011; Sept. 16, 2011; Sept. 20,
2011; and Sept. 22, 2011;

Another subpoena dated Feb. 10 directed Vidal the following


in the case of former first couple Gloria and Mike Arroyo:

- Supreme Court received (with time and date stamp) Petition


for Certiorari and Prohibition with Prayer for the Issuance of a
Temporary Restraining Order (TRO) and/or Writ of
Preliminary Injunction filed by Mrs. Arroyo, including annexes;

- Supreme Court received (with time and date stamp) Petition


for Certiorari and Prohibition with Prayer for the Issuance of a
TRO and/or Writ of Preliminary Injunction filed by Mr. Arroyo;

- Chief Justice Corona's travel order or leave applied for within


the month of November 2011;

- minutes of the Raffle Committee which handled the Arroyo


petitions;

- appointment or assignment of the Member(Justice)-in-


charge of the Arroyo petitions;

- Resolution dated Nov. 15, 2011 in the Arroyo petitions;

- Temporary Restraining Order (TRO) dated November 15,


2011 issued in the Arroyo petitions;

Page 133 of 186


- logbook or receiving copy showing the time the TRO was the consent of the Supreme Court, can testify on matters
issued to the counsel of the Arroyos, and the date and time covered by the prohibitions and exclusions, particularly with
the TRO was received by the Sheriff for service to the parties; respect to matters pending resolution before the Supreme
Court.
- Special Power of Attorney dated Nov. 15, 2011 submitted by
the Arroyos in favor of Attys. Ferdinand Topacio and Anacleto
Diaz in compliance with the TRO;
ISSUE NO. 2
- official receipt dated Nov. 15, 2011 issued by the Supreme
Court for the P2-million cash bond posted by Mr. and Mrs. Whether or not Court Records are considered Confidential or
Arroyo, with the official date and time stamp; privileged.

- Nov. 15 and 16, 2011 Sheriff's Return for the service of the HELD NO. 2
TRO to the Department of Justice (DOJ) and the Office of the
In the Judiciary, privileges against disclosure of official records
Solicitor General (OSG);
“create a hierarchy of rights that protect certain confidential
- certification from the Fiscal Management and Budget Office relationships over and above the public’s evidentiary need” or
of the Supreme Court dated Nov. 15, 2011, with the date and “right to every man’s evidence.” Accordingly, certain
time it was received by the Clerk of Court showing it to be Nov. information contained in the records of cases before the
16, 2011 at 8:55 am; Supreme Court are considered confidential and are exempt
from disclosure. To reiterate, the need arises from the dictates
- Resolution dated Nov. 18, 2011 issued in the Arroyo TRO of the integrity of the Court’s decision-making function which
petitions; may be affected by the disclosure of information.

- Resolution dated Nov. 22, 2011 on the Arroyo petitions; The Internal Rules of the Supreme Court (IRSC) prohibits the
disclosure of
- logbook showing the date and time Associate Justice Ma.
Lourdes Sereno's dissent to the Nov. 22 Resolution was (1) the result of the raffle of cases,
received by the Clerk of Court en banc;
(2) the actions taken by the Court on each case included in
- Dissenting Opinions dated Nov. 13 and 18, 2011 of the agenda of the Court’s session,
Associate Justice Antonio Carpio on the Arroyo TRO petitions;
(3) the deliberations of the Members in court sessions on
- Separate Opinion dated Dec. 13, 2011 of Associate Justice cases and matters pending before it.
Presbitero Velasco, Jr. on the Arroyo TRO petitions;
Rule 7, Section 3 of the IRSC10 declares that the results of
- Concurring Opinion dated Dec. 13, 2011 of Associate Justice the raffle of cases shall only be available to the parties and
Roberto Abad on the Arroyo petitions; their counsels, unless the cases involve bar matters,
administrative cases and criminal cases involving the penalty
- Official Appointment of Chief Justice Corona as Associate of life imprisonment, which are treated with strict
Justice of the Supreme Court; and confidentiality and where the raffle results are not disclosed
even to the parties themselves.
- Official Appointment of Chief Justice Corona as Chief
Magistrate. Rule 10, Section 2 of the IRSC provides that the actions taken
in each case in the Court’s agenda, which are noted by the
Chief Justice or the Division Chairman, are also to be treated
Subpoena Ad Testificandum et Duces TecumAnd Subpoena with strict confidentiality. Only after the official release of the
Ad Testificandum were also issued against Clerks of Court of resolution embodying the Court action may that action be
the SC. made available to the public. A resolution is considered
officially released once the envelope containing its final copy,
In light of the subpoenas served, the urgent need for a court addressed to the parties, has been transmitted to the process
ruling and based on the Constitution, the pertinent laws and of server for personal service or to the mailing section of the
the Court’s rules and policies, we shall now determine how the Judicial Records Office.
Court will comply with the subpoenas and the letters of the
Prosecution Impeachment Panel. Court deliberations are traditionally recognized as privileged
communication. Section 2, Rule 10 of the IRSC provides:

Section 2. Confidentiality of court sessions. – Court


ISSUE NO. 1 sessions are executive in character, with only the
Members of the Court present. Court deliberations
What is Judicial Privilege?
are confidential and shall not be disclosed to outside
HELD NO. parties, except as may be provided herein or as
authorized by the Court.
In fine, there are Philippine laws, rules and jurisprudence
prohibiting the revelation of confidential or “secret” information Justice Abad discussed the rationale for the rule in his
that causes damage to public interest even in judicial and concurring opinion to the Court Resolution in Arroyo v. De
other proceedings such as the sui generis impeachment trial. Lima13 (TRO on Watch List Order case): the rules on
As far as the Court is concerned, its Members and officials confidentiality will enable the Members of the Court to “freely
involved in all proceedings are duty-bound to observe the discuss the issues without fear of criticism for holding
privileged communication and confidentiality rules if the unpopular positions” or fear of humiliation for one’s comments.
integrity of the administration of justice were to be preserved The privilege against disclosure of these kinds of
– i.e., not even Members of the Court, on their own and without information/communication is known as deliberative process

Page 134 of 186


privilege, involving as it does the deliberative process of 2. Confidential information generally refers to
reaching a decision. “Written advice from a variety of information not yet made a matter of public record
individuals is an important element of the government’s relating to pending cases, such as notes, drafts,
decision-making process and that the interchange of advice research papers, internal discussion, internal
could be stifled if courts forced the government to disclose memoranda, records of internal deliberations, and
those recommendations;” the privilege is intended “to prevent similar papers. Even after the decision, resolution, or
the ‘chilling’ of deliberative communications.” order is made public, such information that a justice
or judge uses in preparing a decision, resolution, or
The privilege is not exclusive to the Judiciary. We have in order shall remain confidential.
passing recognized the claim of this privilege by the two other
branches of government in Chavez v. Public Estates To qualify for protection under the deliberative process
Authority17 (speaking through J. Carpio) when the Court privilege, the agency must show that the document is both (1)
declared that - predecisional and (2) deliberative.

[t]he information x x x like internal deliberations of the A document is “predecisional” under the deliberative process
Supreme Court and other collegiate courts, or executive privilege if it precedes, in temporal sequence, the decision to
sessions of either house of Congress, are recognized as which it relates. In other words, communications are
confidential. This kind of information cannot be pried open by considered predecisional if they were made in the attempt to
a co-equal branch of government. A frank exchange of reach a final conclusion.
exploratory ideas and assessments, free from the glare of
publicity and pressure by interested parties, is essential to A material is “deliberative,” on the other hand, if it reflects the
protect the independence of decision-making of those tasked giveand- take of the consultative process. The key question in
to exercise Presidential, Legislative and Judicial power. determining whether the material is deliberative in nature is
whether disclosure of the information would discourage
Justice Brion noted this fact in his Separate Concurring candid discussion within the agency. If the disclosure of the
Opinion in Neri v. Senate Committee on Accountability of information would expose the government’s decision making
Public Officers and Investigations: process in a way that discourages candid discussion among
the decision-makers (thereby undermining the courts’ ability to
Significantly, this type of privilege is not for the Executive to perform their functions), the information is deemed privileged.
enjoy alone. All the great branches of government are entitled
to this treatment for their own decision and policy making Court records which are “predecisional” and “deliberative” in
conversations and correspondence. It is unthinkable that the nature are thus protected and cannot be the subject of a
disclosure of internal debates and deliberations of the subpoena if judicial privilege is to be preserved. The privilege
Supreme Court or the executive sessions of either Houses of in general insulates the Judiciary from an improper intrusion
Congress can be compelled at will by outside parties. into the functions of the judicial branch and shields justices,
judges, and court officials and employees from public scrutiny
Thus, a Senator may invoke legislative privilege when he or or the pressure of public opinion that would impair a judge’s
she is questioned outside the Senate about information ability to render impartial decisions. The deliberative process
gathered during an executive session of the Senate’s can be impaired by undue exposure of the decision-making
legislative inquiry in aid of legislation. In the same manner, a process to public scrutiny before or even after the decision is
justice of the court or a judge may invoke judicial privilege in made, as discussed below.
the Senate sitting as an Impeachment Court, for proceedings
in the performance of his or her own judicial functions. What Additionally, two other grounds may be cited for denying
applies to magistrates applies with equal force to court officials access to court records, as well as preventing members of the
and employees who are privy to these deliberations. They bench, from being subjected to compulsory process:
may likewise claim exemption when asked about this
privileged information. (1) the disqualification by reason of privileged
communication and
While Section 2, Rule 10 of the IRSC cited above speaks only
of the confidentiality of court deliberations, it is understood that (2) the pendency of an action or matter.
the rule extends to documents and other communications
The prohibition against disclosure of confidential information
which are part of or are related to the deliberative process.
is required to be observed by members of the Court under the
The deliberative process privilege protects from disclosure
New Code of Judicial Conduct for the Philippine Judiciary.
documents reflecting advisory opinions, recommendations
Section 9, Canon 4 (Propriety) states:
and deliberations that are component parts of the process for
formulating governmental decisions and policies. Obviously, Section 9. Confidential information acquired by
the privilege may also be claimed by other court officials and judges in their judicial capacity shall not be used or
employees when asked to act on these documents and other disclosed for any other purpose related to their
communications. judicial duties.
The Code of Conduct for Court Personnel in fact provides that This rule of judicial ethics complements the rule of evidence
access shall be denied with respect to information or records that disqualifies public officials from testifying on information
relating to drafts of decisions, rulings, orders, or internal they acquire in confidence in the course of their duties:
memoranda or internal reports. In the 2007 Resolution on
Access to Justice for the Poor Project, the Court excluded the Rules of Court, Rule 130, Section 24. Disqualification by
same information and records from the public by classifying reason of privileged communication. – The following persons
them as confidential: cannot testify as to matters learned in confidence in the
following cases:
Article 1. Definition of Terms.

Page 135 of 186


(e) A public officer cannot be examined during his To state the rule differently, Justices of the Court cannot be
term of office or afterwards, as to communications compelled to testify on matters relating to the internal
made to him in official confidence, when the court deliberations and actions of the Court, in the exercise of their
finds that the public interest would suffer by the adjudicatory functions and duties. This is to be differentiated
disclosure. from a situation where the testimony is on a matter which is
external to their adjudicatory functions and duties.
Under the law, therefore, the Members of the Court may not
be compelled to testify in the impeachment proceedings
against the Chief Justice or other Members of the Court about
information they acquired in the performance of their official When court records are considered confidential
function of adjudication, such as information on how
The "deliberative process privilege" was also stressed, where
deliberations were conducted or the material inputs that the
Justices of the High Court freely discuss and deliberate cases
justices used in decision-making, because the end-result
and matters "without fear of criticism for holding unpopular
would be the disclosure of confidential information that could
positions or fear of humiliation for one's comments."
subject them to criminal prosecution. Such act violates judicial
privilege (or the equivalent of executive privilege) as it pertains The Resolution stated that deliberations in closed-door
to the exercise of the constitutional mandate of adjudication. sessions must remain executive in nature, as the same is also
granted the Executive and Legislative branches of
Jurisprudence implies that justices and judges may not be
government as stated in Chavez vs. Public Estates Authority
subject to any compulsory process in relation to the
where executive sessions of the Senate and the Lower House
performance of their adjudicatory functions. In Senate of the
are recognized as confidential, and in Neri vs. Senate
Philippines v. Exec. Sec. Ermita,31 the Court declared that
Committee on Accountability of Public Officers and
members of the Supreme Court are also exempt from [the
Investigations where it was held that the privilege is enjoyed
Congress’] power of inquiry [in aid of legislation]. Unlike the
by all the branches of government "for their own decision and
Presidency, judicial power is vested in a collegial body; hence,
policy-making conversation and correspondence."
each member thereof is exempt on the basis not only of
separation of powers but also on the fiscal autonomy and the
constitutional independence of the judiciary.
Court records considered 'matter of public record' allowed to
This ruling was dictated in no small measure by the principle be released
of comity mentioned above. Inter-departmental courtesy
demands that the highest levels of each department be As to Court Records:
exempt from the compulsory processes of the other
While the High Court refused the requested examination by
departments on matters related to the functions and duties of
the Prosecution of the rollos of the FASAP vs. PAL, Navarro
their office.
vs. Ermita, Ma. Merceditas Gutierrez vs. The House of
With respect to Court officials and employees, the same rules Representatives, and League of Cities vs. Comelec, it allowed
on confidentiality that apply to justices and judges apply to the issuance of certified true copies of the Decisions, Orders,
them. They are barred from disclosing (1) the result of the and Resolutions it issued in these cases and which have
raffle of cases, (2) the actions taken by the Court on each case already been released to the parties, including the certified
included in the agenda of the Court’s session, and (3) the true copies of the parties' pleadings.
deliberations of the Members in court sessions on cases and
The High Court also allowed the release of certified true
matters pending before it. They are subject as well to the
copies of the letters of Atty. Mendoza.
disqualification by reason of privileged communication and the
sub judice rule. As stated above, these rules extend to As to the Senate Impeachment Court's subpoena, all
documents and other communications which cannot be documents subpoenaed pertaining to the FASAP vs. PAL
disclosed. case were not allowed to be released citing the pendency of
the case.
These privileges, incidentally, belong to the Judiciary and are
for the Supreme Court (as the representative and entity Vidal, however, was directed to provide the impeachment
speaking for the Judiciary), and not for the individual justice, court with certified true copies of the Decisions, Orders and
judge, or court official or employees to waive. Thus, every Resolutions furnished to the parties, parties' pleadings, and
proposed waiver must be referred to the Supreme Court for its Mendoza's letters.
consideration and approval.
As for the subpoena on court records pertaining to the Arroyo
In fine, there are Philippine laws, rules and jurisprudence TRO petitions, the High Court allowed the release of certified
prohibiting the revelation of confidential or “secret” information true copies only of the following on the basis that these are
that causes damage to public interest even in judicial and matters of public record (the rest, privileged and confidential):
other proceedings such as the sui generis impeachment trial.
As far as the Court is concerned, its Members and officials - Supreme Court received (with time and date stamp) Petition
involved in all proceedings are duty-bound to observe the for Certiorari and Prohibition with Prayer for the Issuance of a
privileged communication and confidentiality rules if the Temporary Restraining Order (TRO) and/or Writ of
integrity of the administration of justice were to be preserved Preliminary Injunction filed by Mrs. Arroyo, including annexes;
– i.e., not even Members of the Court, on their own and without
the consent of the Supreme Court, can testify on matters - Supreme Court received (with time and date stamp) Petition
covered by the prohibitions and exclusions, particularly with for Certiorari and Prohibition with Prayer for the Issuance of a
respect to matters pending resolution before the Supreme TRO and/or Writ of Preliminary Injunction filed by Mr. Arroyo;
Court.
- Official Leave of Chief Justice Corona's travel order or leave
applied in Nov. 2011;

Page 136 of 186


- Resolution dated Nov. 15, 2011 in the Arroyo petitions, as To restate the rule, entries in official records may be presented
published; without the necessity of presenting in court the officer or
person who made the entries. Entries in public or official books
- TRO dated November 15, 2011 issued in the Arroyo or records may be proved by the production of the books or
petitions; records themselves or by a copy certified by the legal keeper
thereof. These records, however, may be presented and
- official receipt dated Nov. 15, 2011 issued by the Supreme
marked in evidence only where they are not excluded by
Court for the P2 M cash bond posted by Mr. and Mrs. Arroyo,
reasons of privilege and the other reasons discussed above.
with the official date and time stamp;
The reasons for this rule are necessity and trustworthiness.
- Resolution dated Nov. 18, 2011 issued in the Arroyo TRO
Necessity consists in the inconvenience and difficulty of
petitions, as published;
requiring the official’s attendance as a witness to testify to the
- Resolution dated Nov. 22, 2011 on the Arroyo petitions; innumerable transactions in the course of his duty. A public
officer is excused from appearing in court in order that public
- Official Appointment of Chief Justice Corona as Associate business may not be interrupted, hampered or delayed.
Justice of the Supreme Court; and Where there is no exception for official statements, hosts of
officials would be found devoting the greater part of their time
- Official Appointment of Chief Justice Corona as Chief
attending as witnesses in court, delivering their deposition
Justice.
before an officer. Trustworthiness is a reason because of the
presumption of regularity of performance of official duty. The
law reposes a particular confidence in public officers that it
The High Court allowed the release of certified true copies of presumes that they will discharge their several trusts with
the subpoenaed Opinions of Justices Carpio, Sereno, Velasco accuracy and fidelity; and therefore, whatever acts they do in
and Abad, however, reservations were made as it was pointed the discharge of their public duty may be given in evidence
out that as an institution, the Supreme Court is entitled to the and shall be taken to be true under such a degree of caution
deliberative process privilege and "cannot waive the as the nature and circumstances of each case may appear to
confidentiality of certain portions" of the Opinions. require. Thus, “[t]he trustworthiness of public documents and
the value given to the entries made therein could be grounded
The Resolution also pointed out that the attendance of court on: 1) the sense of official duty in the preparation of the
officials and employees in the impeachment trial should be statement made, 2) the penalty which is usually affixed to a
excused "if the intent only is for them to identify and certify to breach of that duty, 3) the routine and disinterested origin of
the existence and genuineness of documents within their most such statements, and 4) the publicity of record which
custody or control that are not otherwise confidential or makes more likely the prior exposure of such errors as might
privileged." have occurred.”
SC Justices, officials, employees not compelled to testify on
'adjudicatory' incidents
The independence of the Judiciary, separation of powers
In its Resolution, the High Court held that "Philippine laws,
rules and jurisprudence prohibit the disclosure of confidential The High Court stressed that the doctrine of separation of
or privileged information under well-defined rules" and powers "is an essential component of our democratic and
stressed that "Justices and judges may not be subject to any republican system of government" and system of checks and
compulsory process in relation to the performance of their balances.
adjudicatory functions."
The High Court maintained that its mandate, "in so far as these
"Under the law... the Members of the Court may not be constitutional principles are concerned, is to keep the different
compelled to testify in the impeachment proceedings against branches within the exercise of their respective powers and
the Chief Justice or other Members (Justices) of the Court prerogatives through the Rule of Law."
about information they acquired in the performance of their
official function of adjudication, such as information on how
deliberations were conducted or the material inputs that the Principle of comity
Justices used in decision-making, because the end-result
would be the disclosure of confidential information that could Inter-branch and inter-departmental comity was also stressed
subject them to criminal prosecution. as the High Court held that while voluntary, is established
practice, for each branch to perform its assigned constitutional
"Such act violates judicial privilege (or the equivalent of
duties.
executive privilege) as it pertains to the exercise of the
constitutional mandate of adjudication," the Resolution read. "The Judiciary applies the principle of comity at the first
instance in its interpretation and application of laws... the
As a penultimate point, witnesses need not be summoned to
courts tread carefully; they exercise restraint and intervene
testify on matters of public record. These are the records that
only when the grave abuse of discretion is clear and even then
a government unit is required by law to keep or which it is
must act with carefully calibrated steps, safely and surely
compelled to keep in the discharge of duties imposed by law.
made within constitutional bounds," the Resolution read.
A record is a public record within the purview of a statute
providing that books and records required by law to be kept by The High Court recognized that the trial on matters of
a clerk may be received in evidence in any court if it is a record impeachment "has been specifically assigned by the
which a public officer is required to keep and if it is filled in Constitution to the Senate," however, it also held that "where
such a manner that it is subject to public inspection. Under the doubt exists in an impeachment case, a standard that should
Rules of Court, the rule on public records is embodied in not be forgotten is the need to preserve the structure of a
Section 44, Rule 130.

Page 137 of 186


democratic and republican government, particularly the check 3. HAZEL MA. C. ANTOLIN vs. ABELARDO T.
and balance that should prevail." DOMONDON, JOSE A. GANGAN, and VIOLETA J.
JOSEF (G.R. No. 165036; July 5, 2010)
As a last point and mainly for purposes of stress, the privileges
discussed above that apply to justices and judges apply
mutatis mutandis to court officials and employees with respect FACTS: Antolin took the accountancy licensure examinations
to their official functions. If the intent only is for them to identify conducted by the Board of Accountancy (the Board) in
and certify to the existence and genuineness of documents October 1997.1 The examination results were released and
within their custody or control that are not otherwise out of 6,481 examinees, only 1,171 passed. Unfortunately,
confidential or privileged under the above discussed rules, petitioner did not make it. When the results were released, she
their presence before the Impeachment Court can be and received failing grades in four out of the seven subjects.2
should be excused where certified copies of these non-
privileged and non-confidential documents can be provided. Convinced that she deserved to pass the examinations, she
wrote to respondent Abelardo T. Domondon (Domondon),
Acting Chairman of the Board of Accountancy, and requested
that her answer sheets be re-corrected.3 The petitioner was
shown her answer sheets, but these consisted merely of
shaded marks, so she was unable to determine why she failed
the exam.4 Thus, she again wrote to the Board to request for
copies of (a) the questionnaire in each of the seven subjects
(b) her answer sheets; (c) the answer keys to the
questionnaires, and (d) an explanation of the grading system
used in each subject

Acting Chairman Domondon denied petitioner’s request.

Petitioner filed a Petition for Mandamus with Damages, with


application for preliminary mandatory injunction, against the
Board and its members before the Regional Trial Court (RTC),
praying that the Board provide her with all documents that
would show whether the Board fairly administered the exam
and correctly graded her answers, and if warranted, to issue
to her a certificate of registration as a CPA. She later amended
her Petition to clarify that she only wanted access to the
documents requested, not recorrection of her exam, deleting
in the process her original prayer for issuance of a certificate
of registration as CPA.

Petitioner passed the May 1998 CPA Licensure Exam and


took her oath as a CPA. Consequently, the RTC denied her
application for mandatory injunction for being moot. She
amended her Petition a second time to implead the PRC and
to ask, in addition to access to the documents she had
requested, that if warranted, appropriate revisions in the
October 1997 Exam results be made by the Board and the
PRC. The RTC considered the matter moot and dismissed the
petition. On her motion, however, the RTC reconsidered the
dismissal, holding that her passing of the subsequent CPA
examination did not render the petition moot because the relief
“and if warranted, to issue to her a certificate of registration as
Certified Public Accountant” was deleted from the original
petition. As regards whether she had the constitutional right to
have access to the documents she requested, the RTC
resolved to let the parties first adduce evidence, and to have
PRC air its side of the case. The RTC also ordered the PRC
to preserve and safeguard the questionnaire, petitioner’s
answer sheets, and the answer keys for the October 1997
CPA Licensure Exam.

When their motion for reconsideration was denied,


respondents brought the case to the Court of Appeals (CA)
which set aside the RTC’s decision and ordered the dismissal
of the case because: (1) the petition was mooted when
petitioner passed the May 1998 CPA exam; (2) Section 20,
Article IV of PRC Resolution No. 338, series of 1994,
constituted a valid limitation on her right to information and
access to government documents; (3) the Examination
Documents were not of public concern, because she merely
sought review of her failing marks; (4) it was not the ministerial

Page 138 of 186


or mandatory function of the respondents to review and exceptions, among which is when only a question of law is
reassess the answers to examination questions of a failing involved. Whether or not petitioner had a constitutional right to
examinee; and (5) she failed to exhaust administrative demand access to the Examination Papers was one such
remedies when she did not elevate the matter to the PRC question of law which cannot be resolved with finality by the
before seeking judicial intervention. Petitioner, thus, brought administrative officer.
the matter to the Supreme Court.

ISSUES:
3. An issue becomes moot and academic when it ceases to
1. WON petitioner may seek judicial intervention to compel the present a justiciable controversy, so that a declaration on the
re-correction of her examination; issue would be of no practical use or value. In this jurisdiction,
any citizen may challenge any attempt to obstruct the exercise
2. WON petitioner failed to exhaust the administrative of his or her right to information and may seek its enforcement
remedies; by mandamus. And since every citizen possesses the inherent
right to be informed by the mere fact of citizenship, petitioner’s
3. WON the case was mooted by petitioner’s passing the May
belated passing of the CPA Board Exams did not
1998 CPA Licensure Examination; and
automatically mean that her interest in the Examination
4. WON petitioner has the constitutional right to have Papers had become mere superfluity. Undoubtedly, the
access to the Examination Papers. (MAIN ISSUE) constitutional question presented, in view of the likelihood that
the issues in this case would be repeated, warranted review.

HELD:
4. Like all the constitutional guarantees, the right to
1. Any claim for re-correction or revision of petitioner’s 1997 information is not absolute; it is limited to “matters of
examination cannot be compelled by mandamus. In public concern” and is further “subject to such
AgustinRamos vs. Sandoval[G.R. No. 84470, February 2, limitations as may be provided by law” (Section 7, Article
1989 (Minute Resolution)], where therespondent Judge was III, 1987 Constitution). Similarly, the State’s policy of full
questioned for dismissing therein petitioners’ mandamus disclosure is limited to “transactions involving public
action to compel the Medical Board of Examiners and the interest,” and is “subject to reasonable conditions
Professional Regulation Commission to re-correct their prescribed by law” (Sec. 28, Art. II, 1987 Constitution). The
ratings, the Supreme Court held that “(t)he function of Court has always grappled with the meanings of “public
reviewing and re-assessing the petitioners’ answers to the interest” and “public concern” which “embrace a broad
examination questions, in the light of the facts and arguments spectrum of subjects which the public may want to know,
presented by them x x x is a discretionary function of the either because these directly affect their lives, or simply
Medical Board, not a ministerial and mandatory one, hence, because such matters naturally arouse the interest of an
not within the scope of thewrit of mandamus.” ordinary citizen,” and which are, in the final analysis, up
to the courts to determine on a case by case basis
For a writ of mandamus to issue, the applicant must have a [Legaspi v. Civil Service Commission, 234 Phil. 521, 535
well-defined, clear, and certain legal right to the thing (1987)]. National board examinations such as the CPA
demanded. The corresponding duty of the respondent to Board Exams are matters of public concern. The
perform the required act must be equally clear. No such clarity populace in general, and the examinees in particular,
exists here. And despite petitioner’s assertion that she did not would understandably be interested in the fair and
demand re-correction, the most cursory perusal of her Second competent administration of these exams in order to
Amended Petition and her prayer that respondents “make the ensure that only those qualified are admitted into the
appropriate revisions on the results of her examination” belied accounting profession. And as with all matters
this claim.
pedagogical, these examinations could be not merely
quantitative means of assessment, but also means to
further improve the teaching and learning of the art and
2. Like the claimants in Agustin, petitioner’s remedy from the science of accounting. The Court, nonetheless, realizes
Board’s refusal to release the Examination Papers should that there may be valid reasons to limit access to the
have been through an appeal to the PRC. Under Section 5(c) Examination Papers in order to properly administer the
of Presidential Decree No. 223, the PRC has the power to exam. More than the mere convenience of the examiner,
review and approve the policies, resolutions, rules and it may well be that there exist inherent difficulties in the
regulations, orders and decisions of the various professional preparation, generation, encoding, administration, and
Boards, including the results of their licensure examinations, checking of these multiple choice exams that require that
and the decisions of the Boards on administrative cases shall the questions and answers remain confidential for a
be final and executory unless appealed to the PRC within 30 limited duration. The PRC, however, had not been given
days from promulgation. Contrary’s to petitioner’s claim, this an opportunity to explain the reasons behind their
power is not limited to administrative investigations but regulations or articulate the justification for keeping the
encompasses requests for documents. And since the PRC Examination Papers confidential. In view of the far-
itself issued the resolution (PRC Resolution No. 338) reaching implications of this case, which may impact on
questioned by petitioner, it was in the best position to resolve every board examination administered by the PRC, and in
questions addressed to its area of expertise. One of the order that all relevant issues may be ventilated, the Court
reasons for exhaustion of administrative remedies is thewell- deemed it best to remand the case to the RTC for further
entrenched doctrine on separation of powers, which enjoins proceedings.
upon the Judiciary a becoming policy of non-interference with
matters falling primarily (albeit not exclusively) within the
competence of other departments. However, the principle of
exhaustion of administrative remedies is subject to

Page 139 of 186


4. AIR PHILIPPINES CORPORATION v. PENNSWELL, affirms the ruling of the Court of Appeals which upheld the
INC. (G.R. NO. 172835 : December 13, 2007) finding of the RTC that there is substantial basis for
respondent to seek protection of the law for its proprietary
FACTS: Petitioner Air Philippines Corporation is a domestic rights over the detailed chemical composition of its products.
corporation engaged in the business of air transportation
The Supreme Court has declared that trade secrets and
services. On the other hand, respondent Pennswell, Inc. was
banking transactions are among the recognized restrictions to
organized to engage in the business of manufacturing and
the right of the people to information as embodied in the
selling industrial chemicals, solvents, and special lubricants.
Constitution. SC said that the drafters of the Constitution also
unequivocally affirmed that, aside from national security
On various dates, respondent delivered and sold to petitioner matters and intelligence information, trade or industrial secrets
sundry goods in trade. Under the contracts, petitioner's total (pursuant to the Intellectual Property Code and other related
outstanding obligation amounted to P449,864.98 with interest laws) as well as banking transactions (pursuant to the Secrecy
at 14% per annum until the amount would be fully paid. For of Bank Deposits Act), are also exempted from compulsory
failure of the petitioner to comply with its obligation under said disclosure.
contracts, respondent filed a Complaint for a Sum of Money
with the RTC. A trade secret is defined as a plan or process, tool,
mechanism or compound known only to its owner and those
In its Answer, petitioner contended that its refusal to pay was of his employees to whom it is necessary to confide it. The
not without valid and justifiable reasons. In particular, definition also extends to a secret formula or process not
petitioner alleged that it was defrauded in the amount patented, but known only to certain individuals using it in
of P592,000.00 by respondent for its previous sale of four compounding some article of trade having a commercial
items. Said items were misrepresented by respondent as value. American jurisprudence has utilized the following
belonging to a new line, but were in truth and in fact, identical factors to determine if an information is a trade secret, to wit:
with products petitioner had previously purchased from (1) the extent to which the information is known outside of the
respondent. Petitioner asserted that it was deceived by employer’s business; (2) the extent to which the information is
respondent which merely altered the names and labels of known by employees and others involved in the business; (3)
such goods. the extent of measures taken by the employer to guard the
secrecy of the information; (4) the value of the information to
the employer and to competitors; (5) the amount of effort or
Petitioner alleged that when the purported fraud was
money expended by the company in developing the
discovered, a conference was held between petitioner and
information; and (6) the extent to which the information could
respondent, whereby the parties agreed that respondent
be easily or readily obtained through an independent source.
would return to petitioner the amount it previously paid.
However, petitioner was surprised when it received a letter Rule 27 sets an unequivocal proviso that the documents,
from the respondent, demanding payment of the amount papers, books, accounts, letters, photographs, objects or
of P449,864.94, which later became the subject of tangible things that may be produced and inspected should
respondent's Complaint for Collection of a Sum of Money not be privileged. The documents must not be privileged
against petitioner. against disclosure. On the ground of public policy, the rules
providing for production and inspection of books and papers
During the pendency of the trial, petitioner filed a Motion to do not authorize the production or inspection of privileged
Compel respondent to give a detailed list of the ingredients matter; that is, books and papers which, because of their
and chemical components of the following products. The RTC confidential and privileged character, could not be received in
rendered an Order granting the petitioner’s motion. evidence. Such a condition is in addition to the requisite that
Respondent sought reconsideration of the foregoing Order, the items be specifically described, and must constitute or
contending that it cannot be compelled to disclose the contain evidence material to any matter involved in the action
chemical components sought because the matter is and which are in the party’s possession, custody or control.
confidential. It argued that what petitioner endeavored to
inquire upon constituted a trade secret which respondent In the case at bar, petitioner cannot rely on Section 77of
cannot be forced to divulge. Republic Act 7394, or the Consumer Act of the Philippines, in
order to compel respondent to reveal the chemical
The RTC gave credence to respondent’s reasoning, and components of its products. While it is true that all consumer
reversed itself. Alleging grave abuse of discretion on the part products domestically sold, whether manufactured locally or
of the RTC, petitioner filed a Petition for Certiorari under Rule imported, shall indicate their general make or active
65 of the Rules of Court with the Court of Appeals, which ingredients in their respective labels of packaging, the law
denied the Petition and affirmed the Order dated 30 June 2004 does not apply to respondent. Respondent’s specialized
of the RTC. Petitioner’s Motion for Reconsideration was lubricants — namely, Contact Grease, Connector Grease,
denied. Unyielding, petitioner brought the instant Petition Thixohtropic Grease, Di-Electric Strength Protective Coating,
before the Supreme Court. Dry Lubricant and Anti-Seize Compound — are not consumer
products.

What is clear from the factual findings of the RTC and the
ISSUE: WON THE CHEMICAL COMPONENTS OR Court of Appeals is that the chemical formulation of
INGREDIENTS OF RESPONDENT'S PRODUCTS ARE respondent’s products is not known to the general public and
TRADE SECRETS OR INDUSTRIAL SECRETS THAT ARE is unique only to it. Both courts uniformly ruled that these
NOT SUBJECT TO COMPULSORY DISCLOSURE. ingredients are not within the knowledge of the public. Since
such factual findings are generally not reviewable by this
Court, it is not dutybound to analyze and weigh all over again
HELD: The products are covered by the exception of trade the evidence already considered in the proceedings below.
secrets being divulged in compulsory disclosure. The Court

Page 140 of 186


The revelation of respondent’s trade secrets serves no better J. RIGHT OF ASSOCIATION
purpose to the disposition of the main case pending with the
RTC, which is on the collection of a sum of money. As can be
gleaned from the facts, petitioner received respondent’s 1. Boy Scouts of America et al. v. Dale, 530 U.S. 640 (2000)
goods in trade in the normal course of business. To be sure,
there are defenses under the laws of contracts and sales
available to petitioner. On the other hand, the greater interest
FACTS: The Boy Scouts of America is a private, non-profit
of justice ought to favor respondent as the holder of trade
organization engaged in instilling its system of values in young
secrets. Weighing the conflicting interests between the people. At the time of the case, it asserted
parties, SC rules in favor of the greater interest of respondent. that homosexuality was inconsistent with those values.
Trade secrets should receive greater protection from
discovery, because they derive economic value from being When Dale was a student at Rutgers University, he became
generally unknown and not readily ascertainable by the public. co-president of the Lesbian/Gay student alliance. In July 1990,
he attended a seminar on the health needs of lesbian and gay
teenagers, where he was interviewed. An account of the
interview was published and in a local newspaper and Dale
was quoted as stating he was gay. BSA officials read the
interview and expelled Dale from his position as
assistant Scoutmaster of a New Jersey troop.
Dale, an Eagle Scout, filed suit in the New Jersey Superior
Court, alleging, among other things, that the Boy Scouts had
violated the state statute prohibiting discrimination on the
basis of sexual orientation in places of public accommodation.
The New Jersey Supreme Court ruled against the Boy Scouts,
saying that they violated the State's public accommodations
law by revoking Dale's membership based on his
homosexuality.

Among other rulings, the court


(1) held that application of that law did not violate the Boy
Scouts' First Amendment right of expressive association
because Dale's inclusion would not significantly affect
members' ability to carry out their purposes;
(2) determined that New Jersey has a compelling interest in
eliminating the destructive consequences of discrimination
from society, and that its public accommodations law abridges
no more speech than is necessary to accomplish its purpose;
and
(3) held that Dale's reinstatement did not compel the Boy
Scouts to express any message.
The Boy Scouts appealed to the United States Supreme
Court, which granted certiorari to determine whether the
application of New Jersey's public accommodations law
violated the First Amendment.

ISSUE: WON BSA has a right to revoke the membership of


a gay assistant scoutmaster after he publicly announced his
sexual orientation.

HELD: YES. Chief Justice William Rehnquist's majority


opinion relied upon Roberts v. United States Jaycees, 468
U.S. 609, 622 (1984), in which the Supreme Court said:
"Consequently, we have long understood as implicit in the
right to engage in activities protected by the First Amendment
a corresponding right to associate with others in pursuit of a
wide variety of political, social, economic, educational,
religious, and cultural ends." This right, the Roberts decision
continues, is crucial in preventing the majority from imposing
its views on groups that would rather express other, perhaps
unpopular, ideas. Government actions that may
unconstitutionally burden this freedom may take many forms,
one of which is "intrusion into the internal structure or affairs
of an association" like a "regulation that forces the group to
accept members it does not desire". Forcing a group to accept

Page 141 of 186


certain members may impair the ability of the group to express 2. BANK OF THE PHILIPPINE ISLANDS v. BPI
those views, and only those views, that it intends to express. EMPLOYEES UNION-DAVAO CHAPTER-FEDERATION
Thus, "freedom of association ... plainly presupposes a OF UNIONS IN BPI UNIBANK (G.R. No. 164301 : August
freedom not to associate." 10, 2010)
However, to determine whether a group is protected by the
First Amendment's expressive associational right, it must first FACTS: On March 23, 2000, the BSP approved the Articles
be determined whether the group engages in "expressive of Merger executed 2000 by and between BPI, herein
association." After reviewing the Scout Oath and Scout petitioner, and FEBTC.5 This Article and Plan of Merger was
Law the court decided that the general mission of the Boy approved by the Securities and Exchange Commission.
Scouts is clear—it is "to instill values in young people". The
Boy Scouts seek to instill these values by having its adult Pursuant to the Article and Plan of Merger, all the assets and
leaders spend time with the youth members, instructing and liabilities of FEBTC were transferred to and absorbed by BPI
engaging them in activities like camping, fishing, etc. During as the surviving corporation. FEBTC employees, including
the time spent with the youth members, the Scoutmasters and those in its different branches across the country, were hired
assistant Scoutmasters inculcate them with the Boy Scouts' by petitioner as its own employees, with their status and
tenure recognized and salaries and benefits maintained.
values—both expressly and by example. An association that
seeks to transmit such a system of values engages in
expressive activity. Respondent BPI Employees Union-Davao Chapter -
Federation of Unions in BPI Unibank is the exclusive
bargaining agent of BPI’s rank and file employees in Davao
 First, associations do not have to associate for the City. The former FEBTC rank-and-file employees in Davao
"purpose" of disseminating a certain message in order to City did not belong to any labor union at the time of the merger.
be entitled to the protections of the First Amendment. An Prior to the effectivity of the merger, respondent Union invited
association must merely engage in expressive activity said FEBTC employees to a meeting regarding the Union
Shop Clause (Article II, Section 2) of the existing CBA
that could be impaired in order to be entitled to protection.
between petitioner BPI and respondent Union.7
 Second, even if the Boy Scouts discourages Scout
leaders from disseminating views on sexual issues, the After the meeting called by the Union, some of the former
First Amendment protects the Boy Scouts' method of FEBTC employees joined the Union, while others refused.
expression. If the Boy Scouts wishes Scout leaders to Later, however, some of those who initially joined retracted
avoid questions of sexuality and teach only by example, their membership.9
this fact does not negate the sincerity of its belief
discussed above. Respondent Union then sent notices to the former FEBTC
employees who refused to join, as well as those who retracted
 Regarding whether the Boy Scouts as a whole had an
their membership, and called them to a hearing regarding the
expressive policy against homosexuality, the Court gave matter. When these former FEBTC employees refused to
deference to the organization's own assertions of the attend the hearing, the president of the Union requested BPI
nature of its expressions, as well as what would impair to implement the Union Shop Clause of the CBA and to
them. The Boy Scouts asserts that it "teach[es] that terminate their employment pursuant thereto.10
homosexual conduct is not morally straight", and that it
does "not want to promote homosexual conduct as a After two months of management inaction on the request,
legitimate form of behavior".While the policy may not respondent Union informed petitioner BPI of its decision to
represent the views of all Boy Scouts, the First refer the issue of the implementation of the Union Shop
Clause of the CBA to the Grievance Committee. However, the
Amendment "does not require that every member of a
issue remained unresolved at this level and so it was
group agree on every issue in order for the group's policy subsequently submitted for voluntary arbitration by the
to be expressive association." The Court deemed it parties.11
sufficient that the Boy Scouts had taken an official
position with respect to same-sex relationships. The After two months of management inaction on the request,
presence of an openly gay activist in an assistant respondent Union informed petitioner BPI of its decision to
Scoutmaster's uniform sends a distinctly different refer the issue of the implementation of the Union Shop
message from the presence of a heterosexual assistant Clause of the CBA to the Grievance Committee. However, the
Scoutmaster who is on record as disagreeing with Boy issue remained unresolved at this level and so it was
subsequently submitted for voluntary arbitration by the parties.
Scouts policy. The Boy Scouts has a First Amendment
right to choose to send one message but not the other.
The Voluntary Arbitrator concluded that the former FEBTC
The fact that the organization does not trumpet its views
employees could not be compelled to join the Union, as it was
from the housetops, or that it tolerates dissent within its their constitutional right to join or not to join any organization.
ranks, does not mean that its views receive no First
Amendment protection.
CA reversed the decision.

ISSUE: WON “absorbed” FEBTC employees fell within the


definition of “new employees” under the Union Shop Clause.

Page 142 of 186


HELD: YES. All employees in the bargaining unit covered by Right of an Employee not to Join a Union is not Absolute
a Union Shop Clause in their CBA with management are and Must Give Way to the Collective Good of All Members
subject to its terms. However, under law and jurisprudence, of the Bargaining Unit
the following kinds of employees are exempted from its
coverage, namely, employees who at the time the union shop The dissenting opinions place a premium on the fact that even
agreement takes effect are bona fide members of a religious if the former FEBTC employees are not old employees, they
organization which prohibits its members from joining labor nonetheless were employed as regular and permanent
unions on religious grounds; employees already in the employees without a gap in their service. However, an
service and already members of a union other than the employee’s permanent and regular employment status in itself
majority at the time the union shop agreement took does not necessarily exempt him from the coverage of a union
effect;22 confidential employees who are excluded from the shop clause.
rank and file bargaining unit;23 and employees excluded
from the union shop by express terms of the agreement.
In the past this Court has upheld even the more stringent type
of union security clause, i.e., the closed shop provision, and
When certain employees are obliged to join a particular union held that it can be made applicable to old employees who are
as a requisite for continued employment, as in the case of already regular and permanent but have chosen not to join a
Union Security Clauses, this condition is a valid restriction of union. In the early case of Juat v. Court of Industrial
the freedom or right not to join any labor organization because Relations,38 the Court held that an old employee who had no
it is in favor of unionism. This Court, on occasion, has even union may be compelled to join the union even if the collective
held that a union security clause in a CBA is not a restriction bargaining agreement (CBA) imposing the closed shop
of the right of freedom of association guaranteed by the provision was only entered into seven years after of the hiring
Constitution.24 of the said employee. To quote from that decision:

Moreover, a closed shop agreement is an agreement whereby A closed-shop agreement has been considered as one form
an employer binds himself to hire only members of the of union security whereby only union members can be hired
contracting union who must continue to remain members in and workers must remain union members as a condition of
good standing to keep their jobs. It is "the most prized continued employment. The requirement for employees or
achievement of unionism." It adds membership and workers to become members of a union as a condition for
compulsory dues. By holding out to loyal members a promise employment redounds to the benefit and advantage of said
of employment in the closed shop, it wields group employees because by holding out to loyal members a
solidarity.25 promise of employment in the closed-shop the union wields
group solidarity. In fact, it is said that "the closed-shop contract
Indeed, the situation of the former FEBTC employees in this is the most prized achievement of unionism."
case clearly does not fall within the first three exceptions to
the application of the Union Shop Clause discussed earlier.
No allegation or evidence of religious exemption or prior
membership in another union or engagement as a confidential
employee was presented by both parties. The sole category
therefore in which petitioner may prove its claim is the fourth
recognized exception or whether the former FEBTC
employees are excluded by the express terms of the existing
CBA between petitioner and respondent.

To reiterate, petitioner insists that the term "new employees,"


as the same is used in the Union Shop Clause of the CBA at
issue, refers only to employees hired by BPI as non-regular
employees who later qualify for regular employment and
become regular employees, and not those who, as a legal
consequence of a merger, are allegedly automatically deemed
regular employees of BPI. However, the CBA does not make
a distinction as to how a regular employee attains such a
status. Moreover, there is nothing in the Corporation Law and
the merger agreement mandating the automatic employment
as regular employees by the surviving corporation in the
merger.

In sum, this Court finds it reasonable and just to conclude that


the Union Shop Clause of the CBA covers the former FEBTC
employees who were hired/employed by BPI during the
effectivity of the CBA in a manner which petitioner describes
as "absorption." A contrary appreciation of the facts of this
case would, undoubtedly, lead to an inequitable and very
volatile labor situation which this Court has consistently ruled
against.1avvphi1

In the case of former FEBTC employees who initially joined


the union but later withdrew their membership, there is even
greater reason for the union to request their dismissal from the
employer since the CBA also contained a Maintenance of
Membership Clause.

Page 143 of 186


K. EMINENT DOMAIN In an rem proceeding, condemnation acts upon the property.
After condemnation, the paramount title is in the public under
a new and independent title; thus, by giving notice to all
1. MANOTOK REALTY, INC. and MANOTOK ESTATE
claimants to a disputed title, condemnation proceedings
CORPORATION vs. CLT REALTY DEVELOPMENT, provide a judicial process for securing better title against all
CORPORATION (G.R. No. 123346; March 31, 2009) the world than may be obtained by voluntary conveyance."

FACTS: In 1992, CLT Realty Development Corporation (CLT) In annulling the Manotok titles, focus was laid on the alleged
sought to recover from Manotok Realty, Inc. (MRI) and defects of TCT No. 4211 issued in September of 1918.
Manotok Estate Corporation (MEC) (collectively, Manotoks) However, TCT No. 4211 was issued decades before the
the possession of Lot 26 of the Maysilo Estate in an action property was expropriated. Thus, any and all defects that may
filed before the RTC of Caloocan City. have attended that particular title would have been purged
when the property covered by it was subsequently acquired
CLT claims it is the owner of Lot 26 covered by TCT No. T- by the State through eminent domain.
177013 issued in its name by the Caloocan City Register of
Deeds, which title was derived from Estelita Hipolito. Hipolito’s The majority report focused on the alleged flaws and inherent
title in turn emanated from Jose Dimson, the registered owner technical defects of TCT Nos. 4211, 5261 and 35486, ranging
of TCT No. 15166, the latter having acquired the same by from the language of the technical descriptions, absence of
virtue of a Court Order in 1966 issued by the court in a Civil subdivision plan, lot number and survey plan. The imputed
Case. Dimson’s title appears to have been sourced from OCT flaws affect only those certificates of title issued prior to those
No. 994 registered in the name of the Republic. Remarkably, no
specific flaw was found on the MANOTOKS' titles indicating
Manotoks claimed that Dimson’s title, the proximate source of any irregularity on their issuance.
CLT’s title, was irregularly issued and, hence, the same and
subsequent titles flowing therefrom are likewise void. As it is, the validity of most of MRI’s certificates of title should
be upheld because they were derived from the Republic’s
Like CLT, the Manotoks likewise traced its title to OCT No. valid certificates of title. In fact, some of the MANOTOKS’ titles
994. TCT No. 4210, which cancelled OCT No. 994, had been can be traced back to the Government’s titles as a result of
issued to Alejandro Ruiz and Mariano Leuterio who had the expropriation in 1947.
previously acquired the property in 1918 by virtue of an
“Escritura de Venta” executed by Don Tomas Arguelles and Relevantly, the titles of the Republic, as the predecessor-in-
Don Enrique Lopes. In 1920, Ruiz and Leuterio sold the interest of the MANOTOKS, are presumed valid by virtue of
property to Francisco Gonzalez who held title thereto until their acquisition resulting from the exercise of its inherent
1938 when the property was subdivided amongst the power of eminent domain that need not be granted even by
Gonzalez children. the fundamental law. Thus, the alleged flaws concerning the
certificates of title issued previous to the exercise of the State
The properties covered by said seven certificates of title (TCT of its inherent power did not affect or render invalid the
Nos. 1368-1374) were expropriated by the Republic of the subsequent transfers after the forced sale. Indeed, when land
Philippines. These properties were then later subdivided by has been acquired for public use in fee simple unconditionally,
the National Housing Authority [NHA] into 77 lots and either by the exercise of eminent domain or by purchase, the
thereafter sold to qualified vendees. A number of said former owner retains no rights in the land, and the public use
vendees sold 19 of these lots to Manotok Realty, Inc. while 1 may be abandoned, or the land may be devoted to a different
lot was purchased by the Manotok Estate Corporation. use, without any impairment of the estate or title acquired or
any reversion to the former owner.
The trial court, found in favor of CLT by adopting the factual
findings arrived at by the majority commissioners appointed to
resolve the conflict of titles. It was found that that there were
inherent technical infirmities or defects on the face of TCT No.
4211 (4210), from which the Manotoks derived their titles

The Court of Appeals affirmed the decision of the RTC. Hence,


the Manotoks filed a petition for review with the Supreme
Court.

ISSUE: What are the titles acquired by the Government and


whether any of the parties is able to trace its title to the title
acquired by the Government through expropriation.

HELD: Titles acquired by the State by way of expropriation


are deemed cleansed of whatever previous flaws may have
attended these titles.

As mentioned above, the properties covered by TCT Nos.


1368-1374 were expropriated by the Republic of the
Philippines and were eventually subdivided and sold to
various vendees. The fact of expropriation is extremely
significant, for titles acquired by the State by way of
expropriation are deemed cleansed of whatever previous
flaws may have attended these titles.

Page 144 of 186


2. Hacienda Luisita, Inc. (HLI), petitioner, versus HELD:
Presidential Agrarian Reform Council (PARC);
Secretary Nasser Pangandaman of the Department A. The Court stressed that “just compensation has been
of Agrarian Reform (DAR); Alyansa ng mga defined as the full and fair equivalent of the property taken
Manggagawang Bukid ng Hacienda Luisita from its owner by the expropriator. The measure is not the
(AMBALA), Rene Galang, Noel Mallari, and Julio takers gain, but the owner’s loss. Hence, in determining
Suniga and his Supervisory Group of the HLI and just compensation, the price or value of the property at
Windsor Andaya, respondents. (G.R. No. 171101) the time it was taken from the owner and appropriated by
the government shall be the basis. If the government
takes possession of the land before the institution of
expropriation proceedings, the value should be fixed as
FACTS: On July 5, 2011, the Supreme Court en banc voted of the time of the taking of said possession, not of the
unanimously (11-0) to dismiss the petition filed by HLI and filing of the complaint.”
affirm with modifications the resolutions of the PARC revoking
HLI’s Stock Distribution Plan (SDP) and placing the subject The SC, citing Land Bank of the Philippines v. Livioc, said that
lands in Hacienda Luisita under compulsory coverage of the taking is when the landowner was deprived of the use and
Comprehensive Agrarian Reform Program (CARP) of the benefit of his property, such as when the title is transferred to
government. the Republic. It also noted that taking also occurs when
agricultural lands are voluntarily offered by a landowner and
The Court however did not order outright land distribution. approved by PARC for CARP coverage through the stock
Voting 6-5, the Court noted that there are operative facts that distribution scheme, as in the case of HLI earlier decided.
occurred in the interim and which the Court cannot validly Thus, HLI submitting its SDP for approval is an
ignore. Thus, the Court declared that the revocation of the acknowledgment on its part that the agricultural lands of
SDP must, by application of the operative fact principle, give Hacienda Luisita are covered by CARP. However, the PARC
way to the right of the original 6,296 qualified farmworkers- approval should be considered as the effective date of taking
beneficiaries (FWBs) to choose whether they want to remain because it was only during that time that the government
as HLI stockholders or [choose actual land distribution]. It thus officially confirmed the CARP coverage of these lands.
ordered the Department of Agrarian Reform (DAR) to
“immediately schedule meetings with the said 6,296 FWBs Accordingly, Stock distribution and compulsory acquisition are
and explain to them the effects, consequences and legal or two modalities sharing the same end goal of having a more
practical implications of their choice, after which the FWBs will equitable distribution of land ownership, without ignoring such
be asked to manifest, in secret voting, their choices in the right to just compensation. Also, since it is only upon the
ballot, signing their signatures or placing their thumbmarks, as approval of the SDP that the agricultural lands actually came
the case may be, over their printed names.” under CARP coverage, such approval operates and takes the
place of a notice of coverage ordinarily issued under
On November 22, 2011, the Court recalled and set aside the compulsory acquisition.
option to remain as stockholders of HLI, while maintaining that
all benefits received shall be respected with no obligation to What the SC found notable, however, is that the divestment
refund or return them. by Tadeco of the agricultural lands of Hacienda Luisita and the
giving of the shares of stock for free is nothing but an
On December 9, 2011, a Motion for enticement or incentive for the FWBs to agree with the stock
Reconsideration/Clarification by private respondents Mallari, distribution option scheme and not further push for land
Suniga, Supervisory Group of HLI, and Andaya (Mallari, et al distribution. And the stubborn fact is that the “man days”
scheme of HLI impelled the FWBs to work in the hacienda in
exchange for such shares of stock.
On December 16, 2011, a Motion to Clarify and Reconsider
Resolution of November 22, 2011 was filed by HLI.
The Court ruled that taking only when the landowner is
deprived of the use and benefit of his property is not
ISSUES: incompatible with the earlier conclusion that taking took place
on November 21, 1989, and since even from the start,
A. Whether or not SC erred in determining just compensation TADECO seemed to already favour Stock Distribution
by considering the date of taking as November 21, 1989 when Scheme when complying with the CARP when it organized the
PARC approved the SDP (already revoked) since the Notice HLI as its spin-off corporation which facilitated stock
of Coverage of January 2, 2006 may be considered as time acquisition of FWBs. Tadeco assigned and conveyed
FWBs owned and possess the agricultural lands of Hacienda 4,915.75 has to HLI the agricultural lands of Hacienda Luisita.
Luisita because it was the only time when the latter was placed These agricultural lands constituted as the capital contribution
under Compulsory Acquisition in view of failure to perform of the FWBs in HLI. This, in effect, deprived TADECO itself of
their obligations under the SDP, or SDOA, when the owner is the ownership over these lands when it transferred the same
ACTUALLY deprived or dispossessed of his property, and to HLI.
considering taking from November 21, 1989 is a deprivation
of landowner’s property WITHOUT due process of law; and When the agricultural lands of Hacienda Luisita were
HLI is entitled to be paid interest on the just compensation. transferred by Tadeco to HLI in order to comply with CARP
through the stock distribution option scheme under PARC
B. Whether or not Just Compensation for the Homelots be Resolution No. 89-12-2 dated November 21, 1989, Tadeco
given to FWBs as it does not form part of the 4,915.75 was consequently dispossessed of the ownership of the
hectares covered by the SDP, and hence, the value of these same.
homelots should, with the revocation of the SDP, be paid to
Tadeco as the landowner. Furthermore, adherence to the suggestion of HLI that the
Notice of Coverage issued on January 2, 2006 should be
considered as date of taking would in effect penalize the
qualified FWBs twice for acceding to the Stock Distribution

Page 145 of 186


Scheme, (1) depriving them of the agricultural lands they 3. SECRETARY OF THE DEPARTMENT OF PUBLIC
should have gotten earlier, if it were not for this SDP and (2) WORKS AND HIGHWAYS and DISTRICT ENGINEER
making them pay higher amortization for the agricultural lands CELESTINO R. CONTRERAS vs. SPOUSES
that should have been given to them decades ago. HERACLEO and RAMONA TECSON, (2013
DECISION)
The SC maintained that, as it has in fact already ruled on its
reckoning date, that is, November 21, 1989, the date of FACTS: Respondent spouses Heracleo and Ramona Tecson
issuance of PARC Resolution No. 89-12-2, based on the (respondents) are co-owners of a parcel of land with an area
above-mentioned disquisitions. of 7,268 square meters located in San Pablo, Malolos,
Bulacan and covered by Transfer Certificate of Title (TCT) No.
On side note, the SC added that “even though the T-43006of the Register of Deeds of Bulacan. Said parcel of
compensation due to HLI will still be preliminarily determined land was among the properties taken by the government
by DAR and LBP, subject to review by the RTC acting as a sometime in 1940 without the owners consent and without the
SAC, the fact that the reckoning point of taking is already fixed necessary expropriation proceedings and used for the
at a certain date should already hasten the proceedings and construction of the MacArthur Highway.
not further cause undue hardship on the parties, especially the
qualified FWBs.” In 1940, the Department of Public Works and Highways
(DPWH) took respondents-movants' subject property without
the benefit of expropriation proceedings for the construction of
the MacArthur Highway. In a letter dated December 15,
1994,respondents-movants demanded the payment of the fair
B. The court ruled in the negative. As reiterated in the earlier market value of the subject parcel of land. Celestino R.
decision, the distribution of homelots is required under RA Contreras (Contreras), then District Engineer of the First
6657 only for corporations or business associations owning or Bulacan Engineering District of the DPWH, offered to pay for
operating farms which opted for land distribution. the subject land at the rate of Seventy Centavos (P0.70) per
Corporations are not obliged to provide for homelots. square meter, per Resolution of the Provincial Appraisal
Nonetheless, HLI undertook to subdivide and allocate for free Committee (PAC) of Bulacan. Unsatisfied with the offer,
and without charge among the qualified family-beneficiaries respondents-movants demanded the return of their property,
240 sq. m. of homelots to some, if not all of the qualified or the payment of compensation at the current fair market
beneficiaries. value. Hence, the complaint for recovery of possession with
damages filed by respondents-movants. Respondents-
The Supreme Court, by a unanimous vote, resolved to movants were able to obtain favorable decisions in the
maintain its ruling that the FWBs shall retain ownership of the Regional Trial Court (RTC) and the Court of Appeals (CA),
homelots given to them with no obligation to pay for the value with the subject property valued at One Thousand Five
of said lots. Also, since the SDP was already revoked with Hundred Pesos (₱1,500.00) per square meter, with interest at
finality in th earlier discussion of the decision, the Court directs six percent (6%) per annum. Aggrieved, petitioners come
the government through the DAR to pay HLI the just before the Court assailing the CA decision.
compensation for said homelots in consonance with Sec. 4,
Article XIII of the 1987 Constitution that the taking of land for
use in the agrarian reform program is subject to the payment
of just compensation.
ISSUE: Whether or not the just compensation should be
based on the value of the property at the time of taking in 1940
The Motions of both parties were DENIED with qualification. and not at the time of payment?
The July 5, 2011, Decision was modified by the November 21,
2011 Resolution which ordered the government, through the
DAR, to pay just compensation for the 240 sq. m. homelots
distributed to FWBs. This RESOLUTION is now declared
FINAL and EXECUTORY. HELD: Court of Appeals decision is modified. The instant case
stemmed from an action for recovery of possession with
damages filed by respondents against petitioners. It, however,
revolves around the taking of the subject lot by petitioners for
the construction of the MacArthur Highway. There is taking
when the expropriator enters private property not only for a
momentary period but for a permanent duration, or for the
purpose of devoting the property to public use in such a
manner as to oust the owner and deprive him of all beneficial
enjoyment thereof.

When a property is taken by the government for public use,


jurisprudence clearly provides for the remedies available to a
landowner. The owner may recover his property if its return is
feasible or, if it is not, the aggrieved owner may demand
payment of just compensation for the land taken.For failure of
respondents to question the lack of expropriation proceedings
for a long period of time, they are deemed to have waived and
are estopped from assailing the power of the government to
expropriate or the public use for which the power was
exercised. What is left to respondents is the right of
compensation.The trial and appellate courts found that
respondents are entitled to compensation. The only issue left
for determination is the propriety of the amount awarded to
respondents.

Page 146 of 186


Just compensation is "the fair value of the property as 2015 DECISION
between one who receives, and one who desires to sell, fixed
at the time of the actual taking by the government." This rule For resolution is the Motion for Reconsideration1 filed by
holds true when the property is taken before the filing of an respondents-movants spouses Heracleo and Ramona
expropriation suit, and even if it is the property owner who Tecson imploring the Court to take a second look at its July 1,
brings the action for compensation. 2013 Decision, the dispositive portion of which reads:

The Court in the several cases was confronted with common WHEREFORE, premises considered, the petition is
factual circumstances where the government took control and PARTIALLY GRANTED. The Court of Appeals Decision dated
possession of the subject properties for public use without July 31, 2007 in CAG.R. CV No. 77997 is MODIFIED, in that
initiating expropriation proceedings and without payment of the valuation of the subject property owned by respondents
just compensation, while the landowners failed for a long shall be P0.70 instead of ₱1,500.00 per square meter, with
period of time to question such government act and later interest at six percent (6%) per annum from the date of taking
instituted actions for recovery of possession with damages. in 1940 instead of March 17, 1995, until full payment.
The Court thus determined the landowners right to the
payment of just compensation and, more importantly, the
amount of just compensation. The Court has uniformly ruled In view of the contrasting opinions of the members of the Third
that just compensation is the value of the property at the time Division on the instant motion, and the transcendental
of taking that is controlling for purposes of compensation As importance of the issue raised herein, the members of the
in said cases, just compensation due respondents in this case Third Division opted to refer the issue to the En Banc for
should, therefore, be fixed not as of the time of payment but resolution.
at the time of taking, that is, in 1940.

The reason for the rule has been clearly explained in Republic
v. Lara, et al.,and repeatedly held by the Court in recent cases, ISSUE: Whether or not the taking of private property without
thus: due process should be nullified?

"The value of the property should be fixed as of the date when


it was taken and not the date of the filing of the proceedings."
For where property is taken ahead of the filing of the HELD: No. The government’s failure to initiate the necessary
condemnation proceedings, the value thereof may be expropriation proceedings prior to actual taking cannot simply
enhanced by the public purpose for which it is taken; the entry invalidate the State’s exercise of its eminent domain power,
by the plaintiff upon the property may have depreciated its given that the property subject of expropriation is indubitably
value thereby; or, there may have been a natural increase in devoted for public use, and public policy imposes upon the
the value of the property from the time it is taken to the time public utility the obligation to continue its services to the public.
the complaint is filed, due to general economic conditions. The To hastily nullify said expropriation in the guise of lack of due
owner of private property should be compensated only for process would certainly diminish or weaken one of the State’s
what he actually loses; it is not intended that his compensation inherent powers, the ultimate objective of which is to serve the
shall extend beyond his loss or injury. And what he loses is greater good. Thus, the non-filing of the case for expropriation
only the actual value of his property at the time it is taken. will not necessarily lead to the return of the property to the
landowner. What is left to the landowner is the right of
Both the RTC and the CA recognized that the fair market value compensation.
of the subject property in 1940 wasP0.70/sq m. Hence, it
should, therefore, be used in determining the amount due While it may appear inequitable to the private owners to
respondents instead of the higher value which isP1,500.00. receive an outdated valuation, the long-established rule is that
While disparity in the above amounts is obvious and may the fair equivalent of a property should be computed not at the
appear inequitable to respondents as they would be receiving time of payment, but at the time of taking. This is because the
such outdated valuation after a very long period, it is equally purpose of ‘just compensation’ is not to reward the owner for
true that they too are remiss in guarding against the cruel the property taken but to compensate him for the loss thereof.
effects of belated claim. The concept of just compensation The owner should be compensated only for what he actually
does not imply fairness to the property owner alone. loses, and what he loses is the actual value of the property at
Compensation must be just not only to the property owner, but the time it is taken. The Court must adhere to the doctrine that
also to the public which ultimately bears the cost of its first and fundamental duty is the application of the law
expropriation. according to its express terms, interpretation being called for
only when such literal application is impossible. To entertain
Clearly, petitioners had been occupying the subject property other formula for computing just compensation, contrary to
for more than fifty years without the benefit of expropriation those established by law and jurisprudence, would open
proceedings. In taking respondents property without the varying interpretation of economic policies – a matter which
benefit of expropriation proceedings and without payment of this Court has no competence to take cognizance of. Equity
just compensation, petitioners clearly acted in utter disregard and equitable principles only come into full play when a gap
of respondents proprietary rights which cannot be exists in the law and jurisprudence.
countenanced by the Court.For said illegal taking,
respondents are entitled to adequate compensation in the
form of actual or compensatory damages which in this case
should be the legal interest of six percent (6%) per annum on
the value of the land at the time of taking in 1940 until full Velasco Dissent:
payment. This is based on the principle that interest runs as a
matter of law and follows from the right of the landowner to be The State’s power of eminent domain is not absolute; the
placed in as good position as money can accomplish, as of Constitution is clear that no person shall be deprived of life,
the date of taking. liberty and property without due process of law. As such,
failure of the government to institute the necessary
proceedings should lead to failure of taking an individual’s

Page 147 of 186


property. In this case, since the property was already taken, 4. PATRICIA I. TIONGSON VS NATIONAL HOUSING
the complainants must be equitably compensated for the loss AUTHORITY (558 SCRA 56 (2008)
thereof.
FACTS:
For purposes of “just” compensation, the value of the land
should be determined from the time the property owners filed The present Petition for Review on Certiorari raises the
the initiatory complaint, earning interest therefrom. To hold question of from what date should just compensation of the
otherwise would validate the State’s act as one of subject properties sought to be expropriated be reckoned -
expropriation in spite of procedural infirmities which, in turn, whether it is from the taking of the property or on the filing of
would amount to unjust enrichment on its part. To continue the complaint.
condoning such acts would be licensing the government to
continue dispensing with constitutional requirements in taking
private property. Respondent National Housing Authority (NHA) took
possession in 1978, for purposes of expropriation, of
properties belonging to petitioners Patricia L. Tiongson, et al.
pursuant to P.D. Nos. 1669 and 1670. "An Act Providing for
the Expropriation of the Property Known as the 'Tambunting
Estate' Registered Under TCT Nos. 119059, 122450, 122459,
122452 And Lot Nos. 1-A, 1-C, 1-D, 1-E, 1-F, 1-G And 1-H Of
(LRC) PSD-230517 (Previously Covered By TCT No. 119058)
of the Register of Deeds of Manila and for The Sale at Cost of
the Lots Therein to the Bona Fide Occupants and Other
Squatters Families and to Upgrade the Same, and Authorizing
the Appropriation of Funds For The Purpose" (underscoring
supplied), and of properties belonging to Patricia Tiongson, et
al. pursuant to P.D. No. 1670, "An Act Providing For The
Expropriation of the Property Along the Estero De Sunog-
Apog Formerly Consisting of Lots Nos. 55-A, 55-B And 55-C,
Block 2918 of the Subdivision Plan Psd-11746, Covered by
TCT Nos. 49286, 49287 and 49288, Respectively, of the
Register of Deeds of Manila and for The Sale at Cost of the
Lots Therein to the Bona Fide Occupants and Other Squatter
Families and to Upgrade The Same, and Authorizing The
Appropriation of Funds For The Purpose". The two P.D.’s
were thereafter declared unconstitutional by the Supreme
Court they being violative of the therein petitioners' right to due
process of law. On September 14, 1987, the NHA filed before
the Regional Trial Court (RTC) a complaint against Tiongson,
et al. for expropriation of parcels of land which were covered
by P.D. Nos. 1669 and 1670.

The RTC held that the determination of just compensation of


the properties should be reckoned from the date of filing of
NHA’s petition or on September 14, 1987. However, on
appeal, the Court of Appeals reversed and set aside the trial
court’s orders and held that the just compensation should be
based on the actual taking of the property in 1978. Hence, this
petition.

ISSUE:

Whether or not just compensation should be reckoned from


the time of the taking of the property or on the filing of the
complaint

HELD:

In several aforementioned cases, in a situation where a


government agency, in this case the National Housing
Authority, took possession of properties belonging to private
individuals for purposes of expropriation and the laws by virtue
of which such government agency expropriated the subject
properties were subsequently declared to be unconstitutional
by the Supreme Court, the determination of just compensation
should be reckoned from the date of filing the complaint for
expropriation and not from the time of actual taking of the
properties.

In declaring, in its challenged Decision, that the determination


of just compensation should be reckoned from NHA’s taking
of the properties in 1978, the appellate court simply relied on
Annex ―C of NHA’s petition before it, the Order dated June

Page 148 of 186


15, 1988 of the then Presiding Judge of the trial court, and 5. REPUBLIC OF THE PHILIPPINES THROUGH THE
thus concluded that ―the parties admitted that [NHA] took DEPARTMENT OF PUBLIC WORKS AND HIGHWAYS
possession of the subject properties as early as 1978. The vs. COURT OF APPEALS and ROSARIO
appellate court reached that conclusion, despite its recital of RODRIGUEZ REYES CARPIO, J.)
the antecedents of the case including Tiongson, sustained
moves, even before the trial court, in maintaining that the FACTS: Private respondent Rosario Rodriguez Reyes is the
reckoning of just compensation should be from the date of absolute owner of a parcel of land identified as Lot 849-B and
filing of the petition for expropriation on September 14, 1987. covered by TCT No. T-7194. The 1,043-square meter lot is
situated on Claro M. Recto and Osmeña Streets, Cagayan de
The earlier-quoted allegations of the body and prayer in NHA’s Oro City.
Petition for Expropriation filed before the RTC constitute
judicial admissions of NHA—that it possessed the subject On 6 November 1990, private respondent received a letter
properties until this Court’s declaration, in its above-stated from petitioner Republic of the Philippines, through the
Decision in G.R. No. L-55166 promulgated on May 21, 1987, Department of Public Works and Highways (DPWH),
that P.D. No. 1669 pursuant to which NHA took possession of requesting permission to enter into a portion of private
the properties of petitioners in 1978 was unconstitutional and, respondent’s lot consisting of 663 square meters, and to begin
therefore, null and void. These admissions, the appellate court construction of the Osmeña Street extension road.
either unwittingly failed to consider or escaped its notice.
On 20 December 1990, petitioner took possession of private
Following then Rule 67, Section 4 of the Rules of Court respondent’s property without initiating expropriation
reading: proceedings. Consequently, on 4 and 7 January 1991, private
respondent sent letters to the DPWH stating her objection to
SEC. 4. Order of expropriation. - If the objections to and the the taking of her property. Private respondent sent a letter to
defenses against the right of the plaintiff to expropriate the the City Appraisal Committee (CAC) rejecting the latter’s
property are overruled, or when no party appears to defend as appraisal of the subject property. In the same letter, private
required by this Rule, the court may issue an order of respondent requested the City Assessor for a reappraisal of
expropriation declaring that the plaintiff has a lawful right to her property, but said request was denied.
take the property sought to be expropriated, for the public use
or purpose described in the complaint, upon the payment of On 17 March 1992, private respondent filed with the Regional
just compensation to be determined as of the date of the Trial Court (RTC) of Cagayan de Oro City a complaint claiming
taking of the property or the filing of the complaint, whichever just compensation and damages against petitioner.
came first.
On 30 June 1993, the RTC appointed three commissioners to
determine the subject property’s fair market value, as well as
the consequential benefits and damages of its expropriation.
The RTC deemed it just, fair and reasonable to adopt the
market value of FOUR THOUSAND PESOS (₱4,000.00) per
square meter as the highest price obtaining and prevailing in
1990, the time of the taking of the property.

On 13 April 1994, the scheduled hearing was reset to 19 May


1994, to give private respondent (plaintiff) time to consider the
offer of petitioner (defendant) to amicably settle the case and
to accept the just compensation of ₱3,200 per square meter,
or a total of ₱2,212,600, for the 663-square meter portion of
private respondent’s lot. On 16 May 1994, private respondent
filed with the RTC an "Urgent Motion to Deposit The Amount
of ₱2,121,600 in Court. RTC ordered the commissioners to
submit their report as soon as possible, but until the scheduled
hearing on 15 July 1994, the commissioners still failed to
submit their report. Upon motion of private respondent, the
RTC issued an order appointing a new set of commissioners.

On 11 October 1994, the new commissioners submitted their


report, the pertinent facts are as follows:

Taking into consideration, among others, the location of the


property and a research of the prevailing prices of lots
proximate to and/or near the vicinity of plaintiff's property, the
undersigned Commissioners respectfully recommend to the
Honorable Court the following valuation, to wit:

1. Front portion along Recto Avenue with a measurement of


21.52 meters from south to north with an area of 347.66
square meters at ₱18,000.00 to ₱20,000.00 per square meter;

2. Middle portion with a measurement of 21.52 meters


containing an area of 347.66 square meters at ₱16,000.00 to
₱18,000.00 per square meter;

Page 149 of 186


3. Rear/back portion measuring 21.52 meters with an area of to determine the consequential damages for the remaining
347.66 square meters at ₱14,000.00 to ₱16,000.00 per 297-square meter lot
square meter;
Eminent domain is the authority and right of the State, as
On the other hand, the remaining portion left to the plaintiff, sovereign, to take private property for public use upon
Lot No. 849-B-3 will not actually be 297 square meters. If we observance of due process of law and payment ofjust
deduct the setback area from Osmeña Extension Street, the compensation. The Constitution provides that, "[p]rivate
usable/buildable area left to the plaintiff would only be a little property shall not be taken for public use without just
over 50 square meters. This portion would not command a compensation."
good price if sold. Neither is it ideal for purposes of any
building construction because aside from its being a very Just compensation is the full and fair equivalent of the property
small strip of land, the shape is triangular. sought to be expropriated. Among the factors to be considered
in arriving at the fair market value of the property are the cost
On 2 June 1995, the RTC rendered a decision in favor of the of acquisition, the current value of like properties, its actual or
plaintiff and against the defendants, declaring the former as potential uses, and in the particular case of lands, their size,
having the right to retain 590 square meters of the property shape, location, and the tax declarations thereon. The
covered by TCT No. T-7194, and ordering the latter to return measure is not the taker’s gain but the owner’s loss. To be
293 square meters of the 746 square meters taken. Plaintiff just, the compensation must be fair not only to the owner but
herein is ordered to forthwith defray the expenses to be also to the taker.
incurred in undertaking the road construction of the 293
square meters which the defendants will later on provide along Just compensation is based on the price or value of the
the right portion of her property. property at the time it was taken from the owner and
appropriated by the government. However, if the government
On appeal by petitioner, the Court of Appeals rendered takes possession before the institution of expropriation
judgment affirming with modifications the decision of the RTC. proceedings, the value should be fixed as of the time of the
The Court of Appeals found that the commissioners’ taking of said possession, not of the filing of the complaint.
recommendations on just compensation were not supported The value at the time of the filing of the complaint should be
by valid documents. Also, it was unclear in the RTC decision the basis for the determination of the value when the taking of
whether the trial court merely adopted the commissioners’ the property involved coincides with or is subsequent to the
recommendations or the court made its own independent commencement of the proceedings.
valuation of the subject property. Thus, the Court of Appeals
held that a reconvening of the commissioners or an However, we held in Republic v. Court of Appeals that Rule
appointment of new commissioners to determine just 67 presupposes a prior filing of complaint for eminent domain
compensation was necessary. The appellate court further held with the appropriate court by the expropriator. If no such
that the trial court’s order for petitioner’s return of the 293- complaint is filed, the expropriator is considered to have
square meter lot had no legal basis and was no longer feasible violated procedural requirements, and hence, waived the
since the lot was already part of the completed Sergio usual procedure prescribed in Rule 67, including the
Osmeña extension road. Moreover, consequential damages appointment of commissioners to ascertain just
should be awarded in lieu of actual damages for private compensation.
respondent’s alleged loss of income from the remaining 297-
square meter lot.
In National Power Corporation v. Court of Appeals, we
clarified that when there is no action for expropriation and the
The case is remanded to the trial court which is ordered to case involves only a complaint for damages or just
reconvene the commissioners or appoint new commissioners compensation, the provisions of the Rules of Court on
to determine, in accordance with this Decision, the amount of ascertainment of just compensation (i.e., provisions of Rule
just compensation due to plaintiff-appellee Rosario Rodriguez 67) are no longer applicable, and a trial before commissioners
Reyes for the 746 square meters of land taken from her and is dispensable.
consequential damages to the 297-square meter portion left.
Hence, this appeal.
In this case, petitioner took possession of the subject property
without initiating expropriation proceedings. Consequently,
private respondent filed the instant case for just compensation
and damages. To determine just compensation, the trial court
ISSUES: appointed three commissioners pursuant to Section 5 of Rule
67 of the 1997 Rules of Civil Procedure. None of the parties
A. Whether the Court of Appeals erred in ordering the remand objected to such appointment.
of the case to the trial court, to order the reconvening of the
commissioners or appointment of new commissioners to The trial court’s appointment of commissioners in this
determine the consequential damages for the remaining 297- particular case is not improper. The appointment was done
square meter lot; and mainly to aid the trial court in determining just compensation,
and it was not opposed by the parties. Besides, the trial court
B. Whether or not the Court erred in awarding consequential is not bound by the commissioners’ recommended valuation
damages. of the subject property. The court has the discretion on
whether to adopt the commissioners’ valuation or to substitute
its own estimate of the value as gathered from the records.

However, we agree with the appellate court that the trial


HELD: court’s decision is not clear as to its basis for ascertaining just
compensation. The trial court mentioned in its decision the
A. On whether the Court of Appeals erred in ordering the valuations in the reports of the City Appraisal Committee and
remand of the case to the trial court to order the reconvening of the commissioners appointed pursuant to Rule 67. But
of the commissioners or appointment of new commissioners whether the trial court considered these valuations in arriving

Page 150 of 186


at the just compensation, or the court made its own him." The principle of unjust enrichment under Article 22
independent valuation based on the records, was obscure in requires two conditions: (1) that a person is benefited without
the decision. The trial court simply gave the total amount of a valid basis or justification, and (2) that such benefit is derived
just compensation due to the property owner without laying at another’s expense or damage.39 There is no unjust
down its basis. Thus, there is no way to determine whether the enrichment when the person who will benefit has a valid claim
adjudged just compensation is based on competent evidence. to such benefit. As stated, consequential damages are
awarded if as a result of the expropriation, the remaining
For this reason alone, a remand of the case to the trial court property of the owner suffers from an impairment or decrease
for proper determination of just compensation is in order. In in value. Thus, there is a valid basis for the grant of
National Power Corporation v. Bongbong, we held that consequential damages to the property owner, and no unjust
although the determination of just compensation lies within the enrichment can result therefrom.
trial court’s discretion, it should not be done arbitrarily or
capriciously. The decision of the trial court must be based on
all established rules, correct legal principles, and competent
evidence. The court is proscribed from basing its judgment on
speculations and surmises.

B. Petitioner contends that no consequential damages may be


awarded as the remaining lot was "not actually taken" by the
DPWH, and to award consequential damages for the lot which
was retained by the owner is tantamount to unjust enrichment
on the part of the latter. Petitioner’s contention is
unmeritorious.

No actual taking of the remaining portion of the real property


is necessary to grant consequential damages. If as a result of
the expropriation made by petitioner, the remaining lot (i.e.,
the 297-square meter lot) of private respondent suffers from
an impairment or decrease in value, consequential damages
may be awarded to private respondent. On the other hand, if
the expropriation results to benefits to the remaining lot of
private respondent, these consequential benefits36 may be
deducted from the awarded consequential damages, if any, or
from the market value of the expropriated property. We held
in B.H. Berkenkotter & Co. v. Court of Appeals that:

To determine just compensation, the trial court should first


ascertain the market value of the property, to which should be
added the consequential damages after deducting therefrom
the consequential benefits which may arise from the
expropriation. If the consequential benefits exceed the
consequential damages, these items should be disregarded
altogether as the basic value of the property should be paid in
every case.

Section 6 of Rule 67 of the Rules of Civil Procedure provides:

x x x The commissioners shall assess the


consequential damages to the property not taken
and deduct from such consequential damages the
consequential benefits to be derived by the owner
from the public use or purpose of the property taken,
the operation of its franchise by the corporation or the
carrying on of the business of the corporation or
person taking the property. But in no case shall the
consequential benefits assessed exceed the
consequential damages assessed, or the owner be
deprived of the actual value of his property so taken.

An award of consequential damages for property not taken is


not tantamount to unjust enrichment of the property owner.
There is unjust enrichment "when a person unjustly retains a
benefit to the loss of another, or when a person retains money
or property of another against the fundamental principles of
justice, equity and good conscience."38 Article 22 of the Civil
Code provides that "[e]very person who through an act of
performance by another, or any other means, acquires or
comes into possession of something at the expense of the
latter without just or legal ground, shall return the same to

Page 151 of 186


6. NATIONAL POWER CORPORATION vs. APOLONIO payment of which is only 10% of the market-value of the
V. MARASIGAN, FRANCISCO V. MARASIGAN, LILIA properties. The CA held that payment of the full value was
V. MARASIGAN, BENITO V. MARASIGAN, JR., and Just Compensation. It was also held that NPC failed to allege
ALICIA V. MARASIGAN, (November 20, 2017 Tijam the issue of taking in its complaint nor was it raised during pre-
J) trial or even proved during trial. The CA denied their appeal
and affirmed the RTC decision.
Facts: NPC filed an expropriation committee against
Marasigan in order to construct Steel Transmission Lines and
Wooden electric poles. NPC sought an easement of right of
way over the subject properties. Based on the tax declaration ISSUE: Whether or not the reckoning point of the market
which classified the properties as agricultural and based on value of the properties was during the taking of the property
the BIR zoning valuations, NPC offered to pay Php
299,550.50

Marasigan did not object to the expropriation but opposed the


classification since they allege that it is classified as industrial, HELD: The court ruled in the affirmative. The general rule is
commercial, and residential since 1993. They then claim Php that it should be reckoned at the time of actual taking when it
47,064,400 for the affected properties. As counterclaim, they preceded the filing of the complaint. Under Rule 67, Sec 4; the
also seek payment of consequential damages for the areas in value of just compensation is to be determined as of the date
between the transmission lines (dangling portions). RTC of the taking of the property or the filing of the complaint,
issued an Expropriation order and fixed the value of the whichever came first. In NTC v. Oroville: it was settled that just
properties at Php 47,064,400 which was deposited by NPC compensation should be reckoned from the date of actual
with Landbank. RTC then issued a writ of possession in favor taking when such preceded the filing of the complaint for
of NPC . An appraisal committee was formed by the RTC for expropriation. The State is only obliged to make good the loss
purposes of determining just compensation. A reverse trial sustained by the landowner and considering the
thereafter ensued. circumstances availing at the time the property was taken In
some cases, there were deviations from the general rule due
to special circumstances as it necessitated a valuation at the
Marasigan presented the Chairman of the appraisal time the landowners initiated proceedings after the taking of
committee who testified that the appraisal committee the properties.
recommended the total valuation of PhP 49,064,400 based on
the assessor's data and the BIR zonal valuations as indicated
on the 1997 tax declarations. Also presented was the In the case at bar, NPC insists that it took the properties in the
succeeding Chairman of the appraisal committee who testified 1970s despite initiating the complaint only on Jan 23, 2006
that the properties suffered consequential damages which the which is why they allege that it should be reckoned in the
appraisal committee recommended to be computed at 50% of 1970s. NPC's action relative to the acquisition of an easement
the BIR zonal value per square meter or for a total amount of of right-of-way made prior to the filing of its expropriation
PhP 22,227,800. complaint was limited only to the conduct of negotiations with
respondents. The negotiations pertained to the construction
of transmission lines not among the transmission lines subject
On ocular inspection, the appraisal committee found that the of the complaint. This was alleged by NPC and was testified
existence of the transmission lines hampered the properties' to by their right-of-way officer. There being no sufficient proof
potential use such that while the areas before and after the that NPC actually took the subject properties at a date
transmission lines could still be used, the areas it between preceding the filing of the expropriation complaint, the time of
could no longer be utilized. The appraisal committee also the taking should be taken to mean as coinciding with the
noted that the transmission lines produced considerable noise commencement of the expropriation proceedings on January
making the area unsuitable for residential purposes. NPC, 23, 2006.
presented its right-of-way officers whose testimonies sought
to establish that the lots being claimed by Marasigan as
dangling areas were classified as agricultural under the tax The determination of just compensation in expropriation cases
declarations and that NPC negotiated with Marasigan, and is a function addressed to the discretion of the courts owing to
that NPC took the properties between 1996 and 1998. NPC’s the constitutional mandate that no private property shall be
officers alleged that the dangling areas could still be used for taken for public use without payment of just compensation. As
agricultural purposes but agreed that the lines may endanger such, legislative enactments, as well as executive issuances,
people and animals. fixing or providing for the method of computing just
compensation are tantamount to impermissible encroachment
on judicial prerogatives. As such, they are not binding on
On cross-examination, the right of way officer admitted that courts and are treated as mere guidelines in ascertaining the
the properties were classified as agro industrial as stated in amount of just compensation The tax declarations presented
the 1998 tax declarations. He admitted that the classification by NPC is only of of the several factors which the court may
of the properties as agricultural which was used as basis for consider to facilitate the determination of just compensation
computing its value was erroneous. as they enjoy judicial discretion to determine the classification
of lands. The court's discretion in classifying the expropriated
RTC affirmed the recommendation of the appraisal committee land is only for the purpose of determining just compensation
for payment of the Just compensation in the amount of and is not meant to substitute that of the local government's
47,064,400 based on the BIR zonal valuation as of the time of power to reclassify and convert lands through local ordinance
the filing of the complaint on Jan 23, 2006. It also affirmed that
payment of the consequential damages for the properties that
were rendered useless. RTC rejected NPC’s claim that it took
the properties in 1972 when NPC was allegedly allowed by
Marasigan to construct the lines. NPC’s MR was denied by
RTC but the court made modifications by including interest
(6%, 12%) On their appeal to the CA, they alleged that the
award was based on the premise that it sought to acquire
ownership when it merely sought to acquire right-of-way; the

Page 152 of 186


7. REPUBLIC OF THE PHILIPPINES V. LIM (G.R. No. Petitioner, through the OSG, filed with the SC a petition for
161656, June 29, 2005) review alleging that they remain as the owner of Lot 932.

FACTS: In 1938, the Republic instituted a special civil action


for expropriation of a land in Lahug, Cebu City for the purpose
of establishing a military reservation for the Philippine Army. ISSUE: Whether the Republic has retained ownership of Lot
The said lots were registered in the name of Gervasia and 932 despite its failure to pay respondent’s predecessors-in-
Eulalia Denzon. The Republic deposited P9,500 in the PNB interest the just compensation pursuant to the judgment of the
then took possession of the lots. Thereafter, on May 1940, the CFI rendered as early as May 14, 1940.
CFI rendered its Decision ordering the Republic to pay the
Denzons the sum of P4,062.10 as just compensation. The
Denzons appealed to the CA but it was dismissed on March
11, 1948. An entry of judgment was made on April 5, 1948.
HELD: No.
In 1950, one of the heirs of the Denzons, filed with the National
Airports Corporation a claim for rentals for the two lots, but it No person shall be deprived of his private property without due
"denied knowledge of the matter." On September 6, 1961, Lt. process of law; and in expropriation cases, an essential
Cabal rejected the claim but expressed willingness to pay the element of due process is that there must be just
appraised value of the lots within a reasonable time. compensation whenever private property is taken for public
use. Accordingly, Section 9, Article III, of our Constitution
For failure of the Republic to pay for the lots, on September mandates: “Private property shall not be taken for public use
20, 1961, the Denzons· successors-in-interest,Valdehueza without just compensation.”
and Panerio, filed with the same CFI an action for recovery of
possession with damages against the Republic and AFP The Republic disregarded the foregoing provision when it
officers in possession of the property. failed and refused to pay respondent’s predecessors-in-
interest the just compensation. The length of time and the
On November 1961, Titles of the said lots were issued in the manner with which it evaded payment demonstrate its
names of Valdehueza and Panerio with the annotation arbitrary highhandedness and confiscatory attitude. More than
"subject to the priority of the National Airports Corporation to half of a century has passed, yet, to this day, the landowner,
acquire said parcels of land, Lots 932 and939 upon previous now respondent, has remained empty-handed. Undoubtedly,
payment of a reasonable market value". over 50 years of delayed payment cannot, in any way, be
viewed as fair. This is more so when such delay is
accompanied by bureaucratic hassles.
On July 1962, the CFI promulgated its Decision in favor of
Valdehueza and Panerio, holding that they are the owners and
have retained their right as such over lots because of the The Court of Appeals is correct in saying that Republic’s delay
Republic’s failure to pay the amount of P4,062.10, adjudged is contrary to the rules of fair play, as “just compensation
in the expropriation proceedings. However, in view of the embraces not only the correct determination of the amount to
annotation on their land titles, they were ordered to execute a be paid to the owners of the land, but also the payment for the
deed of sale in favor of the Republic. land within a reasonable time from its taking. Without prompt
payment, compensation cannot be considered ‘just.’” In
jurisdictions similar to ours, where an entry to the expropriated
They appealed the CFI·s decision to the SC. The latter held property precedes the payment of compensation, it has been
that Valdehueza and Panerio are still the registered owners of held that if the compensation is not paid in a reasonable time,
Lots 932 and 939, there having been no payment of just the party may be treated as a trespasser ab initio.
compensation by the Republic. SC still ruled that they are not
entitled to recover possession of the lots but may only demand
the payment of their fair market value. While it is true that all private properties are subject to the
need of government, and the government may take them
whenever the necessity or the exigency of the occasion
Meanwhile, in 1964, Valdehueza and Panerio mortgaged Lot demands, however, the Constitution guarantees that when
932 to Vicente Lim, herein respondent, as security for their this governmental right of expropriation is exercised, it shall
loans. For their failure to pay Lim despite demand, he had the be attended by compensation. From the taking of private
mortgage foreclosed in 1976. The lot title was issued in his property by the government under the power of eminent
name. domain, there arises an implied promise to compensate the
owner for his loss. Significantly, the above-mentioned
On 1992, respondent Lim filed a complaint for quieting of title provision of Section 9, Article III of the Constitution is not a
with the RTC against the petitioners herein. On 2001, the RTC grant but a limitation of power. This limiting function is in
rendered a decision in favor of Lim, declaring that he is the keeping with the philosophy of the Bill of Rights against the
absolute and exclusive owner of the lot with all the rights of an arbitrary exercise of governmental powers to the detriment of
absolute owner including the right to possession. Petitioners the individual’s rights. Given this function, the provision should
elevated the case to the CA. In its Decision dated September therefore be strictly interpreted against the expropriator, the
18, 2003, it sustained the RTC Decision saying: ´... This is government, and liberally in favor of the property owner.
contrary to the rules of fair play because the concept of just
compensation embraces not only the correct determination of The recognized rule is that title to the property expropriated
the amount to be paid to the owners of the land,but also the shall pass from the owner to the expropriator only upon full
payment for the land within a reasonable time from its taking. payment of the just compensation. Although the right to
Without prompt payment, compensation cannot be considered appropriate and use land is complete at the time of entry, title
"just"...” to the property taken remains in the owner until payment is
actually made. Clearly, without full payment of just
compensation, there can be no transfer of title from the
landowner to the expropriator. Otherwise stated, the
Republic’s acquisition of ownership is conditioned upon the
full payment of just compensation within a reasonable time.

Page 153 of 186


Of course the Court is aware of the doctrine that “nonpayment 8. LAND BANK OF THE PHILIPPINES v. EUGENIO
of just compensation (in an expropriation proceedings) does DALAUTA (G.R. No. 190004, August 08, 2017)
not entitle the private landowners to recover possession of the
expropriated lots.” However, the facts of the present case do FACTS: Eugenio Dalauta (Dalauta) was the registered owner
not justify its application. It bears stressing that the Republic of an agricultural land in Florida, Butuan City, with an area of
was ordered to pay just compensation twice, the first was in 25.2160 hectares and covered by Transfer Certificate of Title
the expropriation proceedings and the second, in Valdehueza. (TCT) No. T-1624. The land was placed by the Department of
Fifty-seven (57) years have passed since then. We cannot but Agrarian Reform (DAR) under compulsory acquisition of the
construe the Republic’s failure to pay just compensation as a Comprehensive Agrarian Reform Program (CARP) as
deliberate refusal on its part. Under such circumstance, reflected in the Notice of Coverage, dated January 17, 1994,
recovery of possession is in order. In several jurisdictions, the which Dalauta received on February 7, 1994. Petitioner Land
courts held that recovery of possession may be had when Bank of the Philippines (LBP) offered P192,782.59 as
property has been wrongfully taken or is wrongfully retained compensation for the land, but Dalauta rejected such
by one claiming to act under the power of eminent domain or valuation for being too low. The case was referred to the DAR
where a rightful entry is made and the party condemning Adjudication Board (DARAB) through the Provincial Agrarian
refuses to pay the compensation which has been assessed or Reform Adjudicator (PARAD) of Butuan City. A summary
agreed upon; or fails or refuses to have the compensation administrative proceeding was conducted to determine the
assessed and paid. appropriate just compensation for the subject property. In its
Resolution,5 dated December 4, 1995, the PARAD affirmed
It must be emphasized that an individual cannot be deprived the valuation made by LBP in the amount of P192,782.59. On
of his property for the public convenience. One of the basic February 28, 2000, Dalauta filed a petition for determination
principles of the democratic system is that where the rights of of just compensation with the RTC, sitting as SAC. He alleged
the individual are concerned, the end does not justify the that LBP's valuation of the land was inconsistent with the rules
means. It is not enough that there be a valid objective; it is and regulations prescribed in DAR Administrative Order (A.O.)
also necessary that the means employed to pursue it be in No. 06, series of 1992, for determining the just compensation
keeping with the Constitution. Mere expediency will not of lands covered by CARP's compulsory acquisition scheme.
excuse constitutional shortcuts. There is no question that not
even the strongest moral conviction or the most urgent public Dalauta argued that the valuation of his land should be
need, subject only to a few notable exceptions, will excuse the determined using the formula in DAR A.O. No. 6, series of
bypassing of an individual's rights. It is no exaggeration to say 1992, which was Land Value (LV) = Capitalized Net Income
that a person invoking a right guaranteed under Article III of (CNI) x 0.9 + Market Value (MV) per tax declaration x 0.1, as
the Constitution is a majority of one even as against the rest he had a net income of 350,000.00 in 1993 from the sale of
of the nation who would deny him that right. the trees that were grown on the said land. LBP argued that
the valuation of Dalauta's land should be determined using the
Besides, practically speaking, the reversion of Lot 932 to formula LV= MV x 2, which yielded a total value of
respondent will only affect a handful of military personnel. It P192,782.59 for the 25.2160 hectares of Dalauta's land. LBP
will not result to “irreparable damage” or “damage beyond claimed that during the ocular inspection/investigation, only 36
pecuniary estimation,” as what the Republic vehemently coconut trees existed on the subject land; that three (3)
claims.In summation, while the prevailing doctrine is that “the hectares of it were planted with com; and the rest was idle with
nonpayment of just compensation does not entitle the private few second-growth trees.
landowner to recover possession of the expropriated lots,
however, in cases where the government failed to pay just The SAC explained its decision in this wise:
compensation within five (5) years from the finality of the
judgment in the expropriation proceedings, the owners
concerned shall have the right to recover possession of their Going over the records of this case, taking into consideration
property. This is in consonance with the principle that “the the Commissioners Report which is replete with pictures of the
government cannot keep the property and dishonor the improvements introduced which pictures are admitted into
judgment.” To be sure, the five-year period limitation will evidence not as illustrated testimony of a human witness but
encourage the government to pay just compensation as probative evidence in itself of what it shows (Basic
punctually. This is in keeping with justice and equity. After all, Evidence, Bautista, 2004 Edition), this Court is of the
it is the duty of the government, whenever it takes property considered view that the Report (Commissioners) must be
from private persons against their will, to facilitate the payment given weight.
of just compensation. We defined just compensation as not
only the correct determination of the amount to be paid to the While LBP's witness Ruben P. Penaso may have gone to the
property owner but also the payment of the property within a area, but he did not, at least, list down the improvements. The
reasonable time. Without prompt payment, compensation members of the Board of Commissioners on the other hand,
cannot be considered “just.” went into the area, surveyed its metes and bounds and listed
the improvements they found including the farmhouse made
of wood with galvanized iron roofing (Annex "C",
Commissioner's Report, p. 132, Record)

All told, the basic formula for the valuation of lands covered by
Voluntary Offer to Sell and Compulsory Acquisition is:

LV = (CNI X 0.6) + (CS X 0.3) + (MV X 0.1)

Where: LV = Land Value

CNI = Capitalized Net Income

CS = Comparable Sales

Page 154 of 186


MV = Market Value per Tax Declaration The Capitalized Net Income (CNI) approach to land
valuation assumes that there would be uniform
The above formula is used if all the three (3) factors are streams of future income that would be realized in
present, relevant and applicable. In any case, the resulting perpetuity from the seasonal/permanent crops
figure in the equation is always multiplied to the number of planted to the land. In the case of commercial trees
area or hectarage of land valued for just compensation. (hardwood and soft wood species), however, only a
one-time income is realized when the trees are due
for harvest. The regular CNI approach in the
Whenever one of the factors in the general formula is not valuation of lands planted to commercial trees would
available, the computation of land value will be any of the three therefore not apply.
(3) computations or formulae:
Dalauta's sale of falcata trees indeed appears to be a one-
LV (CNI x 0.9) + (MVx 0.1) (If the comparable sales time transaction. He did not claim to have derived any other
factor is missing) income from the property prior to receiving the Notice of
Coverage from the DAR in February 1994. For this reason, his
LV (CS x 0.9) + (MV x 0.1) (If the capitalize net property would be more appropriately covered by the formula
income is unavailable) provided under JMC No. 11 (2003).

LV = MV x 2 (If only the market value factor is Applying the above formula to compute just compensation for
available) respondent's land would be the most equitable course of
action under the circumstances. Without JMC No. 11 (2003),
Since the Capitalized Net Income in this case is available, the Dalauta's property would have to be valued using the formula
formula to be used is: for idle lands, the CNI and CS factors not being applicable.
Following this formula, just compensation for Dalauta's
property would only amount to 225,300.00, computed as
LV = (CNI x 0.9) + (MV x 0.1) follows:chanRoblesvirtualLawlibrary

Whence: LV = MV x 2

LV = (P350,000.00/.12 x 0.9) + (P145,570 x 0.1) Where:

= (P2,916,666.67 x 0.9) + (P145,557.00) [sic] LV = Land Value

= P2,625,000.00 + P14,557.00 MV = Market Value per Tax Declaration*

= P2,639,557.00 plus P100,000.00 for the Farmhouse • For the area planted to com, P7,740.00/hectare

The CA sustained the valuation by the SAC for being well • For idle/pasture land, P3,890/hectare
within R.A. No. 6657, its implementing rules and regulations,
and in accordance with settled jurisprudence. The factors laid
down under Section 17 of R.A. No. 6657, which were Thus:
translated into a basic formula in DAR A.O. No. 6, series of
1992, were used in determining the value of Dalauta's For the 4 hectares planted to corn:
property. It stated that the courts were not at liberty to
disregard the formula which was devised to implement LV = (P7,740/hectare x 4 hectares) x 2
Section 17 of R.A. No. 6657. The CA, however, disagreed with
the SAC's valuation of the farmhouse, which was made of
wood and galvanized iron, for it was inexistent during the = P61,920.00
taking of the subject land – (1) the compensation for the
farmhouse (P100,000.00), as well as the awards for attorney's For the 21 hectares of idle/pasture land:
fees (P150,000.00) and litigation expenses (P50,000.00), are
hereby DELETED LV = (P3,890/hectare x 21) x 2

= P163,380.00

ISSUE: Whether or not the trial court correctly computed the Total Land Value = P61,920.00 + P163,380.00
just compensation of the subject property.
= P225,300.00

As above stated, the amount would be more equitable if it


HELD: Upon an assiduous assessment of the different would be computed pursuant to JMC No. 11 (2003).
valuations arrived at by the DAR, the SAC and the CA, the Moreover, the award shall earn legal interest. Pursuant to
Court agrees with the position of Justice Francis Jardeleza Nacar v. Gallery Frames, the interest shall be computed from
that just compensation for respondent Dalauta's land should the time of taking at the rate of twelve percent (12%) per
be computed based on the formula provided under DAR-LBP annum until June 30, 2013. Thereafter, the rate shall be six
Joint Memorandum Circular No. 11, series of 2003 (JMC No. percent (6%) per annum until fully paid.
11 (2003)). This Memorandum Circular, which provides for the
specific guidelines for properties with standing commercial
trees, explains:

Page 155 of 186


9. Mactan-Cebu International Airport Authority vs. judgment would violate the property owner’s right to justice,
Lozada, Sr. (G.R. No. 176625 February 25, 2010 fairness, and equity.
Nachura,)
In light of these premises, we now expressly hold that the
DOCTRINE: It is well settled that the taking of private property taking of private property, consequent to the Governments
by the Governments power of eminent domain is subject to exercise of its power of eminent domain, is always subject to
two mandatory requirements: (1) that it is for a particular public the condition that the property be devoted to the specific public
purpose; and (2) that just compensation be paid to the purpose for which it was taken. Corollarily, if this particular
property owner. These requirements partake of the nature of purpose or intent is not initiated or not at all pursued, and is
implied conditions that should be complied with to enable the peremptorily abandoned, then the former owners, if they so
condemnor to keep the property expropriated. desire, may seek the reversion of the property, subject to the
return of the amount of just compensation received. In such a
case, the exercise of the power of eminent domain has
become improper for lack of the required factual justification.•
The right of respondents to repurchase Lot No. 88 may be
FACTS: Subject of this case is a lot (Lot 88) located in Lahug, enforced based on a constructive trust constituted on the
Cebu City. Its original owner was Anastacio Deiparine when property held by the government in favor of the former.
the same was subject to expropriation proceedings, initiated
by the Republic, represented by the then Civil Aeronautics
Administration (CAA), for the expansion and improvement of  In constructive trusts, the arrangement is temporary and
the Lahug Airport. During the pendency of the expropriation passive in which the trustees sole duty is to transfer the
proceedings, respondent Lozada acquired Lot 88 from title and possession over the property to the plaintiff-
Deiparine. The trial court ruled for the Republic and ordered beneficiary.
the Deiparine to pay Lozada the fair market value of the land.  Of course, the wronged party seeking the aid of a court of
equity in establishing a constructive trust must himself do
equity. Accordingly, the court will exercise its discretion in
However, the projected improvement and expansion plan of deciding what acts are required of the plaintiff-beneficiary
the old Lahug Airport was not pursued. The plaintiff- as conditions precedent to obtaining such decree and has
respondents initiated a complaint for the recovery of the obligation to reimburse the trustee the consideration
possession and conveyance of ownership on the subject lot. received from the latter just as the plaintiff-beneficiary
On the other hand, petitioners asked for the immediate would if he proceeded on the theory of rescission. In the
dismissal of the complaint. They specifically denied that the good judgment of the court, the trustee may also be paid
Government had made assurances to reconvey Lot 88 to the necessary expenses he may have incurred in
respondents in the event that the property would no longer be sustaining the property, his fixed costs for improvements
needed for airport operations. Petitioners instead asserted thereon, and the monetary value of his services in
that the judgment of condemnation was unconditional, and managing the property to the extent that plaintiff-
respondents were, therefore, not entitled to recover the beneficiary will secure a benefit from his acts.
expropriated property notwithstanding non-use or
abandonment thereof. The trial court ruled for herein plaintiff-
respondents, which decision was affirmed by the CA.

ISSUE: Whether or not there is a valid exercise of power of


eminent domain.

HELD: No. The acquisition by the Republic of the expropriated


lots was subject to the condition that the Lahug Airport would
continue its operation. The condition not having materialized
because the airport had been abandoned, the former owner
should then be allowed to reacquire the expropriated property.
It is well settled that the taking of private property by the
Governments power of eminent domain is subject to two
mandatory requirements: (1) that it is for a particular public
purpose; and (2) that just compensation be paid to the
property owner. These requirements partake of the nature of
implied conditions that should be complied with to enable the
condemnor to keep the property expropriated.

More particularly, with respect to the element of public use,


the expropriator should commit to use the property pursuant
to the purpose stated in the petition for expropriation filed,
failing which, it should file another petition for the new
purpose. If not, it is then incumbent upon the expropriator to
return the said property to its private owner, if the latter desires
to reacquire the same. Otherwise, the judgment of
expropriation suffers an intrinsic flaw, as it would lack one
indispensable element for the proper exercise of the power of
eminent domain, namely, the particular public purpose for
which the property will be devoted. Accordingly, the private
property owner would be denied due process of law, and the

Page 156 of 186


10. Vda de Ouano vs. Republic (G.R. No. 168770 upon a state of affairs that was not conceived nor
February 9, 2011 Velasco, Jr., J.) contemplated when the expropriation was authorized. In
effect, the government merely held the properties condemned
DOCTRINE: A condemnor should commit to use the property in trust until the proposed public use or purpose for which the
pursuant to the purpose stated in the petition for expropriation, lots were condemned was actually consummated by the
failing which it should file another petition for the new purpose. government. Since the government failed to perform the
If not, then it behooves the condemnor to return the said obligation that is the basis of the transfer of the property, then
property to its private owner, if the latter so desires. The the lot owners Ouanos and Inocians can demand the
government cannot plausibly keep the property it expropriated reconveyance of their old properties after the payment of the
in any manner it pleases and, in the process, dishonor the condemnation price.Constructive trusts are fictions of equity
judgment of expropriation. that courts Respondents demanded hat NAPOCOR pay
damages use as devices to remedy any situation in which the
holder of the legal title, MCIAA in this case, may not, in good
conscience, retain the beneficial interest. We add, however,
as in Heirs of Moreno, that the party seeking the aid of equity
FACTS: In 1949, the National Airport Corporation (NAC), the landowners in this instance, in establishing the trust must
MCIAAs predecessor agency, pursued a program to expand himself do equity in a manner as the court may deem just and
the Lahug Airport in Cebu City. Through its team of reasonable.
negotiators, NAC met and negotiated with the owners of the
properties situated around the airport, which included Lot Nos. Expropriation is forced private property taking, the landowner
744-A, 745-A, 746, 747, 761-A, 762-A, 763-A, 942, and 947 being really without a ghost of a chance to defeat the case of
of the Banilad Estate. As the landowners would later claim, the the expropriating agency. In other words, in expropriation, the
government negotiating team, as a sweetener, assured them private owner is deprived of property against his will. Withal,
that they could repurchase their respective lands should the the mandatory requirement of due process ought to be strictly
Lahug Airport expansion project do not push through or once followed, such that the state must show, at the minimum, a
the Lahug Airport closes or its operations transferred to genuine need, an exacting public purpose to take private
Mactan-Cebu Airport. Some of the landowners accepted the property, the purpose to be specifically alleged or least
assurance and executed deeds of sale with a right of reasonably deducible from the complaint.
repurchase. Others, however, including the owners of the
aforementioned lots, refused to sell because the purchase
price offered was viewed as way below market, forcing the
hand of the Republic, represented by the then Civil
Aeronautics Administration (CAA), as successor agency of the
NAC, to file a complaint for the expropriation of Lot Nos. 744-
A, 745A, 746, 747, 761-A, 762-A, 763-A, 942, and 947, among
others, docketed as Civil Case No. R-1881 entitled Republic
v. Damian Ouano, et al.

Soon after the MCIAA jettisoned the Lahug Airport expansion


project, informal settlers entered and occupied Lot No. 763-A
which, before its expropriation, belonged to the Ouanos. The
Ouanos then formally asked to be allowed to exercise their
right to repurchase the aforementioned lot, but the MCIAA
ignored the demand. On August 18, 1997, the Ouanos
instituted a complaint before the Cebu City RTC against the
Republic and the MCIAA for reconveyance.

Answering, the Republic and MCIAA averred that the Ouanos


no longer have enforceable rights whatsoever over the
condemned Lot No. 763-A, the decision in Civil Case No. R-
1881 not having found any reversionary condition.

ISSUE: Whether or not former owners of lots acquired for the


expansion of the Lahug Airport in Cebu City have the right to
repurchase or secure reconveyance of their respective
properties.

HELD: Yes. Providing added support to the Ouanos and the


Inocians right to repurchase is what in Heirs of Moreno was
referred to as constructive trust, one that is akin to the implied
trust expressed in Art. 1454 of the Civil Code, the purpose of
which is to prevent unjust enrichment. In the case at bench,
the Ouanos and the Inocians parted with their respective lots
in favor of the MCIAA, the latter obliging itself to use the
realties for the expansion of Lahug Airport; failing to keep its
end of the bargain, MCIAA can be compelled by the former
landowners to reconvey the parcels of land to them,
otherwise, they would be denied the use of their properties

Page 157 of 186


11. REPUBLIC OF THE PHILIPPINES, represented by the meters of the subject real property at the rate of ₱550.00 per
NATIONAL POWER CORPORATION vs. HEIRS OF square meter and to pay legal interest therefrom until fully
SATURNINO Q. BORBON, AND COURT OF paid.
APPEALS (G.R. No. 165354 ; January 12, 2015)
On January 3, 2014, NAPOCOR filed a Manifestation and
Motion to Discontinue Expropriation Proceedings, informing
that the parties failed to reach an amicable agreement; that
FACTS: In February 1993, NAPOCOR entered a property the property sought to be expropriated was no longer
located in Barangay San Isidro, Batangas City in order to necessary for public purpose because of the intervening
construct and maintain transmission lines for the 230 KV retirement of the transmission lines installed on the
Mahabang Parang-Pinamucan Power Transmission Project. respondents’ property; that because the public purpose for
Respondents heirs of Saturnino Q. Borbon owned the which such property would be used thereby ceased to exist,
property, with a total area of 14,257 square meters, which was the proceedings for expropriation should no longer continue,
registered under Transfer Certificate of Title No. T-9696 of the and the State was now duty-bound to return the property to its
Registry of Deeds of Batangas. owners; and that the dismissal or discontinuance of the
expropriation proceedings was in accordance with Section 4,
Rule 67 of the Rules of Court. Hence, NAPOCOR prayed that
On May 26, 1995, NAPOCOR filed a complaint for the proceedings be discontinued "under such terms as the
expropriation in the Regional Trial Court in Batangas City court deems just and equitable," and that the compensation to
(RTC), seeking the acquisition of an easement of right of way be awarded the respondents be reduced by the equivalent of
over a portion of the property involving an area of only 6,326 the benefit they received from the land during the time of its
square meters, more or less, alleging that it had negotiated occupation, for which purpose the case could be remanded to
with the respondents for the acquisition of the easement but the trial court for the determination of reasonable
they had failed to reach any agreement; and that, compensation to be paid to them.
nonetheless, it was willing to deposit the amount of ₱9,790.00
representing the assessed value of the portion sought to be
expropriated. It prayed for the issuance of a writ of possession
upon deposit to enable it to enter and take possession and
control of the affected portion of the property; to demolish all ISSUE: whether or not the expropriation proceedings should
improvements existing thereon; and to commence be discontinued or dismissed pending appeal
construction of the transmission line project.

In their answer with motion to dismiss, the respondents


staunchly maintained that NAPOCOR had not negotiated with HELD: The dismissal of the proceedings for expropriation at
them before entering the property and that the entry was done the instance of NAPOCOR is proper, but, conformably with
without their consent in the process, destroying some fruit Section 4, Rule 67 of the Rules of Court, the dismissal or
trees without payment, and installing five transmission line discontinuance of the proceedings must be upon such terms
posts and five woodpoles for its project; that the area being as the court deems just and equitable.
expropriated only covered the portion directly affected by the
transmission lines; that the remaining portion of the property
was also affected because the transmission line passed NAPOCOR entered the property without the owners’ consent
through the center of the land, thereby dividing the land into and without paying just compensation to the respondents.
three lots; that the presence of the high tension transmission Neither did it deposit any amount as required by law prior to
line had rendered the entire property inutile for any future use its entry. The Constitution is explicit in obliging the
and capabilities; that, nonetheless, they tendered no objection Government and its entities to pay just compensation before
to NAPOCOR’s entry provided it would pay just compensation depriving any person of his or her property for public use.
not only for the portion sought to be expropriated but for the Considering that in the process of installing transmission lines,
entire property whose potential was greatly diminished, if not NAPOCOR destroyed some fruit trees and plants without
totally lost, due to the project; and that their property was payment, and the installation of the transmission lines went
classified as industrial land. through the middle of the land as to divide the property into
three lots, thereby effectively rendering the entire property
inutile for any future use, it would be unfair for NAPOCOR not
In the judgment dated November 27, 2000, the RTC adopted to be made liable to the respondents for the disturbance of
the recommendation contained in the joint report, and ruled their property rights from the time of entry until the time of
thusly: restoration of the possession of the property. There should be
no question about the taking. There is a sufficient showing that
The price to be paid for an expropriated land is its NAPOCOR entered into and took possession of the
value at the time of taking, which is the date when respondents’ property as early as in March 1993 without the
the plaintiff actually entered the property or the date benefit of first filing a petition for eminent domain. For all
of the filing of the complaint for expropriation. In this intents and purposes, therefore, March 1993 is the reckoning
case, there is no evidence as to when the plaintiff point of NAPOCOR’s taking of the property, instead of May 5,
actually entered the property in question, so the 1995, the time NAPOCOR filed the petition for expropriation.
reference point should be the date of filing of the
complaint, which is May 5, 1995. In the context of the State's inherent power of eminent domain,
there is a "taking" when the owner is actually deprived or
Accordingly, the RTC ordered NAPOCOR to pay the dispossessed of his property; when there is a practical
respondents: (1) just compensation for the whole area of destruction or a material impairment of the value of his
14,257 square meters at the rate of ₱550.00/square meter; (2) property or when he is deprived of the ordinary use thereof.
legal rate of interest from May 5, 1995 until full payment; and There is a "taking" in this sense when the expropriator enters
(3) the costs of suit. private property not only for a momentary period but for a more
permanent duration, for the purpose of devoting the property
The CA made some modification on RTC’s ruling stating that to a public use in such a manner as to oust the owner and
plaintiff-appellant shall pay only for the occupied 6,326 square deprive him of all beneficial enjoyment thereof. For ownership,

Page 158 of 186


after all, "is nothing without the inherent rights of possession, 12. NATIONAL POWER CORPORATION vs. SPOUSES
control and enjoyment. Where the owner is deprived of the RODOLFO ZABALA and LILIA BAYLON (G.R. No.
ordinary and beneficial use of his property or of its value by its 173520; January 30, 2013)
being diverted to public use, there is taking within the
Constitutional sense." x x x. FACTS:

In view of the discontinuance of the proceedings and the On October 27, 1994, plaintiff-appellant National Power
eventual return of the property to the respondents, there is no Corporation (Napocor) filed a complaint for Eminent Domain
need to pay "just compensation" to them because their against defendants-appellees Sps. R. Zabala & L. Baylon,
property would not be taken by NAPOCOR. Instead of full before the RTC, Balanga City, Bataan alleging that Spouses
market value of the property, therefore, NAPOCOR should Zabala and Baylon own parcels of land located in Balanga
compensate the respondents for the disturbance of their City, Bataan and that it urgently needed an easement of right
property rights from the time of entry in March 1993 until the of way over the affected areas for its 230 KV Limay-Hermosa
time of restoration of the possession by paying to them actual Transmission Lines. The Commissioners submitted their
or other compensatory damages. Report/ Recommendation fixing the just compensation at
P150.00 per square meter. Napocor prayed that the report be
recommitted to the commissioners for the modification of the
In light of these premises, we now expressly hold that the report and the substantiation of the same with reliable and
taking of private property, consequent to the Government’s competent documentary evidence based on the value of the
exercise of its power of eminent domain, is always subject to property at the time of its taking. The Commissioners
the condition that the property be devoted to the specific public submitted their Final Report fixing the just compensation at
purpose for which it was taken. Corollarily, if this particular P500.00 per square meter.
purpose or intent is not initiated or not at all pursued, and is
peremptorily abandoned, then the former owners, if they so On June 28, 2004, the RTC rendered its Partial Decision and
desire, may seek the reversion of the property, subject to the ordered Napocor to pay Php150.00 per square meter for the
return of the amount of just compensation received. In such a 6,820 square meters determined as of the date of the taking
case, the exercise of the power of eminent domain has of the property.
become improper for lack of the required factual justification. Napocor appealed to the CA arguing that the Commissioners
This should mean that the compensation must be based on reports are not supported by documentary evidence. Napocor
what they actually lost as a result and by reason of their argued that the RTC did not apply Section 3A of R.A. No. 6395
dispossession of the property and of its use, including the which limits its liability to easement fee of not more than 10%
value of the fruit trees, plants and crops destroyed by of the market value of the property traversed by its
NAPOCOR’s construction of the transmission lines. transmission lines. CA affirmed the RTCs Partial Decision.
Considering that the dismissal of the expropriation
proceedings is a development occurring during the appeal, the ISSUE:
Court now treats the dismissal of the expropriation
Whether or not the RTC erred in fixing the amount of
proceedings as producing the effect of converting the case
Php150.00 per square meter as the fair market value of the
into an action for damages. For that purpose, the Court
property subject of the easement right of way of
remands the case to the court of origin for further proceedings,
Napocor?
with instruction to the court of origin to enable the parties to
fully litigate the action for damages by giving them the HELD:
opportunity to re-define the factual and legal issues by the
submission of the proper pleadings on the extent of the taking, The petition is partially meritorious. CONSTITUTIONAL
the value of the compensation to be paid to the respondents LAW: just compensation
by NAPOCOR, and other relevant matters as they deem fit. Sec. 3A of RA No. 6395 cannot restrict the constitutional
power of the courts to determine just compensation. The
payment of just compensation for private property taken for
public use is guaranteed no less by our Constitution and is
included in the Bill of Rights. As such, no legislative
enactments or executive issuances can prevent the courts
from determining whether the right of the property owners to
just compensation has been violated. It is a judicial function
that cannot be usurped by any other branch or official of the
government. Statutes and executive issuances fixing or
providing for the method of computing just compensation are
not binding on courts and, at best, are treated as mere
guidelines in ascertaining the amount thereof.
The Supreme Court has held in a long line of cases that since
the high- tension electric current passing through the
transmission lines will perpetually deprive the property owners
of the normal use of their land, it is only just and proper to
require Napocor to recompense them for the full market value
of their property.
The just compensation of P150.00 per square meter as fixed
by the RTC is not supported by evidence. Just compensation
cannot be arrived at arbitrarily. Several factors must be
considered, such as, but not limited to, acquisition cost,
current market value of like properties, tax value of the
condemned property, its size, shape, and location. But before
these factors can be considered and given weight, the same
must be supported by documentary evidence.

Page 159 of 186


Under Section 8, Rule 67 of the Rules of Court, the trial court 13. Cabahug v. National Power Corporation (698 SCRA
may accept or reject, whether in whole or in part, the 666), GR No. 186069, Jan 30, 2013
commissioners report which is merely advisory and
Facts:
recommendatory in character. It may also recommit the report
or set aside the same and appoint new commissioners. In this Spouses Cabahug are the owners of two parcels of land in
case, however, in spite of the insufficient and flawed reports Leyte. They were among the defendants in a Special Civil
of the Commissioners and Napocors objections thereto, the Action suit for expropriation earlier filed by NPC before the
RTC eventually adopted the same. It shrugged off Napocors RTC.
protestations and limited itself to the reports submitted by the
Commissioners. The suit was later dismissed when NPC opted to settle with
the landowners by paying an easement fee equivalent to 10%
Lastly, it should be borne in mind that just compensation of value of their property in Accordance with RA 6395, “An Act
should be computed based on the fair value of the subject Revising the Charter of the NPC”.
property at the time of its taking or the filing of the complaint,
whichever came first. Since in this case the filing of the Leyte Provincial Appraisal Committee fixed the valuation of
eminent domain case came ahead of the taking, just the affected properties at P45.00 per square meter. Jesus
compensation should be based on the fair market value of Cabahug executed two documents denominated as Right of
spouses Zabalas property at the time of the filing of Napocors Way Grant in favor of NPC. For and in consideration of the
Complaint on October 27, 1994 or thereabouts. easement fees Jesus Cabahug also granted NPC a
continuous easement of right of way for the latter’s
Petition is PARTIALLY GRANTED. Case is REMANDED to transmissions lines and their appurtenances.
the RTC for the proper determination of just
compensation. Jesus Cabahug agreed not to construct any building or
structure whatsoever, nor plant in any area within the Right of
Way that will adversely affect or obstruct the transmission line
of NPC, except agricultural crops, the growth of which will not
exceed three meters high.
Jesus Cabahug reserved the option to seek additional
compensation for easement fee, based on the Supreme
Court’s 18 January 1991 Decision.
Spouses Cabahug filed the complaint for the payment of just
compensation, damages, and attorney’s fees against NPC,
claiming to have been totally deprived of the use of the
portions of land covered by their TCTs.
NPC’s answer: they already paid full easement fee
RTC ruled for the Spouses Cabahug. NPC’s easement of right
of way which indefinitely deprives the owner of their propriety
rights over their property falls within the purview of the power
of eminent domain.
CA: reversed and set aside the RTC decision finding that the
RA 6395 only allows NPC to acquire an easement right of way
over properties traversed by its transmission lines. MR denied
for lack of merit.
Issue 1:
Whether or not CA reversibly erred in sustaining NPC’s
reliance on Section 3-A of RA 6395 which states that only 10%
of the market value of the property is due to the owner of the
property subject to an easement of right of way. (YES)
Held 1:
Since said easement falls within the purview of the power of
eminent domain, NPC’s utilization of said provision has been
repeatedly struck down by this Court in a number of cases.
The determination of just compensation in eminent domain
proceedings is a judicial function and no statute, decree, or
executive order can mandate that its own determination shall
prevail over the court’s findings. Any valuation for just
compensation laid down in the statutes may serve only as a
guiding principle or one of the factors in determining just
compensation, but it may not substitute the court’s own
judgment as to what amount should be awarded and how to
arrive at such amount. Hence, Section 3A of R.A. No. 6395,
as amended, is not binding upon this Court.
Issue 2:
Whether or not NPC may still be held liable to pay for the full
market value of the affected property despite the fact transfer
of title thereto was not required by the easement.
Held 2:

Page 160 of 186


The power of Eminent Domain may be exercised although title 14. National Power Corporation v. Ibrahim, G.R. No.
is not transferred to the expropriator in easement of right of 168732. June 29, 2007
way. Just compensation which should be neither more nor
DOCTRINE: In determining the just compensation, the
less than the money equivalent of the property is, moreover,
due where the nature and effect of the easement is to impose valuation of the property should be based on the value on the
limitations against the use of the land for an indefinite period date when the landowners discovered the presence of the
and deprive the landowner if ordinary use. huge underground tunnels beneath their lands, not the value
on the date on which the latter constructed the tunnels.
The owner should be compensated for the monetary
Facts:
equivalent of the land if the easement is intended to
perpetually or indefinitely deprive the owner of his proprietary Respondent Lucman G. Ibrahim, in his personal capacity and
rights through the imposition of conditions that affect the in behalf of his co-heirs instituted an action against petitioner
ordinary use, free enjoyment and disposal of the property or National Power Corporation (NAPOCOR) for recovery of
through restrictions and limitations that are inconsistent with possession of land and damages before the RTC of Lanao del
the exercise of the attributes of ownership, or when the Sur.
introduction of structures or objects which, by their nature,
create or increase the probability of injury, death upon or Ibrahim and his co-heirs claimed that they were owners of
destruction of life and property found on the land is necessary. several parcels of land described in Survey Plan FP (VII-5)
2278 consisting of 70,000 square meters, divided into three
DISPOSITIVE: petition is GRANTED and the CA’s assailed (3) lots, i.e. Lots 1, 2, and 3 consisting of 31,894, 14,915, and
Decision and Resolution are, accordingly, REVERSED and 23,191 square meters each respectively. NAPOCOR, through
SET ASIDE. alleged stealth and without respondents’ knowledge and prior
consent, took possession of the sub-terrain area of their lands
and constructed therein underground tunnels. The existence
of the tunnels was only discovered sometime in July 1992 by
respondents and then later confirmed on November 13, 1992
by NAPOCOR itself through a memorandum issued by the
latter’s Acting Assistant Project Manager. The tunnels were
apparently being used by NAPOCOR in siphoning the water
of Lake Lanao and in the operation of NAPOCOR’s Agus II,
III, IV, V, VI, VII projects located in Saguiran, Lanao del Sur;
Nangca and Balo-i in Lanao del Norte; and Ditucalan and
Fuentes in Iligan City.
Respondents demanded that NAPOCOR pay damages and
vacate the sub-terrain portion of their lands but the latter
refused to vacate much less pay damages. Respondents
further averred that the construction of the underground
tunnels has endangered their lives and properties as Marawi
City lies in an area of local volcanic and tectonic activity.
This case revolves around the propriety of paying just
compensation to respondents, and, by extension, the basis for
computing the same.
Issue:
Whether or not respondents are entitled to just compensation
hinges upon who owns the sub-terrain area occupied by
Petitioner.
Held:
The Court sustains the finding of the lower courts that the sub-
terrain portion of the property similarly belongs to
respondents.
Thus, the ownership of land extends to the surface as well as
to the subsoil under it. This principle was applied to show that
rights over lands are indivisible and, consequently, require a
definitive and categorical classification.
Registered landowners may even be ousted of ownership and
possession of their properties in the event the latter are
reclassified as mineral lands because real properties are
characteristically indivisible. For the loss sustained by such
owners, they are entitled to just compensation under the
Mining Laws or in appropriate expropriation proceedings.
Moreover, petitioner’s argument that the landowners’ right
extends to the sub-soil insofar as necessary for their practical
interests serves only to further weaken its case. The theory
would limit the right to the sub-soil upon the economic utility,
which such area offers to the surface owners. Presumably, the
landowners’ right extends to such height or depth where it is
possible for them to obtain some benefit or enjoyment, and it
is extinguished beyond such limit as there would be no more
interest protected by law.

Page 161 of 186


The underground tunnels impose limitations on respondents’ 15. National Power Corporation vs. Heirs of Macabangkit
use of the property for an indefinite period and deprive them Sangkay, G.R. No. 165828. August 24, 2011.
of its ordinary use. Based upon the foregoing, respondents are
DOCTRINE: The Court of Appeals’ restrictive construal of
clearly entitled to the payment of just compensation.
Section 3(i) of R.A. No. 6395 as exclusive of tunnels was
The entitlement of respondents to just compensation having obviously unwarranted, for the provision applies not only to
been settled, the issue now is on the manner of computing the development works easily discoverable or on the surface of
same. the earth but also to subterranean works like tunnels—when
the law does not distinguish, so must we not, and when the
The general rule in determining “just compensation” in language of the statute is plain and free from ambiguity, and
eminent domain is the value of the property as of the date of expresses a single, definite, and sensible meaning, that
the filing of the complaint. meaning is conclusively presumed to be the meaning that the
The general rule, however, admits of an exception: where this Congress intended to convey.
Court fixed the value of the property as of the date it was taken The prescriptive period provided under Section 3(i)
and not the date of the commencement of the expropriation of Republic Act No. 6395 is applicable only to an action for
proceedings. damages, and does not extend to an action to recover just
The exception finds the application where the owner would be compensation.
given undue incremental advantages arising from the use to Facts:
which the government devotes the property expropriated.
Pursuant to its legal mandate under Republic Act
In the present case, to allow petitioner to use the date it No. 6395 (An Act Revising the Charter of the National Power
constructed the tunnels as the date of valuation would be Corporation), NPC undertook the Agus River Hydroelectric
grossly unfair. First, it did not enter the land under warrant or Power Plant Project in the 1970s to generate electricity for
color of legal authority or with intent to expropriate the same. Mindanao. The project included the construction of several
In fact, it did not bother to notify the owners and wrongly underground tunnels to be used in diverting the water flow
assumed it had the right to dig those tunnels under their from the Agus River to the hydroelectric plants.
property. Secondly, the “improvements” introduced by
petitioner, namely, the tunnels, in no way contributed to an The respondents (Heirs of Macabangkit), as the
increase in the value of the land. The trial court, therefore, as owners of land with an area of 221,573 square meters
affirmed by the CA, rightly computed the valuation of the situated in Ditucalan, Iligan City, sued NPC in the RTC for the
property as of 1992, when respondents discovered the recovery of damages and of the property, with the alternative
construction of the huge underground tunnels beneath their prayer for the payment of just compensation. They alleged
lands and petitioner confirmed the same and started that they had belatedly discovered that one of the
negotiations for their purchase but no agreement could be underground tunnels of NPC that diverted the water flow of
reached. the Agus River for the operation of the Hydroelectric Project
in Agus V, Agus VI and Agus VII traversed their land; that their
discovery had occurred in 1995 after Atty. Saidali C.
Gandamra, had rejected their offer to sell the land because of
the danger the underground tunnel might pose to the
proposed Arabic Language Training Center and Muslims
Skills Development Center; that such rejection had been
followed by the withdrawal by Global Asia Management and
Resource Corporation from developing the land into a
housing project for the same reason; that Al-Amanah Islamic
Investment Bank of the Philippines had also refused to accept
their land as collateral because of the presence of the
underground tunnel; that the underground tunnel had been
constructed without their knowledge and consent; that the
presence of the tunnel deprived them of the agricultural,
commercial, industrial and residential value of their land; and
that their land had also become an unsafe place for habitation
because of the loud sound of the water rushing through the
tunnel and the constant shaking of the ground, forcing them
and their workers to relocate to safer grounds.
NPC countered that the Heirs of Macabangkit had
no right to compensation under section 3(f) of Republic Act
No. 6395, under which a mere legal easement on their land
was established; that their cause of action, should they be
entitled to compensation, already prescribed due to the tunnel
having been constructed in 1979; and that by reason of the
tunnel being an apparent and continuous easement, any
action arising from such easement prescribed in five years.
Issue:
Whether the Heirs of Macabangkit’s right to claim
just compensation had prescribed under section 3(i) of
Republic Act No. 6395, or, alternatively, under Article 620 and
Article 646 of the Civil Code.
Held:
The Court upholds the liability of NPC for payment
of just compensation.

Page 162 of 186


Five-year prescriptive period under Section 3(i) of Republic 16. National Power Corporation vs. Tarcelo, G.R. No.
Act No. 6395 does not apply to claims for just compensation. 198139. September 8, 2014.
A cursory reading shows that Section 3(i) covers the DOCTRINE: Execution must therefore conform to that
construction of “works across, or otherwise, any stream, ordained or decreed in the dispositive part of the decision.
watercourse, canal, ditch, flume, street, avenue, highway or
Facts:
railway of private and public ownership, as the location of said
works may require.” It is notable that Section 3(i) includes no Respondents Felicisimo Tarcelo (Tarcelo) and the heirs of
limitation except those enumerated after the term works. Comia Santos (Santos heirs) are the owners of two lots
Accordingly, we consider the term works as embracing all measuring 4,404 and 2,611 square meters, respectively,
kinds of constructions, facilities, and other developments that which are situated in Brgy. Tabangao-Ambulong, Batangas
can enable or help NPC to meet its objectives of developing City.
hydraulic power expressly provided under paragraph (g) of
Section 3. The CA’s restrictive construal of Section 3(i) as Petitioner National Power Corporation (NPC) filed Civil Case
exclusive of tunnels was obviously unwarranted, for the No. 5785 with the Batangas City RTC, seeking to expropriate
provision applies not only to development works easily portions of Tarcelo and the Santos heirs’ lots to the extent of
discoverable or on the surface of the earth but also to 1,595.91 square meters which are affected by the
subterranean works like tunnels. Such interpretation accords construction and maintenance of NPC’s 1,200 MW Ilijan
with the fundamental guideline in statutory construction that Natural Gas Pipeline Project. In other words, NPC’s natural
when the law does not distinguish, so must we not. Moreover, gas pipeline shall traverse respondents’ lands to such extent.
when the language of the statute is plain and free from The Decision of the appellate court which defendants-
ambiguity, and expresses a single, definite, and sensible appellees are entitled for just compensation to the full market
meaning, that meaning is conclusively presumed to be the value of their property not just ten percent (10%) of it, and just
meaning that the Congress intended to convey. compensation was fixed at P1,000.00 per square meter only
We rule that the prescriptive period provided under for the affected area of 1,591.91 square meters, and not for
Section 3(i) of Republic Act No. 6395 is applicable only to an the whole of respondents’ respective lots became final and
action for damages, and does not extend to an action to executory, and entry of judgment was done accordingly.
recover just compensation like this case. Consequently, NPC Respondents moved for execution.
cannot thereby bar the right of the Heirs of Macabangkit to A Notice of Garnishment was served on the Manager of the
recover just compensation for their land. Land Bank of the Philippines, NPC Branch, Quezon City for
the satisfaction of the amount of P5,594,462.50 representing
just compensation for the whole of respondents’ 4,404- and
2,611-square meter lots — or 7,015 square meters — and not
merely the supposedly affected portions thereof totaling
1,595.91 square meters as NPC originally sought to acquire.
NPC filed an Urgent Omnibus Motion seeking to quash the
Writ of Execution and Notice of Garnishment.
Issue:
Whether or not the respondents are entitled for just
compensation for the whole lots and not only the affected
portions thereof.
Held:
The Court held that the respondents are entitled to just
compensation for the affected portions of their land only.
The exercise of the right of eminent domain, whether directly
by the State or by its authorized agents, is necessarily in
derogation of private rights. It is one of the harshest
proceedings known to the law. x x x The authority to condemn
is to be strictly construed in favor of the owner and against the
condemnor. When the power is granted, the extent to which
it may be exercised is limited to the express terms or clear
implication of the statute in which the grant is contained.
Corollarily, it has been held that trial courts should exercise
care and circumspection in the resolution of just
compensation cases, considering that they involve the
expenditure of public funds.
The Commissioners’ Reports in Civil Case No. 5785 indicate
that only the affected areas were intended to be acquired and
compensated.
It has always been the rule that “the only portion of the
decision that may be the subject of execution is that which is
ordained or decreed in the dispositive portion. Whatever may
be found in the body of the decision can only be considered
as part of the reasons or conclusions of the court and serve
only as guides to determine the ratio decidendi.” “Where there
is a conflict between the dispositive portion of the decision
and the body thereof, the dispositive portion controls
irrespective of what appears in the body of the decision. While

Page 163 of 186


the body of the decision, order or resolution might create L. CONTRACT CLAUSE
some ambiguity in the manner of the court’s reasoning
1. LEPANTO CONSOLIDATED MINING CO. VS. WMC
preponderates, it is the dispositive portion thereof that finally
RESOURCES INT’L G.R. NO. 162331 NOVEMBER 20, 2006
invests rights upon the parties, sets conditions for the
J. CHICO-NAZARIO
exercise of those rights, and imposes corresponding duties or
obligation.” Thus, with the decretal portion of the trial court’s FACTS: On 22 March 1995, the Philippine Government and
November 7, 2005 Decision particularly stating that NPC shall WMC Philippines executed a Financial and Technical
have the lawful right to enter, take possession and acquire Assistance Agreement, denominated as the Columbio FTAA
easement of right-of-way over the affected portions of for the purpose of large scale exploration, development, and
respondents’ properties upon the payment of just commercial exploration of possible mineral resources in
compensation, any order executing the trial court’s Decision accordance with EO 279 and DAO No. 63, Series of 1991.
should be based on such dispositive portion. “An order of
execution is based on the disposition, not on the body, of the The Columbio FTAA is covered in part by 156 mining claims
decision.” held under various Mineral Production Sharing Agreements
(MPSA) by Southcot Mining Corporation, Tampakan Mining
Execution must therefore conform to that ordained or decreed Corporation, and Sagittarius Mines, Inc. (collectively called
in the dispositive part of the decision. Since there is a disparity the Tampakan Companies), in accordance with the
between the dispositive portion of the trial court’s November Tampakan Option Agreement. The Option Agreement
7, 2005 Decision as affirmed with modification by the final and provides for the grant of the right of first refusal to the
executory June 26, 2007 Decision of the CA in C.A.-G.R. CV Tampakan Companies in case WMC Philippines desires to
No. 86712 — which decreed that respondents be paid just dispose of its rights and interests in the mining claims
compensation only for the affected portions of their covering the area subject of the agreement.
properties, totaling 1,595.91 square meters — and the
Notice of Garnishment — for the satisfaction of the amount of WMC Resources subsequently divested itself of its rights and
P5,594,462.50 representing just compensation for the whole interests in the Columbio FTAA, executed a Sale and
7,015 square meters — the latter must be declared null and Purchase Agreement with Lepanto over its entire
void. It is a settled general principle that a writ of execution shareholdings in WMC Philippines, subject to the exercise of
must conform substantially to every essential particular of the the Tampakan Companies’ right of first refusal to purchase
judgment promulgated. Execution not in harmony with the the subject shares. Lepanto sought the approval of the Sale
judgment is bereft of validity. It must conform, more and Purchase Agreement from the DENR Secretary.
particularly, to that ordained or decreed in the dispositive The Tampakan Companies sought to exercise its right of first
portion of the decision. refusal. Thus, Lepanto assailed the Tampakan Companies’
exercise of its right of first refusal, alleging that the Tampakan
Companies failed to match the terms and conditions set forth
in the Sale and Purchase Agreement.
Lepanto filed a case for Injunction, Specific Performance,
Annulment of Contracts and Contractual Interference with the
RTC against WMC Philippines and the Tampakan
Companies. Respondents moved for the dismissal of said
case, but was denied.
In the interim, contending that the Sale and Purchase
Agreement between Lepanto and WMC Philippines had
expired due to failure to meet the necessary preconditions for
its validity, WMC and the Tampakan Companies executed
another Sale and Purchase Agreement, where Sagittarius
Mines, Inc. was designated assignee and corporate vehicle
which would acquire the shareholdings and undertake the
Columbio FTAA activities.
After due consideration and evaluation of the financial and
technical qualifications of Sagittarius Mines, Inc., the DENR
Secretary approved the transfer of the Columbio FTAA from
WMC Philippines to Sagittarius Mines, Inc.
Aggrieved by the transfer of the Columbio FTAA in favor of
Sagittarius Mines, Inc., Lepanto filed a Petition for Review of
the Order of the DENR Secretary with the OP. The OP
dismissed the petition.
ISSUE: WoN Sec. 40 of RA 7942 or the Philippine Mining Act
of 1995 which took effect on 14 April 1995, requiring the
approval of the President of the assignment or transfer of
FTTAs, is applicable as contended by Lepanto.
HELD: No. The Columbio FTAA was entered into by the
Philippine Government and WMC Philippines on 22 March
1995, undoubtedly before the Philippine Mining Act of 1995
took effect on 14 April 1995.
Furthermore, it is undisputed that said FTAA was granted in
accordance with EO 279 and DAO No. 63, Series of 1991,
which does not contain any similar condition on the transfer
or assignment of FTTAs.
There is also an absence of either an express declaration or

Page 164 of 186


an implication in the Philippine Mining Act of 1995 that the M. FREE ACCESS TO COURTS
provisions of said law shall be made to apply retroactively,
1. RE: QUERY OF MR. ROGER C. PRIORESCHI RE
therefore, any section of said law must be made to apply only
EXEMPTION FROM LEGAL AND FILING FEES OF THE
prospectively, in view of the rule that a statute ought not to
GOOD SHEPHERD FOUNDATION, INC.
receive a construction making it act retroactively, unless the
words used are so clear, strong, and imperative that no other (Dahil taga Baguio ako, alam ko to. Hahaha. Yung Good
meaning can be annexed to them, or unless the intention of Shepherd kasi sa Baguio ung gumagawa ng Ubeng Halaya,
the legislature cannot be otherwise satisfied. peanut brittle etc. Ung workers nila indigent in a way. So pag
Assuming for the sake of argument that We are to apply the nagwork ka dun ung tipong taga-pack ka or parang factory
Philippine Mining Act of 1995 retrospectively to the Columbio worker, pagaaralin ka ng Good Shepherd. Yung funds ng
FTAA, the lack of presidential approval will not be fatal as to scholars nila nakukuha sa profits nung pagbenta ng Good
render the transfer illegal, especially since the alleged lack of Shepherd)
presidential approval has been remedied when Lepanto Can the Courts grant to our Foundation who works for indigent
appealed the matter to the OP which approved the Order of and underprivileged people, the same option granted to
the DENR Secretary granting the application for transfer of indigent people?
the Columbio FTAA to Sagittarius Mines, Inc.
To answer the query of Mr. Prioreschi, the Courts cannot grant
Furthermore, if Lepanto was indeed of the mind that Section to foundations like the Good Shepherd Foundation, Inc. the
40 of the Philippine Mining Act of 1995 is applicable to the same exemption from payment of legal fees granted to
Columbio FTAA, thus necessitating the approval of the indigent litigants even if the foundations are working for
President for the validity of its transfer or assignment, it would indigent and underprivileged people.
seem contradictory that Lepanto sought the approval of the The basis for the exemption from legal and filing fees is the
DENR Secretary, and not that of the President, of its Sale and free access clause, embodied in Sec. 11, Art. III of the 1987
Purchase Agreement with WMC Resources. Hence, it may be Constitution, thus:
glimpsed from the very act of Lepanto that it recognized that
the provision of the Columbio FTAA regarding the consent of Sec. 11. Free access to the courts and quasi judicial
the DENR Secretary with respect to the transfer of said FTAA bodies and adequate legal assistance shall not be
must be upheld. denied to any person by reason of poverty.

The constitutional prohibition on the impairment of the The importance of the right to free access to the courts and
obligation of contract does not prohibit every change in quasi judicial bodies and to adequate legal assistance cannot
existing laws, and to fall within the prohibition, the change be denied. A move to remove the provision on free access
must not only impair the obligation of the existing contract, but from the Constitution on the ground that it was already
the impairment must be substantial. Substantial impairment covered by the equal protection clause was defeated by the
as conceived in relation to impairment of contracts has been desire to give constitutional stature to such specific protection
explained in the case of Clemons v. Nolting, which stated that: of the poor.
a law which changes the terms of a legal contract between The clear intent and precise language of the aforequoted
parties, either in the time or mode of performance, or imposes provisions of the Rules of Court indicate that only a natural
new conditions, or dispenses with those expressed, or party litigant may be regarded as an indigent litigant. The
authorizes for its satisfaction something different from that Good Shepherd Foundation, Inc., being a corporation
provided in its terms, is law which impairs the obligation of a invested by the State with a juridical personality separate and
contract and is therefore null and void. Section 40 of the distinct from that of its members,[4] is a juridical person.
Philippine Mining Act of 1995 requiring the approval of the Among others, it has the power to acquire and possess
President with respect to assignment or transfer of FTAAs, if property of all kinds as well as incur obligations and bring civil
made applicable retroactively to the Columbio FTAA, would or criminal actions, in conformity with the laws and regulations
be tantamount to an impairment of the obligations under said of their organization.[5] As a juridical person, therefore, it
contract as it would effectively restrict the right of the parties cannot be accorded the exemption from legal and filing fees
thereto to assign or transfer their interests in the said FTAA. granted to indigent litigants.
By imposing a new condition apart from those already That the Good Shepherd Foundation, Inc. is working for
contained in the agreement, before the parties to the indigent and underprivileged people is of no moment. Clearly,
Columbio FTAA may assign or transfer its rights and interest the Constitution has explicitly premised the free access clause
in the said agreement, Section 40 of the Philippine Mining Act on a persons poverty, a condition that only a natural person
of 1995, if made to apply to the Columbio FTAA, will effectively can suffer.
modify the terms of the original contract and thus impair the
obligations of the parties thereto and restrict the exercise of There are other reasons that warrant the rejection of the
their vested rights under the original agreement. Such request for exemption in favor of a juridical person. For one,
modification to the Columbio FTAA, particularly in the extending the exemption to a juridical person on the ground
conditions imposed for its valid transfer is equivalent to an that it works for indigent and underprivileged people may be
impairment of said contract violative of the Constitution. prone to abuse (even with the imposition of rigid
documentation requirements), particularly by corporations
and entities bent on circumventing the rule on payment of the
fees. Also, the scrutiny of compliance with the documentation
requirements may prove too time-consuming and wasteful for
the courts.

Page 165 of 186


1. Dickerson v. United States, 530 U.S. 428 (2000) 2. PEOPLE OF THE PHILIPPINES, plaintiff- appellee, vs.
DOMINGO REYES y PAJE, ALVIN ARNALDO y AVENA
Facts: and JOSELITO FLORES y VICTORIO, accused-
appellants. G.R. No. 178300. March 17, 2009.

During questioning about a robbery he was connected to, Facts:


Charles Dickerson made statements to authorities admitting DOCTRINE: Right to Counsel; The mantle of protection
that he was the getaway driver in a series of bank robberies. afforded by Art. III, Sec. 12 of the 1987 Constitution covers
Dickerson was then placed under arrest. The timing of his the period from the time a person is taken into custody for the
statement is disputed. The FBI and local detectives testified investigation of his possible participation in the commission
that Dickerson was advised of his Miranda rights, established of a crime or from the time he is singled out as a suspect in
in Miranda v. Arizona, and waived them before he made his the commission of the offense although not yet in custody.
statement. Dickerson said he was not read his Miranda
warnings until after he gave his statement. After his indictment The presence of a lawyer is not intended to stop an accused
for bank robbery, Dickerson filed a motion to suppress the from saying anything which might incriminate him, but, rather,
statement that he made on the ground that he had not it was adopted in our Constitution to preclude the slightest
received Miranda warnings before being interrogated. The coercion on the accused to admit something false—the
government argued that even if the Miranda warnings were counsel should never prevent an accused from freely and
not read, the statement was voluntary and therefore voluntarily telling the truth.
admissible under 18 USC Section 3501, which provides that
Facts:
“a confession shall be admissible in evidence if it is voluntarily
given.” The District Court granted Dickerson’s motion, finding An Information was filed before the RTC charging appellants
that he had not been read his Miranda rights or signed a with the special complex crime of kidnapping for ransom with
waiver until after he made his statement, but the Court did not homicide. During their arraignment, appellants, assisted by a
address section 3501. In reversing, the Court of Appeals counsel de oficio, pleaded “Not guilty” to the charge. Trial on
acknowledged that Dickerson had not received Miranda the merits thereafter followed. Appellants, however,
warnings, but held that section 3501 was satisfied because challenge the legality and admissibility of the written
his statement was voluntary. The Court held that “Congress extrajudicial confessions. Appellants claim that their alleged
enacted section 3501 with the express purpose of legislatively participation in the kidnapping of the Yao family was based
overruling Miranda and restoring voluntariness as the test for solely on the written extrajudicial confessions of appellants
admitting confessions in federal court.” Arnaldo and Flores. They maintain, however, that said
extrajudicial confessions are inadmissible in evidence,
because they were obtained in violation of his co-appellants’
Issue: constitutional right to have an independent counsel of their
own choice during custodial investigation.
Whether or not the Congress may legislatively overrule
Miranda vs. Arizona and its warnings that govern the BACKGROUND: The Yao family owns and operates a poultry
admissibility of statements made during custodial farm in Barangay Santo Cristo, San Jose del Monte, Bulacan.
investigation. On 16 July 1999, at about 11:00 p.m., the Yao family, on
board a Mazda MVP van, arrived at their poultry farm in
Barangay Sto. Cristo, San Jose del Monte, Bulacan. Yao San
Held: alighted from the van to open the gate of the farm, appellant
Reyes and a certain Juanito Pataray (Pataray) approached,
NO. In a 7-2 opinion delivered by Chief Justice poked their guns at Yao San, and dragged him inside the van.
William H. Rehnquist, the Court held that Miranda governs the Appellant Reyes and Pataray also boarded the van.
admissibility of statements made during custodial Thereupon, appellants Arnaldo and Flores, with two male
interrogation in both state and federal courts. “Miranda has companions, all armed with guns, arrived and immediately
become embedded in routine police practice to the point boarded the van. Appellant Flores took the driver’s seat and
where the warnings have become part of our national culture,” drove the van. Appellants Reyes and Arnaldo and their
wrote Rehnquist. “Miranda announced a constitutional rule cohorts then blindfolded each member of the Yao family
that Congress may not supersede legislatively. We decline to inside the van with packaging tape.
overrule Miranda ourselves.” Dissenting, Justice Antolin
Scalia, joined by Justice Clarence Thomas, blasted the Appellant Flores and his male companion told Yao San to
Court’s produce the amount of five million pesos (P5,000,000.00) as
ransom in exchange for the release of Chua Ong Ping Sim,
Robert, Raymond and Abagatnan. Thereafter, appellant
Flores and his male companion left the van and fled; while
Yao San, Lenny, Matthew, Charlene and Josephine remained
inside the van. Upon sensing that the kidnappers had already
left, Yao San drove the van towards the poultry farm and
sought the help of relatives. Meanwhile, Chua Ong Ping Sim,
Robert, Raymond and Abagatnan were taken on foot by
appellants Reyes and Arnaldo, Pataray and one male
companion to a safe-house situated in the mountainous part
of San Jose Del Monte, Bulacan where they spent the whole
night.
On the morning of 19 July 1999, appellants again called Yao
San via a cellular phone and threatened to kill Chua Ong Ping
Sim and Raymond because of newspaper and radio reports
regarding the incident. Yao San clarified to appellants that he
did not report the incident to the police and also pleaded with
them to spare the life of Chua Ong Ping Sim and Raymond.
Appellants then instructed Yao San to appear and bring with
him the ransom of P5 million at 3:00 p.m. in the Usan

Page 166 of 186


dumpsite, Litex Road, Fairview, Quezon City. Yao San 3. PEOPLE vs. CHAVEZ, G.R. No. 207950, September 22,
arrived at the designated place of the pay-off at 4:00 p.m., but 2014.
none of the appellants or their cohorts showed up. Yao San
Facts:
waited for appellant’s call, but none came. Thus, Yao San left.
The corpses of Chua Ong Ping Sim and Raymond were When Peñamante (witness) arrived home from work, around
found at the La Mesa Dam. 2:45am, he saw a person wearing a black, long-sleeved shirt
Novaliches, Quezon City. Both died of asphyxia by and black pants and holding something while leaving the
strangulation. On 26 July 1999, appellant Arnaldo house/parlor of Elmer Duque aka Barbie (victim).
surrendered. There was a light at the left side of the house/parlor of Barbie,
Issue: his favorite haircutter, so Peñamante was able to see the face
of Chavez (accused).
NO. The right to counsel is a fundamental right and is intended
to preclude the slightest coercion as would lead the accused The following day, Barbie was found dead, due to stab
to admit something false. The right to counsel attaches upon wounds, in the parlor and the place was in disarray. In a line-
the start of the investigation, i.e., when the investigating officer up to identify the person he saw leaving Barbie’s house/parlor
starts to ask questions to elicit information and/or confessions that early morning of October 28, 2006, Peñamante
or admissions from the accused. The lawyer called to be immediately pointed to and identified Chavez and thereafter
present during such investigation should be, as far as executed his written statement.
reasonably possible, the choice of the accused. If the lawyer Chavez was charged with robbery with homicide.
is one furnished in behalf of accused, he should be competent
and independent; that is, he must be willing to fully safeguard LC: Guilty as charged, based on circumstantial evidence.
the constitutional rights of the accused. A competent and CA: Affirmed.
independent counsel is logically required to be present and
able to advice and assist his client from the time the latter Issue:
answers the first question asked by the investigator until the
Whether or not Chavez is guilty beyond reasonable doubt of
signing of the confession. Moreover, the lawyer should
the crime of robbery with homicide.
ascertain that the confession was made voluntarily, and that
the person under investigation fully understood the nature and Held:
the consequence of his extrajudicial confession vis-a-vis his
constitutional rights. No. There was no robbery. “What is imperative and essential
for a conviction for the crime of robbery with homicide is for
However, the foregoing rule is not intended to deter the prosecution to establish the offender’s intent to take
to the accused from confessing guilt if he voluntarily and personal property before the killing, regardless of the time
intelligently so desires, but to protect him from admitting what when the homicide is actually carried out.” In cases when the
he is being coerced to admit although untrue. To be an prosecution failed to conclusively prove that homicide was
effective counsel, a lawyer need not challenge all the committed for the purpose of robbing the victim, no accused
questions being propounded to his client. The presence of a can be convicted of robbery with homicide.
lawyer is not intended to stop an accused from saying
anything which might incriminate him; but, rather, it was The circumstantial evidence relied on by the lower courts do
adopted in our Constitution to preclude the slightest coercion not satisfactorily establish an original criminal design by
on the accused to admit something false. The counsel should Chavez to commit robbery.
never prevent an accused from freely and voluntarily telling At most, the intent to take personal property was mentioned
the truth. by Chavez’s mother in her statement as follows: “Na sinabi
We have gone over the records and found that the PAOCTF niya sa akin na wala siyang intensyon na patayin [sic] si
investigators have duly apprised appellants Arnaldo and Barbie kundi ay pagnakawan lamang.” However, this
Flores of their constitutional rights to remain silent and to have statement is considered as hearsay, with no evidentiary
competent and independent counsel of their own choice value, since Chavez’s mother was never presented as a
during their respective custodial investigations. witness during trial to testify on her statement.

In the case at bar, appellants Arnaldo and Flores failed to An original criminal design to take personal property is also
discharge their burden of proving that they were forced or inconsistent with the infliction of no less than 21 stab wounds
coerced to make their respective confessions. Other than in various parts of Barbie’s body. The sheer number of stab
their self-serving statements that they were maltreated by the wounds inflicted on Barbie makes it difficult to conclude an
PAOCTF officers/agents, they did not present any plausible original criminal intent of merely taking Barbie’s personal
proof to substantiate their claims. They did not submit any property.
medical report showing that their bodies were subjected to Homicide – All these circumstances taken together establish
violence or torture. Neither did they file complaints against Chavez’s guilt beyond reasonable doubt for the crime of
the persons who had allegedly beaten or forced them to homicide:
execute their respective confessions despite several
opportunities to do so. Appellants Arnaldo and Flores averred 1. The alibi of Chavez still places him at the scene of the
that they informed their family members/relatives of the crime that early morning of October 28, 2006. This court has
alleged maltreatment, but the latter did not report such considered motive as one of the factors in determining the
allegations to proper authorities. presence of an intent to kill, and a confrontation with the victim
immediately prior to the victim’s death has been considered
as circumstantial evidence for homicide.
2. The number of stab wounds inflicted on Barbie
strengthens an intention to kill and ensures his death.
3. Peñamante’s positive identification of Chavez as the
person leaving Barbie’s house that early morning of October
28, 2006.
4. The medico-legal’s testimony establishing Barbie’s time
of death as 12 hours prior to autopsy at 1:00 p.m., thus,

Page 167 of 186


narrowing the time of death to approximately 1:00 a.m. of the 4. PEOPLE OF THE PHILIPPINES, appellee, vs. EDNA
same day, October 28, 2006. MALNGAN y MAYO, G.R. No. 170470. September 26,2006.
DECISION. Judgment by lower court is MODIFIED. Chavez DOCTRINE: Rights of Suspects; Miranda Doctrine;
is GUILTY beyond reasonable doubt of the separate and Extrajudicial Confessions; Requisites for Admissibility.—We
distinct crime of HOMICIDE. have held that the above quoted provision applies to the
stage of custodial investigation—when the investigation is no
longer a general inquiry into an unsolved crime but starts to
focus on a particular person as a suspect. Said constitutional
guarantee has also been extended to situations in which an
individual has not been formally arrested but has merely
been “invited" for questioning. To be admissible in evidence
against an accused, the extrajudicial confessions made must
satisfy the following requirements: (1) it must be voluntary;
(2) it must be made with the assistance of competent and
independent counsel; (3) it must be express; and (4) it must
be in writing.
Facts:
An Information was filed before the RTC of Manila, Branch
41, charging accused-appellant with the crime of Arson with
Multiple Homicide. When arraigned, accused-appellant with
assistance of counsel de officio, pleaded “Not Guilty” to the
crime charged. Thereafter, trial ensued. The RTC considered
accused-appellant to have waived her right to present
evidence, having filed the Demurrer to Evidence without
leave of court. Accused- appellant has not shown any
compelling reason why the witnesses presented would
openly, publicly and deliberately lie or concoct a story, to
send an innocent person to jail all the while knowing that the
real malefactor remains at large. Such proposition defies
logic. And where the defense failed to show any evil or
improper motive on the part of the prosecution witnesses, the
presumption is that their testimonies are true and thus
entitled to full faith and credence.
Accused-appellant questions the admissibility of her
uncounselled extrajudicial confession given to prosecution
witnesses, namely Remigio Bernardo, Mercedita Mendoza,
and to the media. Accused-appellant Edna contends that
being uncounselled extrajudicial confession, her admissions
to having committed the crime charged should have been
excluded in evidence against her for being violative of Article
III, Section 12(1) of the Constitution.
BACKGROUND: From the personal account of Remigio
Bernardo, the Barangay Chairman in the area, as well as the
personal account of the pedicab driver named Rolando Gruta,
it was at around 4:45 a.m. on January 2, 2001 when Remigio
Bernardo and his tanods saw the accused-appellant EDNA,
one hired as a housemaid by Roberto Separa, Sr., with her
head turning in different directions, hurriedly leaving the house
of her employer at No. 172 Moderna Street, Balut, Tondo,
Manila. She was seen to have boarded a pedicab which was
driven by a person later identified as Rolando Gruta. She was
heard by the pedicab driver to have instructed that she be
brought to Nipa Street, but upon her arrival there, she changed
her mind and asked that she be brought instead to Balasan
Street where she finally alighted, after paying for her fare.
Thirty minutes later, at around 5:15 a.m. Barangay Chairman
Bernardo’s group later discovered that a fire gutted the house
of the employer of the housemaid. Barangay Chairman
Bernardo and his tanods responded to the fire upon hearing
shouts from the residents and thereafter, firemen from the
Fire District 1-NCR arrived at the fire scene to contain the fire.
When Barangay Chairman Bernardo returned to the
Barangay Hall, he received a report from pedicab driver
Rolando Gruta, who was also a tanod, that shortly before the
occurrence of the fire, he saw a woman (the housemaid)
coming out of the house at No. 172 Moderna Street, Balut,
Tondo, Manila and he received a call from his wife telling him
of a woman (the same housemaid) who was acting strangely
and suspiciously on Balasan Street. Barangay Chairman
Bernardo, Rolando Gruta and the other tanods proceeded to

Page 168 of 186


Balasan Street and found the woman who was later identified “interrogation” made by the latter —admittedly conducted
as the accused- appellant. After Rolando Gruta positively without first informing accused-appellant of her rights under
identified the woman as the same person who left No. 172 the Constitution or done in the presence of counsel. For this
Moderna Street, Balut, Tondo, Manila, Barangay Chairman reason, the confession of accused-appellant, given to
Bernardo and his tanods apprehended her and brought her to Barangay Chairman Remigio Bernardo, as well as the lighter
the Barangay Hall for investigation. At the Barangay Hall, found by the latter in her bag are inadmissible in evidence
Mercedita Mendoza, neighbor of Roberto Separa, Sr. and against her as such were obtained in violation of her
whose house was also burned, identified the woman as constitutional rights.
accused-appellant EDNA who was the housemaid of Roberto
Separa, Sr. Upon inspection, a disposable lighter was found Be that as it may, the inadmissibility of accused-appellant’s
inside accused-appellant EDNA’s bag. Thereafter, accused- confession to Barangay Chairman Remigio Bernardo and the
appellant EDNA confessed to Barangay Chairman Bernardo lighter as evidence do not automatically lead to her acquittal.
in the presence of multitudes of angry residents outside the It should well be recalled that the constitutional safeguards
Barangay Hall that she set her employer’s house on fire during custodial investigations do not apply to those not
because she had not been paid her salary for about a year elicited through questioning by the police or their agents but
and that she wanted to go home to her province but her given in an ordinary manner whereby the accused verbally
employer told her to just ride a broomstick in going home. admits to having committed the offense as what happened in
the case at bar when accused-appellant admitted to
Accused-appellant EDNA was then turned over to arson Mercedita Mendoza, one of the neighbors of Roberto Separa,
investigators headed by S[F]O4 Danilo Talusan, who brought Sr., to having started the fire in the Separas’ house. The
her to the San Lazaro Fire Station in Sta. Cruz, Manila where testimony of Mercedita Mendoza recounting said admission
she was further investigated and then detained.When is, unfortunately for accused-appellant, admissible in
Mercedita Mendoza went to the San Lazaro Fire Station to evidence against her and is not covered by the aforesaid
give her sworn statement, she had the opportunity to ask constitutional guarantee. Article III of the Constitution, or the
accused-appellant EDNA at the latter’s detention cell why she Bill of Rights, solely governs the relationship between the
did the burning of her employer’s house and accused- individual on one hand and the State (and its agents) on the
appellant EDNA replied that she set the house on fire other; it does not concern itself with the relation between a
because when she asked permission to go home to her private individual and another private individual—as both
province, the wife of her employer Roberto Separa, Sr., accused-appellant and prose ution witness Mercedita
named Virginia Separa shouted at her: “Sige umuwi ka, Mendoza undoubtedly are. Here, there is no evidence on
pagdating mo maputi ka na. Sumakay ka sa walis, pagdating record to show that said witness was acting under police
mo maputi ka na” (“Go ahead, when you arrive your color authority, so appropriately, accused-appellant’s uncounselled
would be fair already. Ride a broomstick, when you arrive extrajudicial confession to said witness was properly admitted
your color would be fair already.”) And when Mercedita by the RTC.
Mendoza asked accused-appellant EDNA how she burned
the house, accused-appellant EDNA told her: “Naglukot ako
ng maraming diyaryo, sinindihan ko ng disposable lighter at
hinagis ko sa ibabaw ng lamesa sa loob ng bahay” (“I
crumpled newspapers, lighted them with a disposable lighter
and threw them on top of the table inside the house.”)
When interviewed by Carmelita Valdez, a reporter of ABS-
CBN Network, accused-appellant
EDNA while under detention was heard by SFO4 Danilo
Talusan as having admitted the crime and even narrated the
manner how she accomplished it. SFO4 Danilo Talusan was
able to hear the same confession, this time at his home, while
watching the television program “True Crime” hosted by Gus
Abelgas also of ABS-CBN Network.
The fire resulted in [the] destruction of the house of Roberto
Separa, Sr. and other adjoining houses and the death of
Roberto Separa, Sr. and Virginia Separa together with their
four (4) children, namely: Michael, Daphne, Priscilla and
Roberto, Jr.”
Issue:
Whether or not the uncounselled extrajudicial confession
given to prosecution witnesses was admissible in evidence.
Held:
NO. Arguably, the barangay tanods, including the Barangay
Chairman, in this particular instance, may be deemed as law
enforcement officer for purposes of applying Article III,
Section 12(1) and (3), of the Constitution. When accused-
appellant was brought to the barangay hall in the morning of
2 January 2001, she was already a suspect, actually the only
one, in the fire that destroyed several houses as well as killed
the whole family of Roberto Separa, Sr. She was, therefore,
already under custodial investigation and the rights
guaranteed by Article III, Section 12(1), of the Constitution
should have already been observed or applied to her.
Accused- appellant’s confession to Barangay Chairman
Remigio Bernardo was made in response to the

Page 169 of 186


5. PEOPLE OF THE PHILIPPINES, plaintiff- appellee, vs. Court needs to ascertain whether or not a “ bantay bayan”
ANTONIO LAUGA Y PINA ALIAS TERIO, accused- may be deemed a law enforcement officer with in the
appellant. G.R. No. 186228. March 15, 2010. contemplation of Article III, Section 12 of the Constitution. In
People of the Philippines v. Buendia, 382 SCRA 714 (2002),
DOCTRINE: Bantay Bayan; Words and Phrases; A “bantay
this Court had the occasion to mention the nature of a “bantay
bayan" is a group of male residents living in an area organized bayan,” that is, “a group of male residents living in the area
for the purpose of keeping peace in their community. organized for the purpose of keeping peace in their
Barangay-based volunteer organization in the nature of watch community, which is an accredited auxiliary of the x x x PNP.”
groups, as in the case of the “bantay bayan," are recognized Also, it may be worthy to consider that pursuant to Section
by the local government unit to perform functions relating to 1(g) of Executive Order No. 309 issued on 11 November
the preservation of peace and order at the barangay level; 1987, as amended, a Peace and Order Committee in each
Any inquiry a bantay bayan makes has the color of a state- barangay shall be organized “to serve as implementing arm
related function and objective insofar as the entitlement of a of the City/Municipal Peace and Order Council at the
suspect to his constitutional rights provided for under Article Barangay level.” The composition of the Committee includes,
III, Section 12 of the Constitution, otherwise known as the among others: (1) the Punong Barangay as Chairman; (2) the
Miranda Rights, is concerned, and an extrajudicial confession Chairman of the Sangguniang Kabataan; (3) a Member of the
taken from a suspect by such bantay bayan without a counsel Lupon Tagapamayapa; (4) a Barangay Tanod; and (5) at
is inadmissible in evidence. least three (3) Members of existing Barangay-Based Anti-
Crime or neighborhood Watch Groups or a Non Government
Facts: Organization Representative well-known in his community.
The appellant was accused of the crime of QUALIFIED The Court is convinced that barangay-based volunteer nature
RAPE. Appellant entered a plea of not guilty. This case is a of watch groups, as in the case of the “bantay bayan,” are
review of a conviction for the rape of the accused’s 13 year recognized by the local government unit to perform functions
old daughter. relating to the preservation of peace and order at the
BACKGROUND: In the afternoon of 15 March 2000, AAA was barangay level. Thus, without ruling on the legality of the
left alone at home. AAA’s father, the appellant, as having a actions taken by Moises Boy Banting, and the specific scope
drinking spree at the neighbor’s place. Her mother decided to of duties and responsibilities delegated to a “bantay bayan,”
leave because when appellant get drunk, he has the habit of particularly on the authority to conduct a custodial
mauling AAA’s mother. Her only brother BBB also went out in investigation, any inquiry he makes has the color of a state-
the company of some neighbors. related function and objective insofar as the entitlement of a
suspect to his constitutional rights provided for under Article
At around 10:00 o’clock in the evening, appellant woke AAA III, Section 12 of the Constitution, otherwise known as the
up; removed his pants, slid inside the blanket co ering AAA Miranda Rights, is concerned. We, therefore, find the
and removed her pants and underwear; warned her not to extrajudicial confession of appellant, which was taken without
shout for help while threatening her with his fist; and tol her a counsel, inadmissible in evidence.
that he had a knife placed above her head. He proceeded to
mash her breast, kiss her repeatedly, and “inserted his penis
inside her vagina.”
Soon after, BBB arrived and found AAA crying. Appellant
claimed he scolded her for staying out late. BBB decided to
take AAA with him. While on their way to their maternal
grandmother’s house, AAA ecounted her harrowing
experience with their father. Upon reaching their
grandmother’s house, they told their grandmother and uncle
of the incident, after which, they sought the assistance of
Moises Boy Banting.Moises Boy Banting found appellant in
his house wearing only his und rwear. He invited appellant to
the police station, to which appellant obliged. At the police
outpost, he admitted to him that he raped. AAA because he
was unable to control himself. The following day, AAA
submitted herself to physical examination.
On the other hand, only appellant testified for the defense. He
believed that the charge against him was ill-motivated
because he sometimes physically abuses his wife in front o
their children after engaging in a heated argu ent, and beats
the children as a disciplinary measure. He went further to
narrate how his day was on the date of the alleged rape.
Appellant went back to work and went home again around 3
o’clock in the afternoon. Finding nobody at home, he prepared
his dinner and went to sleep. Later in the evening, he was
awakened by the members of the “Bantay Bayan” headed by
Moises Boy Banting. They asked him to go with them to
discuss some matters. He later learned that he was under
detention because AAA charged him of rape.
Issue:
Whether or not the confession of the accused with a “bantay
bayan” is admissible in evidence.
Held:
NO. Following the rationale behind the ruling in Malngan, this

Page 170 of 186


6. Miguel vs. People, 833 SCRA 139, 2017 It held that search made on petitioner was valid as it was done
incidental to his arrest for exhibiting his private parts on public
FACTS:
An information was filed before the RTC charging Jeffrey As such, said marijuana is admissible in evidence and
Miguel (petitioner) of illegal possession of dangerous drugs, sufficient to convict him for the crime charged
penalized under 9165 or the “Comprehensive Dangerous Petitioner moved for reconsideration but was denied, hence
Drugs Act of 2002” the petition to the SC
Prosecution alleged that (prosecution version of facts):
Petitioner argues that the search and arrest was illegal and,
Around 12:45 am of May 24, 2010, a Bantay Bayan operative thus, the marijuana purportedly seized from him is
of Brgy. San Antonio Village, Makati, named Reynaldo inadmissible in evidence
Bahoyo was doing his rounds when he received a report of a
man showing off his private parts at Kaong Street! ISSUE:

Bahoyo and fellow Bantay Bayan operative Mark Velasquez Whether or not the there was a valid search incidental to a
then went to said street and saw a visible intoxicated person, lawful warrantless arrest
which they later identified as petitioner, urinating and HELD:
displaying his private parts while standing in front of a gate
NO
enclosing an empty lot
(First, the Court established that the Bill of Rights apply to
When they asked petitioner where he lived, the latter
Bantay Bayan operatives)
answered “Kaong Street”
The Bantay Bayan operatives are not government agents like
Bahoyo then said he lived on the same street but petitioner
the PNP or the NBI in charge of law enforcement, but are
looked unfamiliar to him, so he asked for an ID, but petitioner
civilian volunteers who act as “force multipliers” to assist the
failed to produce one
law enforcement agencies in maintaining peace and security.
Velasquez repeated the for an ID, but instead, petitioner The Bill of Rights generally cannot be invoked against the acts
emptied his pockets, revealing a pack of cigarettes containing of private individuals, however, they may be applicable if such
one stick and two pieces of rolled paper containing dried individuals act under the color of a state-related function
marijuana leaves
In this case, the acts of the Bantay Bayan relating to the
This prompted the Bantay Bayans to seize the items, take preservation of peace and order in their respective areas have
petitioner to the police station, and turn him, as well as the the color of a state-related function. As such, the Bill of Rights
items, over to SPO3 Rafael Castillo may be applied to the Bantay Bayan operatives who arrested
and subsequently searched petitioner.
SPO3 Castillo then inventoried the items, and prepared a
request for qualitative examination of the rolled paper and for
petitioner to undergo drug testing
(Next, the court determined that the arrest and search was
Tests confirmed it was marijuana and that petitioner was NOT validly made)
positive for the presence of Methamphetamine but negative
The Bill of Rights provides that evidence obtained from
for THC-metabolites
unreasonable searches and seizures shall be inadmissible in
Petitioner pleaded not guilty, and presented a different version evidence for any purpose in any proceeding, being the
of facts (petitioner version of facts): proverbial fruit of a poisonous tree. The law requires that there
first be a lawful arrest first before a search can be made – this
He was just urinating in from of his workplace when 2 Bantay
process cannot be reversed.
Bayan operatives approached him and asked him where he
lived A lawful arrest may be made without a warrant, provided the
parameters defined in Section 5, Rule 113 of the Revised
Upon responding that he lived in Kaong Street, they frisked
Rules of Criminal Procedure are complied with.
him, took away his belongings, and thereafter handcuffed and
brought him to the Brgy. Hall In Section 5(a), or in flagrante delicto arrests, 2 elements must
concur, namely:
He was detained for about an hour before being taken to the
Ospital ng Makati and to another office where a bald officer a. The person to be arrested must execute an
questioned him overt act indicating that he has committed,
is actually committing, or is attempting to
He was then taken back to the Brgy. Hall where they showed commit a crime
him 2 sticks of marijuana joints allegedly recovered from him b. Such overt act is done in the presence or
within the view of the arresting officer
RTC Ruling
Section 5(b), requires that at the time of the arrest, an offense
RTC found petitioner guilty for the crime charged
had in fact just been committed and the arresting officer had
They ruled that the Bahoyo and Velasques conducted a valid personal knowledge of facts indicating that the accused had
warrantless arrest, as petitioner was scandalously showing his committed it
private parts at the time of his arrest
In both instances, the officer’s personal knowledge of the
Thus, the incidental search which yielded the seized fact of the commission of an offense is essential
marijuana was also lawful
On the basis of the testimonies, the Court is inclined to believe
Petitioner appealed to the CA that petitioner went out to the street to urinate (note that there
was no CR in his workplace) when the Bantay Bayan
CA Ruling
operatives chanced upon him. That latter then approached
CA affirmed conviction

Page 171 of 186


and questioned petitioner, and thereafter went on to search 7. People vs. Dacanay, 807 SCRA 130, 2016
his person, which purportedly yielded the marijuana.
Facts:
Verily, the prosecution’s claim that petitioner was showing off In 1985, petitioner was the vice-president of the National
his private parts was belied by the testimonies (important Sugar Trading Corporation (NASUTRA). In 1986, a criminal
note: the Bantay Bayans testified in the cross examination complaint for economic sabotage through smuggling, with
that petitioner was showing off his private parts while urinating regard to the importation of raw sugar in 1983 and 1984 by
and turning his back from them. How the hell can you show off NASUTRA, was filed with the Tanodbayan against the
your private parts when you are turning your back and principal officers of the said corporation including petitioner.
urinating? lol) On October 10, 1986, the Tanodbayan approved the
resolution of the team of Special Prosecutors who
Clearly, these circumstances do not justify the conduct of investigated the case. It found sufficient prima facie evidence
an in flagrante delicto arrest, considering that there was against petitioner and his co-accused to warrant the filing of
NO overt act constituting a crime committed by petitioner an information with respondent Sandiganbayan for violation
in the presence or within the view of the arresting officer of Section 3(e) of Republic Act No. 3019, as amended. The
corresponding information was filed with the Sandiganbayan.
Neither do these circumstances necessitate a “hot pursuit” On November 20, petitioner filed a motion to quash but he
warrantless arrest as the arresting operatives do not have any later withdrew the same. On October 14, 1988, a resolution
personal knowledge of facts that petitioner had just committed was issued by Special Prosecutors Margarito P. Gervacio and
an offense Robert E. Kallos, recommending the dismissal of the
complaint against petitioner and his co-accused for lack of
Moreover, if the arrest was made because of the alleged
sufficient evidence and the withdrawal of the information filed
display of private parts, then the proper charge should have in court. The resolution was approved by Acting Special
been filed against him. However, records are bereft of any Prosecutor Jose Ferrer. On January 6, 1989, the resolution
showing that such charge was filed aside from the instant issued by Prosecutors Gervacio and Kallos was reviewed by
criminal charge for illegal possession of dangerous drugs. Special Prosecutor Wilfredo Orencia, who recommended its
This strengthens the view that no prior lawful arrest was made disapproval. The recommendation of Prosecutor Orencia was
which led to a valid search incidental thereto. approved by Acting Special Prosecutor Jose F. Guerrero and
by Ombudsman Conrado Vasquez. On February 22, Jose
There must be a lawful arrest FIRST, before a search can be Unson, a co-accused of petitioner, filed a motion to quash the
made, the process cannot be reversed. information in the Sandiganbayan. The motion was adopted
by petitioner. On December 12, the Sandiganbayan denied
the motion to quash. Likewise, the motion for reconsideration
filed by Unson and adopted by petitioner was denied.
On April 3, 1991 petitioner filed a motion for immediate and
separate trial invoking his constitutional right to a speedy trial.
On April 23, respondent People of the Philippines opposed
the said motion on the ground that a separate trial for
petitioner would entail a lengthy and repetitious proceeding.
Issue:
Whether or not petitioner is entitled to a separate trial.
Held:
Yes.
A separate trial necessarily requires a repetition of the
presentation of the same evidence. But the resulting in
convenience and expense on the part of the Government
cannot be given preference over the right to speedy trial and
the protection to a person's life, liberty or property accorded
by the Constitution. This is particularly true in the case of
petitioner where the prosecutors' opposition to the request for
separate trial was based on the ground that the principal
accused in the case, the former President of NASUTRA, was
abroad and was not yet arrested. If an accused cannot be
placed under arrest because he remains outside the territorial
jurisdiction of the Philippines, with more reason should his co-
accused, who are under arrest, be entitled to a separate trial.
A separate trial is in consonance with the right of an
accused to a speedy trial as guaranteed to him by the
1987 Constitution, more specifically under Section 14(2)
of Article III thereof. As defined in the case of Flores v.
People, 61 SCRA 331 (1974), a speedy trial is one
"conducted according to the law of criminal procedure and the
rules and regulations, free from vexatious, capricious and
oppressive delays." The primordial purpose of this
constitutional right is to prevent the oppression of an accused
by delaying criminal prosecution for an indefinite period of
time. Likewise, it is intended to prevent delays in the
administration of justice by requiring judicial tribunals to
proceed with reasonable dispatch in the trial of criminal

Page 172 of 186


prosecutions. O. RIGHTS OF THE ACCUSED
WHEREFORE, respondents are ORDERED to proceed 1. PEOPLE OF THE PHILIPPINES, petitioner, vs.
with the trial of petitioner in Criminal Case No. 11957, LUZVIMINDA S. VALDEZ and THE SANDIGANBAYAN
separately if it need be. (FIFTH DIVISION), respondents, G.R. Nos. 216007-09.
December 8, 2015.
DOCTRINE: Malversation of Public Funds thru Falsification
of Official/Public Documents; Manalac, Jr. v. People, G.R.
Nos. 206194-206207, July 3, 2013, already resolved that an
accused charged with Malversation of Public Funds thru
Falsification of Official/Public Documents where the amount
involved exceeds P22,000.00 is not entitled to bail as a matter
of right because it has an actual imposable penalty of
reclusion perpetua.
Facts:
The case stemmed from the Joint Affidavit executed by
Sheila S. Velmonte-Portal and Mylene T. Romero, both State
Auditors of the Commission on Audit Region VI in Pavia,
Iloilo, who conducted a post- audit of the disbursement
vouchers (D.V.) of the Bacolod City Government. Among the
subjects thereof were the reimbursements of expenses of
private respondent Luzviminda S. Valdez (Valdez), a former
mayor of Bacolod City.
Based on the verification conducted in the establishments
that issued the official receipts, it was alleged that the cash
slips were altered/falsified to enable Valdez to claim/receive
reimbursement from the Government the total amount of
P279,150.00 instead of only P4,843.25; thus, an aggregate
overclaim of P274,306.75.
Valdez was charged with eight cases four of which (SB - 14-
CRM-0317 to 0320) were for Violation of Section 3(e) of
Republic Act No. 3019, while the remaining half (SB-14-
CRM-0321 to 0324) were for the complex crime of
Malversation of Public Funds thru Falsification of
Official/Public Documents under Articles 217 and 171, in
relation to Article 48 of the Revised Penal Code.
Issue:
Whether or not respondent is entitled to bail as a matter of
right.
Held:
No. The controversy is, in fact, not one of first impression.
Manalac, Jr. v. People already resolved that an accused
charged with Malversation of Public Funds thru Falsification
of Official/Public Documents where the amount involved
exceeds P22,000.00 is not entitled to bail as a matter of right
because it has an actual imposable penalty of reclusion
perpetua.
The Sandiganbayan thus gravely erred in setting aside the
“No Bail” recommendation of the Special Prosecutor and
fixing the amount of bail as prayed for by Valdez. It is settled
that the grant of bail to an accused charged with an offense
that carries with it the penalty of reclusion perpetua is
discretionary on the part of the trial court, i.e., accused is still
entitled to bail but no longer as a matter of right. Indeed, the
determination of whether or not the evidence of guilt is strong
is a matter of judicial discretion. This discretion, by the nature
of things, may rightly be exercised only after the evidence is
submitted to the court at the hearing. The Prosecution must
be given a chance to show strength of its evidence; otherwise,
a violation of due process occurs. As the rule now stands, a
hearing upon notice is mandatory before the grant of bail,
whether bail is a matter of right or discretion.

Page 173 of 186


2. Government of Hong Kong Special Administrative provision on bail will not apply to a case like
Region v. Olalia. G.R. No. 153675; April 19, 2007, 521 extradition, where the presumption of innocence is
SCRA 470, (2007) not at issue.
Private respondent Munoz was charged before the Hong At first glance, the above ruling applies squarely to private
Kong Court with three (3) counts of the offense of "accepting respondent's case. However, this Court cannot ignore the
an advantage as agent," in violation of Section 9 (1) (a) of the following trends in international law: (1) the growing
Prevention of Bribery Ordinance, Cap. 201 of Hong Kong. He importance of the individual person in public international law
also faces seven (7) counts of the offense of conspiracy to who, in the 20th century, has gradually attained global
defraud, penalized by the common law of Hong Kong. On recognition; (2) the higher value now being given to human
August 23, 1997 and October 25, 1999, warrants of arrest rights in the international sphere; (3) the corresponding duty
were issued against him. If convicted, he faces a jail term of of countries to observe these universal human rights in
seven (7) to fourteen (14) years for each charge. fulfilling their treaty obligations; and (4) the duty of this Court
to balance the rights of the individual under our fundamental
The DOJ received from the Hong Kong Department of Justice law, on one hand, and the law on extradition, on the other.
a request for the provisional arrest of private respondent. The
DOJ then forwarded the request to the National Bureau of The modern trend in public international law is the primacy
Investigation (NBI) which, in turn, filed with the RTC of Manila, placed on the worth of the individual person and the sanctity
Branch 19 an application for the provisional arrest of private of human rights. Significant events show that the individual
respondent. The RTC issued an Order of Arrest. The CA person is now a valid subject of international law. In facts, the
declared the Order of Arrest void. On certiorari, the SC United Nations General Assembly adopted the Universal
sustained the validity of the Order and this decision became Declaration of Human Rights in which the right to life, liberty
final and executory. and all the other fundamental rights of every person were
proclaimed. While not a treaty, the principles contained in the
Meanwhile, petitioner Hong Kong Special Administrative said Declaration are now recognized as customarily binding
Region filed with the RTC of Manila a petition for the upon the members of the international community. The
extradition of private respondent. Private respondent filed, in principles set forth in that Declaration are part of the law of
the same case a petition for bail. Judge Bernardo, Jr. issued our land.
an Order denying the petition for bail, holding that there is no
Philippine law granting bail in extradition cases and that Thus, the Philippine authorities are under obligation to make
private respondent is a high "flight risk.” Judge Bernardo later available to every person under detention such remedies
inhibited himself from the case. It was then raffled off to which safeguard their fundamental right to liberty. These
Branch 8 presided by respondent judge. On motion for remedies include the right to be admitted to bail. While this
reconsideration, respondent judge allowed private Court in Purganan limited the exercise of the right to bail to
respondent to post bail. criminal proceedings, however, in light of the various
international treaties giving recognition and protection to
Petitioner filed an urgent motion to vacate the above Order, human rights, particularly the right to life and liberty, a
but it was denied by respondent judge. Hence, this petition. reexamination of this Court's ruling in Purganan is in order.
Issue: First, we note that the exercise of the State's power to deprive
Whether a potential extraditee has the right to bail. an individual of his liberty is not necessarily limited to criminal
proceedings. Respondents in administrative proceedings,
Held: such as deportation and quarantine, have likewise been
Yes. detained. Second, to limit bail to criminal proceedings would
be to close our eyes to our jurisprudential history. Philippine
Section 13, Article III of the Constitution provides that the right jurisprudence has not limited the exercise of the right to bail
to bail shall not be impaired, thus: to criminal proceedings only. This Court has admitted to bail
persons who are not involved in criminal proceedings. In fact,
Sec. 13. All persons, except those charged with
bail has been allowed in this jurisdiction to persons in
offenses punishable by reclusion perpetua when
detention during the pendency of administrative proceedings,
evidence of guilt is strong, shall, before conviction,
taking into cognizance the obligation of the Philippines under
be bailable by sufficient sureties, or be released on
international conventions to uphold human rights.
recognizance as may be provided by law. The right
to bail shall not be impaired even when the privilege If bail can be granted in deportation cases, we see no
of the writ of habeas corpus is suspended. justification why it should not also be allowed in extradition
Excessive bail shall not be required. cases. Likewise, considering that the Universal Declaration of
Human Rights applies to deportation cases, there is no
In Government of United States of America v. Hon. Guillermo
reason why it cannot be invoked in extradition cases. After all,
G. Purganan, this Court, held that the constitutional provision
both are administrative proceedings where the innocence or
on bail does not apply to extradition proceedings. It is
guilt of the person detained is not in issue.
"available only in criminal proceedings," thus:
Clearly, the right of a prospective extraditee to apply for bail
As suggested by the use of the word "conviction,"
in this jurisdiction must be viewed in the light of the various
the constitutional provision on bail quoted above, as
treaty obligations of the Philippines concerning respect for the
well as Section 4, Rule 114 of the Rules of Court,
promotion and protection of human rights. Under these
applies only when a person has been arrested and
treaties, the presumption lies in favor of human liberty. Thus,
detained for violation of Philippine criminal laws. It
the Philippines should see to it that the right to liberty of every
does not apply to extradition proceedings because
individual is not impaired.
extradition courts do not render judgments of
conviction or acquittal. Concept of Extradition
Moreover, the constitutional right to bail "flows from Section 2(a) of Presidential Decree (P.D.) No. 1069 (The
the presumption of innocence in favor of every Philippine Extradition Law) defines "extradition" as "the
accused who should not be subjected to the loss of removal of an accused from the Philippines with the object of
freedom as thereafter he would be entitled to placing him at the disposal of foreign authorities to enable the
acquittal, unless his guilt be proved beyond requesting state or government to hold him in connection with
reasonable doubt". It follows that the constitutional any criminal investigation directed against him or the

Page 174 of 186


execution of a penalty imposed on him under the penal or given the object of extradition law which is to prevent the
criminal law of the requesting state or government." prospective extraditee from fleeing our jurisdiction. In his
Separate Opinion in Purganan, then Associate Justice, now
It is not a criminal proceeding. Even if the potential extraditee Chief Justice Reynato S. Puno, proposed that a new standard
is a criminal, an extradition proceeding is not by its nature which he termed "clear and convincing evidence" should be
criminal, for it is not punishment for a crime, even though such used in granting bail in extradition cases. According to him,
punishment may follow extradition. It is sui generis, tracing its this standard should be lower than proof beyond reasonable
existence wholly to treaty obligations between different doubt but higher than preponderance of evidence. The
nations. It is not a trial to determine the guilt or innocence of potential extraditee must prove by "clear and convincing
the potential extraditee. Nor is it a fullblown civil action, but evidence" that he is not a flight risk and will abide with all the
one that is merely administrative in character. Its object is to orders and processes of the extradition court.
prevent the escape of a person accused or convicted of a
crime and to secure his return to the state from which he fled, In this case, there is no showing that private respondent
for the purpose of trial or punishment. presented evidence to show that he is not a flight risk.
Consequently, this case should be remanded to the trial court
But while extradition is not a criminal proceeding, it is to determine whether private respondent may be granted bail
characterized by the following: (a) it entails a deprivation of on the basis of "clear and convincing evidence."
liberty on the part of the potential extraditee and (b) the means
employed to attain the purpose of extradition is also "the
machinery of criminal law." Obviously, an extradition
proceeding, while ostensibly administrative, bears all
earmarks of a criminal process. A potential extraditee may be
subjected to arrest, to a prolonged restraint of liberty, and
forced to transfer to the demanding state following the
proceedings." Temporary detention" may be a necessary step
in the process of extradition, but the length of time of the
detention should be reasonable.
Records show that private respondent was arrested on
September 23, 1999, and remained incarcerated until
December 20, 2001, when the trial court ordered his
admission to bail. In other words, he had been detained for
over two (2) years without having been convicted of any
crime. By any standard, such an extended period of detention
is a serious deprivation of his fundamental right to liberty. In
fact, it was this prolonged deprivation of liberty which
prompted the extradition court to grant him bail.
While our extradition law does not provide for the grant of bail
to an extraditee, however, there is no provision prohibiting
him or her from filing a motion for bail, a right to due process
under the Constitution.
The time-honored principle of pacta sunt servanda demands
that the Philippines honor its obligations under the Extradition
Treaty it entered into with the Hong Kong Special
Administrative Region. Failure to comply with these
obligations is a setback in our foreign relations and defeats
the purpose of extradition. However, it does not necessarily
mean that in keeping with its treaty obligations, the
Philippines should diminish a potential extraditee's rights to
life, liberty, and due process. More so, where these rights are
guaranteed, not only by our Constitution, but also by
international conventions, to which the Philippines is a party.
Burden of Proof
The applicable standard of due process, however, should not
be the same as that in criminal proceedings. In the latter, the
standard of due process is premised on the presumption of
innocence of the accused. As Purganan correctly points out,
it is from this major premise that the ancillary presumption in
favor of admitting to bail arises. Bearing in mind the purpose
of extradition proceedings, the premise behind the issuance
of the arrest warrant and the "temporary detention" is the
possibility of flight of the potential extraditee. This is based on
the assumption that such extraditee is a fugitive from justice.
Given the foregoing, the prospective extraditee thus bears the
onus probandi of showing that he or she is not a flight risk and
should be granted bail.
An extradition proceeding being sui generis, the standard of
proof required in granting or denying bail can neither be the
proof beyond reasonable doubt in criminal cases nor the
standard of proof of preponderance of evidence in civil cases.
While administrative in character, the standard of substantial
evidence used in administrative cases cannot likewise apply

Page 175 of 186


3. Alejano vs. Cabuay, G.R. No. 160792; August 25, 2005. release of the person. If, however, the detention is proven
lawful, then the habeas corpus proceedings terminate. The
Facts: use of habeas corpus is thus very limited. It is not a writ of
After the Oakwood Mutiny, Gen. Abaya, as the Chief of Staff error. Neither can it substitute for an appeal.
of the AFP, issued a directive to all the Major Service Nonetheless, case law has expanded the writ's application to
Commanders to turn over custody of ten junior officers circumstances where there is deprivation of a person's
involved to the ISAFP Detention Center. The government constitutional rights. The writ is available where a person
prosecutors accused the soldiers of coup d'etat as defined
continues to be unlawfully denied of one or more of his
and penalized under Article 134-A of the Revised Penal Code constitutional freedoms, where there is denial of due process,
of the Philippines, as amended. The trial court later issued the where the restraints are not merely involuntary but are also
Commitment Orders giving custody of junior officers Lt. SG unnecessary, and where a deprivation of freedom originally
Antonio Trillanes IV ("Trillanes") and Capt. Gerardo Gambala valid has later become arbitrary.
to the Commanding Officers of ISAFP.
However, a mere allegation of a violation of one's
Petitioners later filed a petition for habeas corpus with the constitutional right is not sufficient. The courts will extend the
Supreme Court. The Court issued a scope of the writ only if any of the following circumstances is
Writ of Habeas Corpus directing respondents to make a present: (a) there is a deprivation of a constitutional right
return of the writ and to appear and produce the persons of resulting in the unlawful restraint of a person; (b) the court had
the detainees before the Court of Appeals on the scheduled no jurisdiction to impose the sentence; or (c) an excessive
date for hearing and further proceedings. Respondents penalty is imposed and such sentence is void as to the
submitted their Return of the Writ and Answer to the petition excess. Whatever situation the petitioner invokes, the
and produced the detainees before the Court of Appeals. The threshold remains high. The violation of constitutional right
Court of Appeals rendered its decision dismissing the petition must be sufficient to void the entire proceedings.
on the ground that habeas corpus is not available as the Issue no. 2:
detainees’ confinement is under a valid indictment.
Nonetheless, the appellate court ordered Gen. Cabuay, who Whether there has been a violation of detainees’ rights when
was in charge of implementing the regulations in the ISAFP respondent regulated their visits.
Detention Center, to uphold faithfully the rights of the
Held no 2:
detainees. Hence, this petition.
Petitioners admit that they do not question the legality of the No.
detention of the detainees. Neither do they dispute the lawful Petitioners contend that there was an actual prohibition of the
indictment of the detainees for criminal and military offenses. detainees' right to effective representation when petitioners'
What petitioners bewail is the regulation adopted by Gen. visits were limited by the schedule of visiting hours.
Cabuay in the ISAFP Detention Center preventing petitioners Petitioners assert that the violation of the detainees' rights
as lawyers from seeing the detainees - their clients - any time entitle them to be released from detention. This is not correct.
of the day or night. The regulation allegedly curtails the
detainees' right to counsel and violates Republic Act No. It is true that pre-trial detainees do not forfeit their
7438. Petitioners also point out that the officials of the ISAFP constitutional rights upon confinement. However, the fact that
Detention Center violated the detainees' right to privacy of the detainees are confined makes their rights more limited
communication when the ISAFP officials opened and read the than those of the public. RA 7438, which specifies the rights
personal letters of Trillanes and Capt. Milo Maestrecampo. of detainees and the duties of detention officers, expressly
Petitioners further claim that the ISAFP officials violated the recognizes the power of the detention officer to adopt and
detainees' right against cruel and unusual punishment when implement reasonable measures to secure the safety of the
the ISAFP officials prevented the detainees from having detainee and prevent his escape.
contact with their visitors. Moreover, the ISAFP officials Section 4(b) of RA 7438 provides:
boarded up with iron bars and plywood slabs the iron grills of
the detention cells, limiting the already poor light and b) Any person who obstructs, prevents or prohibits
ventilation in the detainees' cells. any lawyer, any member of the immediate family of
a person arrested, detained or under custodial
Issue no.1: investigation, or any medical doctor or priest or
Whether the detainees are entitled to the writ of habeas religious minister or by his counsel, from visiting and
corpus. conferring privately chosen by him or by any
member of his immediate family with him, or from
Held 1: examining and treating him, or from ministering to
No, neither is the prior granting of the SC of the privilege of his spiritual needs, at any hour of the day or, in
the writ determinative of this case. urgent cases, of the night shall suffer the penalty of
imprisonment of not less than four (4) years nor
In a habeas corpus petition, the order to present an individual more than six (6) years, and a fine of four thousand
before the court is a preliminary step in the hearing of the pesos (P4,000.00).
petition. The respondent must produce the person and
explain the cause of his detention. However, this order is not The provisions of the above Section
a ruling on the propriety of the remedy or on the substantive notwithstanding, any security officer with custodial
matters covered by the remedy. Thus, the Court's order to the responsibility over any detainee or prisoner may
Court of Appeals to conduct a factual hearing was not an undertake such reasonable measures as may be
affirmation of the propriety of the remedy of habeas corpus. necessary to secure his safety and prevent his
escape.
The Court of Appeals correctly ruled that the remedy of
habeas corpus is not the proper remedy to address the The schedule of visiting hours does not render void the
detainees' complaint against the regulations and conditions in detainees' indictment for criminal and military offenses to
the ISAFP Detention Center. The remedy of habeas corpus warrant the detainees' release from detention. The ISAFP
has one objective: to inquire into the cause of detention of a officials did not deny, but merely regulated, the detainees'
person. The purpose of the writ is to determine whether a right to counsel. The purpose of the regulation is not to render
person is being illegally deprived of his liberty. If the inquiry ineffective the right to counsel, but to secure the safety and
reveals that the detention is illegal, the court orders the security of all detainees.

Page 176 of 186


In our jurisdiction, the last paragraph of Section 4(b) of RA the personal letters of Trillanes and Capt. Milo
7438 provides the standard to make regulations in detention Maestrecampo.
centers allowable: "such reasonable measures as may be
Held no 4:
necessary to secure the detainee's safety and prevent his
escape." In the present case, the visiting hours accorded to No.
the lawyers of the detainees are reasonably connected to the
legitimate purpose of securing the safety and preventing the American cases recognize that the unmonitored use of pre-
escape of all detainees. trial detainees' non-privileged mail poses a genuine threat to
jail security. Hence, when a detainee places his letter in an
While petitioners may not visit the detainees any time they envelope for nonprivileged mail, the detainee knowingly
want, the fact that the detainees still have face- to-face exposes his letter to possible inspection by jail officials. A pre-
meetings with their lawyers on a daily basis clearly shows that trial detainee has no reasonable expectation of privacy for his
there is no impairment of detainees' right to counsel. incoming mail. However, incoming mail from lawyers of
Petitioners as counsels could visit their clients between 8:00 inmates enjoys limited protection such that prison officials can
a.m. and 5:00 p.m. with a lunch break at 12:00 p.m. Clearly, open and inspect the mail for contraband but could not read
the visiting hours pass the standard of reasonableness. the contents without violating the inmates' right to correspond
Moreover, in urgent cases, petitioners could always seek with his lawyer. The inspection of privileged mail is limited to
permission from the ISAFP officials to confer with their clients physical contraband and not to verbal contraband.
beyond the visiting hours.
We do not agree with the Court of Appeals that the opening
Issue no. 3: and reading of the detainees' letters in the present case
Whether respondents violated the detainees' right against violated the detainees' right to privacy of communication. The
cruel and unusual punishment when the ISAFP officials letters were not in a sealed envelope. The inspection of the
prevented the detainees from having contact with their visitors folded letters is a valid measure as it serves the same
anf boarded up with iron bars and plywood slabs the iron grills purpose as the opening of sealed letters for the inspection of
of the detention cells, limiting the already poor light and contraband.
ventilation in the detainees' cells. Besides, the letters alleged to have been read by the ISAFP
Held: authorities were not confidential letters between the
detainees and their lawyers. The petitioner who received the
No. letters from detainees Trillanes and Maestrecampo was
merely acting as the detainees' personal courier and not as
An American case, Bell v. Wolfish, pointed out that while a their counsel when he received the letters for mailing. In the
detainee may not be punished prior to an adjudication of guilt present case, since the letters were not confidential
in accordance with due process of law, detention inevitably communication between the detainees and their lawyers, the
interferes with a detainee's desire to live comfortably. The fact officials of the ISAFP Detention Center could read the letters.
that the restrictions inherent in detention intrude into the If the letters are marked confidential communication between
detainees' desire to live comfortably does not convert those the detainees and their lawyers, the detention officials should
restrictions into punishment. It is when the restrictions are not read the letters but only open the envelopes for inspection
arbitrary and purposeless that courts will infer intent to punish. in the presence of the detainees.
Courts will also infer intent to punish even if the restriction
seems to be related rationally to the alternative purpose if the That a law is required before an executive officer could intrude
restriction appears excessive in relation to that purpose. Jail on a citizen's privacy rights is a guarantee that is available
officials are thus not required to use the least restrictive only to the public at large but not to persons who are detained
security measure. They must only refrain from implementing or imprisoned. The right to privacy of those detained is subject
a restriction that appears excessive to the purpose it serves. to Section 4 of RA 7438, as well as to the limitations inherent
in lawful detention or imprisonment. By the very fact of their
An action constitutes a punishment when (1) that action detention, pre-trial detainees and convicted prisoners have a
causes the inmate to suffer some harm or "disability," and (2) diminished expectation of privacy rights.
the purpose of the action is to punish the inmate. Punishment
also requires that the harm or disability be significantly greater The ruling in this case, however, does not foreclose the right
than, or be independent of, the inherent discomforts of of detainees and convicted prisoners from petitioning the
confinement. In the present case, we cannot infer punishment courts for the redress of grievances. Regulations and
from the separation of the detainees from their visitors by iron conditions in detention and prison facilities that violate the
bars, which is merely a limitation on contact visits. The iron Constitutional rights of the detainees and prisoners will be
bars separating the detainees from their visitors prevent direct reviewed by the courts on a case-by-case basis. The courts
physical contact but still allow the detainees to have visual, could afford injunctive relief or damages to the detainees and
verbal, non-verbal and limited physical contact with their prisoners subjected to arbitrary and inhumane conditions.
visitors. The arrangement is not unduly restrictive. In fact, it is However, habeas corpus is not the proper mode to question
not even a strict noncontact visitation regulation. The conditions of confinement. The writ of habeas corpus will only
limitation on the detainees' physical contacts with visitors is a lie if what is challenged is the fact or duration of confinement.
reasonable, non-punitive response to valid security concerns.
The boarding of the iron grills is for the furtherance of security
within the ISAFP Detention Center. This measure intends to
fortify the individual cells and to prevent the detainees from
passing on contraband and weapons from one cell to another.
The boarded grills ensure security and prevent disorder and
crime within the facility. The diminished illumination and
ventilation are but discomforts inherent in the fact of
detention, and do not constitute punishments on the
detainees.
Issue no 4:
Whether respondents violated the detainees' right to privacy
of communication when the ISAFP officials opened and read

Page 177 of 186


4. Enrile v. Sandiganbayan, G.R. No. 213847; August 18, Section 7. Capital offense or an offense punishable
2015. by reclusion perpetua or life imprisonment, not
bailable. — No person charged with a capital
Facts:
offense, or an offense punishable by reclusion
The Office of the Ombudsman charged Enrile and several perpetua or life imprisonment, shall be admitted to
others with plunder in the Sandiganbayan on the basis of their bail when evidence of guilt is strong, regardless of
purported involvement in the diversion and misuse of the stage of the criminal prosecution.
appropriations under the Priority Development Assistance A capital offense in the context of the rule refers to
Fund (PDAF). Enrile respectively filed his Omnibus Motion an offense that, under the law existing at the time of
and Supplemental Opposition, praying, among others, that he its commission and the application for admission to
be allowed to post bail should probable cause be found bail, may be punished with death.
against him. The Sandiganbayan issued its resolution
denying Enrile’s motion, particularly on the matter of bail, on The general rule is, therefore, that any person, before being
the ground of its prematurity. Accordingly, the Sandiganbayan convicted of any criminal offense, shall be bailable, unless he
ordered the arrest of Enrile. Enrile voluntarily surrendered to is charged with a capital offense, or with an offense
Director Benjamin Magalong of the Criminal Investigation and punishable with reclusion perpetua or life imprisonment, and
Detection Group (CIDG) in Camp Crame, Quezon City, and the evidence of his guilt is strong. Hence, from the moment
was later on confined at the Philippine National Police (PNP) he is placed under arrest, or is detained or restrained by the
General Hospital following his medical examination. officers of the law, he can claim the guarantee of his
provisional liberty under the Bill of Rights, and he retains his
Thereafter, Enrile filed his Motion for Detention at the PNP
right to bail unless he is charged with a capital offense, or with
General Hospital, and his Motion to Fix Bail. Enrile argued an offense punishable with reclusion perpetua or life
that he should be allowed to post bail because: (a) the imprisonment, and the evidence of his guilt is strong. Once it
Prosecution had not yet established that the evidence of his has been established that the evidence of guilt is strong, no
guilt was strong; right to bail shall be recognized.
(b) although he was charged with plunder, the penalty as to On the other hand, the granting of bail is discretionary:
him would only be reclusion temporal, not reclusion perpetua;
and (c) he was not a flight risk, and his age and physical (1) upon conviction by the RTC of an offense
condition must further be seriously considered. not punishable by death, reclusion
The Sandiganbayan denied Enrile’s motion, hence this perpetua or life imprisonment; or
petition.
(2) if the RTC has imposed a penalty of
Issue: imprisonment exceeding six years, provided none of the
circumstances enumerated under paragraph 3 of Section 5,
Whether Enrile is entitled to post bail.
Rule 114 is present, as follows:
Held:
(a) That he is a recidivist, quasi-recidivist, or habitual
Yes. delinquent, or has committed the crime aggravated by
the circumstance of reiteration;
Bail protects the right of the accused to due process and to
be presumed innocent (b) That he has previously escaped from legal
confinement, evaded sentence, or violated the
In all criminal prosecutions, the accused shall be presumed conditions of his bail without valid justification;
innocent until the contrary is proved. The presumption of
innocence is rooted in the guarantee of due process, and is (c) That he committed the offense while under
safeguarded by the constitutional right to be released on bail, probation, parole, or conditional pardon;
and further binds the court to wait until after trial to impose
any punishment on the accused. (d) That the circumstances of his case indicate the
probability of flight if released on bail; or
It is worthy to note that bail is not granted to prevent the
accused from committing additional crimes. The purpose of (e) That there is undue risk that he may commit
bail is to guarantee the appearance of the accused at the trial, another crime during the pendency of the appeal.
or whenever so required by the trial court. The amount of bail Admission to bail in offenses punished by death, or life
should be high enough to assure the presence of the accused imprisonment, or reclusion perpetua is subject to judicial
when so required, but it should be no higher than is discretion
reasonably calculated to fulfill this purpose. Thus, bail acts as
a reconciling mechanism to accommodate both the accused’s For purposes of admission to bail, the determination of
interest in his provisional liberty before or during the trial, and whether or not evidence of guilt is strong in criminal cases
the society’s interest in assuring the accused’s presence at involving capital offenses, or offenses punishable with
trial. reclusion perpetua or life imprisonment lies within the
discretion of the trial court. Such discretion may be exercised
Bail may be granted as a matter of right or of discretion only after the hearing called to ascertain the degree of guilt of
The right to bail is expressly afforded by Section 13, Article III the accused for the purpose of whether or not he should be
(Bill of Rights) of the Constitution, viz.: granted provisional liberty.” It is axiomatic, therefore, that bail
cannot be allowed when its grant is a matter of discretion on
All persons, except those charged with offenses the part of the trial court unless there has been a hearing with
punishable by reclusion perpetua when evidence of notice to the Prosecution.
guilt is strong, shall, before conviction, be bailable by
sufficient sureties, or be released on recognizance Certain guidelines in the fixing of a bailbond call for the
as may be provided by law. The right to bail shall not presentation of evidence and reasonable opportunity for the
be impaired even when the privilege of the writ of prosecution to refute it. Among them are the nature and
habeas corpus is suspended. Excessive bail shall circumstances of the crime, character and reputation of the
not be required. accused, the weight of the evidence against him, the
probability of the accused appearing at the trial, whether or
This constitutional provision is repeated in Section 7, Rule not the accused is a fugitive from justice, and whether or not
114 of the Rules of Court, as follows:

Page 178 of 186


the accused is under bond in other cases. (Section 6, Rule compelling justification for his admission to bail, but which the
114, Rules of Court) It is highly doubtful if the trial court can Sandiganbayan did not recognize. Dr. Gonzales attested that
appreciate these guidelines in an ex- parte determination the following medical conditions, singly or collectively, could
where the Fiscal is neither present nor heard. The hearing, pose significant risks to the life of Enrile, to wit: (1)
which may be either summary or otherwise, in the discretion uncontrolled hypertension, because it could lead to brain or
of the court, should primarily determine whether or not the heart complications, including recurrence of stroke; (2)
evidence of guilt against the accused is strong. arrhythmia, because it could lead to fatal or non-fatal
cardiovascular events, especially under stressful conditions;
In resolving bail applications of the accused who is charged (3) coronary calcifications associated with coronary artery
with a capital offense, or an offense punishable by reclusion disease, because they could indicate a future risk for heart
perpetua or life imprisonment, the trial judge is expected to attack under stressful conditions; and (4) exacerbations of
comply with the guidelines outlined in Cortes v. Catral, to wit:
ACOS, because they could be triggered by certain
1. In all cases, whether bail is a matter of right or of circumstances (like excessive heat, humidity, dust or allergen
discretion, notify the prosecutor of the hearing of the exposure) which could cause a deterioration in patients with
application for bail or require him to submit his asthma or COPD. Based on foregoing, there is no question at
recommendation (Section 18, Rule 114 of the Rules of all that Enrile’s advanced age and ill health required special
Court, as amended); medical attention.

2. Where bail is a matter of discretion, conduct a hearing Bail for the provisional liberty of the accused, regardless of
of the application for bail regardless of whether or not the crime charged, should be allowed independently of the
the prosecution refuses to present evidence to show merits of the charge, provided his continued incarceration is
that the guilt of the accused is strong for the purpose clearly shown to be injurious to his health or to endanger his
of enabling the court to exercise its sound discretion; life. Indeed, denying him bail despite imperiling his health and
(Section 7 and 8, supra) life would not serve the true objective of preventive
incarceration during the trial.
Decide whether the guilt of the accused is strong based
on the summary of evidence of the prosecution; Granting bail to Enrile on the foregoing reasons is not
unprecedented. The Court has already held in Dela Rama v.
3. If the guilt of the accused is not strong, The People’s Court: [U]nless allowance of bail is forbidden by
law in the particular case, the illness of the prisoner,
discharge the accused upon the approval of the
independently of the merits of the case, is a circumstance,
bailbond (Section 19, supra) Otherwise petition should and the humanity of the law makes it a consideration which
be denied. should, regardless of the charge and the stage of the
proceeding, influence the court to exercise its discretion to
Enrile’s poor health justifies his admission to bail admit the prisoner to bail.
Nonetheless, in now granting Enrile’s petition for certiorari, On the other hand, to mark time in order to wait for the trial to
the Court is guided by the earlier mentioned principal purpose finish before a meaningful consideration of the application for
of bail, which is to guarantee the appearance of the accused bail can be had is to defeat the objective of bail, which is to
at the trial, or whenever so required by the court. The Court entitle the accused to provisional liberty pending the trial.
is further mindful of the Philippines’ responsibility in the There may be circumstances decisive of the issue of bail -
international community arising from the national whose existence is either admitted by the Prosecution, or is
commitment under the Universal Declaration of Human
Rights to uphold the fundamental human rights as well as properly the subject of judicial notice - that the courts can
value the worth and dignity of every person. This commitment already consider in resolving the application for bail without
is enshrined in Section II, Article II of our Constitution which awaiting the trial to finish. The Court thus balances the scales
provides: “The State values the dignity of every human of justice by protecting the interest of the People through
person and guarantees full respect for human rights.” The ensuring his personal appearance at the trial, and at the same
Philippines, therefore, has the responsibility of protecting and time realizing for him the guarantees of due process as well
promoting the right of every person to liberty and due as to be presumed innocent until proven guilty.
process, ensuring that those detained or arrested can
Accordingly, we conclude that the Sandiganbayan arbitrarily
participate in the proceedings before a court, to enable it to
ignored the objective of bail to ensure the appearance of the
decide without delay on the legality of the detention and order
accused during the trial; and unwarrantedly disregarded the
their release if justified. In other words, the Philippine
clear showing of the fragile health and advanced age of
authorities are under obligation to make available to every
Enrile.
person under detention such remedies which safeguard their
fundamental right to liberty. These remedies include the right
to be admitted to bail.
In our view, his social and political standing and his having
immediately surrendered to the authorities upon his being
charged in court indicate that the risk of his flight or escape
from this jurisdiction is highly unlikely. His personal
disposition from the onset of his indictment for plunder, formal
or otherwise, has demonstrated his utter respect for the legal
processes of this country. We also do not ignore that at an
earlier time many years ago when he had been charged with
rebellion with murder and multiple frustrated murder, he
already evinced a similar personal disposition of respect for
the legal processes, and was granted bail during the
pendency of his trial because he was not seen as a flight risk.
With his solid reputation in both his public and his private
lives, his long years of public service, and history’s judgment
of him being at stake, he should be granted bail.
The currently fragile state of Enrile’s health presents another

Page 179 of 186


4.2 Enrile v. Sandiganbayan, G.R. No. 213847; July 12, Admission to bail always involves the risk that the accused
2016 will take flight. This is the reason precisely why the probability
or the improbability of flight is an important factor to be taken
Facts:
into consideration in granting or denying bail, even in capital
This is a Motion for Reconsideration filed by the People of the cases. The exception to the fundamental right to bail should
Philippines on the previous case. People asserts that the be applied in direct ratio to the extent of the probability of
decision unduly and radically modified constitutional and evasion of prosecution. Apparently, an accused's official and
procedural principles governing bail. social standing and his other personal circumstances are
considered and appreciated as tending to render his flight
Issue: improbable.
Whether Enrile is entitled to bail. The petitioner has proven with more than sufficient evidence
Held: that he would not be a flight risk. For one, his advanced age
and fragile state of health have minimized the likelihood that
Yes. he would make himself scarce and escape from the
jurisdiction of our courts.
The Court finds no compelling or good reason to reverse its
decision.
Firstly, the People were not denied the reasonable
opportunity to challenge or refute the allegations about his
advanced age and the instability of his health even if the
allegations had not been directly made in connection with his
Motion to Fix Bail.
Secondly, the imputation of "preferential treatment" in "undue
favor" of the petitioner is absolutely bereft of basis. A reading
of the decision of August 18, 2015 indicates that the Court did
not grant his provisional liberty because he was a sitting
Senator of the Republic. It did so because there were proper
bases - legal as well as factual - for the favorable
consideration and treatment of his plea for provisional liberty
on bail. By its decision, the Court has recognized his right to
bail by emphasizing that such right should be curtailed only if
the risks of flight from this jurisdiction were too high. In our
view, however, the records demonstrated that the risks of
flight were low, or even nil. The Court has taken into
consideration other circumstances, such as his advanced age
and poor health, his past and present disposition of respect
for the legal processes, the length of his public service, and
his individual public and private reputation. There was really
no reasonable way for the Court to deny bail to him simply
because his situation of being 92 years of age when he was
first charged for the very serious crime in court was quite
unique and very rare. To ignore his advanced age and
unstable health condition in order to deny his right to bail on
the basis alone of the judicial discretion to deny bail would be
probably unjust. To equate his situation with that of the other
accused indicted for a similarly serious offense would be
inherently wrong when other conditions significantly
differentiating his situation from that of the latter's
unquestionably existed.
Section 2, Rule 114 of the Rules of Court expressly states that
one of the conditions of bail is for the accused to "appear
before the proper court whenever required by the court or
these Rules." The practice of bail fixing supports this purpose.
Bail exists to ensure society's interest in having the accused
answer to a criminal prosecution without unduly restricting his
or her liberty and without ignoring the accused's right to be
presumed innocent. It does not perform the function of
preventing or licensing the commission of a crime. The notion
that bail is required to punish a person accused of crime is,
therefore, fundamentally misplaced. Indeed, the practice of
admission to bail is not a device for keeping persons in jail
upon mere accusation until it is found convenient to give them
a trial. The spirit of the procedure is rather to enable them to
stay out of jail until a trial with all the safeguards has found
and adjudged them guilty. Unless permitted this conditional
privilege, the individuals wrongly accused could be punished
by the period of imprisonment they undergo while awaiting
trial, and even handicap them in consulting counsel,
searching for evidence and witnesses, and preparing a
defense. Hence, bail acts as a reconciling mechanism to
accommodate both the accused's interest in pretrial liberty
and society's interest in assuring his presence at trial

Page 180 of 186


5. Qui vs. People, G.R. No. 196161; September 26, 2012. 6. Re: Conviction of Judge Adoracion Angeles, A.M. No.
06-9-545-RTC; January 31, 2008.
Facts:
Facts:
Petitioner Cyril Calpito Qui was convicted by the RTC of
violation of Section 10(a), Article VI of Republic Act No. (RA) The RTC convicted Judge Angeles of violation of Republic
7610 or the Special Protection of Children Against Child Act (RA) No. 7610. Senior State Prosecutor Emmanuel Y.
Abuse, Exploitation and Discrimination Act and sentenced her Velasco (SSP Velasco) of the Department of Justice (DOJ)
to two equal periods of imprisonment for an indeterminate wrote a letter to then Chief Justice Artemio V. Panganiban
penalty of five (5) years, four (4) months and twenty one (21) inquiring whether it is possible for this Court, in the public
days of prision correccional in its maximum period, as interest, motu proprio to order the immediate suspension of
minimum, to seven (7) years, four (4) months and one (1) day the respondent in view of the aforementioned RTC Decision.
of prision mayor in its minimum period, as maximum. On the basis of SSP Velasco's letter, the OCA recommended
Petitioner filed her Notice of Appeal. With the perfection of her that she be indefinitely suspended pending the outcome of
appeal and the consequent elevation of the case records to the case or until further orders from the Court.
the CA, petitioner posthaste filed before the appellate court
Issue:
an Urgent Petition/Application for Bail Pending Appeal. The
CA denied petitioner’s application for bail pending appeal on Whether Judge Angeles should be suspended pending the
the basis of Sec. 5(d) of Rule 114, Revised Rules of Criminal determination of her criminal case.
Procedure.
Held:
Issue:
No.
Whether Qui is entitled to bail pending appeal.
The Court cannot fully agree with the recommendation of the
Held: OCA.
Under the present rule, the grant of bail is a matter of It is settled that conviction in the criminal case will not
discretion upon conviction by the RTC of an offense not automatically warrant a finding of guilt in the administrative
punishable by death, reclusion perpetua or life imprisonment, case. Criminal and civil cases are altogether different from
as here. administrative matters, and each must be disposed of
In the exercise of that discretion, the proper courts are to be according to the facts and the law applicable to it.
guided by the fundamental principle that the allowance of bail In Nunez v. Atty. Arturo B. Astorga, the Court held that the
pending appeal should be exercised not with laxity but with mere existence of pending criminal charges against the
grave caution and only for strong reasons, considering that respondent-lawyer cannot be a ground for disbarment or
the accused has been in fact convicted by the trial court. suspension of the latter. To hold otherwise would open the
The CA properly exercised its discretion in denying door to harassment of attorneys through the mere filing of
petitioner’s application for bail pending appeal. The CA’s numerous criminal cases against them.
determination as to petitioner being a high risk for flight is not By parity of reasoning, the fact of respondent’s conviction by
without factual mooring. Indeed, the undisputed fact that the RTC does not necessarily warrant her suspension. We
petitioner did not attend the hearings before the RTC, which agree with respondent's argument that since her conviction of
compelled the trial court to issue warrants for her arrest, is the crime of child abuse is currently on appeal before the CA,
undeniably indicative of petitioner’s propensity to trifle with the same has not yet attained finality. As such, she still enjoys
court processes. This fact alone should weigh heavily against the constitutional presumption of innocence. It must be
a grant of bail pending appeal. remembered that the existence of a presumption indicating
Petitioner’s argument that she has the constitutional right to the guilt of the accused does not in itself destroy the
bail and that the evidence of guilt against her is not strong is constitutional presumption of innocence unless the
spurious. Certainly, after one is convicted by the trial court, inculpating presumption, together with all the evidence, or the
the presumption of innocence, and with it, the constitutional lack of any evidence or explanation, proves the accused's
right to bail, ends. guilt beyond a reasonable doubt. Until the accused's guilt is
shown in this manner, the presumption of innocence
continues

Page 181 of 186


7. People vs. Caoili, G.R. No. 196342. August 8, 2017, G.R. R.A. 8353, with the aggravating circumstance that the
No. 196848. August 8, 2017. accused is the father of the victim and R.A. 7610[.]
Doctrines: The RTC declared Caoili guilty of rape by sexual assault.
Thereafter, Caoili filed his appeal before the CA.
Variance Doctrine; By jurisprudence, an accused charged in
the Information with rape by sexual intercourse cannot be The CA held that although Caoili is clearly guilty of rape by
found guilty of rape by sexual assault, even though the latter sexual assault, what the trial court should have done was to
crime was proven during trial. —The language of paragraphs direct the State Prosecutor to file a new Information charging
1 and 2 of Article 266-A of the RPC, as amended by R.A. No. the proper offense, and after compliance therewith, to dismiss
8353, provides the elements that substantially differentiate the original Information. The appellate court found it
the two forms of rape, i.e., rape by sexual intercourse and “imperative and morally upright” to set the judgment aside and
rape by sexual assault. It is through legislative process that to remand the case for further proceedings pursuant to
the dichotomy between these two modes of rape was created. Section 14, Rule 110.
To broaden the scope of rape by sexual assault, by
Issue:
eliminating its legal distinction from rape through sexual
intercourse, calls for judicial legislation which We cannot WHETHER RAPE BY SEXUAL ASSAULT IS NECESSARILY
traverse without violating the principle of separation of INCLUDED IN RAPE BY SEXUAL INTERCOURSE.
powers. The Court remains steadfast in confining its powers
within the constitutional sphere of applying the law as enacted Ruling:
by the Legislature. In fine, given the material distinctions Rape by sexual assault is not subsumed in rape through
between the two modes of rape introduced in R.A. No. 8353, sexual intercourse.
the variance doctrine cannot be applied to convict an accused
of rape by sexual assault if the crime charged is rape through The Court did not accept the OSG’s argument that based on
sexual intercourse, since the former offense cannot be the variance doctrine, Caoili can be convicted of rape by
considered subsumed in the latter. sexual assault because this offense is necessarily included in
the crime of rape through sexual intercourse.
Leonen, Dissenting
The variance doctrine, which allows the conviction of an
Constitutional Law; Right to be Informed; View that the right accused for a crime proved which is different from but
to be informed of the nature and cause of the accusations necessarily included in the crime charged, is embodied in
against a person need not be alleged with the highest degree Section 4, in relation to Section 5 of Rule 120 of the Rules of
of particularity. It is satisfied as long as facts are alleged with Court which reads:
sufficient clarity that allows the accused to understand what
acts he is being made liable for in order to enable him to make Sec. 4. Judgment in case of variance between allegation and
a defense.—The accused may be convicted of rape by sexual proof.—When there is variance between the offense charged
intercourse without violating his due process rights and his in the complaint or information and that proved, and the
right to be informed of the nature and cause of the offense as charged is included in or necessarily includes the
accusations against him as provided in Article III, Section 14 offense proved, the accused shall be convicted of the offense
of the 1987 Constitution and reproduced in Rule 115, Section proved which is included in the offense charged, or of the
1(b) of our Rules of Procedure. The importance and purpose offense charged which is included in the offense proved.
of this rule has been explained by this Court in People v. (Emphasis ours)
Quitlong, 292 SCRA 360 (1998): First. To furnish the accused
Sec. 5. When an offense includes or is included in another.—
with such a description of the charge against him as will
An offense charged necessarily includes the offense proved
enable him to make his defense; and second, to avail himself
when some of the essential elements or ingredients of the
of his conviction or acquittal for protection against a further
former, as alleged in the complaint or information, constitute
prosecution for the same cause, and third, to inform the court
the latter. And an offense charged is necessarily included in
of the facts alleged, so that it may decide whether they are
the offense proved, when the essential ingredients of the
sufficient in law to support a conviction, if one should be had.
former constitute or form part of those constituting the latter.
The right to be informed of the nature and cause of the
accusations against a person need not be alleged with the By jurisprudence, however, an accused charged in the
highest degree of particularity. It is satisfied as long as facts Information with rape by sexual intercourse cannot be found
are alleged with sufficient clarity that allows the accused to guilty of rape by sexual assault, even though the latter crime
understand what acts he is being made liable for in order to was proven during trial. This is due to the substantial
enable him to make a defense. distinctions between these two modes of rape.
Facts: In the first mode (rape by sexual intercourse): (1) the offender
is always a man; (2) the offended party is always a woman;
On June 22, 2006, First Assistant Provincial Prosecutor Raul
(3) rape is committed through penile penetration of the
O. Nasayao filed an Information against Caoili, charging him
vagina; and (4) the penalty is reclusion perpertua.
with the crime of rape through sexual intercourse in violation
of Article 266-A, in relation to Article 266-B, of the RPC as In the second mode (rape by sexual assault): (1) theoffender
amended by R.A. No. 8353, and R.A. No. 7610. The may be a man or a woman; (2) the offended party may be a
accusatory portion of the Information reads: man or a woman; (3) rape is committed by inserting the penis
into another person’s mouth or anal orifice, or any instrument
That on or about the 23rd day of October 2005, at 7:00 o’clock
or object into the genital or anal orifice of another person; and
in the evening, more or less, in Purok [III], Barangay [JJJ],
(4) the penalty is prisión mayor.
[KKK], [LLL], Philippines, and within the jurisdiction of this
Honorable Court, the above named accused, with full Given the material distinctions between the two modes of rape
freedom and intelligence, with lewd design, did, then and introduced in R.A. No. 8353, the variance doctrine cannot be
there, willfully, unlawfully and feloniously had sexual applied to convict an accused of rape by sexual assault if the
intercourse with one [AAA],7 a minor, fifteen (15) years of age crime charged is rape through sexual intercourse, since the
and the daughter of the herein accused, through force, threat former offense cannot be considered subsumed in the latter.
and intimidation and against her will, to her damage and
prejudice in the amount as may be allowed by law. The Court, thus, took the occasion to once again remind
public prosecutors of their crucial role in drafting criminal
CONTRARY TO Article 266-A, in relation to Article 266-B of complaints or Information. They have to be more judicious

Page 182 of 186


and circumspect in preparing the Information since a mistake 8. RE: PETITION FOR RADIO AND TELEVISION
or defect therein may not render full justice to the State, the COVERAGE OF THE MULTIPLE MURDER CASES
offended party and even the offender. AGAINST MAGUINDANAO GOVERNOR ZALDY
Variance Doctrine Applied AMPATUAN, ET AL.

Caoili had been charged with rape through sexual intercourse FACTS:
in violation of Article 266-A of the RPC and R.A. No. 7610.
On November 23, 2009, 57 people including 32 journalists
Applying the variance doctrine under Section 4, in relation to
and media practitioners were killed on their way to Shariff
Section 5 of Rule 120 of the Revised Rules of Criminal
Aguak in Maguindanao. This tragic incident came to be
Procedure, Caoili can be held guilty of the lesser crime of acts
known as Maguindanao massacre" spawned charges for 57
of lasciviousness performed on a child, i.e., lascivious
counts of murder and additional charges of rebellion against
conduct under Section 5(b) of R.A. No. 7610, which was the
197 accused, commonly entitled People v. Datu Andal
offense proved, because it is included in rape, the offense
Ampatuan, Jr., et al. Following the transfer of venue and the
charged. This echoes the Court’s pronouncement in
re-raffling of the cases, the cases are being tried by Presiding
Leonardo, viz.:
Judge Jocelyn Solis-Reyes of Branch 221 of RTC Quezon
This Court holds that the lower courts properly convicted the City. Almost a year later, the National Union of Journalists of
the Philippines (NUJP), ABSCBN Broadcasting Corporation,
appellant in Criminal Case Nos. 546-V-02, 547-V-02, 548-V- GMA Network Inc., relatives of the victims, individual
02, 554-V-02 and 555-V-02 for five counts of sexual abuse journalists from various media entities and members of the
under Section 5(b), Article III of Republic Act No. 7610 even academe filed a petition before this court praying that live
though the charges against him in the aforesaid criminal television and radio coverage of the trial in this criminal cases
cases were for rape in relation to Republic Act No. 7610. The be allowed, recording devises be permitted inside the court
lower court[’s] ruling is in conformity with the variance doctrine room to assist the working journalists, and reasonable
embodied in Section 4, in relation to Section 5, Rule 120 of guidelines be formulated to govern the broadcast coverage
the Revised Rules of Criminal Procedure, x x x: and the use of devices. President Benigno
This Court holds that the lower courts properly convicted the S. Aquino III, letter addressed to Chief Justice Renato
appellant in Criminal Case Nos. 546-V-02, 547-V-02, 548-V- Corona, came out in support of those who have petitioned this
02, 554-V-02 and 555-V-02 for five counts of sexual abuse Court to permit television and radio broadcast of the trial.
under Section 5(b), Article III of Republic Act No. 7610 even Petitioners state that the trial of the Maguindanao Massacre
though the charges against him in the aforesaid criminal cases has attracted intense media coverage due to the
cases were for rape in relation to Republic Act No. 7610. The gruesomeness of the crime, prominence of the accused, and
lower court[’s] ruling is in conformity with the variance doctrine the number of media personnel killed. They inform that
embodied in Section 4, in relation to Section 5, Rule 120 of reporters are being frisked and searched for cameras,
the Revised Rules of Criminal Procedure, x x x: recorders, and cellular devices upon entry, and that under
strict orders of the trial court against live broadcast coverage,
xxxx the number of media practitioners allowed inside the
With the aforesaid provisions, the appellant can be held guilty courtroom has been limited to one reporter for each media
of a lesser crime of acts of lasciviousness performed on a institution. Hence, the present petitions which assert the
child, i.e., sexual abuse under Section 5(b), Article III of exercise of right to a fair and public trial and the lifting of the
Republic Act No. 7610, which was the offense proved absolute ban on live television and radio coverage of court
because it is included in rape, the offense charged. proceedings. They principally urge the Court to revisit the
1991 ruling in Re: Live TV and Radio Coverage of the Hearing
The due recognition of the constitutional right of an accused of President Corazon C. Aquinos Libel Case and the 2001
to be informed of the nature and cause of the accusation ruling in Re: Request Radio-TV Coverage of the Trial in the
through the criminal complaint or information is decisive of Sandiganbayan of the Plunder Cases Against the Former
whether his prosecution for a crime stands or not. President Joseph E. Estrada which rulings, they contend,
Nonetheless, the right is not transgressed if the information violate the doctrine that proposed restrictions on
sufficiently alleges facts and omissions constituting an constitutional rights are to be narrowly construed and outright
prohibition cannot stand when regulation is a viable
offense that includes the offense established to have been
alternative.
committed by the accused, which, in this case, is lascivious
conduct under Section 5(b) of R.A. No. 7610. ISSUE:
WON the petition for radio and television coverage of the
Maguindanao Massacre should be allowed
HELD:
The Court partially GRANTS pro hac vice petitioners’ prayer
for a live broadcast of the trial court proceedings, subject to
guidelines. Respecting the possible influence of media
coverage on the impartiality of trial court judges, petitioners
correctly explain that prejudicial publicity insofar as it
undermines the right to a fair trial must pass the totality of
circumstances test, applied in People v. Teehankee, Jr. and
Estrada v. Desierto, that the right of an accused to a fair trial
is not incompatible to a free press, that pervasive publicity is
not per se prejudicial to the right of an accused to a fair trial,
and that there must be allegation and proof of the impaired
capacity of a judge to render a bias-free decision. Mere fear
of possible undue influence is not tantamount to actual
prejudice resulting in the deprivation of the right to a fair trial.
On public trial, Estrada basically discusses: An accused has
a right to a public trial but it is a right that belongs to him, more

Page 183 of 186


than anyone else, where his life or liberty can be held critically effects weighs heavily against broadcasting the trial.
in balance. A public trial aims to ensure that he is fairly dealt Moreover, the fact that the accused has legal remedies after
with and would not be unjustly condemned and that his rights the fact is of no moment, since the damage has been done
are not compromised in secrete conclaves of long ago. A and may be irreparable. It must be pointed out that the
public trial is not synonymous with publicized trial; it only fundamental right to due process of the accused cannot be
implies that the court doors must be open to those who wish afforded after the fact but must be protected at the first
to come, sit in the available seats, conduct themselves with instance.
decorum and observe the trial process.
The basic principle upheld in Aquino is firm. A trial of any kind
or in any court is a matter of serious importance to all
concerned and should not be treated as a means of
entertainment, and to so treat it deprives the court of the
dignity which pertains to it and departs from the orderly and
serious quest for truth for which our judicial proceedings are
formulated. The observation that massive intrusion of
representatives of the news media into the trial itself can so
alter and destroy the constitutionally necessary atmosphere
and decorum stands. The Court had another unique
opportunity in Estrada to revisit the question of live radio and
television coverage of court proceedings in a criminal case. It
held that the propriety of granting or denying the instant
petition involves the weighing out of the constitutional
guarantees of freedom of the press and the right to public
information, on the one hand, and the fundamental rights of
the accused, on the other hand, along with the constitutional
power of a court to control its proceedings in ensuring a fair
and impartial trial In so allowing pro hac vice the live
broadcasting by radio and television of the Maguindanao
Massacre cases.
RESOLUTION October 23
Petitioners Tiamzon and Legarta take issue on provisos (g)
and (h) of the enumerated guidelines in the June 14, 2011
Resolution and allege that these must be struck down for
being unconstitutional, as they constitute prior restraint on
free expression because they dictate what media can and
cannot report about the "Maguindanao massacre" trial.
(g) To avoid overriding or superimposing the audio
output from the on-going proceedings, the proceedings shall
be broadcast without any voiceovers, except brief annotations
of scenes depicted therein as may be necessary to explain
them at the start or at the end of the scene. Any commentary
shall observe the sub judice rule and be subject to the
contempt power of the court;
(h) No repeat airing of the audio-visual recording shall
be allowed until after the finality of judgment, except brief
footages and still images derived from or cartographic
sketches of scenes based on the recording, only for news
purposes, which shall likewise observe the sub-judice rule
and be subject to the contempt power of the court
Accused Ampatuan also filed a Motion for Reconsideration,
alleging that the June 14, 2011 Resolution "deprives him of
his rights to due process, equal protection, presumption of
innocence, and to be shielded from degrading psychological
punishment." This Court partially grants reconsideration of the
June 14, 2011 Resolution and deny the Partial Motion for
Reconsideration of petitioners Editha Mirandilla Tiamzon and
Glenna Legarta. The Court is now disallowing the live media
broadcast of the trial of the "Maguindanao massacre" cases
but is still allowing the filming of the proceedings for (1) the
real-time transmission to specified viewing areas, and
(2) documentation. In a constitutional sense, public trial is
not synonymous with publicized trial. The right to a public trial
belongs to the accused. The requirement of a public trial is
satisfied by the opportunity of the members of the public and
the press to attend the trial and to report what they have
observed. The accused's right to a public trial should not be
confused with the freedom of the press and the public's right
to know as a justification for allowing the live broadcast of the
trial. The tendency of a high profile case like the subject case
to generate undue publicity with its concomitant undesirable

Page 184 of 186


9. Go v. People, 677 SCRA 213, 2012 Rules of Criminal Procedure. The pertinent provision reads
thus:
Facts:
SEC. 15. Examination of witness for the prosecution. - When
Petitioners Harry Go, Tonny Ngo, Jerry Ngo and Jane Go it satisfactorily appears that a witness for the prosecution is
were charged before MeTC of Manila for Other Deceits under too sick or infirm to appear at the trial as directed by the court,
Article 318 of the RPC. Sometime in August 1996 accused, or has to leave the Philippines with no definite date of
conspiring, confederating together and helping one another returning, he may forthwith be conditionally examined before
defraud Highdone Company Ltd. Represented by Li Luen the court where the case is pending. Such examination, in the
Ping. Made Li Luen Ping believed that they have chattels such presence of the accused, or in his absence after reasonable
as machinery, spare parts, equipment and raw materials notice to attend the examination has been served on him shall
installed and fixed in the premises of BGB Industrial Textile be conducted in the same manner as an examination at the
Mills Factory located in the Bataan Export Processing Zone trial. Failure or refusal of the accused to attend the
(BEPZ) in Mariveles, Bataan, executed a Deed of Mortgage examination after notice shall be considered a waiver. The
for a consideration of the amount of P21M more or less in statement taken may be admitted in behalf of or against the
favor of ML Resources and Highdone Company Ltd. accused.
Representing that the said deed is a FIRST MORTGAGE
when in truth and in fact the accused well knew that the same Certainly, to take the deposition of the prosecution witness
had been previously encumbered, mortgaged and foreclosed elsewhere and not before the very same court where the case
by CHINA BANK CORPORATION as early as September is pending would not only deprive a detained accused of his
1994 thereby causing damage and prejudice to said right to attend the proceedings but also deprive the trial judge
HIGHDONE COMPANY LTD. The prosecution's complaining of the opportunity to observe the prosecution witness'
witness, Li Luen Ping, a frail old businessman from Laos, deportment and properly assess his credibility, which is
Cambodia, traveled from his home country back to the especially intolerable when the witness' testimony is crucial to
Philippines in order to attend the hearing held on September the prosecution's case against the accused. This is the import
9, 2004. However, trial dates were subsequently postponed of the Court's ruling in Vda. de Manguerra where we further
due to his unavailability. The private prosecutor filed with the declared that
MeTC a Motion to Take Oral Deposition6 of Li Luen Ping, While we recognize the prosecution's right to preserve the
alleging that he was being treated for lung infection at the testimony of its witness in order to prove its case, we cannot
Cambodia Charity Hospital in Laos, Cambodia and that, upon disregard the rules which are designed mainly for the
doctor's advice, he could not make the long travel to the protection of the accused's constitutional rights. The giving of
Philippines by reason of ill health. Notwithstanding petitioners' testimony during trial is the general rule. The conditional
Opposition, the MeTC granted the motion after the examination of a witness outside of the trial is only an
prosecution complied with the directive to submit a Medical exception, and as such, calls for a strict construction of the
Certificate of Li Luen Ping. Petitioners sought its rules.
reconsideration which the MeTC denied, prompting
petitioners to file a Petition for Certiorari before the RTC.
Upon denial by the RTC of their motion for reconsideration
through an Order dated March 5, 2006, the prosecution
elevated the case to the CA. the CA denied petitioners' motion
for Reconsideration.
Issue:

WON CA erred in sustaining the judicial legislation committed


by the MeTC in applying the ruled on deposition-taking in civil
case to criminal cases.
The examination of witnesses must be done orally before a
judge in open court. This is true especially in criminal cases
where the Constitution secures to the accused his right to a
public trial and to meet the witnesses against him face to face.
The requirement is the “safest and most satisfactory method
of investigating facts” as it enables the judge to test the
witness' credibility through his manner and deportment while
testifying. It is not without exceptions, however, as the Rules
of Court recognizes the conditional examination of witnesses
and the use of their depositions as testimonial evidence in lieu
of direct court testimony.
The procedure under Rule 23 to 28 of the Rules of Court
allows the taking of depositions in civil cases, either upon oral
examination or written interrogatories, before any judge,
notary public or person authorized to administer oaths at any
time or place within the Philippines; or before any Philippine
consular official, commissioned officer or person authorized
to administer oaths in a foreign state or country, with no
additional requirement except reasonable notice in writing to
the other party.
But for purposes of taking the deposition in criminal cases,
more particularly of a prosecution witness who would
foreseeably be unavailable for trial, the testimonial
examination should be made before the court, or at least
before the judge, where the case is pending as required by
the clear mandate of Section 15, Rule 119 of the Revised

Page 185 of 186


9. Jaylo v. Sandiganbayan, 746 SCRA 452 (2015) of Court diminishes or modifies the substantive rights of
petitioners. It only works in pursuance of the power of the
Facts: Supreme Court to "provide a simplified and inexpensive
In a decision dated April 17 2007, the Sandiganbayan found procedure for the speedy disposition of cases." This provision
petitioners Jaylo, Castro, Valenzona and Habalo guilty of protects the courts from delay in the speedy disposition of
homicide. During the promulgation of the Sandiganbayan’s criminal cases - delay arising from the simple expediency of
nonappearance of the accused on the scheduled
judgment, none of the accused appeared despite notice.
Thus, the decision was promulgated in absentia and the promulgation of the judgment of conviction.
judgment entered in the criminal docket. Their bail bonds In this case, petitioners have just shown their lack of faith in
were also cancelled and warrants for their arrest issued. the jurisdiction of the Sandiganbayan by not appearing before
it for the promulgation of the judgment on their cases. Surely
The petitioners filed a Motion for Partial Reconsideration of
the abovementioned decision, but the Sandiganbayan took they cannot later on expect to be allowed to invoke the
no action on said motion and ordered the implementation of Sandiganbayan’s jurisdiction to grant them relief from its
the arrest warrants, holding that the 15-dayh period from the judgment of conviction.
promulgation of the judgment had long passed without any of 3. YES. It is well to note that Section 6, Rule 120, of the
the accused giving any reason for their non-appearance Rules of Court also provides the remedy by which the
during the promulgation. Applying Section 6, Rule 120 of the accused who were absent during the promulgation may
Rules of Court, the Sandiganbayan held that the accused reverse the forfeiture of the remedies available to them
have lost the remedies available to them under the Rules of against the judgment of conviction. In order to regain their
Court against the Sandiganbayan’s judgment of conviction, standing in court, the accused must do as follows: 1)
including the filing of a motion for reconsideration. The surrender and 2) file a motion for leave of court to avail of the
petitioners filed a petition for certiorari with the Supreme remedies, stating the reasons for their absence, within 15
Court, arguing that Section 6, Rule 120 cannot diminish, days from the date of the promulgation of judgment.
modify or increase substantive rights like the filing of an MR
under P.D. 1606. Petitioners also argued that even if Section In Villena v. People, we stated that the term "surrender"
6, Rule 120 were applied, the conditions under which the said contemplates the act by the convicted accused of physically
rules may be applied do not obtain in this case because it was and voluntarily submitting themselves to the jurisdiction of the
“it was incumbent upon the Sandiganbayan to take pains to court to suffer the consequences of the judgment against
find out whether their absence at the promulgation was them. Upon surrender, the accused must request permission
without justifiable cause, and only then could the court of the court to avail of the remedies by making clear the
conclude that petitioners have lost the remedies available in reasons for their failure to attend the promulgation of the
the Rules of Court against the judgment of conviction.”. judgment of conviction.

Issue: Clearly, the convicted accused are the ones who should show
that their reason for being absent at the promulgation of
1. WON the petitioners, who failed to appear at the judgment was justifiable. If the court finds that the reasons
promulgation of judgment, has any standing in court and has proffered justify their nonappearance during the promulgation
the right to seek relief? of judgment, it shall allow them to avail of the remedies. Thus,
unless they surrender and prove their justifiable reason to the
2. WON Section 6, Rule 120 diminished or modified satisfaction of the court, their absence is presumed to be
the substantive rights of the petitioners? unjustified.
3. WON Section 6, Rule 120 applies to this case?
Held:

1. No. Section 6, Rule 120 provides that an accused


who failed to an accused who failed to appear at the
promulgation of the judgment of conviction shall lose the
remedies available against the said judgment.
If the judgment is for conviction and the failure to appear was
without justifiable cause, the accused shall lose the remedies
available in the Rules of Court against the judgment. Thus, it
is incumbent upon the accused to appear on the scheduled
date of promulgation, because it determines the availability of
their possible remedies against the judgment of conviction.
When the accused fail to present themselves at the
promulgation of the judgment of conviction, they lose the
remedies of filing a motion for a new trial or reconsideration
(Rule 121) and an appeal from the judgment of conviction
(Rule 122). The reason is simple. When the accused on bail
fail to present themselves at the promulgation of a judgment
of conviction, they are considered to have lost their standing
in court. Without any standing in court, the accused cannot
invoke its jurisdiction to seek relief.
2. No. Section 6, Rule 120, of the Rules of Court, does
not take away per se the right of the convicted accused to
avail of the remedies under the Rules. It is the failure of the
accused to appear without justifiable cause on the scheduled
date of promulgation of the judgment of conviction that forfeits
their right to avail themselves of the remedies against the
judgment.
It is not correct to say that Section 6, Rule 120, of the Rules

Page 186 of 186

You might also like